Download MATEMATICOS

Document related concepts

Triángulo wikipedia , lookup

Triángulo de Kepler wikipedia , lookup

Transcript
Prácticas para Resolver
PROBLEMAS
MATEMATICOS
/
V Gi1sev.
[( Litvinenlf.o.
A. Mordkóuich
Editorial Mir Moscú
Prácticas para Resolver
PROBLEMAS
MATEMÁTICOS
B. A. ryccs, B. H. JlnTB1mem<o, A.
no
r.
Mopp;KOBD'I
DPA}{THHYM
PEll.lEHHIO MATEMATHllECIHIX 3A)J.All
reoMeTpBS!
MocKDa •flpoc1Jell1euuc•
Práaicas para Resolver
PROBLEMAS
MATEMÁTICOS
V.Gúsiev,
V. Litvinenko,
A Mordkóvich
t;eometría
~
Editorial Mir Moscú
Traducido del ruso por el ingeniero
Antonio Ballesteros EH.as
lmprmC> en la URSS
Ha ncnaecKO)! Rab!Ke
ISBN 5-()3.()00669·9
©
©
, npoCJlOm,cunc·• . 1985
traducción al español. editorial Mir, f989
1'13Jl.8TC116CTOO
INDICE
Prólogo
Capltulo l. PLANIMETRIA
§ 1. Sobre los métodos para resolver problemas geométricos
§ 2. Triángulos y cuadriláteros
Problemas para el trabajo individual
§ 3. ClrcunJcrcncin
Problemas para el trabojo individual
4. Areai· de las figuras planas
Problemas para el trabajo individual
5. Transformaciones geométricas
Problemas para el trabajo individual
6. Vectores
Problemas para el trabojo individual
7. Valores máximos y mínimos
Problemas para el trabajo individual
Capftulo /l. ESTEREOMETRfA
§ 8 . Generalidades sobro In constnicciún de. la representación do unn
figura dada
9. Construcciones geométricas en el espacio
Problemas para ol trabajo individual
t o. Rectas cruzadas. Angulo entre una recta y un plano
Problemas para el trabajo individual
§ 11. Ángulos diedros y poliedros
Probfomas para el trabajo individual
§ 12. Secciones de poliedros
Problemas para ol trabajo individual
§ 13. Arcas
Problemas p4ra el trabajo individual
14. Volúmenes
Problemas para el trabajo individual
§ 15. Combinación de poliedros y cuerpos redondos
Problemas parn el trabajo individual
§ t6. Valores máximos y mínimos
Problemas para el trabajo individual
Soluciones e indicaciones
Bibliografía
6
9
2t
29
3:;
42
4!)
60
li7
71
76
87
96
105
108
120
132
13&
14&
149
153
t55
166
169
177
179
187
HlO
1117
201
207
210
245
PROLOGO
El p¡'l¡sentemanual está dirigido a los estudiantes de las facultades matemáticas y fisicomatemátícas de las Escuelas Normales
Supe.riores para las especialidades «MatemáticM y «Matemática y
física» y, además, pnra Ja especialidad «Física y matemática.». Está
confeccionado en correspondencia con el programa en vigor «Prácticas de resolución de problemas».
Al trabnjnr en las «Prácticas» tendíamos a que en ellas encontraran su reflejo los tipos funcl.amentales de problemas geométricos
escolares. En el presente libro hay cerca de 1000 problemas, de
diversa complejidad para la resolución individual. Junto con problemas comparativamente sencillos, con carácter de entrenamiento,
hay problemas cuya resolución requiere serias reflexiones y; en
ocasiones, enfoque no estándar. Aunque no sea de todos, la resolución
de una considerable parte de los problemas, ayudará al estudiante a
la formación de tan importante cualidad profesional para el futuro
profesor de matemáticas como el hábito de resolver problemas geométricos que correspondan a los requisitos de los programas de matemáticas de las escuelas medias de enseñania general y profesional.
Los procedimientos y métodos para resolver problemas geomé tricos SG analiian en dHerentes partes del curso de georuetria estudiado
en las Escuelas Normales Superiores. No obstante, a los métodos
tradicionales se presta insuficiente atención, por lo que nno de los
objetivos planteados en el proceso de la creación de este manual era
completar esos huecos.
Seiialemo's que el libro que ofrecemos a la atención. del lector no
sólo es un compendio de problemas en su sentido habitual, sino,
además, un libro de prácticas para resolver problemas. Es.t o ha
hallado su reflejo en el contenido y en la estructura de nuestro libro.
Cada apartado contiene material teórico y ejemplos examinados
con detalle. Con singular m.i nuciósidad hemos elegido los problemas
acompañados de las sofuciones, tendiendo a que cada solución sea
útil al estudiante, ante todo, desde el punto de vista metodológico,
para que el conjunto de dichos ejemplos sea una aportación lo suficientemente plena y entera en la preparación de los estudiantes de
las Escuelas Normale's Superiores de los problemas del método particular de enseñania de las matemáticas en la escuela. Al iinal de casi
;
todo apartado se aducen problemas para el trabajo individual. EHos
están agrupados según las partes y secciones de la geometría escolar
y por el grado de crecim.iento de la complejidad. 'Para la mayoría de
los problemas, que se resuelven de forma individual, al final del
libro se ofrecen las soluciones y, para parte do ellos, indicaciones para
su resolución. , .
El presente manual const"a de dos capítulos. El primero está dedicad9 a resolver problemas de planimetría. En el § 1, que en cierto
sentido es la introducción a todo el libro, tratan los métodos para
resol.ver problemas geométricos. tradicionales, que con aétivfdad se
emplean en los s iguientes apartados. Aqui se destacan los métodos
p1,1ramen·t e geométricos, algebraicos y mixtos, así como sus casos
patticulares: el método del elemento de referencia (que contiene el
método de las áreas) y el método de introducción de un parámetro
auxiliar. Para que sea más cómodo trabajar, en ese mismo apartado
se ofr!lce la lista. de los teoremas de planimetría, necesarios para
resol ver los proble01as.
En los párrafos 2 - 4 se ha incluido una cantidad bastante grande
de probler!~as. estándares de dificul ~ad media, ya que, como muestra
la práctic'á·; los ptoblemas tradicionales de. planimetria son uno de
los-puntos débiles'en la preparación de los estudiantes, futuros profesores de matemáticas.
El objetivo fundamental de los párrafos 5-6 es la formación en
los futuros profesores los hábitos y conocimientos necesarios para
resolver los problemas geométricos según el mótodo de las transforml\ciones geométricas y el método vectorial. Al mismo tiempo, hay
que remarcar que dichos apartados, fundamentalmente, contienen
problemas geométricos tradicionales, que se resuelven mediante los
métodos indicados, pero no problemas especiales para los transformaciones y vectores, con lo que se tropieza, frecuentemente , en los
compendios de problemas do geometría. Aqui no nos habíamos planteado el objetivo de mostrar la ventaja do los métodos indicados, lo
primero es necesario enseñar al estudiante la aplicación de estos
métodos. Como los problemas geométricos pueden ser resueltos por
diversos métodos, en los párrafos 2-4, 5 y 6 se tropieza. con problemas iguales o análogos.
Dos párrafos (el § 7 del capítulo 1 y el § 16 del capítulo ll) están
dedicados a problemas geométricos para la búsqueda de los valores
mínimos y máximos. Por regla, se considera que de dichos problemas
ha de ocuparso el curso do análisis mawmático. Pero en él, la fundamental aplicación de esos problemas consiste en la domostración <lel
papel aplicado del cálculo diferencial (es decir, se acentúa la resolución do los problemas dentro del modelo matemático confeccionado
y, en menor grado, para la propia composición del ·modelo y su
interpretación). Incluyendo en el presente manual uno u otro problema para determinar el valor máximo o mínimo, teníamos on cuenta
8
Prólogo
que cada uno de los problemas fuera, ante todo, interesante desde
el punto de vista geométrico (es decir, que se acentuara la estructuración del modelo matemático y su interpretación).
El segundo capítulo está dedicado a la resolución de problemas
de estereometría. En él, en forma breve, se recuerdan los daios fundamentales para ln construcción de las representaciones de las figuras
en la proyección paralela. Se trata de la determinación de la plenitud de la representación y su definición métrica. Se examinan las
construcciones geométricas en el espacio y, con ello, se presta singular atención a las construcciones en las representaciones. En este
capítulo muchos problemas son nuevos confeccionados especialmente
para el presente material. Entre ellos, señalemos aquellos que están
li,gados con la determinación del ángulo entre rectas cruzadas, la
distancia entre ellas, el ángulo entre la recta y el plano, el ángulo
diedro y Jos problemas relacionados· con la construcción de las secciones. Según nuestra opinión, la resolución de estos problemas hn
de favorecer al desarrollo en los estudiantes de las representaciones
espaciales.
Los autores
CapCfolo l
PLANIMETRÍA
§ 1. SOBRE LOS ME TODOS PARA RESOl.V.ER
PROBLEMAS G'EOMETRICOS
Al resolver problemas· geométricos, por regla, se emplean tres
métodos fundamentales: geornétrtco (la afirmación requerida se
deduce con ayuda de razonamientos lógicos de una serie de teoremas
conpcidos); algebraico (la demostración de la afirmación o bien el
haliazgo de los valores que se determinan, se realiza con el cálculo
directo, sobre la base de diversa.s dependencias entre las magnitudes
·geométdcaJ!, con a~~<Ja do la composición de ecuncionos o sistemas
de éstasY; mixto (en ciertas etapas la resolución se lleva a cabo por
méJ;odo geom.é trico, en· otras, alge'bralco).
Independientemente de la vía .elegida para la resolución, el
éxito de su utilización,. como es natural, depende del conocimiento
de los teoremas y el hábito de JIU aplicación. Recordemos la enunciación de alguno· d'e los teoremas que se emplean activamente en la
resolución de los problemas. Más adelante, reiteradamente, nos
vamos a referir a esos teoremas.
I. Triángulos y cuadriláteros
1. Te.9rema de la igualdad de los ángulos con lados perpendiculares
entre sí: si LABC y LDEF son ambos agudos o bien obtusos y
e
B
1\,
A
A
D
Ftg. l
_L EF (fig. 1), entonces LABC = LDEF.
2. Propiedades de la línea media del trapecio:
AB ..L DE, BC
Capítulo l. Plattimctria
10
/ 7 \
~
I7
Ftg. 2
Fig. 3
.~~
~
e
Fig. 4
B
A
~
b
C
Ftg. 5
B
A
~
F
Fig. 6
C
a) la linea media es paralela a las
bases del trapecio;
b) la línea media es igual a la semisuma de las bases del trapecio;
c) la línea media (y sólo ella) divide
por la mitad cualquier segmento contenido entre las bases del trapecio (fig. 2).
Estos teoremas son también válidos
para la línea media del triángulo si
consideramos que éste es un trapecio
«degenerado~, una de cuyas bases tiene
una l argura igual a cero.
3. Teoremas sobre los puntos de intersección de las medianas, bisectrices,
alturas del t riángulo:
a) las tres medianas del triángulo concurren en un punto (llamado centro de
gravedad o centroide del triángulo) y se
dividen por ese punto en la razón 2: 1,
contando desde el vértice;
b) las tres bisectrices del triángulo
concurren en un punto;
c) las tres alturas del triángulo concurren en un punto (llamado ortocentro
. del triángulo).
4. Propiedad de la mediana en un
triá.ngulo rectángulo: en un triángulo
rectángulo la mediana trazada a la hipotenusa es igual a su mitad. El teorema inverso es asimismo ciorto: si en un
triángulo una de las medianas es igual
a la mitad del lado sobre el que está
trazada, t.al triángulo es rectángulo.
5. Propiedad de la bisectriz . de\ ángulo interior de un triángulo: la bisectriz del ángulo interno de un tri*ngulo
divido el lado' en el que está trazada en
partes proporcionales n los lados adyaa
a'
:
centes: -¡¡=v (flg. 3).
6. Relaciones métricas en el triángulo rectángulo: si a y b son los catetos, e, la hipotenusa, h, la altura, a' y b',
las proyecciones de los catetos sobre la
hipotenusa (fig. 4), entonces: a) h~ =
.~ J.
=
Sobre los mitodot para rtso!vu probl<ma• ¡¡tom/trtco•
it
ª! .
a'b'; b) a~ =ca'; c) b2 = cb'; d) a 2 + b' =e•; e) h. =
7. Teorema de los cosenos: a 2 = b2 + c2 - 2bc cos A (fig. 5).
8. Teorema de los senos: se: A = sc!s = se~ e = 2R, donde R
es el radio de la circunferencia circunscrita al t riángulo.
9. Definición del t ipo do triángulo por SU$ lados: sean a, b, e
los lados del triángulo, con la particularidad de que e es el lad·o
mayor, entonces:
a) si c2 < a 2
b2 , el triángulo es ocutángulo;
b) si c2 = a 2
b\ el triángulo es rectángulo;
c) si e~> a 2
b2 , el triángulo es obtua
sángulo.
,,..
10. Teoremas ~e Cevn: supongamos que
Ftg. 7
en el t~iángulo ABC, en los lados. AB, BC,
AC se han t.omado los puntosD, E, F, respectivamente. Con el fin de que las rectas AE, BF y CD concurvan
en un punt.o (fig. 6) es necesario y sufícicnte que se verHique la
igualdad
+
+
+
AD BE
CF
BD ' CE • AF = 1.
11. Relaciones métricas en el paralelogramo: la suma de los cuadrados de las diagonales del paralelogramo es igual a la suma de los
CUadtadOS de SUS lados: d~ +
= 2a2 2b2 (fig. 7).
a:
+
U. La circunferencia
12. Propiedades de las tangentes a la circunferencia:
a) el radio trazado al punto do tangencia es perpendicular a la
tangente (fig. 8);
Pig. 8
Ftg. 9
b) dos tangentes trazadas a la circunferencia desde un mismo
p unto son iguales, y el centro de la circunferencia yace en la bisectriz
d11l ángulo eni-re ellas (fig. 9).
12
Capitulo 1 . Planim<trfa
13. Medida de los ángulos relacionados con la circunferencia:
a) el ángulo centr al se mide con el arco sobre eJ que se apoya;
b) eJ ángulo inscrito se mide con la mil.ad del arco sobre el que
se apoya;
e) el ángulo entre la ta ngente y In cuerda se mide con la mitad
del arco entre ellas.
14. Teoremas sobre las circunferencias y los triángulos:
d
F ig. 12
Flg. 11
J.'ig. JO
a) a lodo triángulo es posible circunscribir una circunferencio;
el centro de ésta es el punto en que concurren las perpendiculares
trazadas a los lados por sus puntos medios;
b) en todo triá ngulo es posible inscribir una circunferencia; el
centro de ésta será el punto en que concurren las bisectrices.
15. Teoremas sobro las circunforencias y los cuadriláteros:
M
Flg. 18
a) con el fin do que o un cuadrilátero puede ser circunscrita uno
circunferencia, es necesario y suficienl.-0 que la suma de los ángulos
opuestos del cuadrilátero sea igual a 180° (a.
~ = 180", fig . 10).
b) con el fin de que en un cuadrilátero puede ser inscrita una
circunferencia, es necesario y suficiente que las sumas de sus Indos
opuestos sean iguales (a
e =b
d, fig. ~ 1).
16. Relaciones métricas· en la circunforoncia:
a) si las cuerdas AB y .CD se cortan en el punto M, AM·BM
= CM· DM (fig. 12);
+
+
+
§ 1. Sobre ros método$ para
re.~tv~r
problema1 geométricos
13
b) si del punto M se t,razan a la circwiferencia dos secantes MAB
y MCD, AM·BM = CM·D'M (fig. 13);
c) si del punto M se trazan la secante MAB y la tangente .MC,
AM · BM = CMi (fig. 14).
III. Áreas de l as figuras planas
17. La razón entre las áreas de ligur.as ·semojanies. es igual al
cuadrado de la razón de semejanza.
18. Si en dos triángulos son iguales las bases, sus áreas se ·relacionan como las alturas; si en dos triángulos son iguales las alturas
sus áreas se relacionan como las bases.
1-9. Fórmulas para calcular el área de un triángulo: a) S =
ah
b S abscn e
abe
a-l-b-l-c
=2;
) = -2- ; c) S='Tlf:
d) S=w, d on d o p =
-2- ;
R es el '~radio de la circunferencia circunscrita; r, el radio de la
circunferencia inscrita; e) S= V p(p-a) (p-b) (p-c) (fórmula de
Herón).
e
B
C
a
&g
A
D
Fig. 16
Fig. 15
20. Fórmulas para calcular el área de un cuadrilátero convexo
(fig. 15): a) S = SADC
S,..cD = SAso
S seo = SA.os
i
+Ssoc+ScoD+SAoD; b) S =2AC·BD·sena; e) S =pr
(si en el cuadrilátero se puede inscribir una circunferencia y r es su
radio).
21. Fórmulas para calcular el ároa de un paralelogramo (fig. 1.6):
1
a) S = ah; b) S = ab sen C; c) S = 2 d 1d 2 sen et.
+
+
22. Fórm\lla del área del trapecio (fig. 17): S
+
= a.-i;b
h.
t
23. Fórmula del área do un sector circular (fig. 18); S = 2 R 2a.
(et es la medida en radianes del ángulo central).
24. Fórmula del área de un segmento circul ar (fig. 19): S =
1
2
2 R (a. - sen et).
Capitulo f, P/animetrfa
14
Con frecuencia, al resolver problemas geométricos, es preciso
establecer la igualdad de dos segmontos (o ángulos). Indiquemos las
tres vfas fundamentales de la demostraci6n geométrica de la igualdad
de des segmentos;
a
Fig . 17
Ftg. 10
Fig. 18
1) dichos segmentos se consideran como los lados de dos triángulos y se demuestra que éstos son iguales;
2) dichos segmentos se consideran como lados de un triángulo y
se demuestra que éste es isósceles.
3) el segmento a se sustituye por el a', igual a él, mientras que
el segmento b por el b', igual a él y se demuestra la igualdad de Jos
segmentos a' y b'.
Ftg .. 20
Ftg. 21
Al f!ll!olver problemas geométricos con frecuencia es preciso realizar cons.i.rucciones auxiliares. Indiquemos algunas de ellas: trazado
de una. recta paral!lla o perpendicular a una de las que h11y en la
figura; duplicación. de la mediana de un triángulo, como resultado
de lo que el triángulo se transforma en paralelogramo; trazado de
una circunferencia aU:Xiliar; trazado de radios al punto de tangencia
de una circunferencia y una recta o de dos circunferencias, etc.
EIÉMPLO 1. Dos rectas perpendiculares entre sí cruzan los lados AB,
BC, CD y AD del cuadrado ABCD en los puntos E, F, K, L, respectivamente. Demostremos qun EK = FL (fig. 20).
.§ J. Sobre loa mitodos po.ra rtsalver probl~m.0.1 gtomitr¡coa
l5
soLucJON. Haciendo uso de la primera de las VÍ!lS indicadas para
la demostración de la igualdad de dos segmentos, tracemos FM 11 Cl)
y KP 11 AD, entonces, los segmentos que n~s inter.esan EK.. y .FL
se convertirán en los lados de dos ~riángulos rectángulos EKP y
FLM (fig. 21) y, por consiguiente, es suficiente demostrar la igualdad
de dichos triángulos.
Tenemos: PK = FM (como oltu.i:as de un cuad:cado), LLFM =
LEKP (como los ángulos de lados perpendiculares entre si,
teorema 1). Esto signüica, que 6EKP = D..FLM (según ·el cateto
y ángulo agudo). De la ·igualdad de los triángulos rectángulos se
desprende la igualdad de sus hipotenusas, o sea, de los ·segmentos
EK y FL.
EJEMPLO 2. Los lados de un triángulo son iguales a a, b, c. Calculemos la mediana m 0 trazada hacia el lado c.
SOLUCION. Dupliquemos la mediana hasta convertir el triángulo
en el paralelogramo A CBP (fig. 22) y apliquemos a éste el teorema 11.
=
e
8
A
p
D
Fig. 23
Flg. 22
Obtendremos: CP
= 2b2
+ 2a
2
2
+ AlJ
2
= 2ACí
+ 2BC
1
,
o sea, (2m 0 ) 2
= V2a* + 2b• -
e•
+c
2
, de donde hallamos: 11t0
2
s. Demostremos que el ortocentro de un triángulo ocutángulo coincide con ol centro de la circunferencia inscrita en ol
triángulo formado por las bases de las alturas.
soLUCION. Como centro de la circunferencia inscrita en el triángulo
es el punto en el que se cortan las bisectrices (teorema 14b), el problema se reduce a demostrar que DH, EH, KH son las bisectrices del
triángulo DEK (fig. 23). Para ello, es suficiente demostrar que
LEDH =LHDK.
Examinemos el cuadrilátero DH KC. Tenemos: LlfDC = 90º,
LHKC = 90º, es decir, LHDC + LHKC = 180°, por lo que en
torno al cuadrilátero DH KC es posiblo circunscribir una circunferencia (teorema 15a).
Al circunscribir dicha circunferencia (fig. 24) advertimos que
los ángulos HDK y HCK son iguales por estar inscritos y porque
EJEMPLO
Capitulo l. /Jlnnimetrla
tG
se apoyan en un mismo arco llK . Por analogía, al circunscribir un11
circunferencia al cuadrilátero AEHD llegamos a la conclusión de
que LEAH = LEDH.
Así, pues, LEAH = LEDH, LHDK = LHCK. Pero los
ángulos EAH y HCK son iguales como los ángulos con lados perpendiculares (teorema 1), por lo que,
n
LEDH = LHDK, que es lo que queríamos demostrar.
Para la composición de ecuaciones en
los problemas geométricos se hace uso del
teorema de Pitágoras, las ·relaciones métricas en el triángulo rectángulo (teorema 6),
las dependencias entre los lados y ángulos
A
del triángulo rectángulo, la proporcionalidad do los lados, alturas y porímotros de
PI¡¡. 21
los triúngulos somcjantcs, el tcoromn acoren
do la bisectriz del triángulo (tcoroma 5),
relaciones métricas en el paralelogramo (teorema 11) y la circunferencia (teorema 16), el teorema de los senos (teorema8), el teorema de los cosenos (teorema 7), diversas fórmulas para calcular
las áreas.
El método fundamental de composición de ecuaciones en el
problema geométrico es el del elemento de referencia, quo consiste
en lo siguiente: un mismo elemento se expresa (mediante magnitudes
e
e
~~
AH
F l g. 25
B
HXA
e
B
Flg. 26
conocidas y desconocidas) con ayuda de dos procedimientos diferentes y las ~iepresiones obtenidas se igualan entre sí. Si en calidad de
elemento de referencia se emplea el área, tanibién· se dice que el
problema se ha rj!Suelto según .el' método de las áreas.
EJEMPLO 4. Los la'd os de un triángulo son iguales a a, b y c.
Calcule~os la altura he, trazada al lado c.
SOJ.,UCION. I procedimiento.
La altura he es el cateto común de dos triángulos rectángulos
ACH y CHB (fig. 25). Empleando el teorema de Pitágorns expresemos CH 2 del D.ACH y del D.CHB (CII es el elemento de referencia).
Hagamos AH = x, entonces BH =e - x. Si el D.ACB fuera
obtusángulo, lendríamos BH = c + x (fig. 26). Aquí, vamos a limi-
§ J. Sobre lo• mdtodo• paro rtiower problemas ¡¡<ométrtcos
i7
tarnos al caso mostrado en la fig. 25.
Del b.ACH hallamos: CH 2 = b1 - x 1 , del ó llCH h nllamos:
CH2 = 4 2 - (e - x) 2 •
De la e(.Uación b2 - x2 = a2 - (e - x)2, obtenemos: x =
c•+b• - a•
2c
Del t:,.ACH, hallamos:
CH=
=y' (b
=
V bZ -
{ c•+~:-o•
)2 =
c•+~-o') ( b+ c•+~;-o• } =
i. Y (a2 - (b-c)2){(b +e)"- -
a2) =
= i, V(a+b -c)(a+c-b)(b+c-a)(a.+IJ+ c).
Asi, pues,
V(o+b-1-<) (a+b -c) (a +•-b) (b+c -
a)
2c
I J procedimiento. Hagamos uso del método ele las áreas. Por
un Indo, el área del tríángulo ABC es ígual a Y p (p-a) (p - ú) (p-c)
y, por olro, es igual o
Igualando cstns expr<'..síones, obtenemos:
+chC'
h
._
2 Jf p (p - o) (p-b) (p-c)
e
Poniendo en 1ug11r de p su expresión por los lad os p
= o+b-t-c
,
2
obtenemos:
BJE~IPLO s. L os lados de un triángulo son a, b y c. Hallar la
bisectriz l 0 trazada hacin el lado c.
SOLUCION. J procedimiento (algebraico). Sea CD In bisectriz del
t::,,ABC (fig. 27). Plon para lo resolución: hallemos la longitud de
Jos segmentos AD y BD y , a continuación, empleañdo paro los
triángulos ACD y BCD el teorema de los cosernos (con ello, teniendo
on cuenta que LACD = LDCO), hallamos ln bisectriz l 0 =CD.
2-0200
Capíl1tlo l. Planímtlría
18
Hagamos AD = x, BD =y. E ntonces, :e+ y =e y según l a
propiedad de la bisectriz (teorema 5)
= ~ . Del sistema do
ecuaciones
f
x+y=c,
,;:_=l.
hallamos:
Y
a
{
Aplicando al D.ADC el t-eorema de los cosenos (teorema 7),
obtenemos
e
2 = b'
(1)
:i;
¿z - 2bl cos t
+
~
(aquí, para mayor brevedad, hemos hecho l0 = l y LACD = LDC/3 = t).
Aplicando al D,BDC el teorema do los
cosemos, obtenemos:
A
D
8
(2)
y• = a 2
P - 2al cos t.
27
F ;g.
l\I· u l t1p
· l'1qucmos am bos nuem
. bros d e l a
igualdad (1) por a y ambos m •embros de
la igualdad (2) por (- b) y sumemos l as igualdades obtenidas como
resultado de lns multiplicaciones: a:i:2 - by 2 = ab2 - a2 b + al 2 - blZ, de donde h allamos:
+
(3)
Pongamos en. la igualdad (3) los valores de x e y hallados
2 2
:
· ba O
.
•i
1
( b ca
a•c•o )
b
mas arr1 .
btenemos.
,. = a=b (•+bP - (a+b)' +a =
= ab ( i -
-' ' - ) = ab (a+b-1- e) (a+l> - c)
(a+b)~
(a+b)'
Vab(a+~+c)(a+b - c)
Así pues l
'
'
'
a+b
e
JJ procedimiento. Además de Ja magni tud incógnita que buscamos
1, int,roduicamos otra magnii ud incógnita auxiliar: hagamos :e =
e LACD ::¡:: LDCB y hagamos uso del método de l as áreas. Tenemos: S A se = S ACD
+ S scD·
Po r otro, ya que S ACD
=
1
2 al sen x
.
Es decir,
1
+2
1
Por un lado, S A
= z.1 bl sen x,
S
scD
se
1
=
2 ab sen 2:i:.
= z1 al sen
x,
S ABC =
bl sen x.
2 abscn 2 z=
l (a+ b) <en
Z'
:i:
,
d
e
d
onde
l=
Zab cos .:r
a+b
•
J
1. Sobre 11>1 mrtMOS pora rt...,loer problemas
19
Rtoml/r/to
P11ra hallar cos x empleemos el teorema de los cosenos respecto
2del t riángulo ABC ¡lara el lado AB. Oh\.Cnemos: c2 = a 2
a•+b• - c•
-2abcos2J:, de donde l1alfomos: cos2x....
2a!>
• ~tonces,
7b
_ /t+cos2..r _, / _!. ( 1 +a•+b•-c2 } _ _!,, / (a+b+c) (a+b -c)
l --2- - V 2
2ab
- 2 V
ab
•
COS X-
Como resu l tado, obtenemos:
l=
2ab cos x
a+b
~ ,...!._,/ (a+b+t)(a+li-c)
a+b 2 V
ab
_ Vab(a+b+clca+b-r.)
a+b
_ Durante la confección de las ecuaciones, nl resolver el prohlcma
geométrico, hay que tener en cuenta que, con Irocut>ncin, l'I éxito
de.la resolución depende de la int.roducciún A
ventajosa de l as vatiables. Aclaremos este
~
p,ensamiento en el sigu ii¡ nte .ejemp1o.
EJEMPLO &. E n un t i'Íá'ngulo rectángulo
<:
la hipotenusa es igual a e y la b isectriz de
d'J
r
3
uno de los ángulos agudos es igual a cl 3 .
3
e
"
/1
}{aliemos los catetos (fig. 28).
s0Luc10N. 1 procedimiento. Hagamos
Ft¡;. 28
AC = x, .BC= y, CD = z. Entonces, de acuerdo con el teorema de Pilágoras, xz+ y2 =
c2
y xi+ z: =
(' ·~~ ) ~ _
Además, según el teorema ele ln bisectri7. (teorema 5), tenemos
AC
CD
d . r
:
es cc1r, 7 =y=z
Como resu l tado, obtenemos un sistemn de lrcs ecunciones con
'AB=m,
¡
.xi+ y-i=c'l.,
•
.
tres variables:
.z2
r
+ ¡Z =
~·3
; cuya
.
. .
.
resolución osla hgnda a cons1-
:
c =u=z,
dcrables complicaciones algebrnicas.
JI proccdlmiento. Supongamos que LCAD = LBAD = x. Empl eando el segmen to A C como olemcnto de referencia, compongamos
la ecuación. Del triángulo ABC, hallamos: AC =e cos 2.x; del
cl/3lriángulo ACD, tenemos:
A e = --¡¡- cos .x. Igualando eslns
e>:presiones, obLeoomos la ecuación trigonomótrica e c-0s 2x =
cV3
= -3- cos x.
Capítulo /. Planimetría
Resolvemos esa ecuación:
= cos
O
x,
.
b ICll
2 ·V3- cos
2
lí3 cos 2x
= cos x,
Y3 =
x - cos x -
O, de
y3 (2 cos 2 x-1) =
donde cos x = 0
2
y:.i
= - 3•
Pero, como por el sentido del problema cos x >O, obtenemos:
cos x
=
COS X
~3.
Esto significa que el ángulo BAD es igual a 30° y
cV3
BAC , a 60º. Co mo resultado, obtenemos.. A e = e
2• B e = 2-.
Si en el problema es preciso hallar la razón entre ciertas magnitudes (larguras o áreas), en particular, si e~ preciso calcular algún
ángulo {lo que, por regla, se reduce a la
búsqueda de cierta función trigonométrica
<lcl ángulo y, por consiguíente, la razón de
los lados de un triángulo rectúngulo), corrientemente se opera del modo siguiente:
se considera que uno ele los elementos l ineales es conocido. con él se expresan las
magnitudes necesarias y, a continuación,
l"ig. 29
se confeccionan sus razones. El elemento lineal introducido lleva el nombre de
parámetro auxiliar y semejante método de resolución de los problemas geométricos, métcdo de introducct6n de parámetros auxiliares.
Es aplicable a los problemas donde la Hgura geométrica está definida
con precisión hasta la semejanza.
EJEMPLO 1. En el triángulo rectángulo ABC, desde el v~rtice del
ángulo recto se han trazado una altura y mediana. El ángulo a entre
ellas es igual a arceas~. Hallemos la razón entre los catetos
(fig. 29).
SOLUCION. Ante todo, es tlo advertir que, de acuerdo con el plan le·
.
. cos. .a = 7J,
40 es dec1r
. CK
40 R eso l vamos el pro blema
a.m1entó
1 CM = ¡;¡.
1
según el método· do introducción de un paráme tro auxiliar.
41
,¡
Hagamos CK=h. Enlonces, CM=¡;¡/t, KM= v C1l11--CK2 =
= ~ h. Como en el triángulo rectángulo la mediana es igual a
la mitad .do la hipotenusa (teorema 4), AM=CM= M B=
:¿ h.
Entonces, AK=AM-KM=~h-:Oh = fh, KB=KM+fJM=
=ioh+~h=+h. AC=VAK2+CKZ=v 2 h2 +h2 =_~
V41.
Bc -YBK'Z+CK2- ~ /~h2+12-.!!.y41· ~-h Y~.h ¡141 _ _j_
'-
- V
16
i
-
4
·Be- s ·
4
-
s·
§ 2. T riángulos
v
cuadrll1flcro1
2t
EJEMPLO B. E n el triángulo isósceles ABC el ángulo e en el vértice
es igual a 100°. Se han trazado dos rayos: uno que comienza en el
punto A bajo un ángulo de 30º con relación al rayo A By, el segundo.
que comienza en el punto B bajo
un ángulo de 20° respecto del BA.
.e
Estos rayos concurren en el puot.o
M perteneciente al triángulo ABC.
Hallemos los á ngulos ACM y
BCM.
SOLUClON. Unamos los puntos
M y C y designemos el ángul o
ACM con x. Del pu n to M trace-,..
B
e,
mos perpendiculares a los l ad os del
Fig. 30
triángulo: MC 1 ..L AB, MB1 ..LAC,
M A 1 ..L BC (fig. 30). I ntroduzcamos un par ámeY.o auxiliar , p.ej., CM = a y calculamos MC,
con dos procedimientos, es decir, em pleainos 1WC 1 como elemento
de referencia.
Del triángulo CfltfB,, h11Llamos: M B 1 = MC st!n z-= asen x. Como
LACB =100" y, de acuerdo con el planteamiento, el triángulo
ABC es isósceles, LCAB=LABC =40" y, por consiguionle,
LCAM = iOº. Del t riáng ulo AMB,, hallamos: AM = sen,
M~o'• =
a.senz
p
=sen 10•·
or fin, del t:::,. AMC,, tenemos: MC , = AM sen 30º =
asen x
= 2.sen tOº •
An alicemos el triángulo CMA 1 • E n él , LMCA 1 = 100º - x.
De modo que MA 1 =CM sen (100° - :i:) =a sen (100º - x).
Como LM BC = 40" - 20° = 20° , l os t r iángulos BMC, y BM A 1
son iguales y, por. consiguien te, MC, = MA1 =a sen (100º - x).
Igu alando las expresiones halladas para MC1 , obtenemos ln
ecuación trigonométrica ·2ªS:ntOo = a sen (100° - x).
De esta ecu ación hal lam os consecutivamente: sen x
= 2 sen (100° - x)-sen 10°, sen :i: = cos (90" - x) - cos(H Oº - x),
COS (110º - X) = 0, X = 2()°.
=
§ 2. TRIÁNGULOS Y CUADRILÁTEROS
EJElllJ>LO t. Las bases de u n trapecio son a y b. Hallemos la l argura
del segmento que une el punto medio de l as d iagonales (fig. 31.).
SOLuc10N, Como P es el pu n to medio de l a diagonal AC y K,
el punto medio de le diagonal BD, los puntos P y K yacen en la
línea medía EF del tr apecio (teo rema 2c). Como EK es la línea
22
Capft,./o f . Plantm<tria
media del Lriángulo ABD, EK = ~; ya que EP es la línea media
del triángulo ABC, EP
=~
Como resultndo, obtllnemos: PK =
EP = a~l>·
Conociendo las medianas m 0 , mb y m0 del triángulo
ABC, hallemos el lado AC = b.
soLuc10N. Según la propiedad de las medianas en el triángulo
(teorema 3a), ellas concurren en un mismo punto M y en él se clivi= EK -
EJEMPLO 2.
s
e
B
~
A
A
D.
~
D
C
fig. 92
Ftg. 81
den con una razón 2: 1, contando desde el vér tice (!ig. 32). Por
2
ello , en el D.AMC conocemos dos lados: AM = 3 m-a· MC =
={me y
=fmb.
la mediana MD
Consideremos el í:>. AN!C. Duplicando !SU mediana MD, convertimos el triángulo en el paralelogramo AMCP (fig. 33). Entonces,
de acuerdo con el teorema 11AC2 MP 2 = 2AM 2 + 2MC2,
es
.
decir,
bz+ 4 mt= 2 . 4 m~+ 2 ..4 m~. de don de hallamos: b=
+
=f
9
V2m~ + 2m~-mt.
9
9
EJE~tPLO a. En los lados AB y BC del triángulo ABC, fuera de
él, se han construido l os cuadrados ABDE y BCKM. Demostremos
que el segmento DM e~ dos v1Jces mayor
que )a mediana BP del ti·iángulo ABC
(flg. 34).
SOLUCION . Como hay que demostrar que
DM = 2BP, es conveniente duplicar la
mediana BP y convertir el óABC en el
paralelogramo ABCT y, a continuación,
r
demostrar que DM = BT. Para demosFig. 33
trar que los segmentos DM y BT son
iguales, l1ay que examinar éstos como los
lados de dos triángulos y demostrar la igualdad do éstos. En corrcspondencia con el plan que nos hemos planteado, resolvemos
el problema.
23
§ 2. Trl6n8ulot y cuadrt/61aos
Dupliquemos In mediano BP convirtiendo el ó A BC en el paralelogramo ABCT (Hg. 35).
Examinemos Jos triángulos DMB y BCT. Tenemos: BM = BC
como Jos l ados de.l cuadrado BMKC; DB = CT (con mayor detalle:
DB = AB como los lados de un cuadJ"ado y AB
CT como los lados
opuestos de un paralelogramo, es decir, DB = CT); LDBM =
= LBCT (como ángulos con lodos perpendiculares entre si). Esto
=
M
e
Flg. 34
Fig. 35
significa quo el t:;DBM = t:;BCT (por los dos lados y el ángulo
entre ellos) y, por lo tanto, DM = BT.
Como BT =2BP, de DM =BT se desprende que DM =2BP.
EJEl'lll'u> 4. La altura trnznda a la h ipotonuso. de un triúngulo
rectángulo la divide en partes: V y 16 cm. Del vértice del ángulo
agudo mayor del triángulo se ha trazado una recta quo posn por el
punto medio de la altura. Hollemos la largura del segmento de d iclrn
recta en el iulorior del triángulo rectángulo dado (fig. 30).
SOLUC!ON. Tenemos:
Clf2 = AH -BH (teorema 6n), por ello,
Cll2 = 9-16, es dl'cir, CH = i 2 cm. Dol 6 ADH h allamos: AD=
=V 92 ()Z = 3 13 cm. Trucumns HM 11 AK y llagamos DK. = x .
Como DK es Ja línea media del t::. llCM, HM = 2.i:. Dll In semejanza
de los triírngulos HM.8 y /\KB Uegamos o la concl11s ió11 de quo
HM
+
V
BH
d
AK=Aif• es
= 3 vu+
.
ec1r ,
16
-"+ 32x.11 13 =z·
l d
ce
I
oiue
24
Jf ia
17-
X = --
y
Af"'"~=
2.q /i3 - 75 -VTii
t 7 -17 .
75 ·1/ IJ
Así, pues, AK = - -- cm.
17
s. Ea un triángu lo con los lados de 10, 17· y 21 cm est á
inscrilo un rectúngulo de modo que dos de sus vértices so encuentran
en un lado clel triángulo y los ottos <los, en los otros dos ltl<los del
EJI:IMl'LO
24
C4piluw /. Planimetrla
triángulo. Hallemos los lados del rectángulo si sabernos que su perímetro es igual a 22,5 cm.
SOLUClON. Ante todo de~ermínamos el tipo del triángulo. Tenemos:
102 = 100; 17~ = 289, 21z = 441. Como 2P > 10z
172 , el triáng11lo es obtusángulo (teorema 9) y, por lo tanto, es posible inscribir
el rectángulo sólo por un procedimiento: situando dos vértices en
el lado mayor (fig. 37).
+
B
e
A
&
H
B
A
~
DU
K
C
Fig. 31
Plg ..1a
]fallemos la altura BIJ del triángulo ABC. Aplicando el mélodo
que empleamos en el ejemplo 4 del § ·1 (o bien haciendo uso de la
fórmula obtenida en dicho ejemplo), hallamos: BH = 8 cm.
Hagamos ED = x. Entonces, EF = 11,25 - x (ya que el perímetro del rectángulo DEFK es igual a 22,5 cm), BP = 8 - x.
Los triúng1Jlos BEF y ABC son semejantes, es dcci'l', !~= ffi~
(en los triángulos semejantes Ja ra?.ón entro las correspondient.es
B
alturas es igual a la razón de seme.janza),
25-x = 8-x
o sea, 1t 'Zt
- -, d e donde h allamos:
8
:c = 6.
Así, pues, los lados del rectángulo son
6 y 5,25 cm.
EJEMPLO G. En el triángulo ABC se conoce que el ángulo A es dos veces mayor
A
e que el ángulo C, el lado BC es 2 cm mayor
flg. 38
que el lado AB, en tanto que AC = 5 cm.
Halfcmos AB y BC .
.s0Luc10N. 1 procedimiento. Tracemos la bisectriz AD del ángulo A.
Entonces, obten.ewos que LBAD = LDAC = LACB (fig. 38).
En el b.ADC los ángulos e[! la base son iguales, es decir, este
triángulo es isósceles: AD = DC. Hagamos AB = x, AD = DC =
= y. Entonces BC =;= x Z, BD = x Z - y.
Los t1 iángulos ABD y ABC son semejantes, ya que L BA D =
=LECA y LB es común para dichos triángµlos. De la seme.
d o 1os t nangu
..
l os, Jlegamos a ¡·a concl uswn
. , que AB
AD
1anza
ífC BD
AB
AC ,
+
+
= =
d .
%
es ecir, z+z =
.x+2-y
,,
y
=5 .
25
§ 2. Tridng1tlos y criadr¡l.áteros
Para hallar x e y se ha obtenido un sistemn de dos ecuaciones
=+'
· bles: { z..';. 2
de donde { 5x=xy+2y,
con dos vana
x + 2-Y _J!..
5x+10-5y=xy.
:¡;
-
5'
Sustrayendo Ja segunda ecuación de la primera, oblcnemos:
5y-10-=2y o Y=~· Eslo significa que .:~ 2
osea, x = 4.
=; ,
Asi, pues, AB=4 cm, BC=6 cm.
II
procedimiento. Hagamos L. C = t, entonci;:s L. A= 2t,
L. B = 180º- 3t. Hagamos, asimismo, AB = x, entonces BC = x + 2.
Según el teorema do los sonos (loorcma 8), tonemos:
-
SC: t = ~;t2~ =
5
s~Ji (18Cf-3t) ·.
Hen1os obtenido un sistema de dos ecuaciones con clos variables
~~ t = :c~:i
'
x y t: { _ z_=_s_ (aquí hemos hecho uso de que sen (180º scn t
sen 3t
- 3t) = sen 3t).
Resolvamos este sistema. De la segunrl<i ecuac1on obtenemos:
5 sen t
5 sen t
5
D 1
·
"
1l
x= sen 3t
3 sen t - 4 sen" t - 3- 4 sen• t . e a prinwra ecuac1on <o
.
, es decir, 1
sistema hallamos: -r+2
.,- = ~n21
~en
z2 = 2 cos t. Pon ien-
+
1
do en lugar de x su expresión con t, hallada más arriba, obtene2
mos: i + 6 - 8 ~en ' = 2 cos t. Hagamos en esta ec\tación lrigonoaiÚrica ~ cos t = z. Obtendremos: 1 + 6- 8 ~i-z:) = 2z, de donde
.Z1=+;
Zz=+,
l'.S
decir, cos t = ~ o bien cos t =
+
.
3
hallamos: x = 4.
S1 COSl=4• de 1+.1...=Zcost,
.~
Si cost=+, de 1+...!.=2cost, hallamos: 1+~=1. lo que
$
"'
no puedo ser.
Así, pues, AB = 4 cm, BC = 6 cm.
relación cos t
el triángulo ABC obtenemos que LC
lugar.
ODSERVACJON. La
= 1Zquiere decir que t =
=
60º, LA
~
60°, entonces en
120º, lo que no puede tener
26
Capítulo l. l'lanimetria
K es el punto medio ·del lado AD del rectángulo
EJEMPLO 1.
A BCD. Hallemos el ángulo en Lee BK y la diagonal A C si sabemos que
V2.
AD: AB =
s01,uc10N. Empleemos el método del
parámetro auxiliar (véase el ejemplo 7
del § 1). Hagamos AB = a y, entonces, AD =a V2. He aquí el plan para
resolver el problema: expresemos con a
todos Jos lodos rlel triángulo AM K
(fig. 39) y apliquemos el teorema de los
Fig. 39
cosenos para el lado AK. EsLo nos permite calcular el coseno del ángulo AM K que buscamos; designémoslo con z.
Los segmentos AO y BK son las medianas del triáng\1lo ABD.
De
modo que MK=
MK =J_IJK
3
= !.
3
1
3
2
BK, AM = TAO
(teorema 3a). Tenemos:
VAEP+Af(Z =J_ , /
z.¡.( 11112
)2 = -¡¡-•
a-Vi\
i
t V< a V2
-)"-+<L2 =
=;r-
3 Vª
2
1
AM = TAO=
¡¡-AC=
T1
V,wi+ cv2
a 113
=-3-
En el triángulo AMf(, tenemos: AK =
MK = ª
rií .
Según cl
teorema
de
los
ª
r,
A1lf = "
cosenos
ra ,
(teorema 7)
("112)2 = ("11a)2
3
+
.
A K z=All:[ .i+MKZ-2AM-1v!K-cosz, es dec1r,
- 2-
01/6)2
ai/6
+ ( -¡¡
-2· -al/3
3---¡¡-cos ;i:,
a• 112
- - ·X=
-
3
.
.,
a contrnuacrnn,
a•
a•
a.•
T=:r-+rr-
cosz, de donde hnllaroos: cosx=O y, por consiguiente,
90°.
Así, pues, él ángulo entre BK y AC es recto.
EIB~lrLo s. Demostremos que en todo triángulo ABC la distancia
desde el ortocentro hasta el vértice B es dos veces mayor que lu
distancia desde el centro de l;i circunferencia circunscrita al triángulo
hasta el Jado AG.
SOLUC!ON. Sea ABC un triángulo oc11tángulo, el p1tnto H el ortocentro, el punto O, el centro de la circunferencia circunscrita, los
· ,segmentos BD y AP, las alturas, K y L, los puntos medios de los
Indos, OK y OL, perpendicularns a los lados (fig. 40).
§ 2. Trldngulos y cuadri/dleros
27
Los triángulos ABH y KOL son semejantes (BHllOK, AH\IOL,
BJl
AB
AB ll LK), o sea, OK= LK. El segmento LK es la línea medi.a
del 6. ABC y, por consiguiente, ~! = 2. Pero, ontonces, ~; = 2,
quo es lo que queríamos deroostrnr.
B
B
Ftg. 40
Flg. 41
Sea ABC un triángulo obtusángulo no isósceles, con la pnr tic11laridad de que se han conservado las designaciones del caso anterior
(fig. 41).
De la semejanza ele Jos triángulos ABH y KOL se dm::prcn<lo
Bll
AB
que OK=LK=2, do forma que, BH=20K.
En las figs. 40 y 41 la linea de Eulor está represen tada como OH
(véase el ejercicio 54).
B
EJEMPLO 9. Por el centroide de
un triángulo regular, on el plano
traiado una recta.
de éste; se
Demostremos que la suma de los
cuadrados de las distancias desde
los vériices del triángulo hasta
dicha rl!cta no depende do su elección.
M
SOLUCION. Sea que la recta a
Ptg. 42
la que nos referimos forma con la
base AC del triángulo ABC el ángulo et (lig. 42). Hagamos AO
= BO = CO =a, expresemos las perpendiculares a dicha recto
AD, BK y CE con a y et y, seguidamente, demostremos ql1e ln expreBK<J.
CE~ es constante con toda et.
sión AD 2
na
+
+
28
Capitulo /. Plantmetrla
LOAC
= 30°, por lo que LMAO =
+ 15()°) = 30° - a. Del
15()° y, entonces, LDOA
t;DOA, hallamos: AD
B
= OA sen LAOD =a sen (3D° - ex).
LBOK =LMOP = 90º -o;(delt:.MOP).
Del D,BOK, \1 allamo8: BK = BO x
D
M
X senLBOK =a· sen (90° - et) =a cos et.
LPOE = 90° +et (como ext er no
para el t:,MOP). LPOC = 600, entonces,
LCOE = LPOE - LPOC = (90°
A
H
e
o;) - 60" = 30"
ex. D el D,COE,
hallamos: CE = CO sen LCOE = n X
Fig. 13
et).
X sen(30º
Tenemos: A [)'l. + BK2
CE2 = ai scni (30º - ex)
ai cos2 a
az senz (30º +ex) = a2 ( 1-cos(~0"-2.a.) + cosi IX+ l-cos ~60º+2.a.J ) =
=
= 180° - (et
+
+
+
= ai ( 1-cos (60º+ 2a.); cos (6<>º- 2a.>
+ t+cos2a.)
=a1
2
+
+
+
+
+
+ cost ex) =
a2 ( 1-cos 60° cos 2ex
+
(i -2cos
t
1 +2cos~
1
"-) =2ª.
3 2
2a: + 2
Así, pues, con toda ex tenemos: AVl + BKt+CE2=-}ai.
Hallemos la dependencia entre los lados a, b y e
del triángulo ABC si s abemos que la mediana AM, la altura BH
y la bisectriz CD se cortan en un mismo punto (fig. 43).
SOLUCION. Según el teorema de C.Cva (teorema 10). tenemos:
EJEMl'Lo 10.
AD BM CH
.
BM
Jil}"éiii ..íílf=i. Como AM es la med11ma, BM=CM y CM = 1.
Como CD es la bisectriz,
:V = 7 (teorema
5).
¿·
Coroo resultado,
=T.
la relación dada tom a la forma:
~~ = 1, es decir, ~~
Hagamos CH= at, AH= bt. Entonces, por un lado at +bt.., b,
o sea, t = a!b ; por otro, cmploando BH coroo elemento de re-
feroncin, del t:, ABH ohLonon1os: BH 2 ccc1 -bZt2 y del ¡;:, Bl!C:
a'-c11
BH Z=a2-a2ti. Así, pues, c1 - b2t2 =a2 -a1 t 2 , de dondo t 2 = al-b"
Poniendo en la úllima igua ldad en lugar de t su valor t=
= a~b , obtenemos In dopendencia buscada entre los lados a,
b'
a.2- e'
.
by e: (cz+b)• = ..•-b•' es decir,
b'
a•- e•
a+b ==-;;=-;;-·
§ 2. Triángulos y cua·drlláltros
29
PROBLEMAS PARA EL TRABAJO INDIVIDUAL
t. Triángulo rectlingulo
t. Demostrar que en un ~rlángulo rectángulo, la bisectriz del ángulo recto
divide por la mitad el ángulo entre la mediana y la a ltura \ruadas desde ese
mismo vértice.
.
2. La mediana trazada hacia la hipotenusa de un triángulo rectángulo divide
el ánguto recto. en la razón t : 2 y es igual a m.. Hallen los lados del triángulo.
3. Un p.unto tomado en la hipotenusa de un .triángulo T\!Clángulo y situado
a 1.1na misma distanc41 de sus catetos, divide la hipotenusa e.n ·segmentos de. 30
y 40 cm. Hallen los catetos.
4. ijallen la hipotenusa de los ángulos agudos de un triángulo r.ejftáilgulo
con calotos de 18 y 24. cm.
5. ifol!en la ºl>iaectriz del ángulo recto de un triángulo rectángulo con caietos
a y b.
6. Del ·vértice. del án~ulo recto de un triángulo rectángulo se ha trazado la
J:>isectrlz quo divld!l la lHP!>lenusa en los segmentos m y n. Hallen la allura
trazada ·a la hipo\enu~ .
.7. En un triáng1,1_lo rcctá_ngull• los mc<llnnos tra7.ndus n los cate\os son igunle11 a ·if52..y )f73 cm . .Jialléri la hipotenusa.
.
. 8.' El por1m9tro do'·un ·triángulo rectángulo os igual 11 60 cm y la altura trazada a .l a. llipot.enusa, igual á 1-2. cm. Hallen Jos lados del triángulo.
9. En el triángulo r~ctá!.'igulo AB6, desde el vértice C del ángulo recto, se
han t~azádo Ja bisettriz:CK y !a.mediana Cl\1. Hallemos· tos catetos si CM= m
y KM= n.
.
.
1.0 . En un triangulo· rectángulo hny que hallar el ángulo entre la mediana
y la bisectrh trozadas desde -el vértlco del ángulo agudo, igual a et.
H. En el triángulo rectángulo ABC se han trazado las bisectrices de los
ángulos agudos AD y 'B K. Hallen los ángulos del triángulo si sabemosqueAB2 =
= AD·BK.
12. Demuestren quo si la altura y la mediano trazadas desde uno de lo~
vé.rtices de un triángulo isóscoles yacen dentro de éste y forman con sus lados
Jntorale·s ángulos iguales, dicho triángulo es rectángulo.
U. Triáugulo isósceles
13. Demuestren que si en un triángulo la razón de las tangentes do dos ángulos es igual a Ja razón de los cuadrados de los senos de est-Os ángulos, el triá.ngulo
es isósceles o rectángulo.
14. Demuestren que si en un triángulo se verifican las relo.cioncs
b
'te
..
1cs.
cosa A =co;¡¡
, es
es 1sosce
15. L11 base clo.,un triángulo isósceles es igual a 4 'V2 cm, la mediana trazada al lado lateral es igual a 5 cm. Hallen el lado latera.l.
16. El Jacto lateral de un trhíngulo isósceles es igual a 4 cm, In mediana
trazada al lado lnLeral e.!! i~ual n 3 cm. Hallen In bnse del trián~ulo.
17. La baS<: de un iriangulo isósceles es igual a t2. cm, mientras que el
lado lateral, a i8 cm. A los lados laterales so han trazado alturas. Calcular la
largura del segmento cuyos r.xtremos son las bases de lns alturas.
18. La base de 11n triñngnlo isóscelo11 es Igual a 12 cm y el lado laternl,
a t8 cm. A los lados lat.ernles se han trozado bisectrices. Calculen Ju largura de
segmento cuyos extremos son las bases de las bisectrices.
19. La suma de dos alturas diferentQS d o un triángulo isósceles es igunl a/,
el :íngulo en el vértice, a. Hall1m el lado lateral.
30
Capitulo /. Pla.nimclría
20. En la altura BJf del triángulo isósceles ABC (AB = BC) se ha lomado
el punto de modo que Jos ángulos AMB , DMC y Alt1C son iguales. ;En qué
rntón , contando desde el vérli~, el punto M divide l a altura si el ángulo junto
a la base del triángulo es igun 1 a a?
21. El ángulo ¡unlo a l a base de un triángulo isósceles es igual u et. Hallen
la razón entre la base y la mediana trazada ni l ado lateral.
22. H allen los ángulos de un lriñngulo isósceles si sabemos que el orlo·
c~ntro divide por la mitad la altura lrnzadn o la hase de la figura.
23. Las rectas IJ, 1 y 13 son paralelas, con la particularidnd de que l.
yace entre 11 y l$ y está distanciada do ellns n p y q. r~spoctivnm enlc. Hal l ~n e[
lado de un t rhlngulo regular cuyos vórtices yacun en las roct.as dadas (uno en
cada una de ellas).
24. En el t r iángulo isósceles A BC (A B = BC) en el lado AB so ha torondo
el punto D y en el BC, el punto E, de modo que BD ~CE. Demuestren que
un conjunt o de los puntos medios de t.odos l os segmentos DE coincide con
la línea media del triángulo ABC.
25. En un triángulo isósceles el ángulo en el vértice es igual a 36° y la base
a a. Hallen l os lados laterales del triángulo.
26. En el triángulo isósceles ADC (AH
DC), en e l lado ne se ha lomado
el ))llllto D de forma que DD : DC = 1 : 4. Hallen BM : ME", doudo BE ~s
In altura del triñngulo y M, el punto tle iutcrsccción de BJ:; y AD .
27. La base do un triángulo ísósceles es igi¡nl a a. el ángulo en el vórtice,
a 2C1.. Hallen Ja lar¡¡ura do Ja bisectriz trazada al lado lntcral.
28. Por ol vértice de un triángulo regular se ha trazado un rayo que divide
la liase como 2 : 1 . ¿Qué ángulos forma este. rayo con los lados latera les del tri·
ángulo?
29. El ángulo en la hase de un triángulo isóscolcs es ig11al a arclgf. Hnllcu
=
el :\ngulo entre Ja mediana y la bisectriz trazadas al lado lateral.
30. Hallen el ángulo en el vértico del triángulo ísóscel~s si la mediana,
3
trazada al lado l~tcral, forma con Ja base el ángulo nrcscn .
5
3 1. En el triángulo isósceles ADC el ángulo 8 es ig11al a l!O°. En el intc·
rior del triángulo se ha tomado el punto .M de modo qu e LMAC = 30°,
LMCA = 25°. Hallen ól ángulo DMC111. Triángulo arbitrario
32. Demuestren que si dos lad os y la altura de un triángulo ocutáugulo son
Iguales, respectivamente, a dos lados y a la alturn de otro triángulo cicµ\iingulo, scm~ja ntes t riángu los son iguales (considerar dos casos).
33. Demuestren.que si dos Jados y la me.diana dc ·un triángulo son iguales,
respectiv;uncnte, ·a do~ lados y a In me~iana de otro triá'ngulo, semejantes lrián·
gulos son iguales (considere.o dos Cll$OS)..
34. Dcmu~stron que en t.odo lrj:fogulo Ja bisectriz bien coincido con la
mediana ·Y lo ~ lturn, trazadas desde 111\ misinó· vórtice, o bien yace enlre ellas .
.~., Dcm¡i~str~n que. en t.oao trhing\)lo la suma. de l as medianas es mayor
•
.
quo 3. de1 .penmetr11,
pero menor que este.
4
36·. Por el centrnide del tr.iánguló regular A BC se hn trazado la recta l
qua cruza los lados A B y IJC. Demostrar QUQ lo suma de las dist nn cias desde
A y C hasta 1 es igual n la distancia desde B hasta l.
37. En el triángulo ABC el ángulo lJ es .igual a 115º. Desde el punto medio
.<101. lado A C se ha .tr11zl\do una perpendicular· 11 A C hasta su intersección con el
lado BC en el r 11nto D. El St'gmento.AD divido el ángulo A como 5 : 3, contando
desde el Indo A C. Hnllen los ángulos A y C del triángulo ADC .
:11
§ 2. Trl<ingulos y cuadrll6l•ro1
38. La biscctrh: deJ óngulo de un triángulo divide el lado opuesto en segmentos de largur:i 2 y 4 cm y la altura, trnuda a ose mismo lado, ei; igu11l e
")l'iS cm . Hallen los lad 0$ del triánguJo y determinen su tipo.
39. Hallen la razón entro la suma de Jos cundrndos de lns med iano s do un
triángulo y Ja sumo da los cuadrados do sus lo.d os.
·
40. Determinen el tipo del triáqgulo si sabemos que sus medianas estñn
mt = sna¿.
ligadas con In igualdad m~
,1. Dos lados de un triángulo son Iguales 11 o y b y las mcdionll!, trniadas
a estos lados, son perpe11dfcularc$ entre sí. Hallen ol tercer lado del triángulo.
/¡2. En· el tri ángulo A BC se ha. trazado la bisectriz A. D. Hallen el lado BC
si sabemos que A C = b, Ali - e y AD - DO.
43. En-el triángulo A BC es conocido que DC - f 2 cm.• A C = 8 cm y el
ángulo A es dos VCCC5 mayor quo el ángulo B . HaJlen AD .
44. La aJt.U.ra del triángulo es igual a· G cm y divide el ángul o como 2 : l
y In base de la figura, en segmentos, de l os cu;ilcs ol menor es igual a 3 cm . Hallen
los lados .del .triángulo ..
,5. La al~ora del t r iángulo divide los ángu los como 2 : 1 y la baso, en sogmelit.os, Ja. razón entro los cuales es igual n k (k > t). Hallen el óogul o mnyor
en la bosc del 't riñngu lo·.
46. En el •.rióngu lo oéutllngu lo ABC el úngul o 11g11do entro hts ollurns AV
y C8 es Igual a á. ,Hallen A C si A.D = a y CE = b.
47. La boso da un triángulo es lgunl o 4. La medionR, trazada n In base,
es Igual a Y6y uno do los· ángulos en In baso, R f 5°. Hallen el ángulo
ugudo en\re In mediana y 111 base.
48. Haciendo u~o dol teorema de Ceva demuestren, que :
a) Jas mediaJ111.s del triángulo se c9rtAn on un mismo punto;
b) l as biséctriccs do un trlángulo se intersccon en un mismo punto;
c) las. alturas do un trlóngulo so cortan en un ml su10 punto.
49. La-.reeta DE es paralola a la base del triángulo ABC, con IR porticula·
ridad de quo cJ punto D yoco on el Indo AB y el E, en <ll lado BC . Demucstr~u
quo A E, CD y In mediana BM se cortan en un mismo punto.
50. Demuestren quo si los lorguras de los l11dos de un triángu lo forman unn
progresión aritmética, el centro de la circunferencia inscrita en dich<> lrhingu lo
y el ccutrolde de éste yacen en una recta paralelo al lado del triángulo do lnrgura .media.
5 1. AD es lo altura dol triángulo ABC, el punt.o Jf es el ortocenlro. Demuestren que DC ·D.B
AD ·DJI.
5Z. En el Uiángulo A BC el ángulo A es igunl a 30" y <ll IJ, a 50°. Demuestr en
que Jos lados del t riángulo están ligados con la reloclon e• = b (a
b).
53. DernuestrNl que en tOdo. ~riángulo In dlícrcncln entre lu suma cl r. los
cuoilrados do las larguras do cualesquiera dos do sus Indos y el prnducto d o In&
Jargu.rns de estos lados, multiplicado por el coseno dol ángulo entre ellos, ~s
una magnitud constante (parn ol tri ángulo dado).
54. Demuestren quo en todo triángulo el ortocentro, el ccntroide y el centro
de la circunferencia circunscrita yacen en un:i miama recta (ruto dt Eulu).
55. En los triángu los A BC y A' B'C' los ángu los 8 y B ' son iguales, en
tanto que lo suma de loa óngulos A y A' cons tituye 180°. Demuestren quo los
Indos de estos d os triángulos eslán l igados con lit relación aa' ... bb'
+
v2
=
+
+ce'.
+
56. En el triángulo ABC los ángulos A, 8 y C están rcl ac ionnd os como
4 : 2 : 1. Domueslten que los lados del triángul o c~llin llgndlls con Ja igun ldntl
'I
1
i
a-+11=7·
!i7. La altura , moilinnn y biser.Lriz troznrlns destlo uno dellos vértices de un
triángulo dividen el ángulo en cslc vlárlico en 4 p&rtes Iguales. Hallen los úngul11s
del tri ngulo.
32
Cap1tt1lo 1. Plonimetrlo
58. CD es la altura del triánpulo A B C , Hallen la dependencia entro los
ángulos A y JJ si sabemos quo CD - AD·DB .
59. En el lriángulo A BC el ángulo A es igual a a y el B. a ~. la mediana
BD cruia la bisectriz CE en el punt.o K. Hallen C K : KE.
IV. ParalelogJamo
60. Demuestren que si en un cuadrilátero las di3gonales son las bisectrices
de los ángulos, el cuadrilátero es un rombo.
6t. En un paralologramo con lados a y b (a> b) se han trazado las blsectrlcQ.!I de los ángulos Internos. Definan ol tipo do cuudrilálcro formado d11rnnto la
intersección de las bisectrices y hallon 111 lnrgurn de sus diagonales.
62. La altura de un rombo d ivido ~u lado on los segmentos ni y "· Hallen
las diagonales del rombo.
63. La pcrpendicul:i.r trazada desde el vértice de un paralelogramo huta su
diagonal divide ésta en l!egmentos de 6 y 15 cm. Hallen los lados y las diagonnles
del paralelogramo si sabemos que la diforoncia do los lados es igual a 1 cm.
G~. Dos alturas de un paralelogramo. lra:taclas dll-$()e el vértice del ángulo
obtuso, son iguales a p y q, respedivamonlo, el ángulo ontre ellos es igual a a.
Hallen la diagonal mayor del paralelogumo.
65. La diagonal do un rectángulo divido su ángulo en la razón 111 : n. Jhllen
la ra1ón entre el perímetro del rcetánlJ'llO y su diagonal.
66. El ángu lo agudo do un paralelogramo es Igual a a y sus lados, o. a y b.
Hallen lns l11ngentc.s de los ángulos agudos, que su diagonal mayor forma con
los lados del paralelogramo.
67. Hallen el ángulo agudo del rombo A BCD si una recta, trazada por ol
vértico A, divide el ángulo BAD en la rar.ón 1 : 3 y el lado BC, en In razón 3: 5.
68. La razón entre el perímetro do un rombo y la suma do sus dingono lcs es
igual 11 k. Hnllen los ángu los del rombo.
69. Las diagonales de 11n paralelogramo son proporcionales a sus lados no
par11lelos. Demuestren que los ángulos entro las diagonales son iguales 11 los
ángulos del paralelogramo.
70. En el rectángulo ABCD la base AD eslá dividida por los puntos M y P
en tres partes iguales. Demuesuen que la suma do los angulos AMB, APB
y ADB os igual a 90º si sabemos quo AD =- 3AB.
71 . J,os lados de un pAralelogrnmo son iguales a a y bJa > b). La diagonal
menor forma con el lado menor un ángulo ob\uso y con el la o mayor el Anguio a .
Hallen la diagonal mayor del paralelogramo.
72. Los lados de un paralelogramo están ligados con la raz6n JI : q y l:i.s
d i"gonales, como m.: n. Hallen los óngulos de -la figura.
73. La razón entre el perímetro de un parnlclogramo y.su diagonal mayor es
igual o. k . Hallen Jos ángulos del poralclogramo si se sabe que su diagonal mayor
divide el ángulo do la figura en la rai6n 1 : 2.
Y. Trapecio
N. Demuestren que si los lados de un trapecio son. respectivnmeute,
Iguales a los lados de otro, los trapccíos son iguales.
75. Demuestren el teorema: pnra que el trapecio sea isósceles es necesario
y suficiente: a) que los ángulos en la base sean icudes; b) que las diagonales sean
Iguales.
76. De.muestren que las bisecLrieca de los ángulos ndyacenLos al lado lateral
del trapecio so cruuin formando un ángulo recto y que el punto en quo so cortan
yace en la lineo media del trapecio.
77. La suma do los ángulos en la base de un Lrnpccio es i~unl n !JOO. Demu1utrcn que el segmento que une o1 punto medio de las b3ses es igual n la semidilor eocia do las b:iscs.
78. Lns d iagonales de un trapecio son i11ualos y pcrpendicnlaros en l rt' sí,
la ollura es igual -a t5 cm. H"allen la lnrgur11 de la línea medin del trapecio.
7!1. Una de l ns bases de un trapecio es igual a 24 cm, y In distancia onlre
los punl.os medios de las diagonales es iguol a 4 cm. ffallcn la olra base.
80. Uno de los ángulos de un Lrnpeoio os igual a 30~, l os la dos la l era les son
perp ondieulnres. Hallen el lado lnleral menor <lol trnpecio si su línea med in es
igual n 10 cm y una de las bases, a 8 cm .
81. En urt trapecio r~tángul o los bases y ol lado later11I meno~ son igua les
a a, b .y e, rospectivamento. Hallen la distnncia desde ol' punto de intersección
de l as diagonales hasta la base y el lado lateral menor.
· 82. (;os bisectrlcos de los ángul os obtusos cm In base do un trapecio se oruznn
en su otra bnsc y tienen unas larguras do 13y15 cm . Hallen los Indos del t rapec io
si su altura es igual " 12 cm.
83. Ln nltura de un IJ'apccio isósceles es igual n h, el ángulo agudo entre
las dla11onales es 2ct. .Hallen la largura do lo base media del trnpecío.
84. En el trapecio ABCD los ángulos A y IJ son rectos, AIJ = 5 cm, BC ""
= i cm, AD 4 c:m. Bn él lnclo A 8 so hrt lomntln el punto M olt> modo q11c rl
ángulo AMD sea dos veccs1nay11r quo el á 11gulo M C. ll ollcn l;1 r11zón A M: fof/J .
85. El ángulo en el yérlice A del trapecio A BCD es igual a et, el lado
b teral AB es dos veces mayor que su baso menor BC. Jfallon el ánculo BAC.
8G. La baso m:iyor de un trapecio es igunl na , los Indos laterales son iguales
a b y e (b < e) y la rnóo de Jos angulos en la bnsc mayor os como 2 : 1 . Hallen
la baso mono r .
87. Las dingo nales AC y BD del t rapecio is6seclcs ABCD (AD 11 BC) concurvcu en el punto O, siendo LAOD
(100. l)emuesLrcn que K, M, P, quo
son los puntos medios <lo los segmentos A O, 0 0 y CD, corrcspo ndc11 a los '"érli.ces clo un .triángulo regular.
88. Oomuoslrcn que In suma do los cuod raclos do lns diagonnlc.'S de un t.rn peoio es igunl al producto 1luplic11clo do su8 bn scs, ncllr.ionando In sumn de los
cualltados do los Indos latemlcs.
8!J. Demueslren que la recta que pnsa por ol punlo de inlor3ecríón de lus
continunciones de Jos lados laterales del trapecio y ni punto dl' inlcrsccción
ele sus clingon11lcs, dividen lo. s buses riel Lrupcci o por lo. mitacl .
90. En un Lr:ipecio con lmses a y b se ha traind o por el punto do i11t.e1·sccción
de las diagunales una recln paralela a l:is bases. Hallen la largura clt?I segmento
de esla rceta que se encucntm entre los Indos loteralcs del trnpocio.
91. En ol trapceio A BCD cncla una 110 lns bases AD y BC se CQntinúa ha cia
los dos lados. Las bisectrices de los ángulos externos A y B ~ C-Orla n en <'I pnn·
to K y las bisectrices de los ángulos externos C y O, e n el punlo E. 1-lllllt>n el
perímetro del t ra pooio si KE = 20 cm.
112. Por el p unto O de intersección de ln s diagonales ilcl lr-J¡>c.><:io lsóscclns
ABCD (AD 11 BC) . cuy:is diagunnlcs son pcrpl'ndlcu lnrcs entre si. se ha tr;m1do
lo l"l'Cta MK, perpendicular al Indo CD (el pu nt.<> M ynro t'n AB, el punto K,
en CD). Hn ll cn /lf K, si AD ~ r,o cm, lJC - 30 c m .
=
n
=
vr.
llHcrcntcs pr()blcmns
!13. E n el cunrlrilátcro ABCD. l'. K. E y M son Jos ponlos mrdi os de los
lados AB , BC, CD y D A . Oconucstren q110 el cnadrilñtcro PKFJM es un par11l11loi:romo.
!l4. En los cat~los A C y BC tlcl Lriiíngulo rocl iingulo A BC s e h:in construido
(fuera del tríiingul<1) los !'undntdos ADl\C ~· C8MO. Ot1 IM ponlos V y ¡\/se
bajan los pcrprnolieularcs DI/ y M f' sobro In continuación de la hi110Lcnus:1 A 8.
Oemuostron quo DH
.MP = AB.
+
3-0290
34
Capítitlo l. Plnnimetria
95. En Jos lo.dos de un paralelogramo (fuera de él) se bnn construido cuadrados. Sus centros están consecutivamente unidos. Demuestren que el cuadrilát.cro obtenido es un cuadrtido.
·
96. En un triángulo rectángulo con catetos a y b está inscrito utt cundrndo
que tiene con el triángulo el ángulo recto común. H alJcn el ,perímetro del cuadrado.
97. En un li-iángulo rectángulo está inscríto un rombo do íorma que !.(idos
sus vértices yacen en los lados del triángulo y el ángulo igual a 60º es común
parn el triángulo y el rombo. Hallen los lados del triángulo si el lado del rombo
es igual a 6 cm .
98. En un triángulo está inscrito un rombo de forma que uno de los ángulos es común para las dos figurns, en tanto q uo el vértice opuesto del ro mbo
divido el lado del triángulo en .segmontos entre los que la razón es 2: 3. Hallen
los lados del triángulo entro los que yoce el ángulo común del triángulo y el
rombo si las dia1;1onolcs del rombo son 111 y n.
99. En un truíngulo con l ados latcr~les 9 y 15 cm está inscrito u.n pamlclogramo de forma que uno de sus lados de ·e cm de largura yace en la base del triángulo y sus diagonal es son, respectivamente, para)elns a los Indos lnlerales del
triángulo, }iallcn el otro Indo clr,1 par~l~logr~mo -y 1~ base dC?l triángulo .
100. En el cunclrado A BCD esta rnsc r1t.o el rr1áng11lo 1s•ísr.elcs A KM de
forma que el punto K yace en el lado ne, el pu uto M , en el CD y A M = A K.
Hallen el ángulo MAD si sabemos qu!l ln Lg L AKM = 3.
101. En el lrhíngulo regular .A BC está inscrito el triángulo regulnr DE K
de forma que el punto D yace en el JadoBC , el punto E, en A C y el punto K.
en AB . Hallen AD: DE si sabemos que LDEC = a .
102. Demuestren que los segmentos· que unen los puntos med ios de los
lados opuestos de un cuadrilátero conve:'ío y el segmento que une los p1rntos
medie>s de sus diagonales, s~ .:rui o.n en uu mismo punto y s11 div.iden por él en
ln mitad.
103. Demuestren que en un cuadrilátero convexo los puntos medios de las
tliogonales y de los segmentos q1lc unen l os puntos medios do los lados opuestos
yacen en una misma recta.
101. Demueslrcn que si en un c11odrilá1cro la suma de los cuadrados de los
lados opuestos son iguales , sus dingcmales son perpendiculares entre sí.
105. Demuestren que si en un cuadrilátoro co·nvexo el segmento que une
l os pun los medios de los lados opuestos es ígual a la semisuma de los ot ros dos
lados, semejante cuadrilátero es un trapecio.
1OG. Las bases do los t riángulos regulares con lados a y 3a, yacen en una
misma liocn. Los triángulos están situados por diferentes lados de una recta
y la dista.ocia cnLre los extremos más próximos de sus bnscs es igual a 2a. Hallen
la dislm1cia entre.los vértices do los triángulos que no yacen en la i ndicada recia.
107. En el cuaclrilátero ABCD se conoce qué LA= 4-D = GOº, .AB =
= 1( 3, PC == 3, cp = 2 1/3. Hallcn.lc;is ángulos B y C..
_108. Lns dia*"onales del cunclrllátcro. c<;>.nvexo ABCD se cort11n en el puntó O bajo ángulo .recto de form,¡-_quo AO =r 8 cm,. Bp""' CO = 1 cm, DO=
= 7 cm. Al· col)tinuar los lados AD y CD ellos so cor.l an en el punto M, Hallen
el ángu lo A 11.fD.
109. En el cuadri látero ABCD. el ángulo Bes recto y AD : BD = 2 : 4if2.
Al continuar los la<los .OC y AD ellos .concurren en el punto M. Hallen el ángulo
D/lfC si ~abemo~ que .LADD = 45°.
L10. ·En d rect6ngule ABCD está Inscrito el triángulo AEK de forma
que el punto E yace en el lado BC y· el punto K, en CD. H111le11 el ángulo
. AB
BE
CK
1gL EAK
s1Tc =cs=w=m.
.~
3 . Circunfeuncltt
35
§ 3. CIRCUNFERENCIA
EJEMPLO 1. Demostremos que si a y b son los catetos, e, la hipotenusa de u11 triángulo rectángulo y r, el radio de la circunferencia
.
'ta, r = tt+b- c
mscn
2
s0Luc10f'/. Realicemos las construc.ciones onxiliare·s necesarias:
desde el centro O de la circunferencia inscrito tracemos los radios
OD, OE y Q/•' a los punlos de tangencin. Entonrcs, OD ...L. RC, OE ..L
...L AC, OF ...L AB (fig. 44). ODCI:: es un cuadrado (todos los ángulos,
lJ
,,
Fig. 41
Flg. IS
rectos y OE = OD) , es dr.cir, CB =CD = r, BD =a - r, A E
= b - r. Pero JJD = BP y AE = AF (Lcoromn 12b), por lo tanto,
BF = a. - r,
AF = b - r. AB = lJF + AF, os <lr.r.ir, e =
=(a - r) + (9 - r), de donde h111lnmoR que r = o-1-~-c.
Ol!SERVACION. L o 16rmula obtenida se u U lita con frccucnci11 :il
rrsolv~r
problemas Ugados con triángulos rectángulos:
EJEWPLO 2. A Ja circunferencia inscrita en un t riángulo, cuyo
perímetro es igual a 18· cm, se ha trazado una tangente paralela a
Ja base del triángulo. La largura del segmento do la tangente situado
entre 'los lados laterales del triángulo es igual a 2 cm. Hallemos la
base del triángulo.
SOLUClON. Sean M, P y N los puntos do tangencia (fig. 45). Ento n~
ces, AM =AN, CN = CP, BP = BM (teorema 12b). Hagamos
AN = AM = x, CN = CP =y , /JP = /JM = z. En tal e.aso, el
perímetro del triángulo ABC scrü igual 11 a + 2y + 2z y, por
consiguiente, x + y + z = 9.
Tracemos la tangente DE 11 AC. Entonces, los triángulos ABC
y DBE son semejantes y, por lo tanto, la razón de sus lados es igual
a In de Jos perímetros: ~g = :;:: , es decir,
2
Po11E
.i:+y= - ,s3•
(1)
36
Capitulo 1. Pla11lme1rla
PDss=BD+BE+DE=BD+BE.+(DK+KE) = BD+ BE+
+(DM+EP). (Aquí, hacemos uso de que DM=DK yKE=EP).
Por consiguiento, PonE= (BD+ DM)+(BE +EP)=BfVI BP = 2z
+
2
y, enlonccs, la igualdad (1) puedo reescl'ibirse en la forma - -=
:r:+11
2•
= 18.
ObLcnemos ol sistema de ecuncionos:
:i:+ Y= b,
b+z=9,
=
,
18
t
b= 3 cm, o bien b= G cm.
obt.enemos
¡x+y+z=9,
l z+y- n ·
-2- _ ~
Haciendo
de donde hallamos que
/Jz
bien
F.JEMT'LO 3. Por el punto A de la cuerd a común AB de dos circunferencias se ha t.r azado una recta que cruza la primera circunforencia
en el punto C y la segunda, en el D. La tangente a la primera circunforoncia en el pun to
y la tanM
gonlo a segunda circunferencia en
el punto D, concurren on el punto
M. Demostremos que los pu ntos
Jl1, C, By D yacen en una circunferencia (fig. 46).
SOLuc10N. Es suficiente deroostrur
que
LCMD
LCBD =
= 180° (teorema 15a). LCBA =
= v AC (como ángulo inscrito).
e
+
i
1
+
Pero, osimimo, LMCA = 2 V
v AC (como el ángulo entre In
tangente y la cuerda, teorema 13).
Por lo Lant.o, LCBA = LMCA.
Flg. 16
Por analogía se domuostra que
LABD = LADM.
Del 6AfCDllegamosalaconclusióndequeLCMD
LMCD
LMDC = ·180º. Pero LMCD
LMDC = LCBA
LABD=
+
=LCBD.
+
+
+
+
Por consiguiente, LCMD
LCBD = 1800 que es lo que habíamos de demostrar.
EJEm•Lo· 4. En "na circunforcncia ostá inscrito el triángulo isósceles ABC con baso AC
b y ángulo a en la base. La segunda
circunferencia es tangente a la primera y 11 la base del tri6ngulo en
su punto medio D y se encuentra fuera del triángulo. Hallemos el
radio de l a segunda cirC\mferencin (fig. '17).
s0Luc10N. Hagamos uso do que AD·DC = BD·DK (teorema 16a).
'
.
b2
Como
AD = DC = ¡, , BD = 211 tg ex, OK = 2r, obtenemos:
4 =
=
= 2b
2
b
.tg a·2r, de donde r = 4 ctg a.
37
§ 3. Circu11/er1ncltt
EJENPLo ~. Una circunCcrencia de radio R pasa por los dos véctices adyacentes de un cuadrado. La tangente a In circunferencia, trazada desde el tercer vértice del cuadrado es dos veces mayor que el
lado del cuadrado . Hallemos el lado del cuadrado.
SOLUC ION. I ntroduzcamos los designaciones: AB = x, BM = 2x
(Hg. 48). Continuemos el segmento AB hasta su intersección con
la circunferencia e n el punto K. Entonces BK ·AB = BM 1 (teorema
B
C
R
,.
Jllg. 17
f"lg. 48
Hlc), es decir, BK·x =4x2 , de donde Jinllomos: JJK = 4x, os decir,
AK = 3x. LKAD = 90°, por lo tanto, [(D es el diámelm. Del
triángufo rectángulo ADK, hallamos: AD~
A xz = KD~. o seo,
x-.
+
4R 2 , de donde x = -RVm
5
E,EMPI.o &. El ángulo entre los radios del seclor circufor es igual
o. 90q. Una circunferencia de ese mismo radio tiene su centro en el
extremo del arco del sector y ella divid!l el sector en dos triángulos
curvilíneos. En el menor de éstos está inscdta una circ\IOforcncia.
Hal!emos la razón de los rudios de la cit'C\lllforencia insc.rita y el
sector.
-·
SC1Luc10N. Renlicemos las construcc.ioncs auxiliares necesarias,
las que, por rl'gln, se lwcon cuando sc lral n de la tnngcncia iutcrna
o ex terna de circuníerencias o la t.angencin de una circunferencia con
unn recta: 0:03 es la línea de los centros; B, el punto de tangencia;
0 1 0 3 , Ja línea de los centros; A, el punto de tangoncia; 0 3 C ..L 0 1C;
C, el punto de tangencia (fig. 49). Jnlroduicnmos la design11ció11:
0 10, = R (pará metro auxiliar) y ox11rcscmos con R. el radio r de
Ja circu nferencia inscrita.
Consideremos el t::.Ot0z0 3 • Tenemos: 0 1 0 2 = n, 0 10~ = R - r,
020 3 = R
r. Trar.emos Ja altura 0 3 /1. Entonces, O,H = 0 3 C =
= r, O~H = il - r. Utilicemos 0 3 H colllo elemento de referencia.
+ 9x' =
+
Capitulo 1. Planlm~trfa
38
Del t::.0 10 3 H, tenemos: 0 3 lli = 0 10! ~ 0 1H 2 = (R - r)' - r 1 • Del
t::.03H0 2 , tenemos: 0 3 H2 = 0 20~ __.: OiHt = (R
r)' - (R - r) 2 •
Así, pues, (R - r)2-r2=(R+r)2-(R-r)2, do donde hallamos
+
R
r= 6
r
1
. O sea, -¡¡=ij·
EJE~1rLo 7. Dos circunferencias con radios r y R son t angentes
exteriores. AB y CD son sus tangentes externas comunes. Demostremos que en el cuadrilátero ABCD se puede inscribir una circunferencia y, además, hallemos su rndio.
so1.ucior.i. Rea licemos l as cons trucciones auxiliares necesarias.
Continuemos las tangen tes hasta su concurrencia en el punto O,
tracemos la llnea de los cent ros 0010 1 , tracemos los radios 0 1 D y
OtC en los puntos de tangencia 0 1 D .l.. CD y O,C .l.. CD (fig. 50).
Ftg. ~9
Como Ja linea de los centros es el eje de simetría de la figura, los
puntos A y D son simétr icos con relación o OOi y los puntos B y C
son simétricos respecto o 00:. Esto significo que ABCD es un trapecio isósceles.
Para que sea posible inscribil' una circunferencia en el trapecio,
es necesario y suficiente que se veriHque la igualdad AD
BC =
= AB +CD (teorema i6b) o bien, ya que AB =CD. la igualdad
AB = ADTBC. Es suficiente demostrar que el segmento AB
es igual n la base medía del trapecio.
Tracemos la tangente i n terna eomún KP. Entonces, AK =KM,
BK =KM, DP = PM, CP = P'M (teorema 12b), lo que signüica
que KP es Ja base mcdin del trapecio. A-BCD y ' KP = AB. Así,
pues, en el trapecio podemos inscribir una circunferencia y EF es
su diámetro. Hagamos 0 1E = x, OiF =y. Entonces, de Ja igualdad
MF =ME (la base· media KP divide al segmento EF por la mitad)
llegamos n la conclusión de que R - y = r
x. De la semejanza
+
+
.li 3.
Circti11ftrt11cla
0 E
0 D
1
1
de los triángulos O,DE y O~CF, obtenemos: o,F
= o,c
, o sea,
x
r
-;=If·
R-y=r+x,
n:-rn
= .!....
ha liamos: y= R='r"
{ .=...
y
R
y, entonces, el radio de la circunferencia inscrit.a es igual a
R-y= R2~r.
E1EM'PLo s. El ángulo agu do de un triángulo rectángulo es igual
Del sistema de ecuaciones
a a. Hallemos su hipotenusa si el radio de la circunferencia, tangen te
a la hipotenusa y a las co ntinuaciones ele los catetos, es igual a R.
e
PI¡¡. 51
flg. 5 2
+
soLucrON. Como AB = AK
BK (fig. 51), el problemn se reduce
al cálculo de los segmentos AK (del D.AOK) y BK (del t::,.OIJK).
Consideremos el t::,.AOK. Como LKOF = LIJAC = ex (ángulos
con lados perpendiculares entre si). LKOA
~ t::,. KOA = D.AOF).
= (
Do modo que AK = OK·tg ~
Examinemos
el
t::,. BOK.
=R
tg ~ .
L BOK =
+
LDOK = +(!JOº -
- L f(OP) = 45º - ~ . (Aquí hemos hecho uso de que on el cuadrilátero ODCP, tres ángulos D, C y F son rectos y, por lo uinto,
asimismo, el cuar to ángulo, es decir L DOF, es recto.) Entonces,
BK =OK·tg LBOK=Rtg( 45°- ~), AB=AK +BK = Rtg ~
+
+ R tg ( 45° - .S:..)
=R
2
sen { !:.+45º-~)
2
cos
~
¡¡
cos ( 45º-f)
-
R 112
=
2 cos
Teº' (45º-
~)
.
scn(a. +~. )
( Aquí, se ha hecho uso de 111 fórmula tga+ t.g~= COSO.
C<IS
CapfttLlO J. Planimetría
EJEMPLO 9. Se ha dado un triángulo"1:>cutángulo A·BC con ángulos
LA = a, LB = ~. LC = y. ¿En qué razón divido el ortocentro
la altura trazada desde el vértice A?
sowc10K En torno del triángulo ABC circunscribamos una circunferencia. Designemos con R el radio ele ésta (parámetro auxiliar).
Tracemos OP _t_ BC y hagamos uso <lo que AH = 20P (véase
el ejemplo 8 del § 2), donde Hes el ortoccntro.
Consideremos el t:.OP B (fig. 52). C-Omo LKOB se mide con el
arco BK, uBK =+u BC y LCAB se mide con la mitad del
arco BC, LKOB = LCAB =a. Entonces, OP = R cosa; y, por
lo t anto, AH = 2R cosa .
AC
De ncuerdo con el teoromn de los senos sen LABC = 2R (teorema 8), es decir, AC = 2R sen ~y, en tal caso, del t:. ACD obtenemos que AD= AC son L ABC = 2R sen~ sen 'V ·
B
A
(a)
(b)
Flg. 59
E ntonces, AH= 2R cos CG, HD =AD - AH = 2 R sen ~sen V 2R cosa = 2R (sen f. sen V - e.os (180" (~
v))) =
= 2R (sen ~sen y cos (j} + y)) = 2R (sen j} sen y
c.os ~ cos y - sen ~sen y) = 2R cos ~ cos y.
A ·
·
A·H
2R cosa
cosa
cos~cosv •
· sr , pues, HD = 2Rcos~cosy
EJEMPt.O 10. Demostremos que si la altura y la mediana, trazadas
.desde uno de los v~rtic0$ .de un triángulo escaleno, yacen dentro del
triángulo y · forman co.n sus lados l aterales ángulos igualos, dicho
triángülo es rcctángulQ.
s0Luc10N·. Resolvamos este problema con dos procedimientos.
l procedimiento (geométrico) (fig.. 53, a). Circunscribamos al triángulo dado ABC la circunferencia co y tracemos la al t ura CH y
la m·efüana CM hasta la intersección con· la circunferencia en los
puntos D y ·e, respectivamente.
-
+
+
+
Como LACD ·= LBCE, vAmD = vBnE Y~ por lo tanto, las
cuerdas AB y DE, entre las qu.e yacen los arcos iguales AmD y BnE,
son paralelas. Pero LCHB = 90º. Esto 9ufore decir que también
LCDE = 90º. Entonces, CE es el diámetro de la yircunferencia.
Este divide por Ja mitad la cuerda A B, lo que sólo os posible .cuando
la cuerd11 AB es el diáxp¡¡tro de la circunferencia w. Pero, en tal caso
L..ACB .= 90º , es deci r , el ¡:,ABC es.rectangulQ.
II pro~edimiento (algebraico) (fig. 53, b). Deinos~remos que
LACE = 90°. Es evidente, que la suposición do quó LCAB """· 90°
es inverosímil, ya que entonces el segm_e nto CH coincido con el lado
4.C y, po~ ,lo tanto, ·(lebc coincidir e.o n el lado BC, para que se formen
los ángulos iguales prefijados. No obstante, la mediana no puedo
coincidir c~>n el lado del triángulo.
Se!! LACB = x. Esta es la incógnita 4ue buscamos. Para abreviar, hagamos asimismo LACH =a. Entone.es también LBCM =
' := ·(%. ~continuación, intrpduzcamos el pnrúmotro 1111xilinr: CH =
.= h. Entonces, LBCll = x - a, LMCfl = x - 2a.
Como CM es l~ medi_a na, ÁM ::: 13M. Pero AM =AH + Mil y
BM = 1)1{ - MH. Asi ; ·pues, Ali + Mll "="81/ - MH, o bien
2MH = BH - A}{. D_e lps triángulos rectángulos MCll, BCI! y
A CH tenemos lv!H = .h tg (x - 2a), Bll = h lg (;¡¡ - a) y AH
= h tg a, respectivamento. Obtenemos la ecuación:
2h tg (x -
Za)
2 tg (x -
=h
tg (x - CL) - h tg a o bien
2a) = tg (x - a) - tg a..
Después do resolver esta ecuació11, hallamos:
2 s~n (.T. -2a) _
~os(x - 2a)
-
Como el triángulo ABC
sen (x-2a) =;l=O ,y, entonces,
sen (:r-2a)
cos(x-a)cosa
es
isósceles,
x =;I= 2a,
es decir,
2 cos (x - a) cos a = cos (x - Za) o bien
cos x + cos (x - 2a) = cos (x - Za),
de donde cos x = O,
es decir, x = 90º y el triángulo ABC es rectángulo.
&JElllPLO 11. AD y CM son las alturas del triángulo ocutángulo
ABC, cuyo perímetro es igual a 15 cm, el perímetro del triángulo
BDM es igual a 9 cm , el radio de Ja circunferencia circunscrita al
triángulo BDM es igual a 1,8 cm. Hallemos la largura de AC (fig. 54).
soLucxoN. Ante todo, demostremos que los triángulos ABC y BMD
son semejantes. En efecto, dos triángulos rectángulos ABD y BMC
con el ángulo agudo común B son semejantes y, por esto, ~~
ff~
=
.
Capitulo l.
t,2
Pl1111imetrfa
Pero, en tnl caso, los triángulos ABC y M BD, con el ángulo B común
y los Indos que los contienen, son proporcionales, es decir, semojantcs.
Hagamos uso de que en Jos triángulos semejantes las razones entre
los perímetros y entre Jos radios de las circunferencias circunscritas
son iguales a la razón de semejanza. De acuerdo con el planteamiento,
PA De = 15 cm, P DD~; =· 9 cm, por lo
que la razón de semejanza es igual a
Ya que, según el planteamiento, el radio
de l a circunícrencia circunscrita en torno
del triángulo BMD es igual a 1,8 cm,
obtenemos que el radio de Ja circunferencia circunscrita alrededor del 6. ABC,
A
~
I'
f·
C'
es igual n 1,8·Í = 3 cm.
Sea el punto O el cent.ro de la circunferencia circunscriLn en torno del
6.·ABC, OP ..L AC. Entonces, BIJ = 20P (véase el ejemplo 8
del § 2). Pero BH es el diámetro de la circunferencia circunscrita en
torno del Lriángulo BMD (ya que el ángulo BDH que en él se apoya
es igual a 90º), por lo que BH = 3,6 cm y, por ello, OP = 1,8 cm.
Ahora, en el triáugulo rectángulo AOP conocemos dos lados:
.A0=3cm (el radio de Ja circunforencia circunscrila) y OP=
Plg. 54
12
T )2 =-g-cm
= 1,8cm. Entonces, AP= v í O- ( 9
y, por lo tanto,
AC = 4,8cm .
.PROBLEMAS PARA EL TRABAJO lNDIVIDUAL
I. Circun fetcncía
111 . En el punto A dos circunferencias son tnogentcs ext.eriores, flC <'..S
su tangente comúo externa. Demuestren que LBA C = 90•.
112. O.os circuhlercncias sa intersccan en· los puntos A y B . Estos yacon
por diferente~ lados do la recta l que corta !ns circunferencias en )O.'J puntos
C, D, E y .M, respectivamente. Demu'c strcn que la suma de los ángulos DBE
y CAM es igual a j80°.
,t,f3. :Pos circunferencias ~e intcrsecan en los puntos A y B. Las rectas 11
y 12 son para.lelas, con la par.Ucularidnd de que 11 pasa por el punto A y corta
las circunforoncias en los pµntos E y K, mientras que l~ . v.asa por el punto B
y corta las circunferencias en los puntos M y P . Demuestren que el cuadrilátero EK MP es up paralelogramo.
U4 . Hacia una circunfer encia con centro ~n el punto O, dosde el punto M,
se han trazad<> las t'angentes M A y M B. La recta l es tangente a In circunferencia en el punto Ce intersl)cn las tangentes ftfA y .AfB en los puntos D y E, correspondientemente. Demuestren: a) que el perímetro del triángulo MD E no
depende de cómo se ha elegido el punto C; b) que el ángul1> DOE no dependo do
·1n elección del punto e.
43
§ 3. Circunferencia
115. Los puntos A, B, C y D dividen una c.ircunferencia en partes en tic
las que la razón es 1 : 3 : 5 : 6. Hallen los ángulo~ entro las tangentes a la circunferencia trazo.das d~d·e los puntos A, B, e; y D.
116. Dos circunferencias iguales son tangentes exteriores y co11 una. tercera
circunferencia cuyo radio Cll· igual a 8 cm. El scgment.o que uno .los puntos de
tangencia de las dos circunferencias iguales con In tercera es igual a 12 cm. Hallen los radios de las ·circunf9rencias iguales.
·
t 17. La c.u erda común de c!os circuníer.e ncins. que· se jnter~ecao es Igual a a
y, para una de ellas, es el Jac!o del hexágono regular .inscrito y, ·pa~a ·Ja otra, del
cuadrndo inscrito. .l;laUen Jo distancia entre los centros de las clrcunfere.nclns.
t 18 . .Doa circunferencln.s CQn radios r y R son tangeñtes. exteriores. HaUcn
la .largura de su .tangente. externa comí1n.
t 19. D.o s circunrercncias con ra'dlos r y f!. .spn tangentes exteriores. Ln
rec~a l corta la circunferencia en los puntos A , B, C y D de modo que A 8 =
= BC = CD . Hallen AD .
120. Dos ciTcunferoncias, la razón entre cuyos radios es 1 : 3, son tangentes
ex~eriores, la largura de Ja tangenw externa común es 6 'Jla cm. Hallen el perímetro do la figura formada por las tnngl•ntes externas y los arcos extornos de
lns circuníerencins.
121. Oosde un punto externo se hnn trnr.ado a In circunferencia unn sccnntc
2
de 48 cm de largura y una wngcnle, cuyn lt1rgura constlluyo T Jel segmento
interno. de·. Ja secante. Hallen ol radio de In circunfe1·cncia si snbemos que In
secante se encuentra a una distancia de 24 cm del centro.
122·.- La tangente externa com~n de· <los ciréunlctencins que son tangentes
exteriores formn con la línea de: los centros ol ón·g4 lo a. Hnllen 111 rnzón t.lo los
r ádios .
.
J23. Del punto _A, situado fuera rle un circulo con centr o en O, se han trozado ras ·s ecantes A BC y A M K. (8 y M son los puntos de la circunfercnci3 más
cercanos a A que yacen en la.s secan les). Hallen BC si sabemos que A C
a,
LCAO = <X, LCOK = p y que la secanLe A M K pasa por el centro de la cir·
cuníerencia.
124. Dos circunferencias so intcrsccnn en los puntos A y B, Por ol punto A
se han trazado Jos segmentos A C y AD, cadn uno de los cuales, siend o cuerda de
una circunfcrenola, es tangente a la segunda circunferencia. Demuestren que
AC"·BD = AD'·DC.
125. AB y CD son cuerdas intersecantes, perpendiculares entre sí de las
circunferencias de radio R. Demuestren que A e•
BlJ • = 4R1 •
t 26. Demuestren que la suma de los cuadrados de fas distancias dese!<) el
punto M ,,. tomado en el diámetro de una cirouníerencin, hasta los eictrcmos de
cualcsquicrn de las cuerdas 11arnlelas ni diúmet.ro es una mngnilutl constante
para la circunferencia dada.
127. Dos circunferencias son langcn\.es exteriores en el punto C, A B es
su tangente ex &.erna cumún. Hallen los radíos sí AC = 8 cm, BC = G cm.
128. Las circunferencias de radios R y ~ S<Jn tangcnr.es exteriores. Desde
el centro de In circunferencia menor se ha trazado, bajo un ángulo de 30° con
relación n la linea de los centros, un segmento de lnrgurn 2R. Hallen las largucas do las pnrtos-del segmenlo quo yacen fuera de la circunferencia.
129. Las circunferencias de radios a y b son ~angentes interiores (a < b),
con la particularidad de que el centro de la circunferencia mayor ynco fuera de
la menor. La cuort.111 AB de la circunferencia mayor es tangente a la menor y lorma con la tangente común a las circunferencias el ángulo ex. H!>-llcn AB.
=
+
44
Capitulo l. Pln11im<t~Ea
11. Triángulos inscritos y circunscritos
13(1, En el lri:ingulo regular /JIJC , en lns lacios AB y A C, sa hnn tomn•lv
los punt(ls 1\/ y K de manera q11c AM: MB = 2: l, AK: KC = i : 2. Dcmucst.rco que el segmento K Al es igual al radio ele la circunferencia circunscrita
al triángulo ABC.
131. Al triángu lo ABC (AB =se¡ está circunscrita una circunferencia.
Las continuaciones de las bisectrices de os ángulos A y C intersecnn la circunlerencia en los puntos K y P y t•ntl'C s i en el punto E. Demuestren que el cuadrilátero B KEP es un rombo.
132. AD y CE son las bisectrices del triángulo ABC. La circ11nfcrcncin
circunscrita al triángulo BD E pasa por el centro de la circunferencia Inscrita en
el triángulo A BC. l)('mnstrur <11<0 LA lJC = GOº .
133. Demuestren que el c~ntro ele la circunferencia inscrita en un triángulo
yace dentro do un triangulo Iormndo por las bases med lns del triángulo
dado.
134. La recia les l 11 ngente en d punto Ca la circunfcrP-nda circunscrita al
t.riángulo ABl'. Demuestren quo d cuadrado de la altul'll Cll del irióngulo A BC
es igua 1 al producto lle las d isla nc ias dcsdi! Jn 1·ccla 1 u Jos pun~osA y 13.
135. Hallen los úngul <>s del lti;í11g11Jo si snhc11101< que lo~ ci!nlros do lns
circuufcrcmcins Inscrita y d1·c11nscriln a .íl son simétricos con rclucíón a uno
de los lados del triángulo .
136. Ln hase de un 1.riáng11lo isósceles es 2a. y b altu1·a, /l./\ In circunferencia inscl'Ítn en el lri1í11g11lo s e hn lrarndo 01111 tnn¡:cnle paralcfa a la bnsc.
Hallen la largura del segmento d u esta langcnl.c que yace cnti·o los Indos late rales d~l triángulo.
137. En un tri;í ngulu rect{lngulo <>I punto de tnngtmdn cl o In circunferencia
insc rita dividc:> In hipoll•nnsn "'' S~!?lllcnlos de V. y 36 cm. H nllm los catetos.
t:i8. En un triángulo rectá ngulo 11110 rlc los r.at(ltos l'S i¡n1al a 48 cm y la
proyección dt>I otro en la hipot.,n usa, a ;i,1)2 cm. Halkn la longitud ele la cirCllOÍPrcnci11 inscriln.
l:l!l. En un ll'i:in¡tuln r~rhiiu.:ulo ''ºll <" a lelos clt• Hl y 24 cm hallen la distancia cnt.1·0 los ccnl.1·os de las cil'cunforencias insc rita y cirr.unscl'ita.
140. En un triángulo isósceles la nllum, trnznda a In base, os 1 ,5 veces
menor que el radio de la cÍl'Cllnforencia circunscrita. l!n llen el ángulo en la
ha se.
Ml. Hallen el radio de una circunferencia circunscrita oJ triángulo con
lados a y b y el ángulo y cntn• ellos.
142. En un ll'ilÍngulo isósceles In ba so es igual n b, (') ángulo on ella cz .
A la circunforcncia inscrita en el t riángulo so ha trazado 11na laugcnte paralela
a la bnsc. H nUen In largura de ést;i s ituacla cntru los Indos lnte1·nles del triángulo.
143, En un triángulo isósceles la razón do los radios do 'las circun ferencias
inscriln y circunsctila es iglial a k. Hallen los ángulos dd triángulo.
1H. Demuestren quo paro, todo triángulo roct{ingulo es válida la desigualdad Q,4< ~ < 0,5, dondo r os t>I radio de ln circ11nfcrnncin inscrita; Ir,
la altura baja.da a In hipotel)lisn,
14&. Demuestren que la circunferencia circunscrita a u.n triángulo es igual
a la circunferencia que pnsa por dos de sus vérl-íccs y el ortoccnlro.
146. En unn circunferencia c~tó insc~ito el triángulo r egular ABC. En el
~reo B C se hn· tom9do a 1 nz;ir el pu11 lo Jlf y trazado las cuerdas A M, B M, CM .
.Demuestren que AM = .BM+ Ck{.
l 47. D'e mucstrén que la suma de Jos cuadrados de l as distancias desde un
punto arbitrario de )11 circunfcrencln hasta los vértices de un trióogulo regular
inscrito en ella, es una magnitud conslanle que no depende de Ja posición del
punto en la circunferencia.
45
§ 3. Ctrcun/trcncia
1q8, En una circunferencia est6 lnecriio el triángulo is6scoles ABC (AB =
= BC). En el arco AB se Loina a l azar el punLo K y se une con cuerdas con Jos
vér Lices del trián!fulo. Demuestren que A K · KC = AB' - KB'.
149 . E n un t r u\ ngulo ·oc~tángulo con ladó.s 11 , b y e, desde el centro de l!l
circunferencia circunscrit a, so bajan perpco<!icularcs a los lodos. Las longitudes
de estas perpendiculares son lguolcs a m, n y p, respectivamente. Demuestren
mnp
q uo.l!.+2:..+.L=
a
b
e
abe '
150. Demuestron que las 003cs de la.~ pcr pcnd iculares, bajad!\$ a los lados
do un triángulo o bien a.la continuaci6n de ellos, desde 11n punto arbitrarlo de Ja
clrcunf<!rencia cireunscríta al triángulo yacen en una misma recta.
151. Demuestren que si a y b son los l ados de un triángulo , ~. In bisectriz
dol ángulo entre ellos, a•· y b', los sogmenlos en Jos quo la bisect·riz divide el·
tercor lado, ¡s = ab - a'b'.
152. De.muestren que el radio do la circunforcncl11 circunscrit.n n un tri·
ángulo, trozado a 1100 de los vérllcos do éste, es perpendicular a la rcctn que une
la base de !ns alturas trauidas de los otros dos vértices del triángulo.
153. Al triángulo ABC se ha circunscrito una clrcunforeocia. Por el punto O se b:i lraznclo unn tnngonlo n In ci rcnníorcnci:\ hnstn In in lorsccción con In
contlm1a~i6n del Indo CA trns el punto A en el ¡rnnt.u l). Hallen el pl! TI molrlJ del
triíirigulo ABC si AB +AD""" AC, CD= 3 cm, LBAC = 60°.
154. En ·uno c'iruunforencin 'do racl lo n csLá i11sc1·l lo e l triángulo 1·cgulor
ABC. Ln cuerda BD in terseca A C en el pu nto E do for rnn que A F:: : C/J
2 : 3.
Hallen CD.
''
155. En el trapecio ABCD la bisectriz del ángulo A lnlersecn la base BC
(o bien su conlinuaci6n) en_.e l.punto E. En el triángulo ABE está inscrlt.n uon
circunJerenein tangente al lado AB en el punto M y cl l11<lo BE en el punto P.
Hallen el ángulo BAD si sabemos que AB ·: MP =- 2.
156. La -hipotenusa de un triáo¡rulo rectángulo, se divide con el punto de
tnngencin de la clrcun[erencio ins('.rlta en segmentos, cuyn rnión ~s lgun 1 n k
{k > 1.). Hallen los ángulos del triángulo .
157. Hallon el ángulo en In base de un triángulo isósceles si S11hcmos que
su ortocenlro yace en la ci rcunferencia inscrita.
=
ru.
Disposición arbitrnrla de la clrcun!erencla
y el
l tiángulo
158. t..os sogmenlos AD, 8111 y C/> son las medianas del triángulo ABC.
La circunforencia circunscr ita a l triángulo DMC pnsn por el ccnt roi1lo do! Lri.ángulo ABC. Demuestren que LABJ\l - 41'CB y LBAD = Ll'CA.
159. En un triángulo rcctánt;ulo está inscrita una scmicirc11nforencia de
forma que su diámetro yace cu In hipotenusa, mientras que el centro divide
a ésl.3 en segmentos de 15 y 20 cm. Hallen el radio de la seinicircunfcrencia.
160. Una circunferencia pasa por el vértice ,1 del t riñngulo r.:etóngulo A BC,
es lnngente al Indo OC y tiene su centro en In hipntcnus..' AD. Hallen su radio
al A IJ ~ e, OC = a.
161. E n el calcio BC del triángulo rrn;tángulo ABC. c.111110 en su dhímotro,
s~ hn construido unn circunfo1·cndn 1¡11c lnLr.rsecn 0111•1 p1111Lo V In hipot enusa AB
do forma q,ue AD : DIJ = 3 : l. ll n len los latlos dol ll'iángu lo A BC si 111 nlLurn.
tratada a Ja hipotenusa es i~unl n 3 cm.
162. Los lados de un lr1ángulo son igunlcs n a )' I>, el i\nguln cnlr<? dios,
11120". Hallen el radio de la circunícreucia que pasa por dos vértices del Lcrcer
lado y el centro de la ci rcunfcrcneln Inscrito l'n el trlóngulo 1iado.
163. Unn clrcunlcrcncia pasn ror los vértices A y B tlcl- triángulo A BC
y es l11ngcnle al lado OC en el punto O. El lallv A C se 1livido con la circualcrcn·
cia en las partes AM y MC do for111:1 que AM
/llC
IJC. Hallen OC si
AC ~ 4 cm.
=
+
46
Capitula f. Plan lnittria
164. En el l;ulo AB del triángulo ABC. oomo en su diámetro, ao hn c;onll·
truido una circunlcrencin que inLerseca el lado BC en el punto D. Hallen A C
si sabemos que CD = 2 cm y AB = BC - Cl r111 .
165. En el lado AB del triángulo A 8C, r111 r.1 ° o•n su diámetro, se ha construido una circunlcrcncin que intcrscca A C cu c·f 1111uto D y BC en el punto E.
Hallen AC y BC si &.1 hemos que AB
3 cm, A IJ : DC = ·I : 1 y BE: EC
- 7: 2.
166. El segmento DD es la alturu del triángulo ABC, y DT:. la 1nndinna
del triángulo BCD. En ol triángulo DD8 está inscrita unn circunferencia tangente al lado BE en punto K y a l lado DE en el pun to ilf. Ha llen los ángulos del
ltiángulo ABC si AD - BC ~ 8 cm, KM= 2 cm.
I G7. En el triángulo ABC se bon tra~ado la altura .AD y uno Gircunforcncia con el centro en el punto A y de radio AD . Hallen La largura del arco de
dicha circunfer encia sit uado en el interior del triángulo si BC = a, LB = ~.
L.C ~ '\'·
168. Demuestren que. el rodio de la circu nícrcnc ia, tangente a la hipote1111sn y a la continuación de los catetos de un triángulo rectángul o, e5 igunl a In
sumn de las longi tudes do lo hipotenusa y del radio de In cit-cunforcncia insc1·itn
on el trláng11lo .
169. Las bise>clrJccs AD y CK tlol Lriiíngulo AIJC su cortan l'n 1·\ punto O,
IW = l cm. Ha ll en los ángu los y los dos tM<>s lados del triií ngu lo !\DO si
es conocido quo el punlo B yace Nl In circunlcrcncia ci rcunscrit-0 ni triángulo
=
=
A'DO.
170. Una circunlercnci3 cs lan.i:ent t> a los lados A C y OC clcl triángulo A OC
y tiene su centro en AB. Hallen el rntlio de Ja circunferencia si AC = 48 cm,
Bl" = 140 t m, AR = 148 cm.
171. En el triángulo ABC el J>Unto m¡•dio tic ,t C es D , el 110 BC, F., la
ci rcun ferencia circunscrita a l tríánt!ulo CD& posa por el ccntroido dol triángulo AJJC. Hall en la largura de Ju mediana C /\si AIJ
c.
172. Hnllen In dependencia entre los Indos ce, ú y e del l.rlfingulo ABC
si se conoce quo el vértico C, el ccntroide M y los punlos medios do los l a1lo~ A C
y BC yacen en una misma recta.
l i3. En el lrhlngulo is6scclesABC con úngulo B igual a 120°, cslá inscriln
una semicirouofert'neln clo radio (3 l/3 + fl :tt) cm con centro en A C. A 111
semicirounfcrencia so trozo una l<ingenle que inlcrs!'Ca los lados lateral es AD
y BC en los puntosD y E, r<'spcctivamente. !follen BD y BE si DE - 2 117 cm.
174. En el triñngulo ABC son conoeidos los lodos: AB = BC = 39 cm.
AC = 30 cm. Se han t razado los alturas AD y BE. Hallen el rad io de la circunferencia q\le paS4 v.or los puntos D y E y tangen to n i lado BC.
175. En el truí ngu lo AIJC se han lrtizndo lns nlturas CD y AE. Al triángulo BDE se circunscribe) unn ci rcutifercncin. H¡illcn In longitud dol urco do
ésta que yace dentro del triángulo ABC si A C .,.. b, LABC = ~.
=
IV. Clrcunicrcncia y cuadrilátero
lí6. Demuestren quo al para un trapecio oxislen las cir.cunferencias inscrita y circunscrita. In allura_ del trap ecio es In media proporcional entro sus
bases,
111·. L.as ha scs do un trapecio isósceles son iguales a 2.1 y 9 cm, lo a l tura,
a 8 cm. Hallen ol r11dio de la circunfcrenci o c írci¡nscrlln a dicho trapecio.
178. Las bases de un irapecio isósceles son a y b, el ángulo agudo, ex. Hallen el radio de In circunlercncia circuoscriln n, dicho trapecio .
179. Dós vértices tic un cuadrado yo.ceo en la circunfercocin do rad io n
y las otras dos, en Jo tangente a dicha. circunlcrencia. Hallen el l ado del cuadrado.
§ 8. Circunferencia
47
f80. El ángulo ngudo A del rombo ABCD es igual 11 et. Hallen, }a razón
entre el radío de In circunfonmcía inscrita en ol rombo y el radio de la circunferencia inscJ'ít-0 en el trí&ng\Jlo A BC.
t81. Un trapecio está.-circunscrilo ·a ·una circunlcrcncia. Hallen los ángulos
<!el trapecio si conooemos que la rozón entre Slt lado lateral y su base •menor es
igual 11 k .
182. Un trapecio con ángulos agudos· et y ¡¡ está circunscdlo a \loa circunI~rencia. Hallen la rnz.ón entre el •perímetro del trapecio y .Jn longitud· de la circ unierenc ia.
,.
183. a) · Demuestren el .teorema de Ptólomeo: si los !ados opuestos do un
cuadrilátero inscrito en una eircunforencia son iguales a, b, c.. y (ll y las dingo·
nales, d 1 y d.,, ab
cm
d 1d 2 : b) empicando el· teorema de. Ptolomeo dcmost-rar
la afirmación del problema 1.46.
. 184 • .Demuestren· que .la suma do los proquc;to$ .de l.as altura·s .,de un .trián·
gul:o otu~ángulo por sus· segmentos desde el ortoccntro hast.n los vértic~s es
·igual .a ·1á. semisuma de Jos cuadrados ·de los lados .
.185. En· la hipotenusa de ün triángulo rectángulo, como sobre un lado, se
ha· construido un cuadrado (füera del triángu lo) . El centro del cuadrado está
unido con.el v~rtlce' del_j ngulo recto dol triártg1tlo. ¿En quó segmentos se divide
In hipotenu1111 si los cnteto~ son iguales a 21 y 28 ·c m?
1.8Q.· U.o.a circunferenc ia e~ tangente n.dos.Indos adyacentes de un cuadrado
y divic.lo ca_tl!\ uno de los ·oLros <lo.~ en .segmentos de 2· y 23 cm. Hallen el radio de
la circunferencia.
· .
. -187. .En el ~m:n& ABCD. co.n l.ado ele I¡ crri y ángµlo BAD igual a 60°,
1¡sta inscrita u11a circunf11ren.cia. A ella· se ha t razn do tiria .t angen Le que corta A B
en el punto P y AP en el p1mto Q. Hilllcn PB y QD si PQ ""' 2 cm.
. l l¡8. [.,a razóri eñ tre el radio. de la ci rcunferenc ia, circunscrita a un trape·
cio, y el 1·adio de la circunferencia inscrita es igual a le. }folien el ángulo ng11do
del \ra pecio.
·189. En una cireunforencin está inscrito o! cuadrilátero ABCD, cuyns dingonalcs son perpendiculares entrr. si y concurren en el punto E. 1,,n rccln que
paso por E y es perpendicular n AB , int.crseca CD en el punto M . Hallen /:.'M
si AD= 8 cm, AB ~ 4 cm y LCDB =a..
190. En unn circunferencia está inscrito el cuadrilátero ABCD, cuyas
diagonales.son ferpendiculares cnlre sí y se cortan en el punto E. Ln rect.a que
pasa por E y e. punto medio del lado CD interscc.a A B en el p11nlo JI. Hallen
1f 8 si ED = 6 cm, BE
5 cm y LA DB = (l .
+
=
=
191. En el cuadrilátero con\lclto ABCD el lad o AB es igual n ~;
a i2
~;
CD, n 6
+.
BC,
+,
Snbemos qua <!l ángulo DAn es agmlo, ADC , obtuso,
con la pnrticularidad de que sen L DAB=
cos L ABC= -~. La circunferencia con centro 'en el punlo O es tangente a los lados BC, CD y AD.
Hallen Ja largttrn e.le! segmento OC.
V. Diferentes problemas
192. Desde el punto C se han trazado a una circunferencia dos t.angentes CA
y CD que entre si forman un ángulo lle 00°. En el triángulo curvilinco, formado
con estas tangentes y ul are<) monor AH, está inscrita una cireunferencia . Demuestren quo fa lóngitµd d<' ese arco es igual :1 la de la circunforc.ncla Inscrita.
193. Un rectángulo con lados de 30 y 48 cm s e divide por. la diagonftl en
dos triángulos. En cada uno de ellos ~stá inscrita una circunferencia. Hallen
las distancias entre sus centros.
Capil"lo 1. Pla11 imetr fo
1!14. Dos circunforcncins de rndios IG y 9~m son Langeolus cxlcrioros. Cal·
culeo el radio de la circunferencia inser iln eil. el t riángulo curvilíneo situado
<mire Jos circunferencills y su lnngcnle cxltrua común.
195. Ln cuor<ln de G cm dt' lnrgurn divide una circunferencia on dos &L'g·
mentas. En el menor de eHos está inscrito un cu:ul ratlo cuyo lado es igua l o 2 cm .
Hallen t>l rad io de In circunforcncin.
l!JG. Dos círculos de rndio R están sítuados do tal forma que la dist.,.ncio
entre sus CMlros es igual n R. En 13 Intersección de Jos círculos est-á inscrito
un c11ndrado. Hallen el lado <l cl cuadrado.
1!17. En el sector circular, cuyo ún¡¡uJo es igual a 2a, está inscritn una clrcunlcrr.ncia. H ali en la rnzón ontro los radios !lo la circunferencia inscrita y el
sector .
l!JS. En el sector AOB de un circulo 1lc rndío ll con ángulo central a está
lnscrito un lriángulo regular. Uno do los vértices d~ ·éslo yace en el punlo 1nodio
d el arco A 8 y los olros dos, en los radios OA y OD . Hallen .el lado dol trióngulo.
1!10. i:il arco do una circunforoncia do rndio JI ustá comprendido en el ángui o conti·nl
2a.( a. <
~· ) . Ln cuerda de esto nrco dh·ide la circunlcrencia en dos
"'Cl!mr.nlos. En el mcnPr do ellos ost~ ini;crito un c1rnd rado. Hallen el lnllo del
cu:ul rado .
200. El arco de una clrcnnforcncin do radio 11. está comprendido cu el áu¡:ulo
ccn1ral
2a (a <
~ ) . La cu~rdn
do dicho arco divide In circunforencla en dos
se¡¡nterllos. En el meno r de ellos está Inscrito un trifogulo regular ele modo quo
uno de sus vértices coincide con el punto m~odio dol arco y los otros dos, yacen
en lo cuerda del segmento. Hallen el Indo d11l trián!?lllo.
está comprendid o cu el
201. El nrco de una circnufort>ncin do rn1lio
n
Anguloccntrnl
2.a (a < ~ ) . La cuo1·dn do esto ureo divido la cl rcun íoroncin en
dos so¡¡mcnlos. En el rnnyor do ellos es tá iuscrilo un lriángulo regular do formo
que uno de sus vérlices coincide con el punto medio de 111 cuerda y los otros dos.
yRcc11 en el arco. H.alleri el lulo d••l trhinKulo.
202. En un triángulo is6scclcs está inscl'itn unR circunferencia de radí o a.
Otra circunferencia de radio b es langcnlc a los lados laterales del ttlúngulu
y a JA circunferencia .inscrita. Hallen In base del triángulo.
203. En el segmento A de 12 CID do longilud so ha to ruado el pun lo
do forma quo AB = 1, cm. En A C y AD, como en el diáme~ro, so han lrnkdo
circunferencias. Hallen el radio do la circunferencia tangente a las dos dadns
l ' al segmento AC.
204. La base de un Lriñngul-> lsú!cel cs es !t, el ángulo en la base, a. En el
triángulo está inscriUi una circunfrrcncia. La se¡unda circunferencia es t11ngcnte a la primera y a los lad os laterales del lriangulo. Hallen ol radio de la
segunda c1rcunfcrcnci:i.
205. E 11 una circunferencia de rodio R con centro en O, se hall lra1.o.do
e
Jos radios OA Y. OB de Jonna; que L A0.0-a. ( ; .. <a<
n
1t).
Halloo el rr1dio de la clrcuníoroncia, t~n¡¡911to al u·rco A 8 del sector OA 8, n In
cuerda Ali y a In .bisectriz del ángu lo AOB.
ZOG. Dos círculos igual~s do rudio a' QSt;ín situados do manera que la di slancia entre sus centros es igual o. a. La inlcr~cción de los círcµ los está dividld11 11or lu linea de ·Jos. centros on dos, lriángulos curvilíneos en uno do los cuales cst.8 inscrita una circunfcrencin. Hallen la longitud del segmonlo que uoc
los puntos de ~angcncia de In circuníorcncia Inscrita eón las dos circun!crenclos
dadas.
207. Del punlo A n la circunferencia coo el centro en O y radio d11 2 cm,
se h trazado lo tangente A K. El scgmooto OA intcrscca la circunferencia en
§ 4. Areas de los figura~ planas
el punlo M y forma con la tangente un ángulo de. GOº. Hallen el radio 4c la circunferencia inscrita en el triángulo curvilineo llf KA.
208. Del punto A, alejado del centro. O de una circunferencia de radio r
a una distancia a (a> r), se tra~a un rayo que formn un .áng11lo dn 60º con el
rayo AO y que int.crscca 111 cil'.l:unferencla en dos puntos K y .p "(K yácG entre
A y P). Hallen el radio do In circunferencia inscrita en e.l triángulo curvilíneo
M KA, dondo M es el punto do· intersección de Ja circunforencia con el seg·
mento AQ.
·
209 .. La ·ba.s e do un triángulo isósceles es igual a .b , el ángulo en Ja base, a ex.
En el triángulo e11tá inscrita un·a circunfer~ncia. Otra circunfer encia es tangente
a Ja ptii;llera, a la base y al lado lateral del triángulo. Ha.!len el rad.io ele la segunda circunferencia.
.
210. Una circ.u nferencia nstá circunscrita al triángúlo iiJ&celes de base b
y ángulo en ella ix. La segundn ·Circunferencia es tangente a la primera: 'Y a los
lados lat:eral"es dol triángulo. Hallen el rl!dio de la segunda circunferencia .
211. En el segmento de una circunferencia de radio R y ángulo centra 1
a(ex < n) están inscritas dos circunferencias iguales, tangentes entre sí. Hallen
sus radjos.
21-2. Los puntos D, K y M yacen en los lados AB, RC y AC, respecti\'n•
roen U!, ilel triángulo ABC . Demuestren que lns circunferencias circunscritas
a los triángulos ADM, Bl)K y CKM concurren en un punto.
213, .Dé.s de el punf.9. C se han ·trazado dos ·tangentes A C y BC n la circun·
feroncia de rndlo t~ cm .c on ·~ntt'o en el punto O. En el triángulo ABC está ins·
crita una circunferencia con c41ntro en 0 1 ·tangente n los lados A C y BC en los
puntos K y H. Hallen el LAOB si In distancia desde el punto 0 1 hasta lo recta
KH es igual a ~ cm.
21'\. Desde el centro O de una circunferencia d.e radio U se bon trazado los
radio.s OA y OB de modo qua LAOB =a (ex< n). En el segmento menor del
círculo, cortado por la cuerda AB, está inscrito un triángulo regular. Uno de
los lados do éste es perpendicular a la cuerda A lJ. Hallen el lado del triángulo.
215. En una circunferencia de radio r so ban tra?.ado el diámetro AB y la
cuerda AC. En el triángulo curvilíneo lormad<i estA inscrita una circunferencia.
Hallen su radio si LCAB = ex.
?16. En una circunferencia con el centrn en O, el radio OM y In cuerda K/'
se .cor tan en el punto A, con Ja particularidad tic que LM A K - cr. { cr.< ~) .
E11 el triángulo curvllineo formado MA K. está inscrita una circunferencia. Jfa.
!len su .ro.dio. si OM = r, C>A ""< a ._
217. Desde el punto A do .una olrcun!crcncia de radio r, se han trazado el
diámetro AD y dos ·cuerdas AB y AD. Hallen el radio do la circunforencin tangente ir!as cuerdas Ab y AC y ·c1 arco BC si AB - b, LBAC
cr. y AB > AC.
218. En un lado de! ángulo a se dan dos puntos, desde los cuales la dis·
tnncia hasta el otro lado del ángulo es b y e (b < e). Hallen el T&dio de In ci r·
cunferencia que pasa por estos dos puntos y quo c.s tangente al otro lad o del
ángulo.
219. El ángulo A.OB es igual a ex . La circunferencia 11~ t nngcnlc al lado AO
en el punto C y corta el 111do OB en los puntos D y E. Hallen DE y ul radio de
la circunferencia si sabemos que OC = a. y OD ~ b (b > o) .
=
§ 4. ÁREAS DE LAS FIGURAS PLANAS
El punlo H es el orlocenL1·0 del triángulo ABC. En
la recta CH so ha tomado un punto K tal que ABK es un triángulo
rectángulo. Demuestren que el áre.a del !:::,.ABK es la medin proporciona l entre las áreas de los triángulos ABC y A/Jfl (Cig. 55).
EJEMPLO 1.
4- 0290
50
Capitulo 1. P/anlmetrla
soLUCION. Introduzcamos las anotaclones: SAsx=S, SAnc=Sp
i
<
1
1
SABll =S2 • Entonces, S=2 AB· KD, S 1 = 2 AB·CD, S2 = 2 AB x
x H D. Hay que demosu:in· que
S=Y~.
(1)
es decir, que +AB.KD=y ~ AB·CD·f-AB·lfD o bien, que
KDZ= CD.HD.
(2)
Pero el triángulo ABK es rectángulo y , por eHo, KD2 =BD·AD
(teorema 6a). Así, pues, la igualdad (2) será establecida si demosB
e
8
D
Fig . 55
Fíg. 56
tramos que BD·AD=CD·DH o bien que ~Z = ~~. La últimn
igualdad se desprendo con soficientc evidencia de 1a semejanza de
los triángulos rectángulos BCD y HDA (en éstos, LBCD y LHAD
son iguales como ángulos con lados perpendiculares entre sí, yn que
AE es la altura del triángulo). Esto significa que Ja lg11aldad (2), así como también
la (1), quedan demostradas.
F.JEMPLO 2. Conociendo .las medianas m .. ,
mb y m 0 de un triángulo Cl)lculemos su área.
soLUCI01'. Ante todo, ·s eñalemos que
S AMC =
T
'S AJJC (fig. 56). En efocto, la
base· A C de estos triángu los es común y,
por c.onsiguiente, la razón de sus áreas .será
co·mo Ja de las alturas i'vJK y BH (teorema 18). Pero de la scinejanza de los triángulos M KE y BHE llegamos
a la conclusión de quo ·'~~
ME : BE = 1 : 3 (teorema 3b).
Asf, pues, el área buscada S es igual a 3S AMC•
Examinemos el triángulo AMC (fig. 57). En él conocemos dos
Fig. 57
=;:y
lados: AM = ; m 0 , MC = ; m 0 y ln mediana ME=
!
mb
(de nuevo
§
4c. ÁreM de
ÚH.
51·
figu.ra.J p/aM,•
hacemos uso del teorema 3b) . Duplicamos la medinna hasta convertir ·e l triángulo en el pal·al 0 10gramo MCP Á, después de lo cual
obtenemos: S AMC = S AicP = ; S AMCP· En el triángulo MCP son
.
- ~
2
2
2
eónocidos tres lados: 3 m"' 3 rnb• ¡¡-me, es decir, el átca del
L_riángulo MCP puede hallarse cou ayuda de In fórmula de Herón
(teorema
19e).
De
modo que
S = 3SAMC = 3$Mci> =
V T(mci+mb+me)·+ (m,.+m -m,)-f (m.. +m -mi;)X
V +_(m +m -m.,)=+ 'V(m +m11 +m (1~a+m~-m0) X
=3
X
11
0
X Y(m"
0
0)
0
0
+ m -m,,) (1nb+mc-ni
0
0 ).
EJÉJ\rPLO 3. H11llomos el área de un triángulo con ángulos a:, ~. y,
snbicndo que las di~lnnc.ii1s 1lcsclc el punto arbitrnrio 11-f, t.<mrntlo
dentro tlcl tri{rngulo, lwsLa s 11 s lados son iguales i1 m, n y k, rcs¡¡ecLivnmcnte, (Hg. 58) .
so1,uc10N. EL área S del trifogulo ABC es ¡¡osible tic hallar con
.::+
la fórmulaS
AC·BC·scn1t, pero, para ello, hay que hallar
AC y BC. Hagamos BC = x. Entonces, tic acuerdo con el teorema
AC
BC
AB
de los senos (teorema 8) -.. = - = - - , de don de hallamos:
sen ,.
sen et.
sen I'
AC
=
xsenll
sen a'
AB = xscny
sen a ·
Así, pues, el problema se reduce a la búsqnotla de x. Parn confeccionar las e.c uaéiones empleamos el método de las áreas (véase l,J
§. 1): eleg~mos como elemento de referencia el área S del triángulo
ABC.
· Por. un lado, tenemos:
S
1 AC BC
1 .. s.cnfl
z•scnlJseuy
= 2 .- ~ sen y = 2 · scnc;:- x sen V=
•
2 son et.
t
1
Por otro la d o, S;=SANu+SDuc+SAuc = 2 AB·k+-z BCx
L
t AC
1
"'sen y k , 1
1
sen jl
x n-.- 2
·m =-z·~·
,
2
xn
+ -z·~·ni=
x (k sen y+ n sen a.+ m sen 11)
2scn a.
Así
ucs i•scnllscny = z(kseny+nscna+msenl!l, ele donde
' P
'
2 sen a.
i sen a
¡ U
kscn v+n i;cn a.+m sen ll
sen ll sen Y
Poniendo est.c valor de x en la primera de las fórmulas señaladas
más arriba para el área del triángulo ABC, obtenemos:
z• sen 11 sen y _ (kscn v+n sen a+mscn f\) 1
2 sen a
2 sen a sen 11 sen y
ia amos: x =
s
".
52
C11.pftalo l . PlanLmctri11.
EJEMPLO '· E n el t riángulo A BC, en los lados AB y BC, se han
tomado los puntos K y P de forma que AK : BK = 1 : 2, CP : PB =
= 2: 1. Las rectas AP y CK se cortan en el punt o E . H allemos el
área del t riángulo ABC si sabemos que el área del triángulo BEC es
igual a 4 cm 2 (fig. 59).
B
B
e
A
A
~
Pig. 59
Flg. SS
SOLUCl ON.
cemos PM
C
Hagamos AK = x, BK = 2x, BP =y, CP = 2.y y tra,
BM
BP
t
Segun el teorema de Fales MK = PC = T. De
11 KC.
2.:e
4.r
mo do que BM =a• KM=3
A continuación, los triángulos -AKE y AMP son sP.mejantes,
KE
AK
.
KE
:t
3
.
por lo que MP = AM, es decir, MP = - -4
T y, por coni;1-
-.:-e =
z+3
.
3
.
gu1ente, _K E=·7MP . Por otro lado,
=
MP
BP
t
w
= BC
=a·
oseaMP=
f KC. Como resu l tado, obtenemos que KE=f KCy, por esta razón,
EC = ~ KC.
Analicemos .los triángulos BEC y BKC. En e llos, la aHura
t.razada desde el vért ice 8 es común, lo que significa que sus áreas
se r elaciOnl\Il comQ sus bases KC y EC (teor.ema 18), es decir,
S sr<c = KC
7
p ero S nsc= 4 cm· 2 , por cons1gu1en
·
•
•
t e, S sJCc=
SBEC
E,:c =lf··
·7
14
.
=5·.4=3cm2.
Por fin. examinemos los triángulos BKC y ABC. En ellos, la
altura trazada desde el vértice C es .com\¡n: y sus áreas se rela•
SABC
AB
3z
3
c1onan como las bases: SBr<c =7fl[=z;=2
. 9mo resulta do, o.btenemos: _.ase= 2 sxc = T3 · T14 = 7 cm2 .
C
3
s
s
JOJEMPW b, En el triángulo ABC (AB = BC) el ángulo A es igual
a arctg ~· La circunferenci a de radio igúal a 1 cm es tangente a los
_
§ 1, Áreas d• la• ftgurtu pl!111a$
lados AB y BC e interseca la base AC en los puntos E y K (E yace
entre A y K). JW: es ol punto de tangencia de -la circunferencia y la
recta BA, AM =~cm. Calculemos el área de l triángulo AMK.
soLUCION. Ante todo, hay que hacer cálculos con el fin de aélarac
dói;ide yace el centro de la circunferencia (por ahora, sólo est:á claro
quo 'él se. encuentra en. la altura BH del triá.ngulo isósceles A.BC., ya
qu·e BA y BC son tangentes a la circuruerencia y, por lo tanto, el
!)entro de ésta yace en la bisectriz del ángulo entre las ·rectas) (teorema i2b).
8
8
.A
,1 E
Flg. 60
o
¡.;e
Flg. 61
'In~roduzcamos la anotación: LBAC =a. (Hg. 60). Tracemos el
radio OM al punto de taP'gencia, entonces el ángulo BOM también es igual a et. De acuerdo con el planteamiento tg a.= 185 .
. -
1
15
Empleando la fórmula 1 + tgz et = cos• a. , hallamos: cos et= 17 ,
entonces sen a= tg a cos ex= ~
1
•
Del triángulo BOM hallamos: BO = cos
OMu --..!...--E.
15 - i5 , B/11 =
8
•
.
1f
15
=OM tgct= 15 . Seguidamente, ilB = AM +BM = - 82811
BH AB
289 8
n
=
120 '
8
+-:¡s
=
sena=120·17 = 15·
=
Esto significa que BH = BO y, por esta razón, los punLos O y l!
coinciden y para la posterior resolución del problema hay que hacer
un.a nuevo. figura (correcta) (Iig. 61).
El área del triángulo AMK ha de buscarse con la fó.rmula
i
..
15
8
S = 2 AM ·A K - sen ex. Sabemos que AM = 8 cn1, sen ex=
15 . De
este modo, el problema ' se reduce a buscar el segmento AK.
Hagamosuso de que AM2 =AE·AK (teorema 16c). Hagamos
2
AE = x, entonces AK = 2 :r: y obtenemos la ecuación
~ =
+
~x
(2
+ x),
'
9
de donde x = 8.
Capítulo J . Planimetría
54
En tal
CllSO,
AK= ~ +2= ~cm y, por consiguiente,
1
15
i
=2
25
8
sl'MK=
375
AM-AK-scna= y· s·-s--¡7=272cmi.
EJBNPLO G. Por el punto medio de la di agonal BD del cuadrilátero ABCD se ha traiado una recta paralela a la diagon11l AC. Esln
recta interse<:n e.\ lado AD en c.l '{IUnto E. Demostremos q\ll' la n-..:-t:'
CE divide el cuadrilátero AlJCV
e
en dos partes equivalentes (de
á roas igun les) (fig. 62).
soL'UcION. Hay que demostrnr
que el ál'ea del cuadrilátero
ABCE es igual a la mitad de l
área del cuadrilát ero ABCD, lo
que signHicará que las áreas de
las figuras ABCE y CED son
"~--"-----------:..D
iguales, es decir, que ellas son
equivalentes.
Fig. 62
Hemos do indicar que el
cuadrilátero ABCE es equivalente
a 1 cuadrilátero ABCM, donde Mes cualquier punto en In recta EP:
en ofecl-0, en los triángulos ACE y ACM la base es común y las
alturas iguales, ya que los puntos E y M yacen en una misma rectn
paralela a la base AC. Esta observación nos sugiere sustituir el
cuadl'ilátcrei ABCE por el cuadri látero ABCK, equivalente a él ,
donde K es un punto en EP elegido do modo especial. Como K elija,
mos el punto medio de la diagonal BD (fig. 63).
1
Tenemos: S118 c 1! =
2 AC. BK. sen a, donde ci es el á ngulo entro
las diagonales (teorema 20b). Según el plao\,.eamienlo BK =
= ~ BD. Así, pues,
que es lo que queriamos domoslrnr.
EJEl\IP~o 1. E l, área del cuadrilátero convexo ABCD es igual a
2 ·cm~. Su!! lados han si do con Linu'oilos: el lado AB tras el punto /3
de forma que BL = } AB; el lado BC t ras el punto C de forma que
CP
= 21 BC;
.
1
el lado CD tras el punto D de forma quo DE = '2 CD;
el lado DA tras el punto A de forma que AM = { AD . Hallen el
área del cuadrilátero LPEM (fig. 64).
.~ 4. Arta& dt fa$ flgUcrat plana$
55
s0Luc10N. lnduicnmos las anotaciones: AB=a, BC= b, CD=
=e, DA= m. Examinemos el triángulo AML . En él AM = ~ m,
! ·+
AL,,. ~ a. E ntonces su área Si será igual a
m ~ a sen et,
clonde et = L MAL. Comparemos esta · expresión con el áren del
111.---....,__
e
/j
Flg. G4
Flt. 68
1
t
~
t riángulo ABD; SA 8 n=""f AB·AD.sen(180°-a)=T nm scuet.
3
Vemos que S 1 = T S AD U·
Por analogía, el úrea S3 del tri á ngulo CPE, está ligada con
la del tdángulo BCD mediante Ja relación S 3 = ~ Scsn· De modo
!
!
quoS, +Sa =
(SAs o+Sscv)=
SAaco = ~ ·2= 1,5cm2 • De esta
mi.~m11 manera, s i hacemos S 2 = SsLP• S 4= S.uozP obtenemos: S 2 +
+S4 = 1,5cm2 •
Como
resultado, s~fLPE = SABCD +Si+ s, + S3 + s, == 2 + 1,5 + 1,5 = 5 cm•.
EJtlitl'LO s. Una circunferencia con cen tro en O está circunscrita
a l triángulo AB.C con ángulo obtuso A. El radio AO forma cou Jn
altura AH un ángulo do 30º. La continuación de la bisectriz AF
inlorscca la circunferencia en el punto L y el radio AO intérscca BC
en el punto E (fig. 65). Calculen el ároa del cuadrilátero F EOL si
sabemos que AL = 4 y2 cm, Ali= V2 Yif cm.
SOLUCION. Si en el ejemplo 6 buscamos ol úrea del cuodrilátero
con la fórmula conocida 20b y en él 7, como Ja suma de los áreas de
las partes componentes, en el presento caso es conveniente consiclcrar
el cuadrilátero que nos interesa como la diferencia de los triiingulos
AOL y AFE. Es decir, S 1·EoL = SAoL - SAn· Por ello, lo posterior resolución del problema se reduce, en lo fundamental, al cá lculo de diversos elementos (lados, ángu los) de los t riángulos AOL y
AFE.
56
Capítulo l. Planimetría
Demostremos que OL 11 AH. Con este fin, señalemos que LCAL =
(según el planteamiento) y, por ello, uCL = uBL. Pero,
entonces, son también iguales las cuerdas CL y BL (fig. 66), o sea,
CBL es un triáng.u lo isósceles. Mas el centro O de la circunferencia,
rircunscrita al triángu lo isósceles CBL, yace en su altura KL. Es
evidente, que LK 11 AH y , por esta razón, OL 11 AH. En tal caso,
1
1
LHAF = LALO = LLAO = '2 LHAO = 2 -30° = 15°.
= LLAB
L.
/,
Fig. 66
Fig. 65
Hagamos un resumen pi-cvio. Sabemos que el triángulo Af.,O es
isósceles con ángu los 15º, 15º, 150º y lado AL, igual a 4 Y2 cm.
Esto es suficiente para calcular su á rea.
Según el teorema de los cosenos (teorema 7), tenemos: AL2 = .
= A0~+0LZ-2AO·OL-cos150°, de donde, haciendo A0 =0L =
= R,
=
obt enemos: (4 V Z)z = Rz + RZ
+ 2R
2
1l2li , R 2
=
32
,/ _ =
z+ V
3
32 c2-·V3).
A continuación, tenemos: SAo1.. = {-AO . OL -sen150• = +R 2 x
v-
1
1
X 2=4·32
(2- ,vf 3)=8(23)cmz.
Ahora, .calculemos el áren del triángulo AFE. Tenemos: HE =
= ·AHtg ·30·=Vz:V·3 .
~.3 , Jf.F=AH
tg
15°=V2y3 (2-:V3),
FE=llE~. HF = v2y3 ( ~ª - z+v3)= v .zva 2 ( 2 ·~-Xª>.
i ,f
SAu= 21 FE-AH=yV
2
=
,Y; 2 ,v¡-:-3= 2 (2- y
,f-3) cm.
z
Como resultailo, SFEo1..=SA01..- SAEF=8(2- Y3)-2 (2-V~) =
6 (2 .,- 'V3) cmz.
EJEMPLÓ
BC
v-3 2 (2)la
-)/3)
=
o. En el pentágono AB.CDE sabemos que AB =Y2.
= 30°. Calculemos el área de
CD, LA BE = 45º y LDBE
57
§ 4. Árt<U dt las figura• planas
la figura si a .ella es posible circunscribir una circunferencia con radi'~
de 1 cm (fig. 67).
s0Luc16N. Calculemos el área del pentágono como la suma de
las á reas de los triángulos ABE, BED y BCD. _De .acuerdo con el
teorema de los senos, aplicado al triángulo ABE, hallamos que
~ ... 2R, es decir, AE =V 2. Esto significa qué ABE es un
een 45º
Flg. 67
Flg. 68
triángulo isósceles reclángulo en el que AB = AE=)/2, por lo
1
que BE=2 y SAsP. -= 7Afl·AE=1.
Como BE ·-= 2, esta recta es el dfámcntto de la circunferenci:l..
Así, pues, el triáng11lo BDE es rectángulo; de él luulamos: DE =
,,t
113
=1. BD= V 3, SsD!l=z BD DE=-2-·
Para finalizar, consideramos el triángulo BCD. En él BD=
0
= V -3.
.
&egun el teorema de lo:s senos
3Cn
DD
L- Bco· =
2R, o sea,
V}.
sen LBCD=
De aquí hallamos qui! LBCD=120º y LCBD=LCDB=30°.
BC
Ya que sen 30• = 2R, hullamos que BC ~CD= 1 y
~
.)scD = z
Como
,
resultado,
v3
1
11a
BC -CD·sen L BCD =- :¡ .1.1. - 2- = -4- .
obtenemos:
s~
4+3lt'3
- =
= 1 + T113 + -113
4
4
EJEMrLo 10. Los centros de cuatro drculos están situados en los
vérlíces de u n cuadudo con lacio a; los radios son iguales a a. Calculen el área de la intersección de los cfrculos (fig. ü8).
soLuétON. Por consicJornciones de simetría sigue que el cuadrilátero EKM P es un cuadrndo. Esto significa, que la figura buscada es
la unión de un cuadrado y cuatro segmentos iguales. Para calcular
Capítulo J. Planimetría
58
el área de un segmento, ante todo, hay que hallar el correspondiente
ángulo central. Como el triángulo AKD es equilátero, LKAD =
= 60º, es clecir, LBAK = 30º. Por analogía, obtenemos queLMAD=
= 30º, y, por el~o, LKAM "" 30º. Con el fin de hallar el área del
a2 X
segmento, aplicamos el teorema 24 y se obtiene: S.egm =
T
x(~--}).
Para liallar el lado del cuadrado E[(M P, aplicamos al triángulo
AKM el teorema de los cosenos: KM 2 = AK2 + AM2 - 2AK X
x AM·cos 30º, es decir, KM2 =- a1 +a2 - 2a\ 3= a2 (2 - y'3).
1i
Como resultado, obtenemos: S =Scuad +4S,egm = a2 (2-V3) +
+ 2az ( ~
= az ( 1 + ~ - lf3) .
EJEMPLO 11. Una circunforcucia es tangente a los lados AC y llC
del triángulo ABC en los puntos D y E, respectivamente, y tiene
-+)
e
8
A
A
b
e
Fig. 70
FCg. 69
su centro en el lado AB. Hallemos el área del suctor DOE si BC =
= 13 cm, AB = 14 cm, AC = 15 cm (fig. 69).
soLuc10N. Para buscar el radio del sector empleamos el método
de las áreas (véase el § 1). Por un lado, el área S del triángulo ABC
puede ser hallada con ayuda de la fórmula de Herón (teorema 19e);
S = 84 Cil\~. Por
otro,
S = SAoc
BBoc = { AC·DO +
t
"1
4
+ BC·OE = (15 13) r = 1 r . Asi, pues, 14 r = 84, r =
+
=
2
2
+
6 cm.
Con el ffn de hallar el ti.rea del sactor, hny que conocer su ángulo
central, o sea, LDOE. Considerando el cuadrilátero ODCE llegamos
a lá concl.usi6i1 de que LDOE = n - y, dondo y = LACE. Según
el teororna do los cosenos AB 2 = AC 2 + BC 2 - 2AC·BC·cos y.
Esto significa, que 142 =132 +15 2 -2·13·15·cosy, de donde
99
99
hallamos que cos y = 105 , por lo que 1' = arccos 195.
Como resultado, obtenemos que el ángulo central del sector
59
§ ' · Ár<as dt las fi(lu.ra• pla11at
es igual a
11 -
00
arccos 19 :;·
Do acuerdo con
x r2
1
el teorema 23, obtenemos: $s~tor = T
-!lr)
X
-Ns).
n-arccos
= 18 ( ¡¡-al'ccos
1'1 . Dt'ntre> d\'ll triánsrulCl ARC. ron lndo.ll a. b y c. lit\
h :1 h1matl.1 t'l p11111,, .ll ,1,, (,1nn:1 :\Üt' 1lt'~1h' ,;\ '-':t h\,h'.~ 1\,1\ t1·\;\11i:11\.•
so ven bajo diferentes ángulos. Hallemos AM JJM CM (Hg. iO).
SOLVCION. A d iferencia de Jos anteriores, en esto problema 110 se'
trnta del cálculo d o! área de una figura j>lana. Pero, a pesar de esto,
como veremos, el área del triángulo será el medio para su resolución.
Hagamos AM -= x. BM = y y CM == z. De acuerdo con el
planteamíento LAME= LBMC = LAMC = 120º. E l empleo
del teorema ·de los cosenos con relación a cada uno de los triángulos
A/tfB, lJMC y A'M"C nos permite ob tener el sistema de ecuaciones:
(
r.Jr.:-tirte>
+
+
+ y'2 + yz,
+ z + xz,
,2= :r;• + y' + :ry.
a2 = z2
2
b
= x2
{
Seguidamente,
S =SABC =SA'l>fC +saMc+ SAMB =
tenemos:
1
i1a
2 xu;en 120º + 2 y;; sen 1200+-z xy sen 120º = 4 (xy+yz+ xy).
J\sl, pues, xy + xz + yz ""' ~ij , donde S ="V p (p-a) (p-b) (p-c)
=
1
1
(P = a+~+c).
Hay que hallar el valor de 111 suma x +y+ z. Tenemos:
(x +y+z)1 =xi+11i+.12 +2xy +2xz +2y::. Sumando las tres ecuaciones ·del sistema,
obtenemos:
zt + yz z2 = a•+b•+c•
2
+
1
- 2 (xy+xz+yz).
Así,
2
,,,,,
2
2
11
(x+ y+ z)2 =
pues,
2
+ b= +c 2
0 +b +c +2-·~= a'+b•+c~
2
2
113
2
2
+ 2a
+2SV3
(xy
+ xz + yz) =
v por lo tllnto, x +
. '
+ y+z= 1/
SJSMPLO
a•+~•+ c• +zs y3.
u. En ol triángulo ABC conocemos que AC : BC
=
= 2 : 1 y LC = arccos {. En el lado A C se toma el punto D de
formo que CD :_AD ... 1 : 3. Hallen la razón entre el radio do la
c ircunferencia circunscr ita a l triángulo ABC y el radio de la circunferencia inscrita en el triángulo A BD.
SOLVCtON. Introduzca mos el pa rámetro auxiliar CD = a. Entonces, AD = 3a, AC = ~a. BC = 2a (fig. 71).
Capítul<> J . Planlm<lrla
60
Para hallar el radio R de la circun[erencia circunscrita al triángulo A BC calculemos el lado A B según el teorema de l os cose u os y,
a conti nuación, hagamos uso del t.eorema de los senos. Tonemos:
AB~ = A C' + BC 2 2.AC·BC·cos C, es de3
A
cir, AB~ = 16a~ + 4ai - 2-4a.2a· :a.e don4
de hallamos que AB ,,,. 2a Y2.
De acuerdo con el planteamiento, cos C =
3
,r
Jl 'f_
=4. o sea, sen C= y 1-cos2 C=--¡-·
Según el teorema de los senos sen
ABc= 2R,
de moclo que
2av'li 4
117
·
.
""' 2R, de donne halla-
1oavi
=-vr·
mos: n
El radio r de 111 circunferencia inscrita
Ftt. 11
en el triáng ulo ABD puede ser calculado
con la fórmula r = li.. , donde S es el área, p ,
el semi perímetro ri el triángnlo ABD.
sabemos que AD= 3a,
AB = 2a.
El lado BD se halla con ayuda del triángulo BCD ,
según el t.eorema de los senos: BD2 = a.2 4a.2 - 2· a· Za· ~ 1 du don do
, ¡,
3a+2all i+a 112
3a
BD = a v 2. Asi, pues, P =
=--z+
-3a'V2
2 . El área
2
S del triángulo ABD se <'.alcula con la fórmula de Herón;
e
Ya
V2.
+
S= Vp{p-AD ) (p-AB) (p-BD) =
Asf. pues, r=f_=
P.
v
a 7
2(112+ 1)'
.!!..-..!
(2+ VZ)
r - 7
•
PJlOBLEllfAS PARA EL TRABAJO INDIVIDUAL
L Arca del lritlngulo
220. Demuestren quo ol :írea do uo triángulo es igual a .§.mcmb sen ex.
donde: m , m 11 son Las medianas, a, el i\ngulo entre ellas.
221. "En el triángulo ABC son conocidos A C ... 3 cm, LA
30°, el radio
de la circunferencia circunscrita, 2 cm. Demuestren que el área del 6A BC
e' menor quo 3 cm•.
=
§ 4. Áreo1 dt /01 / igu.ras p/anat
61
222. Demuestren que S ..:; -t>'+c•
- - , donde b y e son Jos· lados del t r ián·
4
gulo, S , su• área.
,
223. Los lados de un triángulo son iguales a 55, 55, 66 cm. H allen ol 6.rea
del triángulo cuyos vértices .sirven de base a las bisectrices de la figura dada.
224. E n e l tr16.ngulo .ABC son conocidos los lados: AB ¡::: f3 cm, BC
.., 15 cm, A C
14 cm. Sé han trazado la altura BH, la bi~triz BD , la mediana BM. Hallen: a) el áre!l del triángulo BHD; b) el 6rea del triángulo BMD;
e) el área del triingulo» BHM.
,.
225. En cada una de las medianas do un triángulo se há tomado un punto
que divide la mediana ep la ru6n 5 : 1, contando dosd!' el vértice. Hallen el
área .d el triángul o con los vértlcj!s en dichos puntos ei el área ilcl tri ángu lo ini·
clal es ·igu<!l 11 64 cm•.
, .
.
· 2.26. En un triángulo con base Igual a 11 está inscrito ur¡ cuadrado. Hallen
el área· del triángu lo si conocemo.s que el lado del cuadrado 'es mayor que Ja·
mitad de la base del tri4ngulo y el 6.rea del cuadrado constiLuye { parLe do la del
\tiángulo.
227. Unn clrcun forcncia do rnilio 4 cm estli c ircunscrita al triángulo A BC
con ángu.ld: B - 60°. El d iámetro do In circunforonc in, perpendicular ni lado
D C, "inte.rsccn A B en. el p'u nto M de lorma que A M : B M = 2 : 3. Hallen el
área del ' triángulo.
..
228. Hallen el área de un triángulo rectángulo con hipotenusa e s i sabemos
quo la ~ma do los eenos de sus ángulos agudos es Igual a q.
229: Hallen el áréa de un Lriángulo rectángulo con ánguhi agudo a. si sabemos que la dl.s tancia desde el v6rtice del et.ro ángulo agudo hasta el centro de
la circunferencia lnS<:rlta es igual 11. m.
230. En el triángulo ocutángu lo ABC son conocidos AB = e, la m ed iana
BD .., m, LBDA' ..,; ll (ll < 90°). Hallen el ár,e a dol 6.ABC.
231. Por el vértice del· ángulo a. en la base de un ttl&ngulo isósceles sa ha
t razado. una recta baJo. ol ángul <1 ll o la bese (ll < a.) que divido el tri ángu lo en
dos part-es. Hallen fa razón entre lo.a áreas do dichas par-tos.
232. Por el punto medi o dol lado de un triángulo regul ar se ha tratado una
recta guo forma eon el mencionado lado el ángulo agudo a.. Hallen la rnión
entre las· áreas de aquellas parLes en los que la recta ha dividido el Lrláng-uJo.
233. En el trl,ngulo ABC son conocidos dos ángulos: LA =a., LC = y .
Se han trazado la bi5-0ctriz BD, 111 Altura BH y In mediana 81\f. Hallon: a) la
r azón entre las 6.ro.a s do los triángu los BD M y A B C; b) la razón entre las áreas
do los triángulos Blf M y A B C; e) la razón entre las nreas de los trián gu los BHD
y ABC.
234. liallen el áre11 del triángulo si se conocen sus Lados a y b y l a bisectriz
=
=
'• =235.
l.
La mediana A D del triángulo ABC ín ter.seca en el punLo E la circunferencia ci~unscriLa a l triángulo . Hallen el área dol triángulo ABC si sabemos
que LBAD = 60º, AB +AD ~ DE, AE = G.
236. Demuestren que S <
11R•, donde S es el áit!a del t r iángulo, R.
4'
el radio de ln circunferencia c ircu OBcrita a él.
237. Uno de los ángulos do un tri ángulo es igun l 11 600. El punto do tangen·
ele de la circunforcncia in!cri ta divide el lado opuesto a dicho ángul o en los
~gmentm 11 y b. Hallen el área del triángulo.
238. En el triángulo ABC, del punl-0 /lf al lado AD, so han trazado la.s
rectos MQ JI AC y MP 11 B C. HaJlon el área dol tri ángu lo ABC si sabemos que
el área del triángulo 81\fQ es igual a St y la del triángulo A M P. a S 1 •
239. Por un punto tomado en ol interior del triángulo . so han traudo r ectal!
paralelas a sus lodos. Ellos· dividen el triángulo en 6 partos entre las que hay
tres triángul os con 8rons S 1, S, y S,. Hnll'en el ñrc11 del triángulo Inicial.
Capítulo l. Pl.animttrin
62
240. En un triángu lo con lados de IG, 30 y 34 cm ~tá ins~ita una circunforcncin. Hallen el área del triángulo cuyos vértices son l<Js puntos de tangencia.
241. En una circunferencia está inscrito el triángulo A BC con lado A e
20 cm . Por el punto B está tr:iznda una tangente a ln circunferencia distanciada de los puntos A y C a 25 y 16 cm, tespectivarocnte. Hallen <!I área del
triángulo A BC.
242. Del punto M, situado en el interior del triángulo ABC, se bajan las
p~rpcndicularc;1 MD. ME y MF a los lados AB, BC y AC. respecti\•amonte.
Hallen la razón entre las áreM de los triángulos ABO y Dlff sl sabemos que
AB = c. BC = a, AC = I>, ME= k, MF = · m y MD = 11 .
243. En el triángulo ABC so han trozado lns nlturt1s AD, BE y CF. Hallen
la razón entre Jas áreas de los triángulos JJEF y·ABC sí son conocidos los ángulos del triángulo ABC: 0;, ll y y.
244. La cuerda A B conlieno el arco de una circunferencia cuya longitud
es igual a
do la circunferencia. En dicho arco se toma el punto C, mientras
=
=
-Í
que en la cuerda AD, el punto D. Hnllen el área d'cl triángulo ABC si sabemos
que AD= 2 cm , DD = 1 cm y CD = 112 r.m.
245. En el triángulo A IJC el ángu lo C es igual n (i()º y el rn<lio do la circunícre11cia circunscrita es igun\ a 2 1( 3 cm. En AB se toma el punto D de lorma que AD : DB
2 : 1, CD
2 ')12 cm. Hallen el árM del triángulo ABC.
246. En el triángulo A iJC (A n = BC) o\ ángulo A. ~s igun\ a arcsen
=
=
Ta,
La circunferencia, cuyo centro está distanciado a~ cm del vértice B, es tangente a los lados AB en ~l punto f( y..BC, en 'l "y corta en la base AC el segmen-
!
to EF. Hnlleo el área del triángulo EPC si- sabemos ·que PC = ¡¡ cm.
24 7. Al triángulo A"BC' está ciq:unscrita una circ1infercnci~. La tttogcnle
a ésta en el punto B interscca Ja rcctn .-! C el) l'J punto D (el punto C yace entre
A y D). Hallen el área del triángulo BCD si sabemos 1¡ue L.. BDC =
nrcco.!!~,
BD = 29 cm ~ )a distancia ae5de· el centro de In circ\ID(erencia hasta A C es
igual a 10 cm.
11. Area del c1111drilálero
248. Por ·Jos vértices de un cundrilñtero se trazan rectns paralelas a su~
diagonales. D~muestren que el área del parnlelogTamo obtenido es dos veces
mayor que la , del cuadrílát.c.ro ·dn<lo.
.
249. Los lados do un paralclograino sori ·a y b, el ángulo entre ellos, ~­
Hallen el á~rea d'el· pai'al~logtamo formado p,or las bisectrices de los ángulos
internos de la: figura . ·
250. La Hilen media <le un trarecí'o isúscclcs e.s igual a a, las diagonales
son pc,r péndictllares en'tre s'í., .Hal len el ñrea del trapecio.
251. El p,l!ríniotro del trapecio os igüal a 52 cm, ln baso menor, a 1 cm.
Hal\en el árcá ;del ~rapecio si ·sabemos que sus di'agonales son la.s bisectrices de
los ángulos obtusos.
252. La11. ~lrcunl~rencias de ·radi~ 4 y 8 cm, con sus ~ntros en }os punto~
0 1 ~ 0 2 , se cortan en .los.puQtos Cy D, ~ll es la tangente cxte,na comun. -Hallen
el areil del cuadrilátero O,BA0.2 si. sabemos qú¡¡ las tangentes a las circunferencias, trazadas ~en el punto C, SQn perpendiculares· entre FÍ .
253. '09.s circunferencia~ lgu~les de rad.i~ R, con s11..- "''lr.os en Jos P)lntos
0 1 y ·O,, son t~ngentcs exteriores. ·La recta l 1ntcrscca d1~has c1rcunfcrenc1as en
§ 4. Arta• de las figura• plo11ai
=
63
los puntos A, B, C y D de forma que AB
BC "" CD. Hallen el área del
cuadrilñtero 0 1A.DOJ.
·
,
25li. Los Indos e un triángulo son igua\es 11 ,20, 34 y ;2 cm. La aUur11 que
se encuentra en ol in~erior del triángulo está dlviilido en la, rniqn 3 : 1 , contando desdo el vértice. Desde el punto de división está' tra.zada una recta pcr·pendicular a la mencionaaa altura. Hallen ol área del trapel:io obtenido.
255. Los lados de un t rifogulo son iguales n 20, 34 y 42 ~m. Hallen el á.rea
del paralelogramo inscrito ai sabemos que su ~rlmetro es igual n 45 cm.
256. Las boses de un trapecio son Igual es a 62 y 20 cm, loe lados , a 45 y
39 cm. Hallen e.I área de la figura.
,
257. Las bases de un trapecio BOn iguales e SO y 12 cm, las dingonales,
;
_
e 20 y 34 cm. Hallen el área de In .figura.
25~. Una de las bases de 1in trapecio es lgubl u 7 cm. _La circunlorencin
inscr.itn en 11i divide uno de sus lados en segmentos do 4 y 9 ·cm. Hallen el área
del trapecio.
259. En el trapecio ABCD K es el punto medio do la ba..<e AD, /lf. el punto
modio .do la base BC, BK, la bisectriz del ángulo ABC, DM, la bisecLriz del
ángulo ADC. ffollcn el área del trnpecio ABCD si su perímetro es igual a ilO cm
y LBAD. = (10°.
261;). En el cuadrilñícro ctmv11xo ABCD 1:;, !"., /'y K so n los puntos merlios
de los ·1ados AD, DC, CD' Y AD, rcspcctivamcnt.o. Sobemos que El'~ Kf'.
l:lallcn ' el área do! cuad rilátero AJJCD si A,C = 15 cm y BD = 20 cm.
261. !.fallen el área .de· un paralelogramo si sus lados son a y b (a> b}
y el ángulo ontre las. i:liegonal e..s, a..
262: Hallen el área de un trar«io con ángulo ngudo a en In base si sabemos que una do las bases do Ja figura es el diámotro de la circunforencia de
radio R circunscrita al trapecio.
263. En un trapecio con ángulos agudos a. y ji esU inscrito un circu lo.
Hallen la razón entre las árens del trnpecio y el ci rculo.
264. En el triángulo· ABC so conocen los ángu los: LA = a., LB - ~.
L-C =y y la altura BD =JI. Como sobre su diámetro, on BD so trnia unn cir·
cun.ferencio quo C<lrta los lados AR y BC en los punt<ls E y F. Hallen .,1 área
dcl cuadrilátero Bl"DJ:,'.
265. La rect11 l, pRra lcla a la base AC del triángu lo ABC, corta de él E<l
triángulo BED. En el lado A C se toma al 87.Br el punto M. Demuestren que el
área del cuadrll6loro BEMD es la media propnrcional cotTe las árcns de l os
triángulos :ABC y,DB~.
266. ·En ~I trape~io. ABCD (AD 11BC) las diagonales concurren en el punto O, Hallen el área del trapecio sl. so conoce que ol Arca dol triángulo AOD
es igunl. a ii• y In del triángulo BOC, a b~.
267. En el rombo ABCD .M , N, P y Q son los puntos metlíns de los lados
AB, BC, CD y AD . r!l!pectlvamente. Hnllco el áre11 tlcl cuadrilátero limitado
por las rectas AN , BP, D/lf y CQ si el área del rombo es igual a 100 cm•.
268. Dos circunferencias de radios a y b son tftngeo~ exteriores. A ellas
o han trazado tangentes externas comunes. Hallen el ñrna del cuadrllálcro,
cuyos vértieca son 11\S puntos de U.ngencia.
269. Lns diugonalcs do! cuad rilátero Al/CD concurren en el punto O.
Ha llen el tírca del cuadrilátero si sabemos quo lns áreas de l os triángul os AOB,
BOC y CO.D son iguales a 12, 18 y 24 cm2, rcspecUv11mcnte.
270. Una circunferencia es tangente a los lados A B y AD del rer.láo¡¡ulo
A BCD, p11sa por el vértice C y cortn el lado DC en el punto K. Hallen el arna
del cuadrilátero A.BKD si AD - 9 cm, AD = 8 cm.
271. Dentro del recUíngulo ABCD so toma el punto 111 de forma que A M =
- 112. DM - 2 y CM~ G. F111llcn el área do rectángulo ADCD si sabem05 que
AD= 2AB.
Capitulo l. Plan lmelrfo
111. Área del polígono
272. En los catclos A C y BC y Ja hipotenusn AD del t r iángulo rectángu lo
ABC, como en los lados, están construidos los cuadrados (fuera del tri6ngulo)
CMPA, BEFC y ADKB. Hallen el área del hexágono DKEFMP si AB = e
y sl>.Anc
=s.
273. En los lados AC, BC y AB del triángulo ADC est.án construidos los
cuadrados CMPA , BE:FC y ADKB. Hallen el área del hexágono DK8FMP
si ¿;abemos quo AB = 13 cm, AC = t4 cm, BC - 15 cm.
274. El cuadrado prefijado con lado a, está cortado por los ángulos do
forma que se obtiene un octágono regu lar. Hallen su área.
275. Sen dado un cuadr11do oon lado a. En cada uno de sus lados, fuera de
él, está construido un trapecio do forma q ue las bases superi ores do éstos y sus
lados forman un dodecágono regular. Hallen el área do esta figura.
276. Una circunferencia esU dividida en ocho part.es con los puntos A, B,
e, D. E, F, p y K. Es conocido que vAB "'" vCD = · vEP vPK y
vBC = vDE - vFP = vKA; además, vAB = 2 vBC. Hallen el área
del octágono ABCDEFPK si el área del circulo es Igual a 289 re cm:.
277. En un circulo de radio R osLá inscrito un Lriángulo regular y un cuadrado que tl6nen un vórtice común. Hallen ol Arcu de su intcrscccl6n.
278. Cada uuo de los larlos do un Lriángl•lo e~tá dividido en tres par les en
la razón 3 : 2 : 3. Hallen In raión entre el área dol hoxágono, cuyos vértices son
los punt os de división, y el área del triángulo.
279. El área del cuadrilátero ABCD es igual a 12 cm2 • En los lados AB,
BC, CD y DA se toman los puntos F, K, M y P do forma que AP: PB - 2: t ,
BK.: K.C
i : 3, CM: MD = t : 1 y DP: PA - t : 5. Hallen el 6rea del
hexágono AFKCMP.
=
=
I V. Áreas de figuras combinadas
280. Los Indos del triángulo son iguales a 20, 34 y 42 cm. Hallen Ja razón
de las áreas do los círculos inscrito y circunscrito.
281. El lado de un trióngulo regular es í~al a a. En él, como en su diá·
metro , se ha construido un circu lo. Hallen el área de aquella parte del triángulo
que yace fuera del circulo.
282. Un circulo eslá 'inscrito en un Lriúngulo regular. Con centro en uno
de los vértices del triángulo so trata el segundo circulo. cuyo radio es igual o Ja
mitad del lado do triángulo. ¿Qué parte del área dol trilingulo constituyo el área
de la intersección de los círculos?
283. Dos circunferencias con radios a y b (a> b) son tangentes oxieriores.
A ellas se trata unR tangento oxt.erna com.ú n . Hallen: a) el área del triángulo
curvilíneo obtenido; b) el arca del círculo inscrit.o en ese triángulo.
284. El lado de uu uiñngulo regular es igual a o. ·El centroide dol triángulo
es el centro d41_11n círc11lo de radio
.¡. Hallen el área de la par\e del triángulo que
se
encuenLrn !11ora ·del círéulo.
285. Donlro de un cundrndo con Indo a, sobro cada lado como en su diámetro,. so construyen semicírculos. Hallen el área do la roseta obtenido..
286. Cad' una de las n circunferencias Iguales es t.an¡¡cnte a las dos vecinas.
Hallen el á.roa de la figura limitada por los arcos más proximos entro s[ Je estas
circunferencias si sabemos quo el radio de la circunferencia, con la quo todas las
circunferencias dadas son tangentes interiores, es igual a R y que: a) n
3;
b) n = 4; c) n - 6.
287. Desdo un punto Lomo.do. en una circunferencia de radio R se han trazado dos cuerdas Iguales; el ángu lo entre éstas es Igual a a. Hallen el úrcn de la
par.Le del circulo contenida entro esos cuerdas.
=
------------'~"-~-4_._A
_~~ai d~ las flgiira3. p/ana1
65
288. Sea ABCDEK un hexágono regular con lad o a y co~ su centro en el
punto O. Se han tro7.ado trcs circun!crcncias: la primera, con centro en el punto A,
pasa. po¡: los puntos C r:-'E; la segunda,, con ,c~ntro en el punt~ B, P,1184 por los.
puntos.o y C; In tercera, con centro en el punto K, pn~a por los puntos O y E.
Holloil el área do Ja figura .limitada por las tres circunferencills mencionadas
.
s ituada en el interior del hexágono.
289. Dos circunferencias do radios R y 2ll están situadas !le tal forma que
fo distancia· entre sus centros 0 1 y 0 2 es igual a 2ll lf3. A olllls se hftn trazado
tangente'! comunes que concurren en cierto punto def segme~to 0,0 2 • Hallen
el área ·de la figura limitada por los scgmimtos de las tangen~es y los grandes
arcos de los circuntoroncias qµe unen los puntos de tangencia;.
290. La baso do un, tri~ngulo es Igual n a, lo,s ángulos en. \~· base, 15° y 45º.
~Con centro en ol vórtice opuest.o a 111 base del trláng11lo y CQil. r.adlo igual n la
altura tratada desde díelio . vértlcc se. constniyo un circulo. ijnllen el área de
la parte de ésto éituada dentro dol triángulo.
·
291. Dos círculos de igual rRdio están situados de forma que la distancia
entre sus centros a Igual al radio. Ho lleo la razón entre ol área de la intersección
de los círculOll y la de un cundrado inscrito en clln.
292·. Se dn una scmicircunfcrcncin tln di:íinctro A 8. C es un pnnto arb itrarlo ~n.. el .diúmatro A O. Desdo el punto C se fovnntn unll pcrpondiculnr CU al
d iám~tro, )Jn~la su lntc~sccción con In scll!ici rc1111rerc:ncin en el punto D . En
AC y .CB., .como en los ·diámetros, so construyen somicircunfercncins situada~
en el interior llo la dada .. D'iimuostron que el' área do lo. figura' limitada por las
t~ scmiclrcunforcncias es igual a In del círculo con~lruldo cil CD como on su
diámetro. ..
293. En el trll\ngulo ABC LA =ex, LB~ ll y AC ~ b. !,as nlturas AD
y BE concurren en el punto JI. Al triángulo /IDH cst.A circunscrito un circulo.
Hallen el área de la mtcrsccci6n del circulo y ~1 triángulo. ·
296. En el Interior de un 11-gono regular con lado a eslán situndos n círculos iguales do forma que c:.uln uno de ellos es lRngontc 11 dos otros y al lado
de'l n-gono. Hallen el área de 111 •estrella, formadn on el centro del 11-gono.
295 .. En el interior de un 11·gono regular con lado a están situados " circulOll igualas de rormn que cadn uno de ellos es tnngcnte a dos lados adyacentes
del n-gono y a otros dos círculos. Hnllcn el ;irca de la rcstrell:n formada en el
interior del 11-gono.
V. DUorontcs problemas
"'ª
296. El órea de un triángulo os igual n 16 cm', las mcdienns
y 1111, son
iguales a ,6 y 4 cm, respoctivamcnte. Demuestren que c,,tas medianas son rorpendlcularcs.
297. En el lnlerior de un n-gono regular so toma al unr uu puntó. Desde
111 se bajan perpendiculares a los Indos o sus conlinuaciono5. Demuestren quo la
suma de estas perpendiculares es una magnitud constante.
298. Por ol centroide del trifogulo re.guiar ABC se trazn una recta 1111ralela
ol lado AD. En osta recta, dentro del triangu lo, so tomn ni n7.nr el punto M
y desde él se b11jan las perpendiculares llfD, llfF. y MF n los lados AB, AC y 8C,
r cspcctivnmenl-0. Demueslrcn quo MD = i-<ME +Mil).
.
1
1
1
l
299. Dcrnuestron, que h;-+ -¡;-+ h=r, tl o1ulo 1,., h*, h• son ln.s alt11rns
del triángulo; r, el radio de la o iicunlc~ncia inscrito.
300. Sea D un punLo interi or del lado AC en el óARC, r 1 y r 9 , los radios
de las circuníorenolas inscritas en ltts triángulos ABO y BDC. rcspectlvt'llne11te,
5-02011
Capitulo l. Planinutría
60
r, el rodio de la circunferencia i11scriLa en el triángulo A OC. Dcmu<>stren quo
r < r1
r.,
301. E l área del cuadrilátero convexo ABCD es igunl a 3024 cmª y las
diagonales, a 144 y 42 cm. Hallen la longitud del segmento que une los puntos
medios de los lados AD y CD.
302. El área de un triángulo isósceles es igual a S y el ángul<> entre la3 medianas, Lrazadns a los lados laterales, a a. Hallen la base del triángulo.
303. Los lados de un triángulo son a y b, el ángulo entre ellos y. Hallen:
a) la bisectriz le: b) la altura h •.
304. La baso de un tríánguló es ·igual a a, Ja altura a 11. Rallen la suma de
los lados later\J.lee si sabemos que el ángulo entre ·ellos· es igual a a.
305. En uh triángulo u no do. l os á~los es igual a la dHerencia entre los
otros dos, la. longitud del fado menor es igual a t cur y .la suma de las áreas de
los cuadrados, construidos en los ot.ros dos lados, es dos veces mayor que el área
del círculo cii:cunscrltti al. triángulp. Hallen la longitud del lado mayor del
triángulo.
306. En un triángulo ron conocidos dos Indos a y b (a> b) y el área S.
!-follen el ángulo entro In altura y In medianR trnzndas desde el ·vcrticc común
de cl<is lados chulos.
307. Conociendo el órell S y l os írngulos a, ~ y y dd triángulo, hallen la
longit11d de Ja altura trazada desde ol vértic~ Jol áng11lo a.
308. b:n el triiing11lo ABC está inscritn lo circunfcrencin t;mgente ni Indo
AB en el punto M y ni lado A C en el N. Hallen el ángulo lJAC y ()1 rndio de la
circunferenci a inscrita si AM = l cm, BM = 6 cm y CN
7 ero.
309. El área del rec\ángulo .4 B CD es igual a 48 cm• y In diagonal n 10 cm .
El pun.lo O está nlejndo de los vértices B. y D n unn distancia do 13 cm. Hallen
la distancia desde el.punto O hRst~ el \'értice del rectángulo más alejado de él.
3IO. ·L os Jedos de un triángulo a, b y e formdri una progresión aritmútica
crecieJ)te. Demue5tren que ac
6Rr, dondci R y r, son los radios de las ciccunferencias c;ircunscrit'a e inscrito:.
3 11. Las b¡¡ses de ún trapecio ·so11 a y b. Hallen- la longitud del segmento,
paralelo a las1 bases, contenido entre los lados laterales y que divide el trapecio
en dos partes equi:válen'tes.
312. En el trn¡¡ecio ABCD Ja base AD= JJC = 12 cm. En la continuación
de BC por el punto C se toma el punto M de forma que el área del triángulo, corlado del trapecio ABCD por la recta A Jlf, es igual a
del área del trapecio.
+
=
=
f
Hallen la 1011gi tud dol segmento CM .
313 .. pcs'de los vértices A. y e d.e l triángulo. ocutángulo A BC se ban bajado
las alturas AD y 'CE. Es conocí.d o que el área. del tl'iángulo ABC es igual a
18 cm• y ·la d:e l triángulo BDE, a ·2.· cmi. La.'longitµd del ~egmen to DE es igual
a 21/2. Calculen el radio de la· circunferencia..circU:n~rita al triángulo ).BC.
314. Desde los vértices A y e·del triángulo ocut~ngulo ABC''sc. bájan las
aliuras AD y ·ce. Se ·sab.e que el áre.a del triártgt!IÓ .ABC .es igual· a 64 cm•
y .)a.del triá)lgulo IJDE, a i6 cm•. Hallen lq ·lollgÍtuil del ~gmento DE _!i el
radio de la ·circunferencia circunscrita ni Lri~rigulo A.OC cs ·igunl a 161/3 cm.
315. Eu l eL tdángulo ABC ·cuya .área es igual á 6 cm•, en. los· lados AB
y.AC so han tj:¡mado los puntos Ky /tf, respcctívamol\te, de forma que A K : B K=
2: 3, AA(: CM= 5 ;_3. Las rec.~s CK y BM concurren en el punto P.
Hallen AB si la distancia desde e! punto P hasta In recta AB es igual a 1,5 cm .
.316. En , el triángulo is65\:elos AD C (AB
BC) se ha trazado In bisectriz
AD. Hallen i AC si St>Alll>
S1 Y Sti.ADr;""' S,.
317. En el triángulo A BC el ·punto Ji es el ortocentro. Hallen el segmento
Ali si AB
13 cm, BC = ~4 cm y A·C
15 cm.
31 El·!ccntro' de In circunferencia inscrita en un tri~ngul.o está unido por
segmentos cQn los vértiées· del triángulo. Se obtjenen tres triángulos con áreas
4 ,13 y 15 cmt. Hallen Jos lados del triángulo inicial.
=
=
=
s: =
=
§
5. Tr4nsformaclones geomt!lrlcrur
67
=
. . 31~. EJÍ !)l triángulo AB.C so conoce quo BC: AC
3, ,L..e =-y . .En AB
se Lpinan los puntosD y K de.forma que LACD = LDC'K = LKCB. Hallen.
la relación CD. : c K.
1
320: En .il triángulo ABC so ha trazado la mediana BD. Hallen ln razón
entre éf radfo de In circunferencia. circunsÚita al triángulo ÁBD y el radfo de
·. la-..circun[créncia insnitn en el tr.iángulo ABC si AB = 2·, A C: = 6 y LBAC =
·= 60°.
32t. ;En el triángulo ABC se conoce que AC: OC -oi 1 ·; 3, Li!CB =
' )15
j,
.
•
= arctg
En el lado A C se ha tomndo ol JIUnlo D do forma· quo A C ·= CD.
Hallen ·1a razón entre el área del círculo circunscrito ni triángulo A CD' y el área
·deJ círculo inscrito en el triángulo ABD.
z:-·
§ 5. TRANSFORMACIONES GEOMETRICAS
Aducimos ejemplos de aplicación de la simetría central, así como
de la composici6n de la simetría central.
E.TEMPLO. t. P.01· un punto que ynce dentro de 1111 círculo, t.razamos. una cuerda 'd·e n1odQ que ésta se divida por la mitad con el
punto dado.
.
soi;uc·10N. Coñstrnynrnos un.n circunferencia sirnótricn n In prefijada con relación
punto dado. La cuerda común de eslas circunferencias será la buscada.
EJ"EMPLo 2. C-Onstruyamos un pentágono según los pu11los medios
p~efijados de sus lados.
s0Luc10N. Designemos los puntos medios de Jos lados del pentágono, q~e hemos de construir, con /11, N, P, Q y K. Tomemos al
e
azar el punto A y consideremos la composición de las simetrías cenlrales: ZK o ZQ o
o Zp o Z 11 o Z M· ¿Qué hace esta composición
con. el punto A? Si designamos la composición con ll, entonces 6 (A) = A (fig. 72).
D
Sea Zr o ZN o Z·M (A}= Zs (A), entonces A
Zx 0 ZQ o Zs (4) .,= ZA (A). Construyan los
paralelogramos MNPS y SQKA y, seE
guidamente, la construcción del pentágono
Fíg. 72
está clara.
Aducimos ejemplos de simetría axial.
EJEMPLO 3. Se dan dos circunferencias y ln rcct.a z. Construyan
un triángulo equilátero de forma que dos de sus vértices pertenezcan
n las drcunforencias dadas y otro do los vértices, a la recta l.
soLUCION. Supongamos que el t::,,ABC es el buscado (fig. 73).
Como la altura AD del triángulo equilátero ABC pertenece a la
rect-a l, Jos puntos B y C son simétricos con relación a dicha recta
•
y yacen en las circunferencias dadas roJ y ro 2•
Corno el punto C pertenece a la circunferencia <>>~ y es simétrico
ni punto B, pe1·tenecientc a la circ11nferoncia ro 1 , con relación a la
recta L, el punto C pertenece, asimismo, a Ja imagen de la circun-
al
Capítulo l. Pla11imttrta
08
fcreucia w 1 durante la simetría rcspect-0 a la recta l. Por consiguiente,
el punto C es el punto común de la circnnferencia c.>z y de la imagen
de la circunferenéia c.>1 con. la simetría 8 1• De este modo, después
de construir la circunferencia c.>t> simétrica a co>; con relación a la
recta l , hallamos el punto C.
A continuación, constrnimos el punto B como la imagen del
punto C con la simetría S 1 y, después, el punto A.
Las construcciones realizadas a l., resolver este problema pueden
rcaliiarse en el siguiente 01·den: 1) consir\Jimos Ja imagen de Ja circunferencia :c.>1 con Ja simetría S 1 ; 2) hallan1os los puntos de intersecciéin de las c ircuníerencius w; y w2 ; 3) bailamos en la circunferencia
Fig. 73
Fig. 74
w1 las preimágenes de los 1rnntos de intersección de las circunierencins
y Cilz; 4) ?onstr11imos.el triángulo equilátero ABC, cuyo vértice A
yace en lp lí1101.1 l .
Si las ciicunferencias oo; y c.> 2 se cortan, el problema tiene 4 solu-
w;
ciones. Si ellas son tangentes, el problema tiene 2 soluciones. Si la
coinci~e con w,, el problema tiene una infinita
circunferencia_
cantiilod de soluciones. Si las circunferencias (>); y (>)~ no t ienen
puntos comunes, el problema no tiene solución.
EJEMPLO ~ - En el diámetro AB de un semicírculo se da el punto P
y en sn semicircun[ei:encia, 1os puntos M, M 1 y N, N 1 , tal es que
LMPA = LM¡PB; LNPA = LN1 PB. Demostremos que el punto . Q, punto: .de intersección de las cuerdas llfN1 Y. M 1 N, p~rtenece
a la porpcn:dieulor trozada al .diámetr.o AB. ppr el punw P.
sowc10N~ Cpnstruyamos los puntos M' Y. JY' .simétricqs a. M y N
con relación a la recia AB (!ig. 74). Entonces, 05tará dado que los
puntos M' ,'P, M1 yace11 en una {llisma recta y, por analogía, los
puntos N', ..P, N 1 pertene·cen a una m'isma recta. Al cuadrilátero
PQN1 ML pJede circunscribirse una circunferencia y, por lo tanto,
LQPN1 = ?N1 M 1Q (ya .que su suma. es Jgual a 180º). A continuación , consid'eremQs e.l cuadrilátero PQMN que posee la misma propiedad: LÍ'{PQ = ·LNMQ. De la igualdad' de los ángulos NMQ y
N 1-M1Q se <\'.educe que LNPQ ·= LN¡PQ y que PQ ....L AB.
Aducimo,s ejemplos de la aplicación 'del glro.
w;
§ S. Tran.sformaclonu g~omilrlco•
69
EJEMPLO s. Construyamos un triángulo equilátero uno de cuyos
vértices.coincide con el punto .dado A y los otros do~ per tonecen a dos
circunferepcias . pre!ijadas.
soLupION. Construyamos la imagen do una de las circunforencias
al gl_rar a 60° COl.l centro de giro én el punto A. El -punto do in ter.. sección de la segunda de las circuníerencius prcfiia<tas con l a cirun {ere!lcia ,.construida es el segundo vértice del tri~ngulo.
EJEMPLO s. En los lados AB y BC del triángulo ABC, como sobre
us bases, se construyen los cuadrados A BMN y BCQP, orientados
N.-- - ----,M
I'
Q
M
N
(b)
(ll)
Fts. íS
del mismo modo. Designemos sus centros con 0 1 y 0 2 , el punto medio
do AC con K y el del segmento MP, con L. Demostremos que el
cuadrilátero 0 1 W 2 K es un cuadrado.
s0Luc10N. Consideremos el caso cuando los cuadrados pre[ijallos
están situados en el ámbito exterior cou rel11ci6n al triángulo ABC.
"' a1emos, que l a cornpos1c1on
. . . R210•
Se..
ºi º R27n•
o 1 traspasa el punto A
00
al C. por lo que R5! 0Ri~o· = R;! •. De aqui se desprende que el
triángulo 0 10 2 K es rectángulo e iSóscolcs.
Por analogía, Rb:· o R~~· = R~80º, por lo que el triángulo 0 10 2 L
lnmbién es rectángulo o isósceles (LL = 90°). Por lo tanto, 0 1 L01 K
es un cuadrado (fig. 75, a).
En el caso c1,1ando los cuadrados están si~ui1dos por ol otro lado
de AB y BC, el problema se resuel ve de modo análogo (íig. 75, b).
Aducfmos ejemplos de ln aplicación de la traslación paralela .
EJEMPLO 1. 'Dos rectas para lelas p y q se cor tan poi: una tercera
recta s. Construyamos un triángulo equilátero con el lado prefijado
de n1odo que sus vértices pertenezcan a las rectas p, q y s.
Cap!tulo I. Planimetría
70
soLUCION. Del punto arbitrario A 1 , mediante la recta g con un
radio igual a la longitud del segmento dado tracemos una circunferencia y hallemos el punto Ct de intersección do ésta con la recta p;
construimos el triángulo equilátero A 1 B 1 C1 (fig. í6). Por el punto B1
trazamos la recta h 11 p y designamos c-0n n el punto de intersección
de las rectas h y s. A continuación, efectuamos la traslación paralela
Ü del triáng_ulo A 1 B 1 Ct> donde
= B 1 B. El problema puede toner
o dos soluciones, o bien ninguna.
EJEMPLO s. Se dan las circun[erencias úl 1 y úl 2 y la recta l. Tracemos una recta paralela a l, en la que las circunferencias w1 y c.> 2
cortan iguaics cuerd·as.
SOLUCJO:-:. Supongamos que la recta l' (fig. 77) corta en las circunferencias dadas iguales cuerdas AB y A' B'. Entonces, los puntos
v
Ftg. 77
Píg. 76
A y A', B y /J' pueden considerarse corno los correspondientes con la
traslación paralela :r,,,u;• clonde 0 1 0; es un vector, cuyo origen es
el punto 0 1 , es decir, cJ centro de la circunferencia úl¡, en tanto que
o; es el centro de la circunferencia (!)~.
Como el punto A' es la imagen del punto A, perteneciente a la
circunferen¡;ia úl1 , A' per~encce a la imagen de la .circunferencia w;.
Por consigl(ientc, A' es un pupto común de las circunférencias w2 y
ro; con la traslación paralela T o,o',·Despuéslde.tons.truit el punto ·4 ', e11 la ·circunferencia w1 hallamos
su preimagen. L~ recta AA' será la recJ;_a· l' :buscada:.
Si las é;ircunferencias
y w.2 coincid~n, _-el pro-blema tiene un
co.njunto iJ\íinito da solucione~. En .todos los demás casos, el prol>leu1a tiene nll más de nná solución.
· Aduci~bs ejemplos de ap_licación de la homotecia.
<;JEMl'LOj o. En el trapecio A/JCD trazamos las diagonales AC
y ·BD que se corta¡i en el punl-0 M C4B y CD son las bases del tra·
·pecio). Dei:iiostremos que los áreas pe los triángulos ABM y CDM
son igualei, correspon'dientcm'ente•. a S.2 y S 1 y que con el área S
del trapecio están ligadas c.on la relación Y.81 +
= Vs.
w:·
ys,
§ 5. Transformacloncr geomltrtcas
íl
SOLtlCION. Sea N eJ punto de intersección de la r!)cta A B con ln
iecta que pasa por el punto C. paralela a DB (fig. ?8). El :írea del
triángulo ACN es i.g ual al área S del trapecio dado. !fracc mos BF 11
11 AC. El área del .t r ráng ulo BPN es igual al área $ 1 del triá ng ulo
DMC. Los triángulos AMB y BFN son homotétic"os nl triángulo
·A CN co11 coeficientes k 1 y k~, con la particularidad de que k 1
k~ =
1S
·
k
V..f,
k
y
s,
1
'
,,-S •
P
= .1. ero = Vs y 2 = y s , por o que r 1 ¡
i = y
+ V-S
1
+
Eri el triángulo ABC está inscrita la¡circuofercncia,
tangente en el punto M a Ja recta A B . Sea que el plinto Mi. es d'iaEJEMPLO 10.
C.
·D
A
C
~~
~
8
N
o
A
Fig. 79
Ftg. 78
metra lmente opuesto al punto M en la circunferencia inscrita . Demost remos que la recta CJl-f1 cor ta la recta AB en tal punto C 1 quo
AC + AC1 = BC + BC1 •
SOLVt:ION. Construyamos una tangente a la circunferencia en el
punto M 1 que interseca AC en el pun to A 1 y BC en el punto B 1
(fig. 79) .. En tal caso, queda claro que CA 1 + A 1 M 1 = CB 1 + B1 M •.
A continuaclón, hacemos uso de quo los t riángulos AlJC y A,l)1C 1
son homotéticos, ya que las roct:is A B y A 1 8 1 son perpendiculares
al diámetro MM1 y, por lo tanto, AB 11 A1 B1•
PROBLEMAS PARA E L T RABAJO lNDlVIDUA L
J.
~lmctría
rospccto a un punto
322. Están dados unn recta. un segmento y el punt.o O. Construyan el s~g­
m cnto de forma que sus extremos pcrtcneicnn n In recta y nl segmcnt.o dndos
y quo O sen su punto medio.
323. En el triángulo ABC se han \ra:r.ado las mcdianns AA 1, Dll 1 y CC 1
que se cortan en el punto AJ. P, Q y ll son los puntos med ios de 1os segmentos
AM, DM y CM. Demuestren que el e..A}B1C1 =- t:.PQR.
324. 'Conslruynn un triángulo scgiin ctos de sus Indos y la mediana ni tercero. ¿En q uó limitces puede variar In longitud do In mcllinna si las longitudc~
de los lados del t r iángulo son a y b?
.
325. M, N r K son los puntos modios de los segmentos. un c.xtr~mo du
los cuales es el verHcc dcl triángulo ADC y el otro. el punto de intersección cll!
l ns mNllnMs. Dcmul'slttn que ,.¡ triñn¡n1lo ouyns n\rt irl'S son los puntos dr
72
Capítul" 1. Planimetría
inlcrsección de las r~ctas que contienen los puntos M , N y K ,' parnlelas a los
respectivos lados del triángulo ABC, es igual n éslc.
326. Están dodos dos circunferencias y el punto P. Construyan un paralelogramo de forma que sus vértices porteneican a las ci rcunferencias dadas y el
puolo P sea Ja intersección de las diagonales del paralelogramo.
32i. La r~ta que contiene el punto de intersección de las diagonales del
paralelogramo ABCD corta en sus lados los s~gmcntos BE y DF. Demuestren
que éstos son iguales.
328. Construyan una recta que divida un paralelogramo en dos portes equivalentes.
329. De los extremos d~I diámetro BC de la circunferencia con el centro
en O, se han trazado dos cuerdas iguales BA y · CD de forma q11e BA y CD no
se intersecan y yac~n por diferentes lados de ·B C •. Demuestren que OA y OD
pertenecen a una misma rectn y que DO = OA.
330. A una circunferencia se circunscribe un hexágono con lados op\1eslos
paraldos. Dem\lestren que estos lados. del hexágono son iguales.
331. Los lodos opuestos del hexágono convexo A .B CDEF son paralelos
e iguales a pares. 'Que parle del área del hexágono constituye el ~rea del triángulo ACE?
332. En una circ11nfcrencia se dan los puntos A y B y en la recta 1, el punto M . Hallen en la c ircunforencia semejante punto X 1¡110 lns rcctns A X y B X
corten 1a recta i en puntos situados a iguales distancias del punto M.
333. Por el punto ftf del ángulo ABC, que no perlencce a sus°lados, tracen
11na secante de modo que se obtenga un hiángulo de la menor árc:1.
334. A una·circunforencia se ha circunscrito un octágono cuyos lados opuestos son paralelos n pares. Demuestren que los lados opuestos del octágono son
iguales a pares.
335. i>e dan el triángulo ¡'.BC y cierto punto X. Construyan el paralelogramo BXCY y, a continuación, otro paralelogramo YXAZ. Demuestren que
~xistc la homoteda que trasto.da el p11nto X al Z y hallen la razón de homotecia y el cent~o.
·
336. Inscriban en el euadrihítcro dado un paralelogramo a condición do
que dos vértices de éste están fijados y pertcnccén: a) a los lados opuestos;
b) a los lados 4dyaccrites dd cuadrilátero,
337. La me.diana CM del triángulo ABC forma con los lados A C y BC
los ángulos a y ~. respectivallicnlc. ¿Cuál de estos ángulos es mayor si A C < BC?
II. Simetría respecto a una rec la
338. Construyan un pentágono que tenga: a) un eje de simet.ria; b) más de
un eje de sim.c tna.
33!». Por el punto dado ·traccn µria recta que corte las dos ·rectas ·dadas baju
diferentes ángtil!>s.
340. Construyan un triángulo según un lado, la diferencia de los otros
dos lados y el ángulo entre el pri¡ner ladQ y el ionycir de Jos .o.tros dos.
341. Cónstri.t}•an un t:riángulo"seg'ún dos lados y fa diferencia entre.los ángnlos
opuestos a cllcls.
342. Oeotr,o de un án~ulo agudo se da el p.unto M. Construyan .eJ trián~lo M-AB .d.e perímetro m1oimo, cu):os: #rticlis A y B yacen en. Jos lados del
angulo.
i
·
343. Construyan el. cundri.l átero convexo ABCD que.sólo tenga un eje de
simet'ria: la recta BD.
34~. ¿Es: pos.ib.Jc .construir un pentágono tal q!)e su diagoMI se encuentre
en su ·ejo de s1¡\¡etría? Fur.idnmentcn la solución.
345. Demuestren que en un. polígono convexo con número impar de vértices y que tiene eje de .simetría, ninguna de sus diagonales puedo encontrarse
en el eje de sim:etría.
73
346. Construyan un t riángulo según el ángulo, el lado veeino y la diferencia de los otros dos lados.
,
347. Construyan un triángulo según la dlforcncia pre1ijadá no igual n ·cero
de sus 6nirulos -y las longi tudes de !ad.os opuestos a dichos ángulos.
348. l>e dan :dos élrcunlercncias concéntricas. Construy1111. un rombo diferente de 1;10 cuadrado, -~~ f9rma quo: a) .clo.s 'vértices porlene~can a unn c!r~un­
fareucla .y los otros .dos,, a otrn; b) tres vcrt1ces portcnozcan a upa circunforcncla
y uno. a ol ra.
,
·
349, Construyan el triángulo A BC según tres Jlcrpcnd lcularcs dadns p, q
y r bajadas a los puntos mod ios de sus· }odos.
'
aso. ·En_ la c1rcunJerencla dada inscriban un triángulo • .cuyos lados son
paralelos o tres reetas dndas.
.
351. Al triángulo ABC está circunscrita una circunfcre~cia que interseca
la bisectri~ del ángulo C en el punto ftf. Desdo el ortocontióil/ del triángulo
se bn tra-zado la perpendicular JID a la bisectr ii de modo qt1e el punt-0 D 11crteoeco a l . Demuestren que CD : CM
cos C.
352. itn. una eircunferencin con centro en O ostá inscrito ol cuadrilátero
ABCD. E'st.á n traiado& ,loa rayos 0/11, ON, OP y OQ, donde /lf, N, P y Q
aon los puntos medios do las cuerdas AB, BC, CD y DA. Demuestren que
LMON
LCQD
o hicn LMON ... Ll'OQ.
353. A lo- clrcunfcroncln con centro «.>n O cstll circunscril1> ol cuntlri ltí tcro
ABCD . Demuestren que LAOB
LCOD - 180°.
3M. En Ja ·circunlcrencin dadn inscriban un pentágono, cuyos Indos s~nn
paralelos a cinco rectas dadas.
355. Sobre una mesa do billar do formn rectangular yace una bola. ¿En
qué dirección es preeiso impulsar la. bola para que después de rdlcj;irsc do todos
los criotos, ella pa~o por su posición inicial?
356. Demuestren quo el punto de intersección do las rectas 9uo contienen
los lados laterales de un t rnpecio isósceles, el punto de interscc;cion do su~ din¡¡onales y los puntos medios do las bases del Lrapecio pertenecen a unn misma
recta.
357. Demuestren que la recta que contiene los puntos medios de dos cuNd~s
par"lclas de una circ:unlcrcncla pasa por su centru.
358. La circunferencia F 1 corta las circunforenclos concéntricas r, y f~
en los puntos A. B y C, D. respeetivamcnte . Dcmur.strcn que las cucrons A n
y CD soo paralelas.
359. Tres circunferencias iguales tienen un punto común. Demuestren que
la circunferencia t razada
los segundos puntos de int(lrsccción do las tres
circunfcrencias dadas es 1¡rwil a las primeras.
360. En un pl nno se dan cuatro circunforcncins Iguales quo pasan por uu
punto y se corla.o , por llC8'lllda vez, en seis puntos. Dcmue$tren que cuotro
circunforencins, que -pusnn por cado tres de esos seis ponlos, tomando un punto
en cado una de las c1rcunforonclas dadas, so cortnn an un mismo punto.
3(1t . En un plano se dnn \loa recta y un punto que no ynce en ella. Hallen
el lugar geométrico do los ecn lros de triángulos regulares. en los que uno do los
vértices se halla en el punto dado y otro, en la r·c<:la dadn.
862. En un plano se dan una recta y un punto que no P<'rtenccc o ella.
Hall en el luiar geométrico de Jos terceros vértices de t riángulos regulnres, en
Jos que un vcrtíco se halla en el punto dado y el otro, en la recta dada.
=
+
= tao•
+
.Pº'
111. Giro
363. Construyan ol cuadrndo ARCD según su centro O y los puntos J\f y N
quo pert enecen a las rectas AD y BC, respectivamente, Ot.f +. ON .
a64. Construyan un t riángulo equilátero tal quo uno de sus vértices coincida coo el punto dado O, mientras que los otros dos pertenezcan a dos circunferencias dadas.
Capitu.lo l. Planime.tría
74
365. Por ól punto dado en el interior de uoa circunferencia trazar una cuerda
de Ja longitud prefijada.
'
3G6. En los fados BC, CA y AB del triángulo equilátero ABC están dados
los puntos M, .N y P, rcspootivnmente. Se conoce que BM: MC - CN: NA =
= AP: PB
k. n) Demuestren que MNP es un triángulo· equilátero.
b) Calculen MN si BC = a, k
2.
367. En los lados BC, CD, DA y AB del cuadrado ABCD se dan los puntos P. Q, R y S, respectivamente. Es conocido que DP: PC - CQ: QD =
= DR: RA =AS: SB =/t. n) Dcmue&trpn que PQRS es un cuadrado.
b) Calculen PQ si AB = a, h = 3.
368. En los lo.dos AB y BC del triál)gufo ~B.C, como sobre sus bases, se
han construido los cuadrados ABMN y BCOP orientados en un mismo sentido.
Designemos sus centros con 0 1 y -o,, el punto ·medio del l11do AC, con K; el
punto medio del segmento MP, con L. Dcm).leStren que ·~¡ cuadrilátero 01 L01 K
es un cuadrado.
369. En los lados A C y BC d.e l triángulo-ABC.Juera .de él, se ban construido los triángulos equiláteros ACB1 y BCA 1 .' Hallen 'los ángulos del trián~ulo
MA 10, donde /11 es el punto medio del la-do AB, él punt o O, el centro del triángulo ACB¡.
. .
.
370. En la contínuaci6n dG los lados del. triángulo rectángulo A BC se han
mnrca<lo los segmentos AD y AE, Iguales n lQs-.:atotqs AB y AC, respcctivnm~uLc, del t.riúngulo ABC. ·D emuestren que ln.rcctn que contiene la mcdinna A M
del lriángulo ,.AB9 es p·e rpendicular al segm~ntO' DE.
371. Bsla dacio el cuodrnclo ABCD. Por·cJ cenlro <le éste se han trazado dos
TQ.ctas perpendicular~s cnlrc sí, diferentes de µis·.rect hs A C y BD. Demuestren
que las figuras, formadas por la intcrs.ecr.16n: de, e.stas rectas con el cuadrado,
son ig,1alcs.
· .
372. Por el c~ntro O del· triángulo reg11l¡ir A'BC se han trazado dos rectas
que crean entre sí un ángulo pe CP".. Demucs(ren. que los .segmentos de estas
melas. situados dentro del. t.rllíngulo,, son .lguO:lcs.
373. Con~truyan un tri~ngulo cquil'átqro . de .forma que uno de sus vértices
sen el punto 1!, el scgu11(,lo pcrl~n~zca .n Ja: rec,ta- á, eJ tercero, a la recta. /1.
37~. En los lado¡; AP y .A C del triángµlo ABC, .p·ero fuera de él. se han
construido In~ cua.ctra_dós ABNM y ACQ!J.. ·Deinueslren q1ul MC .L BP .
375. Están dados dos ciiadraclos MP0l1' y MUVW orientados en un mismo
sentido. Dcmucstrc.n q·uc los segmentos/' U y R W son iguales y perpendiculares.
376. En l~s ladns AB v BC del triángulo ABC se lian constroi<Jo c11adrados
con Jos centros D y E, c·on fn par.tic.u laridad i;lo que los puntos C y D están situados por un lado d.o AB , en tanto que los púntos A y 8, por diferentes lados de BC.
Demuestren qi.10 el ángulo entre las. rectas AC .y DE es igual n 45°.
377. Construyan el cundri\30 A.BCD l!egún su centro O y dos p untos M y N
que pertenecen a las r!)cta~ BC y C.D, OM. ..¡. ON.
·
=
=
IV. Traslaclóp páralcla
378. Se clan cuatro diferentes puntos A , ·B....c y D. Trazar por el.Jos cuatro
recta¡¡ pnrole!irs a, b, .e y d, respectivamente, de .forma que la anchura · de las
bandas entre las rec tas a y b sea igual a la. 'anchura entre la.s r~t.ns e y d.
379. Construyan un tro.pcclo según sus diagonales, el ángúlo eñtra ellas
y uno ·de los la.Jos.
380. Deriiuó.s tren quG si uña recta que pasa por el punto medip de las bases
de un tropecfo forma ig1inles ángulc:is con lns rectas que r,onticnen sus lados laterales, el trapecio es isóscelo.
·
·
381. Dos C:ircunforcncins iguales son .tangentes exteriores en el punto K.
La secante paralela a la línea· dó los centros corta, consecutivamente. la circunferencia en los ·pun~os. A, 8, C y D. ·Demuestren ·que el valor del ángulo
A KC. ·no depende de ln elección de lt1 soonnle.
75
38?. :Qoterminen · e1. iírea :de un trapecio si todos S\IS lados son .conocidos.
383. En una -circun!erencia con.centro en O ~e dan· tres puptos A., B y e
tales· .que LAOB .= t.JJOC::. 6.0º. Demuostren. quo la dis~i¡cia desde..cl punto B hásta un diamet,::o tomo.do a l a.zar Clc la ·circupforenc1a es Igual bien a la
suma, bien al v:ilor absol.u to do In diferencia ao l¡i distancfo desde los puntos
A· y e hasta dicho diámetro .
.
384. _.Por· el punt!l:. M ; que yace fl,iora do la .circunferencia (1), tracen In· recl~ m .<Jlle corla (1) cn··dos ;puntos.A ·Y ·B ele forma que J!B
.BM .
.385 . . Cuatro . circunfere!lcius igualés w., w 2 , c.> 3 y <s>4 ~RSl\!l'.por. el punto ·M y ile nuevo se cortan en scls..Runtos: A·12 es el punto elo ;1nterseccion de <s> 1
y (l)j; , A~, .de (1) 2 y <s> 3 , ot.é .; A 0 ~orrespondo a ci> 4
y ro,. uomuestren que los segmentos AiiA 4 .,
A 23 A.1 4 , A 13tlu tienen .Rtmto medio común .
386. ~as r.ect;is a las que pertenecen los lados
late~ales de un trapecio son perpendiculares. Deo,
o~
muestren que la longitud del segmento. cuyos
extremo~ son. los puntos medios ele las bases del
tT.apecio, es igua l a la semid ifercncla de las longi tudes do las basl)s.~
Fig . 80
38i. ,ÍAI sumn do. In(! longitudes clo lns bases de
11n t rapecló es igual a ·21 cm, ch tanto quo las longitudes do lus cliagona lcs son
Iguales u t'3 .y 2Qicm. Calculen el área del trapecio.
388. La distnncia enire los centros ·de dos circun ferencias, de igua les radios.
que se cort.n n es igua l 11 .d . Una: ·rcctn J?Bral el~ ·a la líneo. de l:os centros corta la
primera. circunferencia en. los puntos· k y B y la segunda, en C y D. Ha llen la
longitud del segmento ti C. (fi¡r. 80) .
·
389. Construyan el cuadrilatero .ABCD conociendo la longitud de sus Indos y la del segmento MN que uno los puntos medios de los lodos AB y DC.
390. Las diagonales de un trapecio eón bases a y b son rcrpendiculares
entre si. ¿Qué va.lores puede tomar Je a ltura del t rapecio?
o
=
A<J!fj)
V. Homotccla
39l. Dcmuestfén quo en el triángulo arbit rario ABC el punto fJf de intersección do los medianas; el punto JI de intersección de las al.turas y ol centro O
de la circunferencia. circunS(;rita, pertenecen a una. misma recta (recta de Euler)
0111
1
Y que MJI
2·
392. Se da el ángulo ABC y en su interior ol punto Jlf. Tracen por éste una
recta do forma que Sil segmento, contenido dentro del ángulo ABC , se divida
por el pun to M en la razón 1 : 2.
3lM. 'Pór el punto de tangencia M dP. las circunlcrenc ias c.> y (1) 1 se han lraMdo las secantes h y l que cortan la clrcunlcrencin (1), además del punto M,
en Jos puntos A y B y la circunferencia (1) 1 en los puntos C y D. Demuestren
que lns rectas AB y CD son para lelas.
394. Demuestren que si por el p unto de tangencia do dos circun lerencias
se traza un recta arbilraria, ella corta las circunferencias por segunda vez en
tales puntos que los radios 1.ra1.a<los desdo olios son paralelos.
395. $e dnn tres segmentos paralelos MN, PQ y RS, diferentes a pares.
con La par.ticularidad de que los rayos MN, PQ y RS son codirigidos. Demuestren
·que 1'1:Hrcs puntos de intersección de los pares de rectas MP y NQ, MR y NS ,
.P R y QS pe~t.cnccen a una roo ta; los puntos ele intersceóión de los paros de rectas MQ y NP, QR y PS, MP y NS también pertenecen a una misma recta
=
(fig. 81).
Capitulo T. Pla11tmetri11
396. Las circunforcncias e~, y ro 1 llenen tAngcncia interior en el punto A
(fig. 82). La secanto a corta ro, en los puntos /11 y N y w2 en los puntos P y Q.
Ocm11estren que LPA Jlf
LQA N.
=
p
Q
""
Flg. 82
Flg. 81
397. Las longitudes <le los segmentos, uno de cuyos C:'ltremos es un punto
común y el otro, el punto de una recta, cst.án divididas en una misma raión.
Dcmuestrnu que Jos puntos de división pertcocceu a una línea.
§ 6. VECTORES
Suma de vectores. I\ecibe el nombre de suma de los vectores a
t b2 el vector
con coordonnd as a, + bl t
ª2
bz, es decir,
(al> a2}
b (b1' bz}
(a¡
b¡, ª2
b2).
Para cualesquiera vectores a(au a2). b (b¡, b2). c-(c¡, C2)
¡; =
e
a
+
=e +
+
a+
=-¡; + ª· ;; + (ii + c; = (a + i>> + c.
Regla del triángulo. Para cualesquiera tres puntos A, B y C es
y
b con coordenadasª•' ª2 y b¡
+
válida la igualdad AB + BC=AG.
Regla deL ,para.lelogramo. Si ABCD es un paralelogramo, AB
+
+AD =AC..
Multiplfca~ióri 11!r un v.ector por un número. Recil¡e el nombre
!le prodt1cto ~ril vector (~j por el número A. ol vector (Xa 1 A-<t2 ),
es decir (a 1, a2)>.. = (/...a. 1, Aa 2 ) . Poi' definición, (a. 11 a 2}}.= /... (a 1, a 2).
Para cualhuier vcetor·
los· números A. y µ, (A.+¡.¡.)
=>..a+µa. '
a )'
P.ara cuale,s quiera vectores
+A.b; ¡>..¡l=*l"I 1a 1.
ii
a=
y
b .Y números /... /...(a+ b) =/...a+
a
El sentido del vector· ·Ali, con =FO, coincide con el .sentido
del vector ~i A> O y es opuesto al sentido del vector si /...<O.
a
a
§ 6. Vectores
77
Dos vectores no nulos Hevan el nomb re de coli~f.ales si ellos
yacen en .. una misma rec't a o en las rectas paralelas.
Si a(a 11 a 2 ) llb (b., b2 ) , 04 ' =
También es cierta la afirma-
'
~ión
.
i1-.
ipycrsa.
,
Un vector se denomina unitario si su valor absol u(o e.s igual a la
u nidad . .Los vectores unitarios q ue t i.ep.en el sen ti do de los semiejes
positivos de co.~rc!,enadas,_Tl!Cib~n el ·nombre de !ector~s coordenados
o versores y se designan e1 (1, 0) en el eje Ox y. .e2 (0 1 t.) e n eLeje Oy.
Todo vector ii (a1 , a 2 ) to lera su representación en ~a: forma li =
=
a;e¡
+ ª2e;.
Producto escalar de vectores. Recibe el nombre de producto escalar
+
ª
de los vectores a(al> a,) y b(b., b2) el número a1b1
a2b2.
= a:2, ¡:¡2 = 1 ¡2.
P nrn cunl<'.squiorn tres vectores a(at, a:), b(b 1 , b 2 ) y (e, , e~).
(a b) = ac 5c-:
Se llama ángulo entre los vectores no nulos AJJ y AC el ángulo IJAC.
Recibe el nornb.r e de á ngulo en tre los vectores y ¡;el ángulo entro
vectore11 iguales a ellos con origen com.ún. El ángulo entre vecto res
de un mismo s.1.m.~ido se considera nulo.
El producto escalar de vectores es igual al producto de sus valores
absolutos por el coserJ,o del ángulo enLre ellos.
Si a.Lb, a·b =O y si ;¡.¡; =0, donde a ~O. b ~o. a .1. b.
El aparato del á lgebra vectorial ha permitido crear \tn método
singular para resolver diversos problemas geométr icos. No obstante,
hay que tener en cuenta que dicho método no es un iversal, para
resolver .ciertos problemas no puede ser empleado o es poco eficaz.
En l a siguieQte tabl a se ad·ucen ejemplos do empico del lenguaje
vectorial para enunciar y demostrar algunas afirmaciones geométricas o bien para calcular magnitudes geométricas.
Las aplicaciones concretas en los tres primeros casos pueden ser
observadas por el lector al considerar los problemas afines y en los
últimos tres, al examinar problemas métricos.
a·a
+ e
e
+
a
l. P roblemas a fines
Destaq\1emos varios tipos de pro blemas a(ines que es conveniente
resol ver empleando vectores. (Sólo se t rata de aquellos problemas,
en cuyo texto no hay ningún concepto de álgebra vectorial.)
Al primer tipo se refieren los problemas ligados con la demostración del paralelismo de ciertos segmentos y rectas. En tos problemas
·de este tipo, para resolverlos, es preciso demostrar la colincalidad
de lo~ vectores representados por ciertos segmentos <latlos, es tlecir,
Capitulo 1. Plattlmetria
i8
Qut hay que
d~mostrar (en
geom~trlco)
lenguaje
1
QuC es surtclcntc demostrar (en lengunJe
·
vectorial)
AB= kCJ), donde los segmentos AB
y CD pertenecen a las rectas a y b, res-
1) a 11 b
pectivamente, k es un níimero. En iw1ción
de lo. elección do AB y CD surgen dh•ersas
relaciones ''lliltoriales Mtrc las que S<l
cllgcn los convcnlcntcs.
2) Los p.u ntos A , B y C pertenecen a Ja rllilta a
a) cstab lcccr In justeza de una de las
siguientes igualdades: AB= kBC o AC=
=kBC, o bien AC- kAB;
h) demostrar Ja igualdad QC= pQA+
qQB, donde p + q = 1 y Q es un punto
nrbilrari.o;
e) clcmoslrnr Ja iguahlutl a.OA-1+ ~0B+ vAc~o. donde a +~ + -v=O y
Q es un punto nrhilrario
+
3) El punto C pertenece al
segmento AB, donde AC: AB=
= m.: n (división del segmento
en la raión aadn)
AC=m/nCB
nt
+ m+,.
o
QC= -"-QA+
m + 11
-
QB pnra cierto punto Q.
AB ·CD= O, donde. los puntos A y B
perlene1:en a la recta a y los puntos
y ]) ,. a la recta b.
'•l a J.. b
c
1
5) Có.lcuJo de la longitud de
un segmento
a) se eligen dos vectores no colineales
báslcos (<> bien tres no complanare:i). en
los que son conocidas !ns longitudes y el
ángulo entre ellos;
b) por ellos se clesc<>mponc el yeetor,
cuya longitud se calcula;
e) re halln el cund ratlo escalar de
e~te :vector cmpl~ndo la fórmula
a•=
= 141 2 •
6) C,álcu1o
ángulo
del
valor: del
a) se· eligen dos vectores no colincoles
básicos para los que se conocen In ra?.ón
' de las 1<1ngitudcs y los ángulos entre
el Jos;
b) se eligen los ve<:torcs que prefijan
e\ ángulo bu5cado y se cicscomponcn por
los vectorca básicos:
- -
4-~
1" 1 1b1
c)' se calcula cos L.. (a, b) = - - ---- .
§ 6. Vectores
7!)
a=
demostrar que
kb, donde k es cierto número. Ex:~minemos la
resolución de los problemas del primer tipo en los siguien:tes ejemplos.
EJEMPLO 1. En el plono están dados el cuadriláterq, ABCD y el
puntó M. Demostremos que los puntos simétricos a Af,con relación
a los punto& medios de los lados del cu.adrilútero son los vértices de
un paralologramo.
.
'
soLucróN. Sea ABCD el cuadrilátero dado (fig. 83), N, P, Q y
R , puntos simétricos a M con relación o los segme~tos AB, BC,
CD y DA.y
Según la regla del paralelogramo, tenemos: MN = ;MA + MlJ,
MP = MB+Mc,MQ = MC +MD , MR =MD -f MA .
·' '
e
B~{
R
¡>
IJ
Flg. 83
A
~
Por definición la direroncia de los vectores NR
y PQ =MQ - MJ'.
Como NR - PQ = (MR -
O
Fl¡¡. 84
= MR
- MN
MN) - {MQ - MP), haciendo uso
de las igualdades iniciales nos cercioramos do que N R - PQ =O, es
dec.lr, N R = PQ. Por analogía, se demuestra que N P = RQ. Por
consiguiente, N R = [flf y N P = RQ, lo que s ignifica que el c uadrilátero NPQR es un paralelogramo.
EJEMPLO 2. Se da el cuadrilátero ABCD. La recta trazada por el
vértice A paralela al lado BC cor ta la diagonal BD en el punto M.
mien t ras que la recta trazada por el vértice B paralela a1 lodo AD
corta la dingonal AC en ol punto N. Demuestren que MN 11 DC.
SOLUclóN. Para resolver el proble ma es suficiente demostrar la
colinealidad de los vectores DC y MN (fig. 84), o sea, hay q11c demost rar que DC = kMN, do nde k es cierto número. Para cerciorarse
de Ja colinealidad· do los vectores DC y MN expresemos cada uno de
eJlos con ayuda de otros vectores. Así, el vector DC se expresa con los
vectores OC y OD y el ,;eclor MN, con los vectores OMy ON, donde
Capitulo J. Planimetria
80
O es el punto de intersección de las rectas AC y DD. A continuación,
los vectores OC y ON pueden ser expresados con el vector AO, los
vectores OD y OM, co n el vector /30. Supongamos que
(1)
AO : OC = p : q y BO : OD = m : n.
Entonces podemos expresar el vector DC con AO y JJO median te
las sustituciones sucesivas: DC = OC-OD =.!LAO-..!:.. BO =
1
-
-
m
P
=-;;p(mqAO-npBO).
Por otro lado, del paralelismo de los segmentos BE y AD, se
desprende que
AO: ON =DO: OB = n: m.
(2)
En toncos, dll acuerdo con la figura y de la igualdad (2), se
desprende que ON = .!!!.. AO. Por analogía, del paralelismo do los
segmontos AM y BC" se deduce c¡ue BO: OM = CO: AO = q: p y
OM = .!!.. BO. En tal caso, es posiblo cxpresor el vector MN con
-
'l_ .
-
-
-
AO y BO :rnedinnto las sustituciones sucesivas: MN = ON -OM =
=E.BO+.~ AO=_..!.. ( - npBO +mqAO). Do donde, DC=.!3!1...MN,
q
· "
nq
mp
lo q\lc, nl pasar ni lenguaje geométrico, significa el paralelismo
de los segmentos MN y DC. ·
Al segundo ttp·o se refieren los p1'oblemas relacionados con la demostración de que el. µ.unto dado divide un segmento en una determinada razón .(en particular, es su puuto medio).
divide el segmento !lB
Con el fin -~e demostrar que punto
en ciorta rnzón AC :. CB = m: n, es Sltficicn t.e: a) demostrar la igual~
- · m.
n
dad A_C "':"
CB o bion b.) demostrar la fg ualdaq QC = n + m QA+
e
n
+
m
.,¡__
•
n-:1-m QB,. donde
Q es un punto arbitrario.
La derfiostración de la sQf.iciencia de 1o. condición ox puosta en
el punto fo cs. sencilla. Soa QC=--7---QA·+-;"'
QB, entonces
m
m -:-.-71
-n
QA+J....QB,.
_L(QC-QA)
=
( ..,!._+.!..~
m
n ·) QC= .l...
tn
n.
·.
m
1 Te
i·icn
1
.
'[.!
A
..
e
CB
m
m ="jit.
o que s1gnt ca qui)
: . =n-·
...!...(QB-QC),
n..
1
1
•
Seiíaleinos, además-_. qu·o al :realizar 113 demostración en el orden
in:vcrso" ~odemos asimismo cerciorarnqs. en la necesidad de l a_condición b) ,para dividir el se·g mento AB ·con el punto C en la razón
m: n.
.
§ 6. l'ectore•
lit
Resolvamos v.arios problemas del segundo tipo.
s,1EMPLO a. El seg01ónto que une Jos puntos mediQs de las dingonales de un cuadrilátero arbiirario pasa. por el punto de intersección
de las líneas medias. Demostremos. que
e
el mencionado segmento se divide por la
mitad.
SOLUCIÓN. El becho de que
(fig. 85)
es ól pu'n to· medi:o del segmento EF puede
;l'f r--'-'7"4°'l1CT""~--\
ser demostrado de distinta forma .
P.ej.:
1) demostrar que EP = QF, lo que sig11
n ifica que El'.,FQ es un ¡)aralelogramo y
f)
como EF es su diag·onnl, ella pasa por el
punto O y éste la divide por In mitad;
Fil(. 85
2) demostrar que EO = OF;
o
-t-
-t--
-
3) d'e mostrar que Q0= 2 (QE+ QF) o bien NO .= -¡;(NIZ+ NF);
-t --1-4) demostrar que C0= (CE+CF) o bien D0= 2 (DE+DF).
2
Consideremos el primer ¡iroccdimicnto de In dcmostr.nciún que,
en el caso dado, es el más sencillo.
En el triángulo ABC el segmento EP es la línea media, de donde
EP =~AH. En el triúngulo ABD el segmento QF es la línen media ,
-
1-
-
-
de donde QF =
AB. Esto quiere deci r que EP = QF. El proble2
ma está resucito.
EJEMI'LO '· En el paralelogramo ABCD el lado AD está dividido
en n. partes iguales y el primer punto de división (el K) se uno con el
vértice B (fig. 86). Hallemos en qtió partes divido la semirrecta B K
la diagonal AC.
SOLUClON. Sea DC = b, DA=
y AP = aAC. Expresemos el vector
AP con los vectores
y b, haciendo uso da dos procedimientos:
1f) AP=aAC=a(b-a) = ab-aa; 2) AP=A[(+KP = -ña+
a
a
+aKB= _..!._a+a I ..!._a+&)= .:!:..:=2.a+ab(l<P= a;J\.B,
11.
=
\ n
1t
va que
•
el t::. APK
t::. BPC).
Como :;ólo es posiblo la única representación del vector mediante
dos
.
v~ct.ores
· 1es, -n
o:-t
1
no eohnc11
- = - a, de dorua
a=,.+1 1 . E•sto
significa quo AP =-· n~-i .iiC y, entonces, como es f;íc.i l ccrciorar$e,
AP:PC=1 :n.
Capitulo l. Plar1im..1ria
82
E.IENJll,O 5. En el lado A e del triángulo ABC se ha tomado el
p11nt.o i l:f ele forma que AM = ~AC y en la continuación del lado
BC, tal punto N que BN = CB. Hallemos en qué razón divide los
segmentos AB y 11-IN el punto P de su intersección.
s0Luc:10N (fig. 87). Supongamos que
NP: PM =a:~. AP: PB =y: 6.
(1)
Así, pues, es preciso hallar a : j3 y v : 6. Con este objeto, confeccionemos varias ecuaciones que contengan a, ¡3, y y 6.
N
e
B
0
J.: ª
A
e
M
A
O
Fig. 87
Fig. 81!
Dos ecuaciones semejantes se pueden escribir empleando el teorema acerca de la división de un segmento en la razón dada ;
Sea Q un punto arbitrari o en el plano, entonces para los segmentos AB y MN, teµemos:
-
fl
-
a.
-
QP = a:+¡rQN+ a+~ QM,
(2)
6 '\' QP=v+6 QA+ v+ó QB.
(3)
Las igualdades (2) y (3) que hemos escrito contienen cinco vectores diferentes. Disminuyamos s\1 cantidad sustituyendo los vectores que !tenemos p.or otr.os', haciendo de nuevo uso del teorema
acerca de la. división de un segmento en la .raz·ón ·dada. ·Para los segt ·n1entos NC y AQ, tendremos: Q!l = -:¡(QN + QC),
·-
2 -
4
1-
Q1lf=3'QA+3QC.
()
Poniendo en las igual<lades (2) y (3) los valores de QB y QM de
(4). obtenemos:
li y
y
(5
QP="i.+6QA+2(y,+li)QN+z(y-li)QC,
)
.Q P
~
3(«+1l> Q.A+
ll
-
a.
-
a+~ QN + 3(a+~> QC.
(6
)
§ 6. Vutores
83
De aquí, tenemos:
,,
6
2(v+~l= cx+~
6
2a
v+6 =
y
2
Ja+~
'
'
a;
<-v+6> = 3 <a:+í\>
Resolviendo esto sistema de ccuacionr.s, obl.Cnc{no.': I' = o
1
~=
a.. Asi, pues, AP=BP y NP:PM=3:1.
y
3
Analicemos los problemas del tercer tipo. Refiramos n esto tipo
ta-les problemas en los quo se requiere demostrar la pertenencia de
tres puntos a una misma recta. Est os problemas ,podrían ser considerados como un
D
caso particular del segundo tipo. No obstan~e. en su rCs.olución hny cierto c11rácter específico debido al empico de la coutlició n de pel'teñeúcin do lrns pil ntos n unn
misma rect a.
EJEMPLO o. En el Indo AD del triángulo
ABC se da el punto P por el quo se han
trazado rectas parolel11s e sus medianas A
e
AM1 y BM 2 y que cortan los correspondientes lados del triángn lo en los puntos
Flg. IJIJ
A 1 y 8 1 • Demostremos que E es al punto
me(!io del segmento A 1 8 1 , así como que el pun to P y el punto G
de intersección de las medianas del t ri ángulo dado yacen en una
misma recta.
SOL1JcION'. Variemos la conclusión del problema de forma c¡ue,
pa ra :su r esolución podam.osemplear vectores. Esto es posible ele realizar c:on !os siguientes proctidimientos (fig. 88):
i ) Estobleié~~os que EP = kGP (se puedc11 tomar tnmbién
ot ros vectores).
2) Para ciQrto punto Q establezcamos quo QE = kQP (1. -k) Y.
X QC (condición de pertenencia do t res puntos a 01111 recta).
La primera vía de resolución ya es conocida de los ejemplos,
aducidos más arriba, para resolver los problemllS del primer tipo.
Analicemos la segunda vía. Con este fin , deduzcamos, primen1mente, la condición de pert enencia do tres puntos a una recta.
Para que los puntos A, B y C pertenezcan a uno recta es necesario y suficiente que para cierto punto Q se cumpla la igualdad QC =
= pQA
qQB, donde p
q = 1.
lJl!.MOSTll.ACION. Sen que los puntos A, B y e pertenecen a una
mism:1 rcc~11. En~onccs es posible escribir: AC: CB = m: n. Esto
+
+
+
Capítulo [. Planimetría
manifiesta lu cert eza de las siguientes ig ualdades: AC : CB =
= m : n , de donde QC = ~ QA
+n QB, QC = pQA
+m
m-rn
+ qQB, p + q =
+
n
1.
Los razonamientos realizados aqui demuestran la necesidad y la
suficiencia de la condición.
La resolución de este problema se reduce, por decirlo así, a establecer la dependencia entre los vectores QP, QE y QG. Si el punto Q
se toma al azar, la resolución del problema será muy complicada.
Es mejor tomarlo de forma que coincida con el punto C. Con ello,
los vectores CP, CE, CG son.expresados con facilidad mediante CA
y CB. En efecto, sea
A P : PB
( 1)
= rn : n.
Entonces
AB1 : B 1C
= m:
(m
+ n + n) =
m: (2n
+ m)
(2)
(ya que M 2 es el punto medio de AC) y
BA 1 : A 1C = n: (m+ m+n)=n: (2m+n)
(3)
(ya que M 1 es el punto modio de BC). De Ja propiedad de l centro
-
21
-
. -
i-
-
_22¡~~:)
CA 1,
de gravedad G se desprende: CG=rz(CA+CB)=a-(CA+CB).
De las relaciones (2) y (3) es posible escribir: CB 1 =
CA
2m+"
{m+n) CB
1= 2
1• Entonces, partiend9 de la propiedad del punto
medio E del segmento A,8 1, podemos escribir:
CE =j_
(CB 1 +CA 1) =.!.(~
CA+~CB).
2
4
m+n
111 + n
Según .el teorema acerca de la división de un segmento on Ja
razón dada, tene.m os: CP= ,,.¡,,CA+ m'.'.;.n. CB.
Col\ el, fin de en~awr .Jos vectores CG-,
el
vcc~of: CE:
+-+n
CA+
m. tt 1
CE=.f
(~,-¡~' CA +
CE y CP,
:.t:
2
transformomos
CB) =i-{ CA+CB+
+"' CB)=~(3CG+CP)
= !.
m· · n
-.
4 CG+~CP, es
'J
decir
CE.=-¡;C<:;+-¡;CP., perocomo 4 +4 =1 , los puntos E. G y P
pertenece!\ ~ o una misma reeta y ~G: PE= 1 : 3. El problema queda
-
3-
t~
3
i
resuelto.
Los tipos considerado~ de problemas afines en el plano no agotan
la gran diver~idad de éstos. 'Pero ellos forman los grupos más numerosos de probremas, lo qúe justifica su especial examen.
§ 6. Vectores
85
JI. Problemas métricos
Al resolver Jos problemas métricos se hace uso del producto escalar de 'los.. vecto·res. Sin ciasmcarlos en tipos, vamos a aar vnrios
ejemplos.
·
.
EJElllPLO 7. En la ba~e AB qel triúngillo isósccies ABC se da el
2
.punto .P. a) Demostremos que PC = AC 2 - AP-BP .. b) Aclaremgs
cómo varía esta fórmula si el punto P yace en la cont~nuación de la
base AB.
sóLVc10N. a) Escriba-mos l a iguald ad a demostrar CI'.\ forma vectori,a l. Tomando en consideración el sentido rlc IOs ·véctores AP .Y PB
(fig. 89), obtenemos: P~ = AC2 - AP· BP. Demostremos esta
igualdad. T ransformemos su segundo miembro de In forma si¡.plicnlc:
AC2 -
AP·BP = AC2
-
(AC
- AC· PC-· íiC-cB + ·CP
2
-
+
CP)·(PC
+ CJJ) =
= cAc~
"CP.ca
AC• -
- XC.Pe) -
-~ c .~ +~:~+~ =E0c-~-~0c+
2
+ CP) + CP' = (AC + CP)-(AC - CB) + epi =
-CB) + CP
2
AP (AC
-
•
Si ahora el vector CB' = AC. AC- CIJ = eB' - CB= BIJ'.
El trÜngulo .AB'Bes rectángulo y, por lo tanto, AP (AC - ClJ) =
""' kP·.B]3·.= O. De forma qu!lAC2 - AP· PB = CP 2 •
:)J.) Si el pun to P pertenecc :al segmento AB, al pasar de la igual.d.a d· véct9t.ial a la escalar, tendremos: PC2 = l PC 12 = Pe 2 , AC2 =
..: 1A C t~ == AC2 , AP.· Pl) = I AP 1· I PB I ·cos L (AP, PB) =
= AP·PB·cosóº = AP·.PB, es decir, PC!"= AC 2 -A P-Pll.
Pero sí el punto P no pertenece al segmento AB, los vec tores AP
y PB tienen sentido opuestoyAP · PB = 1 AP !· I PB ¡. cos180º =
= -AP · PB. Así, pues, en este caso, la igualdad buscada tiene lá.
forma: PC~ = AC'
AP·PB.
.EJEMPLO s. Hal.Jemos la suma de l os cuadrados de las medianas
de un triÍíngulo si conocemos sus lados a, b y c.
SOLUCJON. Sea que en el t::. ABC AB
BC=
CA= b (!ig. 92l·
+
=e,
a,
-
Entonces, por definición, la suma de los vectores AD =
-
-
b
~
-
;
BE=a+ 2 , CF = b+2·
-
e+ 2:a ,
Capl1111o l . Planlm1trla
86
Utilizando la propiedad del cuadrado eselllar, obtenemos:
(-c+ á )2 + (~)2 =
AD2+BE2+CF2=
a+ 2¡; )2 + (b+ -¡
2
-
- -
;;i
- -
b•
-
- -
(t
=c2 +c·a+ 4 +a·b+4+b?+b·c+4=
-- -- -= "f5 (ai+ b2+ c2) + (c·a+
a·b+ b-c).
a+b+c=O, (a+b'+c)2= 0. Así, pue~. a2+1i2+c2+
+2(c·a+a·b+b·c)= 0, es decir, a2+b2+c2 = - 2(c·a+a·b+
+F·c).
Como
- -
- -
--
De mnncrn que c·a+a·b+b·c = -
o•+v•+c• •
2
Pouiondo el VAior llallado de In CX!ll'()!;ÍÓll
llClllOS: AD2 +BE2+CF1-=
f (a2+if!+ci) , ya
c·a+a·b+b-c,
ohlu-
que de acuerdo con
la propiedad del cuadrado cscalnr AD2 = ADt, BE'l = BE'l, CF2 = cF:..
&IEMPLO D. Demostremos q110 la~ alturas de un triángulo arbilrario se corta n en un solo punto.
6
e
A
B
p
Flg. 89
J~\C
B
F lg. 90
B
A$ C
Ftg. 91
SOLUCIO N. Sea-n AP. BQ y CL Jasnlturnsdol t::.ABC y O el punto
de intersección de fas a.lt,ura~P y_BQ (fig. ~1)._ _
Para abreviar, hagamos OA = a, OiJ = b, OC = c.
Por definición
de la diferencia do vectores A FJ = b - i, BC =
_._ = e - b, C.A = a - c.
Como, sciguidamento, OA ..L. BC, a (é - b) = O, es decir,
__
-a.<: .... a.."b.
Por anal'ogía, como OJJ ..L. CA,
¡;.-¡,
=
b (a - C)
;;.e:
( 1)
= O, de donde
(2)
§ 6. l'eclort!
87
~.!_as
i!u!Jdades {!)_Y (2).'.... d·ebido a la transitividod. se deduco
b·c· o bien e (a - b) = O.
Esto último significa que OC ..J.. AB o bion que CO ..J.. AB. Como
por el punto dado pasa un·a so1a recta perpendicular a ta d¡i.d a. y, cn·to,nces, {il!1 hecho gúe (:0 ..J.. AB .y CL ..J.. AB se desprende que CL
coincide con CO. Así, pues, lns tres· nltu ras del triángulo se cortnn
en un punto.
que a·c
e::;
PROBLEMAS PARA EL TRABAJO INDIVIDUAL
J. Adicl6n y switraeclón de vectores. lllultlpUcact6n
de un sector por un número
398. Para que el cuadrilátero ABCD sea un pnrnlelograrno ca necesario
QC y suCiclento ·qua para c11alquior punle> Q so verifil(UO In igua ldad (JA
QB·
QD.. Demuestren esto.
399. .En .el cuadrilátero A BCD, M, N, l' y Q son los corl'('!!JlOndir.nLos puntos medios de Jos lados sucesivos. Demucst1-en c¡ue ol cuadrilatcro :lf N l'Q es
un paralelogramo. ..,.
400. En el lado AB del cuadrilátero A BCD 8-0 ha eonsLruido el parnlcJngramo ABCC' y tomado ol punto O que es el pun to medio del segmento C'IJ .
Demuestren que si M y N son l os yu.ntos medios de los Indos AB y CD, rospcellvamente, el s egmento A O es igua y paralelo a l segmento /\! N'.
401. En el cuadrilátero ADCD M y N son l os pu.otos mcélios de l os l"dos
AD y BC, respectivamente. Demuestren que 2MN .s;; AB +CD .
402. Demuostren que In longitud del SllgmonLo que uno los puntos 1nedio~
do las dia~onales de un trarceio es igual a la semldiíorcnci11 dn sus boscs.
403. Si lh longitud de segmento que uno los 11u11tos me(lios tl o ({Is ln<lo~
opuestos de un cuadril átero convoxo es igual a In semisuma do los otros tlos
lados, este cuadrilátero es un trapecio o un paralelogramo. Dcmuéstrcn lo.
404 . ..ftf y N son los punt os medios de los Indos AD y CD, respcctivntnenlo.
1IPI cuadril6tero A BCD. Demuestren quo los JlUDl t>S rnotlios de las dingonales
do los cuadrfüiterosA.MND y DMNC son los vert iccs efe 110 parl!lelogrnmo (o quo
yacen en una misma rectal .
405. Con el fin de que el punto Q sea cJ conuu de gravedad del triángulo
ABC qs necesario~ y' sulicionto que QA
.QB
QC =O. Dcmuéstrcnlu.
406. · o~l- punto M , situado dentro del ·triángulo ADC, -sc .tra7.an pcrpcmlicularea a J0.s ·Ja dos B.C, A C, A.B y en oll"os se m11rcon los s1:gmentos l•fA 1 , /11
y MC1 .iguales a los eorre~pond.l<!ntcs lados del trl8ngulo. Demuestren quo e
punto Mes el cent.ro do gravedad del triángulo A 1 C18 1 •
407. Demuestren quo en un cuadr-llátcro orbilrario, el segmente> q uo UM
los puntos medios de fas diagonales pasa por el punto de intersección do las
lineas modios y se divide con ól por Ja milad.
408. EsLán dados el t riángulo A BC y el punto Q tomado al aZ<1r. Demuestren guo si construimos los pnrnlelogramos Q.BJJ¡C y QAA 1B., Ja difl¡¡onnl QA 1
dol último pasa por el centro de gravedad O de triángulo dado y qur. QA 1 -
=
+
+
+
+
/Jl
= 3QO.
409. Demuestren que la recta que pasa 11or el vértice A del triángulo A BC
y el pup.lo medio de la m otl iano BD divide el lnci o BC ~n la razón 1 : 2 .
410, Se dan dos segmentos iguales AB y A 1B1 • ¿Cuñl debe s·o r el ñng11lo
entre las rectas, a las que pertenecen dichos scgrn~ntos, pam que In dlstnncin
entro los puntos medios de los segmentos AA 1 y 88 1 sea igual a -}AB?
CapEtulo J. P lanimetrfa
88
411. Dcnt~o de un paralelogramo se ha lomado el p\10L-O M y por .íl, so
t.raznn rectas pnraldas a los lados de la figurn. Ellas corlan Jos lados del paralelogramo en los puntos A, C y /J, D. Dcm11oslten que el pu ni.o de intersecc ión
ilc !ns líneas medias del cuadrilátero ABCD es el punto medio del segmento OM,
donde O e~ el centro del pamlelogramo dado.
412. Demuestren que In rectl\ que une los puntos medios de las bases do
1111 trapecio, pasa por el punto de intersección de las continuaciones de los lados
latcrnlcs y por el punto de intersección de las diagon ales.
413. Está daclo el trapecio A BCD. La recta paralela a las bases AB y CD
cortn los lados laterales AD y BC en los puntos M y N , respectivamente. Dcinucstr~11 que si AN 11 CM, DN 11 BM.
414. ${' da el trapecio ABCD en el cual AB y CD son las bases y .11-r y N.
los puntos med ios rle sus lados AD y OC. Demuestren que In recta AN no t:S
pnral~a a la rectn C /11.
4 15. En In circunferencia con centro en O, se han lrnzadodoscucrdas: Jo~
puntos perpend iculares AB y CD que concurren en el punto M. Demostrar que
medios dl' las cucrtln~ AC y BD, el punto M y el centro de la circunferencia
dada son lo.5 vérLiccs de un paralelogramo.
li16. ~;n el tríúngulo ABC se ha t.razado la mctlí:iua CC,. llcmucstren
1
1¡11c CC1 <z(CA+CE).
417.
J>sf,¡i dado el
triáng11lo ABC . l)omucstrc11 <1ne OM
1
<a
(OA+
+OD+OC). donde M es el punto de ínt.erseccíín1 de las medianas del triángu lo. O, un punt.o arbitrario en el plnno.
418. Dos paralel(1gramos ABCD y AB 1C 1D 1 tienen un \•érticc común A.
Demuestren que CC 1 ,,¡;;: BD¡
DD 1 •
4l!l. Se <lñn dos paralelogramos ABCD y A 1B 1C 1D 1 • Demuestren qn c, on
el c11 s,o general , los puntos medios de los segmentos AA, , BB 1 , CC , y DDi.. son
los vcrtices del paralelogramo A ~ 0C 0 D 0 • Construyan dos semejantes parnl el ogramos de modo. quo Jos puntos il o.• B 0 , (:• y D 0 coincidan o pertenezcan a una
recia .
420. Está dado el pa ralelog ramo ABCD. Los puntos P, Q, R y S dividen
Jos lados AJ), BC, CD y DA en iguales razones. Demuestren que el cuadrilátero
PQRS es un paralelogramo.
<\21. SQnn dados dos triángulos ADC y A 1B, C,. Demuestren que si las medianas del primer triángulo son paralelas n los l ados del segundo, las medianas
de é,~te son parillelas a los lados del primero.
422. Sea dado el cundrilát.ero A BCD. Se ha construid o el segundo cuadrílá lero con los ''érliccs on los .puntos de intersección d·e l.ns medianas dtt los triángulos BCD.,.CD'.A, DA,B y ,(pe. Dem.uostren qlle las lín<>as ine·d ias de los cuadriláteros concurren en un mismó punto.
423. Ep uii plano se dan cuatro rectas, do. las cualo~. ni una do tn¡s de ellas
pasa p0 r u-q p1into y ningu 0 as dos ~on p¡irale,las. Ocinuest;en que si una de las
cuatro ·recuis e~ paralela ll 'lo. median(! de un ·triángul o., rlefmido por las t res r estant es, cada una de las demás tros rccta"s poseen propi.cd¡ides análogas.
424. Por lbs vértices A , B y C d él triángulo A BC S!) hnn trntndo las rectas 1,
m· y il., rcspectiyamente, que se cortan cri el punto S. Demuostren que las rectas
l ¡. 111 1 y n 1 que. pasan, respectivamente , por los puntos medios A 0 , B~ y C0 de
los lados BC, c A Y· A B paralelas n las líneas t;· m y '!• Lambién se cortan en un
mlSIJ\O. punto.·
~25 . Sean! dados e l triángulo A·BC y el punt,o llf, los puntos medios A1 ,
B 1 y C1 de sus }ndos B C, CA y AJJ. Por.A, B y C se han trntndo rec~s paralelas
.n las re!:fns M..t 1, M B 1 y A C 11 respcctlvamcnlc. Demuestren que chchas rectas
conc\lrrcn en un punto.
+
§ 6. Vectores
89
426. D.cmuestren que sUa longitud do lll línea m.edia.Jl(N del cuadrilátero.
ABCD és ·igual' n la semisuma de las longitudes de sus lado¡¡ A B y CQ (¡lf pcrlcneco a BC y N , a DA), :A B OD es un tr~pccio o poralelogramo.
427·.. E.n !os lados Mi lriánl;\ulo ABC, fuera de él, cstáit const.ruidos lo$
triángulos•. equiláteros· A~C,, BA¡C y .<!ABl. Demuestren q\Jc los puntos de
i.ntcrseccion. do las llled 1anas de os. tr1angu os
8,
ABC.y A,B1Ci..·c·oinciden.
42.8. En 1a continuación de los alturas
AA 1. y BB 1 iicl triángulo ·:11·sc·, ~ras sus v~r­
tice~ A y" B., se J1an trazado los scg1hentos AA 1
y BIJ,, ctm la part.i cularidad de que AA,=
BC y 08 1 = AC. Demuestren que CA 2
CB.:
y quo ·cA. J. en,.
429. En los lados CA y CD del triángulo
ABC,_ fuera de él, se construyen los cuadrados
CAA 1C 1 y CBB ,e;. Domuestron que Ja mediana del triángulo
trazada por el vérFlg. 92
tice C es porpondlcular ni o.do AD () íguo.l n su
mita<l.
430. :Bn l()s lados del c11ndrilátcro ABCD, fuera de él, so constru)'Cn los
cuadrados ABB.1 A,, ncc;B; , CDD,C' y DAA ;D; con sus ccnlros P, Q, J¡ y S,
·rcspec~ivamcnfo. Deinucstren que Jos segmentos P R y QS son iguales y vcrp~n­
dicularcs.
431. Est,; dndo lli cundrilát.cro A BCD. Sus líneas medias concunen en el
punto Jlf. Se. ha lrazailo una qn~brada MÁ UY, donde Av=
üV = MC.
Demuestren que /\1 es el puntó medio ·del segmento VD. Hall:en In razón entre
el área del c:ua<lriláter<> ABCD y In del cuadrilitiero MA UV.
132. Demuestren que al cortarse lns medi:irins de un triángulo se dividen en
.la. razón 2 : 1.
433. Eu ol Indo AD y In diagonnl A C del pnrnlclogramo A BCD se han
=·
e,
cc,c·\
'MB,
=
lomado l9s puntos M y N do rorma que A M = -?;AD y AN
-¡fA C. Doinucstren que los puntos B, N. M yacen en una n1ismn recta. ¿En qué ra7.Ón dív icle
el punto N el segmento MB?
434. Demuestren que en el cuadril6te1·0 arhitrario ADCD Jos sngmentos.
c\1yos ex.tramos son l<>s puntos medios de los lados opu~tos (P, K, H. y L .ron
los puntos' medios do los lados AB, BC, CD , AD , resp<,ctivamenle) y el
segmento, cuyqs cxtromos son los puntos medios de las di11gona les (M eft el
punto mediu do BD). concurren en un punto y en 61 se dividen por la mit.~d.
435. Están dadasdosrcctas!\yl•ydospa1·.,s <le puntos A 1, A 2 y B 1 , B,.
Hnllcn, corrcspondienlemenw, r.n ns rectas tales puntos C 1 y C 2 que A 1C1 11 A .c•.
B 1C 1 11 B 2 C ••
436. En Jos lado$ dc.>l triángulo A BC est.fo cnns~ruidos los paralologramos
ADB,A 1 , BCC: 2 B 1 , ACC,A, (fi¡r. 92}. (.Es posible construir un t.r iángulo, cuyos
!al.los sea u ígual.-;s n los segmentos ll 1B., C ,C•· A ,A.?
u. Producto cscnl11r de
"Cclorcs
437. So dan dos la dos A B = a y CD = p del cunclrilátcro A BCD y el ángti.l o ~ entre estos lnd<>s. !fallen la longitud del segmento que une los puntos
medios de los ot1·os dos Indos del cuadrilátero.
438. En el triángulo ABC con lados AB = 5, IJC = 2 y AC = 4 cnlculen
el ánguJo rABC.
.
439. Demuestren quo lns nltmas 1fo 11n triángulo ohlu~ángulo se cortan.
4~0 . De1m1eslren que en un paralelogramo fa suma de los cuadrados do sus
d íngonnles es igual n la suma ele los c1rnd rados do sus lados.
90
Capitulo l. Plariim<lrfa
441. Demuestren que si ABCD ca un paralelogramo, para todo punto M
es válida la igualdad MA 2 +. MC 2
MB•
MD• .
442. Demuestren que el ángulo C dc,l triángulo A BC será ogudo, recto u
obtuso en función de si es la mediana CD mayor, igual o menor que }AB.
443. Dcmucstrl'n que en el triángulo A BC, con el centro de grave.dad en M
es válida l a relación ABª
BC•
AC1
3 (MA 1
MB 2
}11C').
444. Demuestren que si el centro .de gravedad del triángulo ABC coincide
con el punto do intersecci6n de las alturas, el triángulo es equiltl.tero.
445. Expresen cada mediana de un triángul9. con sus lados.
1¡4s, En un trapecio con diagonales perpendiculares éntrc sí, la base mayor
cs i~ual a 4 , la inenor, a 3. Hallen su lado lateral si sabemos que él constituye
un angulo de 60° con la base tnayor.
447. Se da el triángulo ABC en el que.AC
4, BC
3 y LABC = 120°.
Hallen la distnricla desde ol vér,Uce C hasta ol punto M quo divide el Indo AB
en Ja raión 1. : 3, contando desde e\ vérfice .A.
448. Demuestren que sí en el triángulo rectár\.gulo ABC desde el vértice
del ángulo r~to se traza la altura CD: a) CD' = AD ·BD; b) A e• = AB ·AD;
e) nci
BA · BD.
4"9. Las di'ag<inalcs de un trapecio rectángulo son pcrpcntliculnrcs cntfll si .
D11m\tQstr~n que' In altura del trapecio es la media proporcional entre sus bases.
4á0. Demuestren que en el trapecio ABCD. cuyas bases s<>n AB y CD,
se veriCica la desigualdad A e•+ BD 2 >AD'+ BC'
2A B ·DC.
451. Con el fin de que las dlago·nnJes de un cuadrilá,toro sean perpendiculares entre sí es nccesarir>·y suficiente que las.sumas de loscm¡drados de los lados
opu~stos de la figura sea n iguales. Domuéstre,nJo:
t,52. Demuestren quo si en un triáµgulo dos ·medianas son perpend iculares
entre sí, la suma de sus cuadrados es· igual al cuadrado de la tercera med iana.
453. Hallen la dcpcndcncfa entro los lados del triángulo ABC si sus medianas AA 1 y 88 1 'son perpendiculares.
454. Los c~lclos dd trián¡tulo ·rectángulo son iguales a a y b. Hallen la
biscctri?. trazad11 desde el vértice del ángulo recto .
455. Desd'e·el punto medio D d·e la base Al! ·del triángulo Isósceles ABC so
ha trazado la perpendicular DM al lado BC. N es el punto medio del segmento
MD. Demuestren que los segmentos Allf y CN son _perpendiculares.
456. Por el vértice del ángulo recto. C del triángulo A BC se ba trazado una
rectn .a la quu ilcsdo los vértices· A y 8 se trazaron las 'Perpendiculares AA 1
y 88 1 • El vértice C cst:í reflejado en el punto C' con relncion al punto medio M
=
+
+
+
+
+
=
=
=
=
+
n
del segmento ALB,. Demostremos que 4..AC,B ~ 2·
457. En el Jedo AB. deHriángulo A-BC, por diferentes lados de-la recta A B,
se ·bnn construido Jos triártRulps equiláteros A '.IJC¡ y Af!Cs· Hallen~\!' dependencia entre ·los la~qs .d'el· triangulo dado -si las rectas. CC 1 y ·cc 2 son perp'éndlcu·
lores (C :#e_,, C :F C2J.
TU. Diferentes problemas
.
458. Fuera: de un paralelogramo se han construido en sus lados cuadrados .
.Demuestren que los centros de éstos són los vertíces de uli cuadrado.
· '159. En los .lados AD .y B.C del triangulo ÁBC, fuera: de él, se han construido lo~ c;úadrado~ A BD'E y BC K'l!, Demuestren que el segmento l>'F es dos ve<:es
mayot: que la mediana BP del· triángulo ABC y perpendí.cular n ·olla.
460. En lo~lados AB .y Be 'del t.riángulo A'BC, fuera de él·, se han colistrui. do. lo'$. triángulos cqullé.t'.eros AB'C1 y B'C.A,. D'emuelit'ren quo el segmento que
une. los¡untos ~nedíos de los segmentos AB· y AAB1 ·es igual a la mitad del segmento · C y cons'i-ituye con él un ángulo do 60 .
§ 6. Ve<lorts
91·
461. En los Indos AB y BC. del. triángulo ABC, fuera de c!l; están construidos· los triángulos equiláte1·os ABC,,_ y ECA·i· Demuestren qµe si M, .N y 1!
son los puntos medios ·de Jos lados A c.;, C1B y BA,, respcctiv!lm~nte, el triái:igulo
MNP es ec¡uilátcro.
:
462. Las diagonales AC y BD del l,ra¡:¡ccío isósceles ABCD (AB ti CD)
concurren en el punto O .bajo un á ngulo de so•. Demuestren que )j>s puntos mcd[os,
do los segmentos OA, OD· y BC son lo.s v§.rticcs ile un triángt1lo ec¡uilá.tero.
4-03. En ·el trapecio ABCD In diagonal A C corta ,eJ. Lriá)lgulo equilátero
ACD. De~de el punto E en la diagonal AC (o d.c su contlnua~ión) la base BC
so v.o bajo un ángulo do 50•. Demuestren que los puntos medios .•d e los segmentos
AE, BC y CD son los vértices de un trilingul<> eq11üátero.
;
464. Si en d'o s lados de un paralelogramo .Q11e salen de µn mismo vértice
se const.t uyen (por íuo1·a 0 por dentro) triángulos rcgulorcs, el vé~Uc'e opuesto
del parnlelogramo y los vértices libres de los triángulos lorn¡an un triangulq
regula.r. Demuéstrenlo.
·
46". Sean dados dos triángulos equilátoros A,B1Cl y A 2 8,CJ orientados ·
en el mismo sentido. Los segmentos A LA,, 8 18 2 y c,c, están divi idos por los
puntos A, B y C en iguales razones desde los extremos A,, 8 1 ·y C1 , rcspcctivamo."te. Demuestron que el triángulo ABC es equilátero.
466. En. el t1·npecio rcctil!igulo ADCD con ángulo llg\Hlo de 1,5•, In din.!!o11al A C os )gua! al lad.o CD. Humos de demostrar q110 ()] punto .rncdio Je la l¡¡¡sc
rnenor es ,e q.u idis'tantc .del. .vértice A y del punto medio del lado CD .
467. En los segmentos AB y A e ~e ciert.a rrcla están cons.truídos los trián¡:ulos rectángulos isósceles ABC1 y A CB , (LCf = LB 1 = 90°) orientarlos eu
sentidos opuestos . .Dcm11estren que el punto mello del segmen~o BC y los ¡>untos 8 1 y C, son los vértices tle un triángulo rectáng11lo isóscqlcs.
468. En el triángulo ABC con ángulo B de 45•, se han trjlzado las alturas
CC, y AA¡ que concurren en el p11nto O. Vamos a demostrar que los puntos
medios de os segmentos B C, A 1C, y CD son los vértices de un trián.rsulo rectángulo isósceles.
469. En los lados AB y BC del triángulo ABC, foern de ~l. se han construí•
do los triángulos equiláteros A BC1 y BCA 1 con los centros en 0 1 y 0 2 , respectivamente. Demuestren ijue el segmento 0 1 0 , es <los veces más largo que el segmento que une los puntos medios de los segmentos 0 1C y C10 2 y que forma con ellos
un ángulo de oo•.
HO. En los lados AB, IJC v CD del rectñngulo /l B CD, {1,1ora <fo oíl, se han
construido los triángulos equiláteros AB01 , BCO~ y CD0 3 • Denlm•slrcn que
las distnncins entre los puntos 111edios de los segmentos AB, 0 10 2 y BC, 0 20 0
son iguales.
471. Puera !l-01 cuadrilútcro convexo ABCD, se han construido en sus lados
AB y CD ·los c11ndrados ABMN y CDKf,. Domucstron que los puntos modios
de las dingonales di? los cuaclrilátoros ABCD y MN KL son los vértices de un
cuadrado o coinciden.
472. Sean dados dos cuad.rados A 1 B 1C 1 D 1 y A 2 B,C2 D 2 orientados en el mismo sentido. Los segmentos A 1 A 2, 8 18 2 •
y D 1 D 2 están divididos p<1r los
puntos A 0 , 8 0 , C 0 y D 0 en unn misma rn7.ón, partiendo desde los vértices de
uno de ostos cuadrados. Demuestren que el c11adrilátcro A 0 8 0 Cof1 0 C!S un cuadrado.
~73. Sean dados dos polígonos regulares homónimos A 1As .. . A n y
E 18 2 ••• Bn de~ igual orientación. Los segmentos A ,A 2 • 8 1 8 2 , •• ., AnBn
están divididos por los puntos C 1 ~ C 2 , • • ., Cn, correspondientemente, en una
misma razón comeninndo desde 1os vértices de uno de los polígonos dados.
Demuestren que el pol}gono C
Cn es rcgul:ir.
474 . Fuera del triangulo ABC, se han co11stru1~0 en s11s Indos AB y BC
los triángulos rectiíngu.los ABD y BCE (LB = LC = 90") orient.ndos en un
mismo sonti<lo. Demuestren que los puntos medios de los segmentos AB , BC
y ve son los vértices de un tr iángulo rectángulo Isósceles.
c,c.
e, ...
92
Caplt.ulo 1. Plnni111etr"1
475. En el cuadrado ASCD el punto O es ~I centro. M y N son los puntos
medios de los segmentos BO y CD. Demueslren que el triángulo A M N .es isósceles y rectángulo.
476. En los lados del cuadrilátero ABCD, fuera de él, se han construido
los triángulos rectángulos Isósceles ABM, BCN, CDI' v DAQ (LM = LN =
= LP = LQ = 900). Demuestren que los puntos medii.s de los segmentol5 MP
y NQ y los puntos medios de las. diagonales del cuadrilátero son los vértices de
un cuadrado.
477. En triángulo rectángulo ABC, desde el vértice del ángulo recto, se
ba trazado la altura CD. Los puntos M y N div iden los lados AC y CB, respectivamente, en ratones iguales (par~iendo de los extremos .A y C). Demuestren
que el triáng\)lo DMN es semejante al prefijado.
478. En la base y en uno de los lados laterale.s de un tri&ngulo isósceles,
fuera de él, han sido construidos cuadrados. Dcmúestr~n que los centros de
éstos y el punto medio del o~ro lado lateral son los vér~ices de un triángulo rectángulo is6scoles.
·
479. En el, rectúngulo .,4 BCD sc ha trazado la perpendicular 8 K a la diagonnl A C. Los:puntos l\J y N 11ividen por lo. mitad los segmentos A K y CD,
rcspecti:vamcnl~. Domucstrcn que LBl!f.N = 90°.
480. Con relnción 11 lo hipolcnusa AB del triáng11lo rcctángulv ABC ~e
!in construido él triángulo ABC1 simétrico al primero. Si .M es el p11nlo medio
de la alt11ra C1D del triángulo ABC y N, el punto medio del lado BC, el triún gulo AMN es semejante ni 6ABC. Dcmucstrenlo.
·
~81. Sea dado ci¡araielogramo A BC. En las rectns A B y D C se han elede forma que los ·~riángulos KAB y llCB son isósceles
gido los puntos H y
(KA = AB y HC = CB). Demuestren qu() el triángulo KDH también lo es.
482. El cuadrilátero ABCD está girado en torno a cierto punto 0, que
yace en el plnn'o, a 900 a la posición A 1B¡C1D 1 • 'Demuestren que si.P, Q, R y S
son los puntos medios de los segmentos A1..B, B 1C, C1D ·y D'1A, respec\lvamente,
1os segmcntos, PR y QS son ¡,erpendicuJa,r~s e igúal~s.
483. Fuer!! de un cuadrilateru se. l)an con.s ttuido en sus Jo.dos c.u adrados.
Demuestren qüe sus cen~ros son los vérticc.s de ·un c\111drllátcro con diagonales
iguales y perpendiculares muluamchlc.
484. En el triángulo rectángulo A·BC, desde el vértice del ángulo recto, se
ha tratado la allura. CD y construido el punto Di aimétrico al punto D con relación al cateto 'AC. Demuestren que el punto A y los puntos medios de los segmentos D C y C!J son los vértices de un triángulo semejante al dado.
485. 'in el triángulo rectángulo ABC se ha construido el¡unto D,, sii:nétrico a cierto punto D del cateto BC, ~n relación a hipotenusa B; E ei; el punto
de intersección tfo lQs. scgmcr¡tos:D.0 1 y AB 1 ; M y N , correspondientemenle, los
puntos me!lios ·de lps segmentos AD 1 y CE. DcmullStrcn que L:..M N.B = · 90º.
486. En e"l trláng11lo ABC se han trazado lns alturas AA 1 y !1B1 y construido. el punto A; sipiétrico al punto A i con .rel.a ci6n a la tectn. A C; M y N. son los
pur¡tos medí(!~ de .los scgmimtos .l!i.Ai Y· AD, respeeUv.amcnte . .'Qeml!.estren que
el triángulo QMJI' e.s ·rec.tángul.o.
·
487. En ci1 Indo. AB del 'triángulo ABC, ,com.o sobro un diámetro, esln circunscrita una i;ircu¡úercncia que corta fas rectas· A e y f!C en los puntos A1 .Y B,
réspcctiVamcritc. Dent11estren que los lJUillQS medios de las cuerdas AB 1 y BA 1,
la baso de la '.a ltura tr:\Z?da en este · friángulo .desd.e el vértice e, forman un
\riángulo setjiejantc al dado .
488. óesde ·el punto /lf, tomado al azar en Ji¡. círcunferencia circunscrit.a
al triángulq A_.BC, S& traza.n fas pelj>cndiculares MAi y MB 1 e los lados JJC
'! A C.; I' y Q¡ ·son, ~cspcchva!fici:itc, lo~ puntos medios do los segmentos A B
y A. B 1• I!e.niue·s~rcn que LPOM :=: 90°.
. .
~811. Desde el v~rti~c A di:) ttlángt1lo.ABC se troza la. biscct:r1z AD hasta
su intersccció!> con .la ·circunferencia circunscrita :i dicht> triángulo en'. el punto
§ 6.
l'ectore.•
!13
A1 ; M y N son los puntos medios de lo.s $egmentos.CD y ·A 1B1 ; respectivamente.
.
Demuestren ql\e los triángulos A CA 1 y A MN son scn;i,ojpntc~..
490. La cuerd'a .común de dos circunferencias intersecanies es el ·diámetro
.de una do ellos. Por uno dc ·'l'Qs exú.cmos do este diá.metro s~. lraz:aii langen(es
a .la!! .mencionadas circunferencias.. Deínucstr~n gue· el otro: e:dremó de!
dtámctro y los Pll.ll~os medios ile los segmentos <hi los tangentes
tráiadas, co.rtailns por las circunferencias, son los.. xér,tic~s de un triangulo
rectángulo.
· 49L En el triáng1,1lo ABC las alturas AD y IJE ~o han continundo tras ·
los vértices A y B y en sus contiriuí!-cioncs ·se marcan tales· ~cgp1entos AM y .B N'
quo AM
BC y BN
AC . .Demuestren que )jrs segmcntos·cM y .GN son. perpe·n diculares o iguales.
.
.
. 492. Fuera del triángulo JfBC se cons,rurcn en. sus ·lados A C y BC los
triángulos equiláteros ACB_1 y BCA 1 ; Mes el punto medio del lado AB y O,
el centro del triángulo A CB 1 • Determinen los ángulos del triángulo M A 10.
498. Fuera del trhínguJo ABC 11e construyen on sus Indos A e y BC los
cuadrados ACDA 1 y BCEB¡. Demuestren que las rectos AB¡ y BA, so cortan
en la altura del triángulo< ado traiada al l acio AB .
.S.91i. Scon dndos tres triángulos oquihí.tcros A 1BC, A 2 D E y A ,FQ con igual
oricntaci6n, con ln pnrticnlnr.idacl d11 que los puntos A,. A• y A, son los vértices
do un triá.ngulo equilátero orientado 1m el mismo scnLldo. Demuestren cinc los
puntos m~dios de los segmentos CD, EP y QB son los vfrticcs de un lriiíng11lo
equilátero..
495. Se da el paralelogramo ABCD. En sus 1<1dos CD y BC, pero fuera de
él, están construidos los triángulos scmejan\es CDE y FBC orientados en el
mi11mo sentido. Dem11estren ·que el triángulo f A E es.semejante n ellos y su orientación es la misma.
496. En los lados AB., AC y BC del triángulo ABC, como .sobre sus bases, st•
han construido los triángulos isósceles semejantes A BP, A CQ y BC R . Los dos
primeros están lucra del triángulo dado, por el C·•mtrario, el tercero, por el mismo
lado de BC que el triángulo dado (o bien a la invers11). Demuestren que el cuadrilátero APBQ es un paralelogrnmo.
497. Fuera del trián!!'tlo ABC se han construido en sus lados A C y BC
los parale.l ogramos seme¡antcs A CMN y BCPQ. Demuestren que las rectas
N 8 y QA so cortan en la altura del triángulo (o bien en su c.onlínunción) traznda
desde el vértice c.
498. En él extremo A de la cuerda A B do Ja circ11nferóncia O se traza a
ella una tangente a la cual, desde el punto Q, se baja In perpendicular BM que,
por segunda vez, tropieza con la circunferencin en el punto (:. J?emuestrcn que
el centro O, el punto N, que di vide la cuerda AB en In razón AN : ND = 1 ; 2
y él punto C', simétrico a C con relación ni punto M , yacen en UM misma recia.
499. Los lados opuestos AB y CD del cuadrilátero ABCD están dividirlos
por los puntos M y N, respectiv•uncntc, en igunles razones r.ontando desde los
puntos A y C. Demuestren que el segmento MN divide In línea medin del cuadrilátor11 en Ja misma razón y se divide por la linea mcclin por la mitad.
500. Lc¡s puntos P , Q, R y S dividen los lados 1lel cuadrilátero A BCD de
forma que AP: PB = DQ: QC = m y AR: RD = BS: SC = n. Demuestren
que los segmentos l'Q y R S se dividen entre sí on ellas 1nisnms razones.
50t. En el t riángulo ABC está inscrito el pnrnlclogramo ADEF el~ formn
que los vértices D , E y P yacen, correspondientemente, en los lodos AB, BC y
AC. Por el punto medio /lf del lado BC se traia la recla AM que corta la recta
DE en el punto K. Demuestren que el cundrilñtcro CPDK es.un pMalclogramo.
502. Por los vértices opuestos de un paralelogramo se han trazado rectas
que cortan sus ledos o sus continuaciones en cuatro puntos. ·Demuestren que
dichos puntos son los vértices de un trnpecio o pnra lolo¡rramo.
503. En el trapecio A BCD el punto nrbilrnrio il1 del lado latera l .A.B está
unido con los vértices C y D. De los vértices A y B se trazan las rectas A N y
=
=
CapCt11/o l. />lnnlmttrla
BN pHralelas o las rec:las CM y DAf, respCGtivamcntc. Demuestren que el punto
N de su intersección rertenl'Ce al lado (;O.
50~. Sea dado el cuadrilátero A BCD. J,a recta t.razada p<>r el vértice A,
paralela al lado BC, cortn la diagonal BD en el punlo M, m.ientrae que lo
recta lraiada por el vórlico B, paro lela ni lado AD, cort.o la diagonal A C:: 9 11 el
punto N. Ocmucslrcn que llfN 11 CD.
505. Est~ d'a do un hoxlígono arbitrario de slmclrla ccnLral. En sus lados,
romo sobre sus bases, es~ n construidos por el exterior triiÍng11los rcgu l~rcs. Demuestren que los puntos medíos de Jos se_gmcntos que unen Jos vérlíccs de Jos
1riáni¡ulos vecinos S<>n los vértices de un hcxiigono regular.
f>OG. En el Indo A C del triángulo A BC se lonrn un punto M tal quo A M -
= ~ AC y en la continuación del Jado BC \al punto N que BN = CJJ. ¿En
qué razón divide a cada uno de los segmentos AB y MN su punto do intersección?
507. Están dados tres segmentos A 1Ai, B 1B: y C1 C,. Designemos eus
puntos medios con A., B 3 y C3 , rcspcc1.ivomente. Seon /lf 1 , 1"1 2 y M 3 los centros
<le gravednd ll e. los trl~ngulos A ,B 1C1 , A ,B 2 C: y A 38
rcspcctivnmm11c.
Demuestren que· M • t.s el pu ni.o medio del ~~gmcnlo M 1 M 2 (o hlcn qnr los puntos M,, JIJ_¡ y M'• coincid1•n).
508. t;n In mcdi:tna CM del triáug11 ln A RC sg tia el punto N . Por cllll ~
lrunn las rCGtos A N y BN que cortan los lados BC y AC en los puntos A 1 y 0 1 •
rcspt\CLivnmenLc. Demuestren quo el se¡,'11lento A 1 lJ 1 so divide con IA mcdinnn
Cftt por la mitad y es ¡inrelelo ni lndn AJJ.
509. En el lado A 8 dd triángulo A BC so da el p1111to /' por el q11c so trazan rectas paralelas n sus mcrlianns A M 1 y B M 1 y qnu corta n los correspondiente s lados del triángulo en Jos puntos A 1 y 8 1 • Domucstren que el punu1 medio
del segmento A 18 1, el punto P 1 y. el punto de lntcrsoccíóo Q de las medianas del
t riángulo dado yacen 011 una misma .recio.
510. La di stoncia desde el punto do interscccl611 ele las 01odl11nAs do un
triángulo hast.n el centro do la círcuo!crcncla circunscrita n él es igua l n un tercio del radio de dicha clrcunícreocia. Demuestren que el mencionado trínngi1lo
es rt>etángulo.
511. En do.s rectas ao dan los corrcspondic11tcs segmentos All y CD. Con
los puntos M y M 1 el segmento AD ~divide en !ns razones AM : AB - 8111 1 :
: A B - A C : BD, A C .,i. DD, en lnnto que el segme11Lo CD está dividido con los
puntos N y N, en es.is mismas rnzoues, rcspccth•amenle. Demuestren quo el
segmento MM, es perpendicular ni segmcnt<> NN,.
512. Dcmur.strm que si los Indos lateral es do un trapecio son porpc11dic11lnres, la suma de Jos cu111.lrndos de sus bnscs os lgunl n suma d11 los cuactrados
de lps diagon·ales.
:¡t a. Si en lun' cuauril1Hcro In suma de los cuadrudos de sus diúgonales
es i~al a la suinn. do los cu11dra·d os de Lodos sus •I ndos, esto cuadrilátero os un
paralelogramo. , Demuéstronlo.
Sl 4. Están.dados dos cuadrados·A.BCD y A>BJ.C1D, orientados en el mismo
sentido. Demuestren que AA?+ cq = BB? -r vD•.
SIS. En la 1mediann CM• del triángulo A OC so Ja el punto />, por ol cual
se trazan las rectos A P y 81' que cortan los Indos CB y A c. en los punt"s A 1
y 8¡, res¡icelivemcnte. Demuestren que si AA 1
BB h enton&l)S el tri6ngulo
es isósceles. ,
f>t6. En. la 'ba~c AB del triángulo isósceles ABC se da 61 punto P. Demuestren que PC2· = A C2 - A P ·B P . Aclaren cómo variará la fórmula sí el punto
P se eñcontrara: en la conlinuncíón do la bnso A lJ.
517. Demuostr~n que sí del punto arbitt'nrlo /11, tomado dentro dol triñ ngulo rectángulo 'ABC (el ángulp C es recto), so trnznn los porpcndicularcs M X,
MY y MZ e los lados BC, CA y AB, respectivamente, lieno lugar la relación
,c.,
=
AY·AC
+ BZ·IJA +ex ·CB -
AD•.
§ 6. Vectorr..s
.%
518. En 1'a continuación de los lados AB, BC y CA del triángulo ABC se
han toma~o, éorresporidientemenl.e, los· puntos M , !{ y 'P d~ r1rma quo 81'1 ==
= AB, CN = BC ·y A P = CA. Ca,l.culcñ la riqón entre la suma-. ~~ los. c.uodta~·
dos do los lados del triángulo PMN y· la suma de los lados del triangulo ABC.
519, I:.os lados Taleralés l)C y AD del trape~io ABCD ~st~n girados en
torno de sus puntos meilioa en· sentldó positivo a ~Oº, después: de lo cunl ellos
ocup~ Ja posición de los ~egiilentos B 1C1 y A 1 D 1 • Domuestrcn quo.
=
D,-c,
= A520.
1.B ,.
_E n
.•
.
1
,
•
los lados AB.. CD y EF de. un hexngO!lO de sunotria central, se
han construido los triánl{U]os equil'á teros ABP" CDQ y EFR <:Qn i_g ual orientación. Demuestren que el triángulo PRQ· es equilátero (en particúlar él puede
.
. .
.
degenerar· en. un pun,t o ).
521. El lado .AC del triangulo ABC esta girado alrededor :del vértice A
a +oo• y ·Ól lado BC .: en torno ·al- 'vértice B a ~so•. Demuestren qu!) lo posición
del puntó niedii> del seg_m ento 0 10 1, que une los. extre.mos C 1 y Gt de los segmentos girndos, no depenáe de In posición del vértice C.
522. En loe lados de un cuadrilátero, como sobre sus diámetros, se construyen >Sem·lcircunferéncias, ·con la pnrticubridad de que dos circunferencias
opuestas está.O dii:igidas. al interior dol cuadrilátero. y las otras dos, al exterior.
Demuestren que .los puntos medios do dichas serilicirclinfcrenelas son los vérti.
ces ,do . l!n :paralelogramo.
523~ ,E stá. dado :uil cuadra4o. Se-.construye11 diversos triáni¡ulos rectangulos
isósceles. El vértice de uno d'e los ángulos agudos de aquéllos coincide con el
vértice del cu·a drado, on ·tan~o que_(11· vértice del ángulo recto pertenece a sus
di8gpnnlcs . .Hallen el conjµnto de ' los terceros \·érticcs do los triángulos que
cóns1der¡¡mos.
524. Fuora de un triángulo arbitrario ae construyen en sus lados cuadrados. Demuestren que las alturas do ·un triángulo, cuyos vé-rti~os son los centros
de dichos cuadrados, pasan, respéctivamenle, por los vértices del triángulo
dado.
525. Én el triángulo ABC se han tra~ado las al t uras AA 1 , 8B 1 y CC 1 ; A 0 ,
B~ y Co, són los puntos medios de las mencionadas alturas. Dem11estrl\n que los
triángulos A 0 8 0C 1, B 0 C.,4 1 y A 0 C0 Bf son semcjanlr.s.
526. Estilo dadas las recias para elas q1 y q 2 y dos pares de puntos A 1, A z
y 8 1 , B 2• Hallen en los rectas. rcs11cctivamente, tales puntos C, y C~ de forma
que A 1C1 U A 2 C2 Y B 1C1 11 B 2 C, .
527. En ol cuad rilátero ABCD los lados DC y AD están divit!idos en partes iguales con los puntos ,81 , 8 1 yA 1 , A 1 , Tespectivamente. ¿Es siempre posible
trazar una r~ta. de ·modo que su segmento contenido entre los lados AB y CD
se divida .con Ja·s rectes A.1B , y A .81 en parles iguales?
528. Son dados tres .p u.!llos
tJ 1 y é,. Considerándolos CO!JlO los puntos de
división de los cort'cspondióntes lados do cierto triángulo ABC en la 'razón 2 : t,
por una misma dlrcccí6n dn circuloción, construyan el triá.ngulo ABC.
529. En la hip'o tenusa ele un triángulo rectángulo o en ·su continuación
hallen tal punto que la rectn que une sus proyecciones en los catetos sea perpendiculor a fo hipotenusa.
530. Sean OA, OB y OC tres rnyos que cortnn rlos rectas a y b en los puntos
0
A;.
A,B,CyA 1 ,B,,C,,re~pectivamente,deformaqueA8;
8C = 11,0A : AA,=
= m. HaUen la dependencia entre las relaciones 08: 08 = x y OC: oc, -
y.
1
531. Los lados opuestos AB y DC, AO y BC del cuadrilátero ABCD se
cortan <'n los punlCIS E y F, respectivamente. Demuestren que l os segmentos
.
.
AE·CE
AF·CF
formados satisfacen la 1gueldad 7JifJ5E=liiT.ñF.
532. Una r<'ctn que pasn por ~l centro de gravedad de un triángulo nivid e
sus Indos en cícrtos ·sogmcntos, Hallen la dllpcnduncia cnlrc> fo l'1lZÓn do lns longitudes de> los segmentos de un Indo y la rclnci6n rle los segmentos del otro lado.
Capflulo l . 1'1011imdria
533. Do.muestren que si en un cuadrilátero !ns continuaciones do los lado$
opuestos a pares se corLM, el punto medio del segmento que uno d ichos p untos
d e iotcrse<:ción yace en una misma recta que los puntos medios de las dlago·
nales.
5~. Demuesl.ren que la suma do las cuartas potencias de las distancias
de un punto dado, situado en el plaoo de cierta circunferencia. hasta los v~r\i­
ces de todo cuadrado inscri to en ella u const;iote.
§ 7. VALORES l\fÁXJMOS Y MlNJMOS•
Es cómodo resolver los problemas para determinar los valores
máximos y mínimos, según el siguiente pion:
1. Se reve la la magnitud a opti mizar (es decir, aquella pnra la
que hay que hallar ol valor máx o nún).y se designa, p.ej., con la
letra y (o S, P, r, R, ot.c., en función de la fábula dol problema}.
2. Ona de las magnitudes incógnitas (el lado, ángulo, etc.) se
doolara variable independiente y se designa con In letra x; so cstablo<:-en los límites reales de variación rle x (de acuerdo con el planteamiento del problema).
3. P artiendo do las condiciones concretas del problema dado se
expresa la magnitud y con x y las magnitudes co no cidas (ot apas de
la resolución geométrica del problema).
4. Para la función y = f (x) obtenida en lo etapa anterior se
halla el má:t o mín (en dependencia de lo quo requiere el problema)
por el intervalo do lo variación real de x hallado en el pun to 2.
5. So. interpela el resultado del punto 4 para el problema geométrico a resolver.
En las tres primeras etapas se confec<:,iona, como se suolo decir,
el modelo matemático, es decir analítico, del problema geométrico.
(El éxito en la resolución del problema dependo con frécuencia de la
elección acertada de la variable independiente. Es deseable que dicha
elección cond uzca a una expresión analítica sencilla comparativamente de y con x.) En l a cuarta etapa el modelo matemático elaborado
se investiga con la mayor frecuencia mediante procedi mientos del
análisis matemático, en ocasiones con métodos elementales. Al realizar semejante investigación, el propio problema geométrico que
fúe punto:de partida pa ra el modelo matemático no intereso al investigador. Solo cuando finaliza la resolúción. del problema en los
márgenes élel modelo matllmático confeccionado, ol resultado obtenido se intqrprete para Ja solución del problema geométrico inicial
(quinte e~apa}.
Recordemos el plan para. resolver, co n los proced imientos del cálculo diieiyncia1, el problema de In de terminnclón del valor máximo
o mínimo · de la función y = f (x} que se deriva en el i ntervalo X:
• Para '.a breviar aquí y en lo adelant.c, por mh y mío se desigruin Jos
valore., superiores máximos e inferiores mínim05 de los !unciones. - (N. del R.)
9;
§ 7. Valores má;r.lt¡ios y mCnLmos
1) se halla f' (x);
.
2) ~ hallan los punto!! estacionarios y críticos para la función
j (x), es decir, correspondkniemen~e. lo!¡ puntos en.los que f' (x) e: O
O· bien f' '(x) no exis't e; de ellos SC eligen aq uellos puntos quo pertenecen. al intervalo x.;
, ~)' se ,cónfeccipna 'la tabla de los valores d~ la funé~ón y = f (x);
en.está tabla se incluyen los-valores de la.función ~n los 'pilutos hallodos ·Qn ll} punto 2, así como en .los extremos del in~erválo X. Si éste
n.o contiene sus extremos, en la tabla se i'ncluyen los limites de la
función f (x) en sus extremos.
.J.
Hay que tom1¡1.r asimismo en consideración que el problema puede
ser resuelto con mayor facHidad por vía puramente geométrica (véase
más adelante el ejemplo 5, II procedimiento).
'EJEMPLO 1. En una circunferencia do radio R están dados los
puntos A y B, ·entre los que la distancia es igual a a y el punto C
tomado al ¡izar. ¿A qué será igual el valor máximo do In expresión
AC2
BC2 ?
"
s0Luc10N. (Fig. 93.) 1. L¡¡ magnitud que optimizamos es Ja suma
Acz + sr;2; de~ignéinos l a con y.
2. Elegimos la variable independiente, hacemos LCAB = x. Los
límites reales Cle varia~ión de esta. variable son: O < x < n - y,
donde y = L.,ACB (este ángulo no depende de la elección del punto
C, ya que siempre se mide con la mitad del arco lijado AmB). Es
fácil cerciorarse de que, según el sentido del problema, el punto C
ha de elegirse en el arco AnB, que es mayor que la mitad do la circunferencia.
3. Expresemos y con x, a y R . De acuerdo con el teorema de los
AC = -2R sen (n - :e - y)= 2R sen (x +y),
BC =
senos
= 2R sen x y AB = 2R' sen y o bien a = 2R sen y. Entonces, sen
'\' = 2~. Como 'resultado obtenemos: y=AC 2 +BC2 =(2R sen (x+
+y))2 + (2R sen x)2 = 4R 2 (sen 2 (x
+ sen2 x), así se lin
confeccionado el .rt1odelo matemático del problema.
4. Consideremos ahora la Iuncíón obtenida y= 4R2 (sen'' (x +y)+
+ :>11nz x)•· Hay que hallar su valor máximo en el intervalo
) O; n- y f. Hagarnos ciertas transformaciones de la expresión que
prefija la función. Tenemos: y =4RZ ('1- cos ~2.%:+2v)
i - ~os 2z } =
= 2RZ (2-cos (2x:+2y) -cos 2x) = 4RZ (1-cos (2x+ y) cos y).
En el ejemplo que consideramos el valor máximo de la expresión
obtenida es posible de hallar sin determinar la derivada: es evidente
que él se alcanza ·cuando cos (2x +y) tiene ol valor mínimo, es decir, cuando cos (2x +y) = -1. Esto será con 2x + y = n, o sea,
.
¡ ·
- pertenece n incos x = n-'I'
- -. Jndtquemos
que e1 punto -n-v
2
2
tervalo JO; n - y[.
+
+ '\')
+
7-0200
Capitulo I. Planlmelrla
Calculemos el valor máximo do Ja función y: y= 4R2 (f - ( - i) COS '}') = 4Jl2 (1
COS y)= 4fi'l- (1
Y1-sen2 -y) = IJl(Z X
+
X (
i+
+
Vi- 4~.) =2R (2R+ V 4R2-a
2
).
5. Efectuando la interpretación de este resultado llegamos a la
conclusión: el valor máximo de la expresión AC 2 +oci es igual n
2R (2R
+V 4R 2 -
a 2 }; él so nlcama cuando LCAB = n;-v, es
decir, cuando el triángulo ABC os isósceles (AC = CB).
EJEMl'l.o 2. Por el punto fijado M dentro de un ángulo, tracemos
una recta que corta del ángulo el triángulo de menor área.
'"
o
Fig. 94
Flg. 93
so1,.uc10N (Iig. 94). 1. P or el punto M trazamos la secante A 8. Ln
magni tud que optimizamos es el área del triángulo AOB. La designemos con S.
2. Tracemos DM 11 OB 11 KM 11 OA. Hagamos KB = x; los limites reales de variación de x son: O < x < +oo.
3. Como Mes un punto fijado, los segmentos DM y KM también
están fijados; hagamos DM =a, KM = b y expresemos S con x,
a y b.
Consideremos los triángulos MKB y AOB. De su semejanza se
KB
.
b =a+-=
:r . E sto s1gn
· if'1ca que
ded uce quo KM
OA =<JF,
es dec1r,
OA
OA
= b (a:z).
A continuación tenemos: S=fOA· OBsena, donde,
1
·
d o a= L AOB
Ca+.t:) X
,Paro a brev1•·ar ,:. s~ h a toJ)1a
· . Así , pues, S = 2 b--z--
x(a+z)sen~=bsen a . (a+zl' (está confeccionado el modelo mate!
2
%
málico del ptoblcnrn).
4. AnaliCQmos la función S=k (a+:r)' , donde 0< x<+ oo y
"'·
k = b sen cx . Hallemos su valor mfoimo.
2
§ 7. Va}ctes m<himo.< 11 mfoimos
99
i) S' ·<=k2Ja..f-x)z;<a.+ z)•=k (a+•>;~-a).
.
X
d~rivada no
lo's.~puntos x = -a, x
2) La
Z
existe en el punto x ·= O· y se te~i·~e a cero en
= a. . De estos tres puntos sólo eJ punto x = a
pe:rteJ!ece.-al intervalo )0; +ex>!.
' 3) tJ;alie·mos_los límites unilaterales do l¡r función e11 los cxtremO's del .Interv¡i:Jo:
lím k(a-f-.r)'
X•"+O
=
+oo,
X
La tabla do valores de la función tiene el aspecto:
Es decir, cl*vafor mínimo de 111 htnción se alcanz:1 en el punto
a.
5. Retornemos al problema geométrico inicial. Si x = KB = a.,
entoncés, como OK = a y M K es la línea media del triángulo AOD.
por lo tanto, M es el punto medio do AB. Así, pues, con el fin de
cortar en los lados del ángulo un triángulo del área mínima, hay
que trazar por el punto Muna recta de modo que su segmento cnll'e
los lados del ángulo se divida en el punto M por la mitad.
EJEMPLO 3. En los Indos iguales AB y BC del triángulo isósceles
ABC se han tomado los puntos D y E de modo que DE 11 AC. Sobre
DE, como en la base, se const ruye un cuadrado de forma que éste y
el punto B yacen por diferentes lados de la recta DE. Hallemos el
valor máximo del área de la intersección del triángulo con el c11a·
drado, si AC = b 'Y la altura BH del triángulo AIJC es igual a h.
soi.uc10N (fig. 95). 1. La magnitud que optimizamos es el área S
de la intersección del triángulo con el cuadrado.
2. Designemos con x el' lado del cuadrado: x =DE. Hallemos los
limites reales de variación de x. Está claro, que de todos los cuadr.idos que yacen onteramonte dentro del triángulo tiene la mayor área
fil cuadrado inscrito, es decir, aquel cuyos cuatro vértices yacen en
Jos lados del triángulo. Si x es mayor quo el lado del éuadrndo inscrito, el cuadrado y el triángulo se disponen como se muestra en la
fig. 96, en esto caso la intersección del cuadrado con el triángulo será
el rectángulo inscrito DEPT (el cuadrado inscrito es un caso particular). Es decir, los límites reales de variación de x: desde el lado del
cuadrado inscrito hasta el lado AC.
X=
1•
Capitulo l. Planimetría
100
Hallemos el lado del cuadrado inscrito. De la semejanza de los
triángulos BDE y ABC (fig. 95), obtenemos: ~=''-;;"', de donde
ha 11 amos: x=b+JI
bl•
A s1,
, pues, b+Ji~x<
bh ~
b.
3. Expresemos el área S del rectángulo inscrito DEPT con x,
a y h. Do la semejania de los triángulos ADT y ABH, obtenemos:
B
B
T
A
1-1
e
p
Flg. 96
Fig. 95
b
DT
AT
.
DT
BiT = -;r¡¡ , c.<; decir, T
. .
S hz (b-x)
cons1gu1ente, =
--b--
4.
Analicemos
[b~h; b [
la
%
2-2
= - -0-
,
d d d DT h(b-.rl
e on o
= - - b - - y, por
2
función
S
"
=-¡; (bx-x?.) en el intervalo
y hallemos su valor máximo:
1) S'=.t(b-2x); 2) S'=Ü con
x=}.
-Ahora, -hay que aclárar si el .punto
{v~l/;b[, ' os
Tyace dentro del intervalo
decir, si se cumple Ja desigualdad
b~h.<~ .
Con
este objeto :compongamos la diferencia b~h-f y hallamos Ja dependencia :entre b y h, con. la cual ésta es negativa. Tenemos
2
b1•
b '2bh-b -bh
¡;:¡:;;2=
Z(b+hl =
b (h-b)
h
1
2 (b+li> . A ora está e aro que l a
dºf
.
1 erenc1a
es -negativa si k< b. Sin embargo, si h>b, entonces dentro del
intervalo [b~h;b[ no hay puntos estacionarios.
§
7. Valores
m~itnO$
10~
·v mlnímos
3. Elab9remos la ~abla de. los yalores do las fun.c iones entre los
que hay que. buscar el máximo. Ante todo, señalet)lós que
lí.m S (x) = lfm
x ... tr-o
x-+b-O
,
•
"
·'A"'- continuac1on,
t
(bx- z2) =O.
s (b+h
bh ) = ( b+h.
bh ) 2 ,
.
,
Por f'in,
cuadrado inscrito.
h < b, la tabla tieno la forma:
bh
11+1i
:r.
bh es e l la do .d el
ya quo !>+ir
s (2b ) =-¡;h ·(
b.
b
( ;t
.b ) 2 ) ·=4·
hb
·28..1
b
2
bh
s
o
T
)2
.
bh
Demostremos c¡uo Tbh > ( b+h
. Ella se reduce a In des1gualded
2
(b-1- h) > 4bh, e!! decir, (b- h)2 >O, lo que es unn. evidente desi-
gualdad.--
Así, pues, si h < b, ol vofor máximo do la función S es igual
M
a T y se alcanza en el punto
Si
h~b,
b
x=-z·
la tabla tiene la forma:
bh
b+h
s
b
o
En este caso el valor máximo de la función Ses igual a (b~h)2
bh
y so alcanza en el punto x = b +h.
5. Retornando a l problema inicial llegamos a la siguiente conclusión. Si la altura del triángulo es menor que la b'aso, el área máxima la tendrá la intersección del triángulo con el cuadrado construido en la .Jínea media del triángulo: Pero si la altura del triángulo no es menor que la hase, la máxima será el área del cuadrado
inscrito en el triángulo. ··
Capltulo f. Planimttrla
102
EJEMPLO '· En una circunfetencia do radio R está inscrito un
trapecio del área máxima. Hallen los lados lnterales del t rapecio si
u na do las bases es igual a RV 3:'
soLuc10N (!ig. 07). 1. Ln magnitud que opUmizarnos es el área S
del t rapecio.
2. Designemos con la letra x el ángulo en la base conocida d o!
trapecio. Con x = 60° el trapecio degenera convirtiéndose en un
triángulo regular inscrito. Como sabemos, su Jado es igual a RV'3.
Por otro lado, x debo ser menor que 120", ya que el arco, sobre el
quo se apoya el ángulo inscrito en la baso del trapecio es me nor que
240º (fig. 98).
Así, pues, hemos establecido los límites reales pa ra la variable
independiente introducida: 60° < x < 120°.
Flg. 97
Flg. 98
3. Sabemos, que el área riel trapecio S,.,aco=
+
Anrsc .fl[{,
+
donde
+
BH es la altura del trapecio. Pero AD
BC = (Ali
DH)
(AD - 2AH) = 2DH. Así, pues, S A acD = DH·BH. Exprese·
mos ol área S con x y R. T enomo·s: AD = R V3, BD - 2R sen x,
LABD se ·mielo median t e } u AD, es decir, LAllD = 00º,
L BDA = 120° - x, Bll = BD· sen (120º - x) = 2R sen x X
X son (120º :- x).
Dlf = BD cos (120º - x) = 2Jl sen x cos (120° - x). Por lo tant o, S = .DH·BH = 2R sen x cos (120º - x).2R sen x sen (120º - x) = 2R 2 sen 2 x sen (240° - 2x).
+
4. HaJIJmos el valor máximo do ln función S = 2R' sen~ x X
sen (24Qº, - 2x) en ol intervalo \60º; 120º(.
Í) S' =i 2R2 (2 sen X COS X sen (240° - .2x) - 2 Seni X X
X cos (240° - 2x)) = 4R: sen x (sen (240º - 2x) cos x - son x X
X cos (240° - 2x)) = 4R 2 son x sen (240" - 2x - x) ,,,. ltR 2 son x X
X sen (240° - 3x).
X
,
§
1Q3
-7. Valoru máo:tmos y mínimps
2) En el intervalo.. ]60º¡ i2()º[ S' se i:educe a cero sófo en el punto
80°.
X=
3)
%
1
S
1
60°
3
R~lfa
1
J 120°
80º
l 2R' sen• 80° 1
En esta tabla so entiende por S (120°) el
O
lím S.' Comparemos
~ ...... 1.20°
an•lf3 y 2R2 se ns 80°. Su pongamos que 2R2 sen3 80º >
los valores ~
an 2 -va
> --.
4
1raEn tonccs, son 3 80• >8
es decir, sen 80° >
~a
,
rlc donde
f;o113
80º >
(z1rs )3 ,
o b¡en que son 80º :>.sen 60º. Esta última
desigualdad y, funto con ella, la ·suposición son ciertas. O sea, el valor máximo do la función S se alcanza con
x=80º.
8
5. Así, pues, el área máxima la tiene
~
el trapecio con ángulo de 80º en la base.
[:
Ahora hallemos el lado l ateral de tal trn2
pecio. Del triángulo ABD (fig. 98), tenemos: AB = 2R sen (120º - x). Con x = 80º
obtenemos: AB =. 2R sen 40°.
E.JEMPLO 5, Demostremos que de todos
A
D
e
l os triángulos con la base y el ángulo en el
Flg. 99
vértice dados la bisecti:iz máxima del ángulo
en el vértice la tiene el· triángulo isósceles.
soLllcJóN. l procedimiento. 1. La magnitud que optimizamos y es
la bisectriz BD (fig. 99).
2. De-acuerdo-con el planteamiento AC y LABC son constantes,
bagamos AC = b, LABC = ~· Introduzcamos la variable indopenrlíente x = LADB.
Hallemos los límites reales do variación de x. Por un lado el
ángulo x, como cxtcrioi: para el triángulo BDC, es mayor que cualquier ángulo de este triángulo que no sea adyacente con el ángulo
BDA, es decir, x_
> {.Por otro lado, del triángulo ABD deducimos
:2
que x
<
:rt -
{.
Así, pues, {
<
x
<
n -
!·
3. Expresemos BD con x, b y 13. Scfinlemos que L BAD = :rc- x-~ LBCD=x-4 .
104
CapltuÚJ 1. Planimetrla
Sogú n ol teorcmA de los senos, del triángulo ABC obtenomos:
AD
.
_ b_ =
AB
d o donde hallamos:
AC
scn B = sen C , es decir,
sen~
sen
(.r-f) '
bsen ( z-{-)
AB =
scnf\
·
Do modo análogo, según ol teorema de los senos, clol triángulo
A BD obtenemos:
~
BD
.
AB
SCü'1f = SCñA , es decir, sen;-=
(
y
sen n - z - 11
de donde hnllamos:
.r+{)
úscn (x-f) sen {x+{)
sen .r sen~
ABscn (
!/ = - --sc-n'-:i:----'-
COJI j\ -
b
= 2 sen 11
4 . Hnllaroos ol va lor
•
CflS 2z
s<'n :r
má ximo do
X cosf\-cos2z en ol intervalo
sen :z:
i)
J!
2
la
función Y =
!>
:l sen
ll "/,
·' n.-.!2 [.
= --b-. 2sen2zrenz-cos.r (cos ll-cos2:r) =
Y
2sen 11
sen• .r
b
(cos 2:z: cos :z:+scn 2..- sen :z:) +sen 2z sen :r - cos f\ cos:z:
= 2sen 11 ·
sen= :r
b
cos :z:+ 2sen•.r cos:z:-cos ll cos:z:
= 2sen~ ·
sen•:r
b cos z (sen• -11 +l!Cn•
2
h
cos .r (l-cos 11+2 sen• .r)
= 2scnll'
sco•.r
senllsen•.r
1
) ,
2
,
=
r,}
2) y' = O si cosx=O, es decir, cos x= ~
(otras soluciones on
ol ~otcrvalo ] {; n-{ [ la ecuación cos x= O no tiene) ¡ y' no
J{ ;
existe si senix"" O, poro on el inlorvalo
n-{ [ estll ecuación
no tiene soluciones.
3) Con· e! fin de elaborar· la "tabla parn hallar el va lor m áximo
de lil !unció~, ante todo, calculemos los límites unilaterales de la
función que ~consideramos cos x-+{+o y con z-+ n-~-0.
lím b(cosll - cos2:z:)= b(cosjl-cosjl) =O
6
'
11 +o 2 sen ji sen :z:
2senllsen2
2
lím I> (cosll- cos2.1') = b (cos 11-cos (2n -jl)) =O.
x->
11
x-=--i-o
2 sen 11 sen :z:
(
11 }
2sepllsen n- 2
§ 7.
Valor~$
m.tizimO$ y mlnimo.5
i05
Ahora ya est.á claro que la función y.(x) tiene el valor máx'imo
con. x= ; .. Este ·\'11lor es igual a:
b
2scnJI ·
cos~+i
1
~
él·2 cos• 2
4scw
11
2
cos
b
/'i
~ =2ctg2·
2
·
·
5. Si :e = ]-, LADB = 90°. Esto significa. que en- el triángulo
ABC la bisectriz BD es la altura y, por lo tanto, ABC es un triángulo isósceles. Así, pues, de ·todos los triángulos con J'a baso y el
ángulo en el vértice dados, l a mayor biscctriZ
del ángulo en .el vértice la ticno un triángu lo
isósceles.
JI proccdi1níimw. Demos la demostración
geométrica que, como veremos, por su brevedad y elegancia gana considerablemente en
comparación con el primor procedimiento.
Circunscribamos una circunferencia al
triáng11lo ABC con la bisectriz BD (Hg. 100).
Los vértices de los demás triángulos con la
base y el ángulo en el vértice dados yacen en
M
el arco ABC. Tomemos el triángulo ísóscclcs
AB1 C, tracemos en él la bisectriz B 1D 1 y deFtg. 100
mostremos qu e BD < B 1D 1 •
Continuemos ambas bisectrices BD y B 1D 1 basta su lnte.rsccción
con 13 circ11níerencia. Ambas cort.an a ésta en un mismo punto M,
que es el pun to medio del arco AC. Como B 1 M es el diámetro ele la
circunferencia, entonces BM < B1M. Del triángulo DD1 M conclnimos que DM > D 1 /lf. De estas desigualdades se desprende que
BM - DM < B 1M - D 1M, es decir, c¡\10 I3D < B 1D 1•
PROBLEMAS PARA EL TRABAJO INPIVlDUAJ,
53.">. Demuestren que c11 el tr iángulo rectñngulo
:i :>;; VZ-1, donde
r
es el rntlio de ia circunfcrcncin inscritn, R, de la circunscrita.
536. Demuestren que en un triángulo isósceles la razón enlrc los radios de
las circunferencias inscrita y circunscrita no es mnyor de
537. Demuestren que de todos los triángulos con igual ángulo en el vértice
y suma constante de los lados iguales, el isósceles tiene la menor baso.
538. Demuestren que de todos los triángulos con Ja base y ángulo en el
vértice dados, el isósceles tiene: n) la mayor área; b) el mayor ·perimctro.
539. Demuestren que de todos los triángulos isósceles, inscritos en In circuuíerencía duda, el equUátero tiene: a) lo menor área; b) el mayor perímetro.
+.
100
Capttufo J . Planlrtttlrl o
540. El tiunto A yace ont~ dos rectas p:iralolas 11 y 4, est:í alojado de ellos
a las distancias o y b y es el vórtice del ángulo recto del triángulo rectángulo
ABC; el punto D yace en la recto 11 , el C, en 11 • Demuestren que de todos estos
triángulos el de catetos a Y2 b 1! 2 Uene la menor área.
541. En un t riángulo est Inscrito un rectángulo. Demuestren quo el ároo.
dol rectángulo no cs ma yor de la mitad del á rea del triángulo .
MZ. Demuestren que de todos los pnralelogra mos, inscritos on el triángulo dado de Cormn que ellos tengan un ángulo común, el paral elogramo que
Llene la mayor área es aquel cuyo vértice divide por In mitad el lado del triángulo opuesto al ángulo común.
~- Hallen el va.lor
543. En el triángulo A BC el área es igual a S y LB
mlnimo: a) de la suma do los lados AB y BC; b) dol lado A C; c) del perímetro
del triángulo.
544. De todos los tr iángulos isósceles 1 con l argura conslll.llte de In mediana
trazada al lado lalcral, hallen aquel que llene la mayor área. ¿A q ué es igual el
á ngulo en el vcrlice do semejante triángulo?
5'15. En el triángulo ABC con lados a, by e, los Indos AB y A C se lian continundo tras los vértices D y C n lns disfoncfo s AD y A E do forma quo BD
-f- CH = A C. Tfol1611 lt /) y A Ji 1lc lllOtlo quo ol ~o¡¡mcnt.o DE sen el 1111ni111u.
:;46. ~n el lad o A C dol triángulo ADC se !tu tomndo ni oiar un punto y
desde él están bajadas perpcndicuJarcs a los Indos A B y DC . ¿A qué serán iguales
los valores mlnimo y máximo de lo suma do estas pcrpendicularrs si sobemos
que AB > BC7
547. Inscriban en el triángulo d.ado un rcetlingulo que tenga con el p rimero
el éngulo rocto común y la menor diagonal.
548. Una eslntun do 4 m de nlt-ura está situada sobre una columna de 5,6 m
do altura. ¿A qué dlstnncio do la colurnnil ha de encontrarse una persona para
q_ue vea In columna bajo ol máximo ángulo, si la distancia. desd e el suelo hastn
el nivel de sus ojos es igual a 1,6 m?
549. Los l:ulos latoralos y unn do las .hnscs de un trapecio son iguales o
i S cm. ¿ Con qué bese el liren de la figura será In máxima?
5r>O- Do un trapecio r ectangulnr con bases a y b y altura Ji se corta un rcclán¡rulo de la máxima árcn. ¿A qué será igual csla área ei: a) a
80 cm, b - 60 cm,
11 = 100 cm; b) a
24 cm, b - 8 cm y /1
12 cm?
551. El lado dol cuadrado ABCD os igual a 8 cm. En los lados AB y BC
so han tomado los runto5 P y E, respectivamente, clo modo quo BP - !JE ""
,.. 3 cm. Hallen en los lados CD y AD los vuntos M y K de forma que el trapecio PEMK t.cngn el área máxima. ¿A quo es igual el valor máximo del área
élcl trapecio?
552. En el penlhgonó ABCDE los ángul os A, By e son rectos, AB "" a,
A E.., b, BC = e y D E=- 111. Inscribir en el pentágono el rectángulo do árou
máxima. si: n) ¡i
7 cm , b - 9 cm. e
3 cm y m
5 cm; b) a ""' 7 cm, b b
·9 cm., e = 3 cm y m. ~ 4 cm.
553. En una circunferencia se dan dos puntos: A y D. 1falleo en In circunferencia tal pu_nto C quo: a) el producto, A C ·BC sea el máximo; h) la suma
AC
BC sea la mhimo.
554. a) De todos los sectores con el pcrlmclro P dado hallen aquel quo
tiene el área máxima.
b) Do todos Jos sectores con el área S dadn bailen aquel quo tiene el perímetro míniiilo.
555. La· s~clón ele un túoel tiene formo de rccllÍn~ulo terminado por arriba
qn formn. l!e semiclréuló. ¿ Con qué radio el e ésto el aren de la sccollin sertl. lo
.
•
máxi mn si ~l perímotro de éstn, os igual o, JI?
SSG. La Cl!.crdo A B ostil ¡ilcjndn del centro O do In clrcunícrenc1n do rodrn
Rala distancia i.. E n el menor do los doncgmcnlos formados por la cuerda AB
Inscriban el réctángulo de área ináxima.
¡
=
+
=
=
+
=
=
=
=
=
§ 7. Valore• m6zimos r¡ mlnfmoi
557. En la i;ircunfereocla de radio R está inscrito un trapei;io, una de cuyas
bases es lgual al diámetro. Hnllen. el área máxima de d Icho trn¡iecio.
558. a) Demuestren quo de todos los triángulos con el áogjilo agudo en el
vértioo y con la base dados, el triángulo isósceles tiene la máxi~a mediana trozado a la baso.
.
.
b) Démue.s tren :quo do todos los triángulos con el ángulo obtuso en el vórtice
y 111 base dados el triángulo isósceles tiene la mediana mininja trazadq 11 l~
baso.
559. Por el vértice O del triáogulo dado ABC so traza la rcc(al. Do los puntos A y C se bajan perpendiculares a dicha recta. Demuestren ·.q ue Ja suma de
las porpendiculares será la minlma si la rec:la t es perpendicular a la mediana
Bftf dol tri{ngulo ABC.
.
SGO. a) De todos los triángulos isósceles del área S dada, hallen aquo} en
ol quo se pu~de inscribir la Gircunferencia de radio máximo. Calculen su r odio.
b) A la circunferencia de radio r circunscriban un Lriángulo isósceles del
6.rea mloima. Hallen esta área.
561. El lado del cuadrado ABCD es igu.a l a G cm. En los lados AD y AB
30 han tomado los puntos K y P do forma quo A K
3 em, A P
2 em. E n
el cundrudo csLá inscrito un trnpocio con la base KP. ¿Cuál será el área máxima
del Lmpc<:io?
562. La cuerdo AB es Igual al radio do Ja Gircunforoncia. La cuerdo C/) se
ha traudo paralelamonto a AB de modo que el trapecio ABCD tione el árP.a
máxim11. Hallen el valor angular del menor do los orco& cortados por la cuerda
=
CD.
=
Capitulo II
ESTEREOMETRÍA
§ 8 GENERALIDADES SOBRE LA CONSTRUCCIÓN DE LA
REPRESENTACIÓN DE UNA FIGURA DADA
· 1. En planimetría la representación de la figura dada <I> 0 , llamada original, puedo ser toda figura <1> semejante a © 0 •
P. ej., si la figura <1> 0 dada es un triángulo con cate.tos iguales a
5 y 30 cm, es posible considerar que su represen tación es cualquier
triángulo rectángulo en el que fo razón entre sus catetos sea .') : 30,
es decir, un triángulo rectángulo con catetos iguales n i y 6 cm.
Hemos do señalar que en la práctica no es siempre sencillo construir la representación de la figura prefij ada, incluso con una precisión hasta Ja semejanza. Así, p. ej., si para resolver el problema es
preciso construir la representación de un .triángulo rectángulo según
la hipotenusa y la bisectriz de uno de los ~ngulos agudos dados, como
ninguno de Jos ángulos agudos del triángulo es conocido, con el fin
de construir su representación ·hasta la semejanza sería necesario
realizar cierta construcción auxiliar, es decir, resolver un nuevo
problema. Es natural, que en tales casos se intenta resolver el problema planteado en un dibujo «aproximado» y después de haber
hallado la resolución, ejecutar el dibujo con una precisión hasta la
semejanza.
2. Cuando se trata de construir la representación de la figura
dada en estereometria, el problema es mucho más complicado.
En la geometría descriptiva se han elaborado con detalles diversos métodos para construir las representaciones, en particular, diversos tipos de proyección paralela. No obstante, en las lecciones de
geometr.}a: en l.a escuela media, la con~ección do planos cin cierta
proyeeción poral~J a no está justifü;oda. Al .resolver problemas geométricos la construcción. d~ das represen tac.iones do las. figuras se
realiza en ·una. proye<;eión paralela arbitraria, es decir, ln posición
del or~gina-1 . COI"! relación al plano en d que se éfcctúa lo proy~ceión
y la .dirección de la ·prop\a pi'oy~ió_n .respecto a dicho. plano, son
indeterwinados. La posibilidad de aplic.ar este procedimiento de
construcc.iól} de la representación,., proyectada se desprende del teorema de Polke-Sc:hwarz,. de acuerdo con el cual todo cuadrilátero plano
ABCD, junto con sus diagonales, puede ser tomado por la proyecct6n
paralela de ~n tetraedro seinejante al tetr11edro .A 0 B 0C0 D 0 de forma arbttr:aria. Según la x:epresentación ·obtenida durante semejante proyec-
§ 8. Generalidades sobr6 la construcción de la represe11tación
109
ción pa.ralela arb.itraria; n~ es posible restablecer el original, ,pe:i:o al
resolver los problemas .d el cttrso escolar esto no es necesario.
3. A 'los dibujos .de proyección que se elaboran- nf resólver problemas ep la escuela medi a se planteiln los siguien~s ,requisitos:
1) }a repi:esentaci6n debe .ser cierta, es decir, ella debe ·representar una :figura semejante .a la proyección paralela del ' original;.
2) la representación, en la medida de lo posible, debe ser evide1ite,. es decir, ha de provocar la representación tridimensional de la
forma del original .(en algun!).s casos este requisito no se observa.
Sobre esto se hablará más abajo);
(a)
(11)
(e)
(d)
Fig. 101
3) la r~presentación debe elaborarse con facilidad, o sea, las reg las
de construcción han de ser lo más sencillas posible; una· gran cantidad de const rucciones auxiliares sólo dificulta In asimilación del
contenido del problema.
Hay que distinguir con claridad las nociones de representación
eicrta y evidente.
111 certeza de la representación es una noción matemática rignro·
samente determinada, mientras que ol concepto de evidencia se refiere a los subjetivos, ya que está relacionado con la percepción
individual de la figura representada.
P. ej., todas las mostradas en Ja fig. 101, a, b, e, d son representaciones ciertas de un cubo. Pero nos parece que la evidente es la
de la fig. 101, d. En la fig. 101, e, f, g, h todas las representaciones
son las ciertas de una pirámide cuadrangular. La evidente sólo es
la mostrada en la fig. 101, h.
HO
Copftulo /l. E1ttreo1nelrla
Para que la representación sea cierta es suficiente construirlo en
correspondencia con las leyes de la proyección paralela.
4. Con la noción de representación cierta ostá intimamento ligado
el concepto de plenitud posicional de la representación (o bien, con
brevedad, plenitud de la representación). Recibe el nombre de comp le t.a. Ja imagen de la figura cD 0 si cada punto A0 , perteneciente a elln,
está prefijado en el dibujo de proyección.
Recordemos con brevedad cómo se define esta noción.
El plano a: sobre el que se proyectan las figuras que estudiamos,
lleva el nombre do plano de proyecci6n o bien de representaci6ri (en
realidad es el plano del dibujo), en tanto quo la propia proyección
sobre el plano a., exteri.cr. Con el
"0
fin de cons truir la representación
de cierta figura puede realizarse la
proyección central o bien paralela. Cuando ltnulamos de la proyección exterior, en adelante, sólo
vamos a tener en cuenta la pro4
yección paralela.
Examinemos en el espacio cierto plano n 0 distinto de et y una
nuova dirección de proyección paralela no paralela al plano n 0 ;
la proyección en la mencionada diFlg. 102
rección va a llevar el nombre de
auxiliar (paralela). Para cada punto A 0 del espacio construimos el punto A; que es la proyección del
punto A 0 en el plano Jt 0 (proyección auxiliar) y, a continuación,
proyectamos ambos puntos A 0 y A~ en el plano et (fig. 102).
Obtenemos los puntos A y A' que reciben el nombre de proyeccUm
y proyecci6n secundaria del punto A 0 , respectivamente.
La correspondencia A - A ' en el plano de proyección se puede
considerar (élaro est6, de modo convencional) como cierta clase de
proyección; i¡e le aa o1 nombre de inter.tor paral§la, ya <¡ue.se efectúa
dentro d~l plano de representación. Es evidente.• que cualesquiera
que sean Jos· puntos A 01 B 0 , C0 , • • • en el espacio, en dicho plano:
AA' 11 B B' IÍ CC' 11 •.• •
Entendemo·s por proyección de una figura tridimensional ol conjunto de las proyecciones de todos sus puntos.
Con el fin de obtener la proyección de la figura tridimensional
<I> 0 (del original) no es oblig.a torio en el caso general la proyección
de cada uno :do ·sus puntos. P. ej., si <D 0 es un poliedro, ól está limitado por un qúmero finito de caras (!}.guras pl'anas), cada una de éstas
está limitad~ por una q uebrac\a, en la quo los eslabones son las ai:iS:.
tas dél poli11dro (segmentos). A su vez, cada arista está limitad\l con
un par de vértices del poliedro. Si hallamos todas las proyecciones
§ 8. Generalidades sobre la co11slrucci6n .de la rep~e.se.11tac!6n
1H
de los. vértices del poliedro, estarán asimismo detcnni11adas las ptoyecciones de todas sus aristas ·y caras, es decir, ln p,royecci:ón de)
·
poliedro.
. El punto A 0 pertenecienie a la. figura <!> 0 se llama prefijado er:i .el
dibujo de proyección (con breyedad, prefijadQ) si C.onocemos sus
proyecciones y la proyección secundaria, es decir; :un par de puntos
A y A'.
.
Así, pues, dos pares de puntos A, A ' y B, B' deter~i.lian la i m agen completa de -la recta A 0 8 0 a condición de que AA' 11 BB.1 •
Por analogía, si AA' 11 BB' 11 CC', los pares de ·pu'ntos A,, A';
B, B ' y C. C' determinan la image;o completa del J.Jl.aoo A 0 B 0 Co.
Con el fin de fundamentar la plcniiud de la .rep~esentaci.ón de
algunas figuras, es conveniente considerar la proyección central
auxiliar de los puntos A 0 , B 0 , C 0 , • • • do 111 figura original <]) 0 en
el plano n 0 • Habiendo realizado dicha proyección y, a continuación,
como habitualmente, la proyección exterior (paralela) rlc los puntos
Ao y 4;, B0 Y'B¿, Coy .C¿, ... en el plano
a , obtenemos las proye¡;ciones y las pros
yecciones auxiliares de los puntos A 0 , B 0 ,
C0 , • . • . En semejante caso, la c9r,respondencía A - A' recH:1e el nombre de proyección interna central (claro está, convencionalmente). Es evident.e, que cualesquiera
que sean los puntos\Ao. B 0 , C 0 , • • • en el
espacio, los rectas AA', BB', CC', . . . en
B
A
el plano de representación se cortan en 1111
mismo punto.
Ahora, mostremos que si prefijamos las
e
proyecciones de los vértices de la pirámide
S 0 A 0 B 0 C0 , es füfcir, los puntos S, A, B
Flg. 103
y e (sin las proyecciones secundarias),
la repi:~entación de · l a: pf.rámide será
completa. En efecto, tomando como centro de proyección auxiliar el
punto S 0 y como n 0 el plano A 0 B 0 (:0 , para cada punto M 0 de la pirám ide es posible, según su proyección M, construir su proyección
secundaria, o son, ol punto M'. En la fig. 103 tal construcción se ha
hecho para el punto M 0 pertenecí en to al plano S 0A 0 K;. Con ello,
cualquier punto del triángulo ABC puede ser considerndo como proyección del punto S.
La representación de un cono en forma de una figurn constituida
por una elipse y w1 par de tangentes a ella , trazadas desde cierto
punto exterior, es completa. Parn cerciorarse de ello hay quo considei·ar el cono jun~o con una Jlirámido inscrita en él.
5. Si lo representación de la figura <1> 0 es complota,-en ella es resoluble cunlquicr problema de posición, es decir, el problema de la
construcción do los incidencias do l as figuras dadas (p. ej., e l pro-
112
Capitulo /J. Eslruomctría
blema de la determinación de los puntos de intersección de la recta
prefijada con el plano dado).
6. También está ligada con Ja representación cierta la noción de
su definición métrica.
La representación de la figura <Do se denomina de/ inida métricamente si de acuerdo con ella es posible (en principio) restableéer la
figura <I> 0 con una precisión hasta la semejanza.
En el caso general, la representación completa no está aún definida métricamente, pero a determi-nadas condiciones puede convertirse en tal. P. ej., si se .indica ·que .el prisma representado en la
fig. 101, d es regular, su representación no estará aún definida métricamente. Si a.Dadimos adicionalmente que la arista lateral del
prisma es dqs veces mayor que el llldo do la base, se asegurará la definición méhica de la representación.
Recibe el nombre do condicional la representación acompañada de
tales condidoncs que permitan restablecer el original co·n una precisión hasta la semejanza. Son precisamente lns representaciones condicionales las que se emplear1 en las lecciones de matemáticas en la
escuela.
Semejantes condicjones (contenidas en el planteamiento. del ¡1roblema y que, por lo general, se indican con brevedad en la anotación
del problenja) son muy diversas y dependen, en particular, de la
figura que se representa.
P. cj., si en él original l a: figura <l> 0.e!! un cubo, es suficiente acom-'
pañar su imagen con la condición: la f\gura <Des un c\,l:bo. En efecto,
tal condición., aplicada a la figura <D, da la p.osibilidad de rcstablece.r
el original con una precisión hasta la semejanza, ya que todos los
. ·
cubos son semejantes entre sí.
Si la figur¡i Cl) 0 es una pirámide cuadrangular regular, es ·insúfi<:icnte acompañar su ropresen~ación con la condición de que )a figurn
<l> es una pirámi!le cuadrangul.ar regular-, ya que en el caso· dado no
podemos to.davía n¡stablecei: el original con una precisión hastl). la
semajanza. JCon el fin de.-que en, este ·tiicmplo ,la imagen sea definida
métricameñte, hay que ii1dic¡;¡.r ~demá~, p. ej., 'la razón eñt~. Ja:. .nltura
de. \.a. piráll;1"id~ y el lado de la base o bien el ángulo entre la arista· lnteral y el J?lano de la base, o. bie·n el· ángulo. entre la cara_lateral Y
el.plano di$ la base, et~.
·
.
7. Las const'ruccio.nQs q.ue sil ·realiza.g en el dibujo de proyección
pueden ser ,de: posic~ón y. métricas. ·L.as primeras t ransmiten las propi¡¡qades d·el:original que se conservan durante lá .proyección paralela
exterior. · . ·
'
. '
·
·
Las constr,uccion.os m~trii:a.s; por regla, transmiten las.propiedades
·
del original que no se cc;m5ervan· d·urante la ·próyección paralela:.
Ciertas ·construcciones que, a primera vista p·a.recen qiétricas, e.n
un problema concreto pueden convertirse en ·tas de posi.c ión. P. ej.;
la construéción de h: altura del iriángulo en el caso general es me-
§ 8. Gene[l!l_ldode& 1obre la construccl6n de la rcpresenlacl6n
113
trica, ya que no se conserva la propiedad de las rectas ·de ser perpendiculares con la proyección paralela. Pero s i en el or~ginal los lados
A 0 8 0 y B 0 C0 del triángulo A 0 l10 C0 son iguales, en el original la
~!tura B 0D 0 es al mismo tiempo la mediana, mientras que la propied ad cl,e nn segmento de ser mediana de un triánguló con la· proyección
pa_ralela se conserva y, por lo ta11to, la representaciÓ!l. <!e la altur11
8 0 D 0 de dicho triángulo será la mc~Hana BD del triángli;Jo ABC. Así ,
pues, en el caso q,ue consideramos, la construcción d·e. la representación de la altura es de posición.
8. Con el fin de fundamentar la definición mét'ric:a do la representación co~~leta hay que distinguir las propiedades a üues y m6tricas de las iíguras. He aquí algunas de ellas.
Propiedades afines (se conservan con la proyección paralela):
1) propiedad de la figuro. de ser un punto, una recta, un ¡>lano¡
2) propiedad de las figuras de tener intersección;
3) división rlel segment o en In razó11 dado;
4) propiedad de Jos rectas, los planos, lo recta y ol plano de Sl'r
paro lelos;
5) propiedad. de In Ugura de ser triáng nlo, JJarnlelogramo, l rnpec.io;
6) razón de las longitudes de Jos segmentos paralelos;
7) razón de· las 6reas de dos figuras.
Propiedades métricas (no se conservan con la proyección pnralola,
pero sí con las transformacio nes de la semejanza):
1) propiedad de 111s rect.as (planos, recta y plano) de formar ontrc
si un determinado ángulo (en particular. ser perpendiculares entre
sí);
2) razón entre las longitudes de segmentos paralelos;
3) razón ent re los valores de los ángulos e ntre Jas rectas (en
particular, propiedad de la recta de ser bisectriz de un ángulo);
4) razón entre los v11lor<!S de los ángulos diedros;
5) razón entro los volol'CS de los ángulos entre las rectas y los
planos.
Así, pues, p. ej., lo base (le un prisma .c uadrnngulat regular, c¡ue
en el original era un cuadrado, puede ser representada por un pa1·alelogra mo arbitrario, yo q uo la razón entro las longitudes de los lados
no paralelos riel cuadrado (igual a la unidad) y la perpendicula ridad
do sus lados adyacen ~es son propiedades métricas, mientras que el
paralelismo de los lados opuestos es un n propiedad afín.
En general, un paralelogramo arbitrario, puede ser la represen·
tnción tan to de un par11lelogramo como de un rectángu lo, rombo
cundradQ.
9. En la representación constru ida según las reglas de proyección
paralela, como es nntural, no se advierten las condiciones m6Lricas
indicadas en el texto del problemn. P. ej. , en la represontaC',ión de
un cubo vemos l11s caras que son p11ralelogramos y no c uadrados.
8 -0200
1 f4
Capítrtlr> Jl. EstereomelrCa
Por regla, dichas condiciones se indican aparte, acompañando el dibujo. Completando éste con las condiciones métricas, es como si
consumiéramos los parámetros.
P. ej., si la fig. <1' 0 es el romho A 0 8 0 C0D 0 y el paralelogramo
ABCD, su representación, acompañando el dibujo de las palabras
oAIJCD es un rombo» imponemos a la representación una condición
métrica o bien, como suele decirse, consumimos un parámetro.
En efecto, en el original A 0 B 0 = B 0 C0 , o sea, A 0 130 : B 0 C0 = 1 : 1,
pero la razón entre la longitud de los segmentos pnl'alelos no se conserva durante la proyección paralela y , por lo tanto, la igualdad
A oBo = B 0 C0 expresa una propiedad métrica del original. De modo
análogo, al construir en el dibujo u·n paralelográmo y añadiendo la
anotación de que está representado un cuadrado, consumimos dos
parámetros.
Cuando con cierta figura lD, que sólo tiene las propiedades afines
de original, reprci;cntamos 111 íigorn © 0 no consumimos pnrámetros,
ya que lns mencionndas propiedades del original se conservan al
efectuar la proyección paralela.
10. S i al realizar la construcción de la representación de la figura
plana <1> 0 se han consumido dos parámetros, de forma: unívoca se ha
definido la representación de cada punto que yace en el plano de dicha
figura (y, por consiguienle, las restantes construcciones métricas en
el plano de la figura Cll, necesarias para resolver el problema, no
pueden verificarse de for"ma arbitraria).
De manc·r a análoga, 's i al comtruir La ·representación. de la f tgura
tridimensional Cl1 0 se han co1isumidc cü1co parárnetros, de este modc se
ha def ln.ido un¡i:ocamente la representact6n de cada. pu.nto del espacio
(y, por lo tanto, en C!'te dibujo las construcciones métricas no se pueden efectuar arbitrariamente).
11. En el proceso de la resolución de un problema estereométrico
surge la necesidad de realizar asimismo diversas construcciones adicionale1; en la representación, 1>- ej., la construcción del ángulo lineal
del diedro dado, el á ngulo entre la recta dada y el plano prefijado,
la bisectriz \le cierto ángulo, etc. Cuando se efectúan construcciones
auxiliátes h~y que .tomar. en consideración rio sólo el número _paramétrico do lll re.presentación (el número ·de pa·ri\metros consum.i do·s),
sino tliml>ién la U-amada región de das posiciones tolera-bles».·
·p. ·cL .ta; representación del ·centro de una circunferencia, inscrita
\?n el triá.ngil)o A 0 B 0C 0 ,. no puede ser el~gidn si"n tener en cuenta In
reg'i ón ~e rtjp1-csentnción de dicho ce·n tro quo, como sabemos de la
geometría descriptiva, es una plirtc del plano situado en el interior
del triangulo, cuyos lados son las líneas medias del triángulo
A,BC.
r;.JEMPLO 1. Una de las caras de una pirámide triangular es perpendicular t1l plano de Ja base. Tanto· l a cara mencionada como la base. de la pi-rámide son tri-ángulos i·egulares. Tomando el cuadrilátel'o
§ 8. Gtnt[a.lidades sabre la <on&trucción dt la rcpresi11tación
115
arbitrario SBAC y S\.\S diago.nales como la representación do la pirámide, 1,a1lemos el . n~.meró para métrico de .la ·reprcscn~ación. ·
SOLOCION (fig. 104). La represcntnción del lri~ngluó regular, que
. yace en .ta. ·base de la- pirámide, con un triángulo arQftrnrio conduce
al fonsum_o <.le !:\os ·parámetros. La representación de .la .cara, que en
~l .órigihal :es un tr.iángulo regular, me.diante un triángulo a.rbitrarió
(como. .es· nat\lral, un lado de éste. es ·también él lador del triángulo
que yac(l en la base de· JA pirámide) tambi~n. conduce al consumo de
dos parámetros. Y, por fin , consi'derando
'
s
que lo!! triángulos construidos son las representaciones de los triángulos, cuyos planos
son perpendiculares en el original, imponemos a la imagen una condición métrica
más, .e s decir, consumimos otro parámetro.
e
Po! fin, considerando que los triángu- f1
los construidos son los representaciones de
los triángulos, cuyos planos son perpendi·
A
culares en el original, imponemos a la imnFlf<. 104
gen una condí_ci!>¡t ·métrica más, es dcch:,
consumimos otro parámetro.
Así, pues, para representar la pirámide dada se han consumido
cinco parámetros, es decir, el número pnramétrico de .la representación p = 5 y, de este modo, en esta representación ya no podemos al
azar llevar a cabo construcciones métricas.
OBSERVACION. Para abr·c viar, en la fraso •Sea dada la rcprcsenlación do \IIlll
fígura», con frecuencia se omito la paJnbra •represcnlllción• y se escribe, simpfemcnte, tia figura dada».
De modo q.ue está dada la pirámide SABC en la que la cara lateral SAB y la ba&e ABC son triángulos regulares, con la particularidad de que el plano SAB os perpendicular ~I plano AlJC.
Sea más adelante· e·n este ejemplo el lado de la base de la pirámide
igual a a y sea, P.reciso hallar el área lateral de In figura.
, I
a~ V 3
Esta c aro que S t.SA». = - 4- ·
Con el fin de hallar S 6 sAC es necesario determinar la altura SK
del triángulo SAC.
Para resolver el problema hay que efectuar construcciones nuxiliares, con Ja particularidad de que como la imagen es métricamente definida (para ella se han consumido los cinco parámetrós), la nltura
S K no puede trazarse al au1r, es decir, no es posible decir ~~ea
SK ..L AC» si hemos tomado en AC el punto arbitrario K.
La construcción auxiliar requerida puede efectua~o del modo
siguiente. Tracemos la mediana BM del triángulo ABC. Como éste
es regular, la mediana es, simultáneamente, 111 altura, o.~ decir,
BM ..L AC. Por analogia, al trazar la mediana SD del triángulo
8•
116
Cap!tu.fo l/. Esiertornelria
SAB, tenemos: SD .L AB. Es fácil demo~trnr que el ~egmen­
to SD es perpendicular al plano ABC. TracemosDK 11 BM, entonce!.'
DK .L AC. Unamos los puntos S y K. Como el segmento SD es perpendicular al plano ABC, DK es la proyección de S K en el plano
ABC y , por lo tanto, S!( J_ AG. Ahora, nos queda por efectuar sencillos cá lculos. Hallamos que SD = BM =
11113
a11í5
,
= - 4- y SK =-¡;-- . As1, pues,
11
1~3 ,
DK =
t
8
=
EJEMrw 2. Del vértice B del triángulo equilátero ABC se ha levantado la perpendicular B !( al plano ABC, con la particular idad do
que BK = Afl. Hallen la tangente del ángu lo agudo entre las roe·
tas AK y BC.
s0Luc10N. Construyamos la representación de la figura dada y hallemos su número paramétrico (fig. 105). Considerando que el triángulo arbitrario ABC es la representación de un triángulo equilátero, consumimos dos parámetros. Consid~rando que el segmento l3K
es la representación de l a ¡¡erpendicular al ·plano ABC, también consumimos dos parámetros y, por fin, tenien!lo en cuenta que
BK : AB = 1 : 1., consumimos un parámetro más; así, pues, p = 5.
De forma quo la representación prefijada está métrii;amente definida
y, en adelante. no son to lerables otras construcciones métricas.
Realicemos las siguientes construcciones auxiliares: tracemos en
el plano ABC la recta l 11 BC por el punto A. Entonces, el ángulo
entre las rectas AK y BC será igual al ángulo entre las rectas AK y l.
Con el fin de hallar el ángulo entre las rectas A K y les conveniente
incluir dicho ángulo en algún triángulo. Operamos del modo siguiente.
Del punto K bajamos la perpendicular KD a la recta l. Con ello la
recta KD .L l puede construir.se como una ob\icua, cuya proyección
es pcrpcndicuJár a l . Para traza.r esta recta; es decir BD, es posible
hacer uso ;del l1ecl10 que el triápgulo ABC es equHátero y, por lo
tanto, su rile~iana AM es asi.mismó la perpendicular a) !legmcnto BC.
De (lstc m~do 1 después dE> construir· la !lledia,na AM del triángulo A.HC
y, seguida¡nente, BD 11 AM y segmento DK, obtcnclr.emos .el triángulo rectápgulo ADK. La. razón KD : AD = tg L D,AK será la
tiuscada. ,
11 7
hallamos: BK =a, BD = a
DK =
'
Haciendo AB =
ª·
AD= Bili[ =
tangt'nte
T y,
·~ntre
por consiguiente, tg L DAK =
r
Yª .
y'7, así que la
las rectiis AK y BC también es igual a
y7.
§ 8. Gentralldatks sobre la eonslrijcc16n de la reprt1cn!acl611
il7
EJEMl'LO ~. Uno de los catetos de un triángulo rectángulo isósceles
yace en el plano a y el otro. forma con él un ángulo ~ igual a 45°.
Constr\)yamos la representación.de la figura dada, hallemos su número pn:ramétrico y, n continuacióq, el ángulo que fori'Q.a su hipotenúsa con ·el ·plano a.
so1iu,crON. Considerando que el triangulo ar bitrnrjo ABC (Ug. lOG)
es la· representación de un t riángulo rectángulo, para su· representación consumimos un parámetro; suponiendo que AC- y BC son l11s
repre.sentaciones de segmentos iguales, consumimos un pará metro
más; considerando, seguidamente, que BC es la representación de In
K
f'ig. 106
Flg. 105
'r ecta que forma con el plano a. un ángulo igual n 45º consumimos asimismo para su represen'.taclón otro pnrámetro; as{, pues, p = 3.
Por ello, para las posteriores construcciones métr icas, en nuoslrn
representación aún nos quedan parámetros libres.
Ahora, de~ermi nemos el ángulo que forma la hipotenusa AB con
el plano a. -Tomemos en este plano cierto punto B' y _consideremos
que BB'
la representación de. la perpendicular al plano a. Do
este modo hemos consumido los dos parámetros libres que nos quednhan. La representaéión está métricamente determinadn y, por l o to.11to, no podemos efectuar const rucciones métricas arbitrarías en In
representación obtenida. Por cierto, que en el caso dado esto 110 es
necesario. Construyomos B'C y B'A. Hagamos AC = a y, entonces,
es
a'Vi
,/ -
hallaremos: Be= a, BB' = - 2 - y AB ""' a y 2.
Así, pues, en el t riáng ulo rectángulo ABB' tenemos: BD' : A B =
.,. 1 : 2, es decir, LBAB'
30°. Pero BB' ..L a, o sea, LBAB'
es el ángulo que forma la hipotenusa AB con el plano a.
=
O,D!!lmVAClO~.Ellgiend o al aur el plmto B', perte1•cclcnlc al plano Cl, guiándonos por cousldcracloncs do aumonto r la evidencia de Jo representación obtenida, lo hicimos de formn que la recta BB' sea paTaleln al borde <le In página. Scmejant.e representación provoca la idea de que en el originnl B 0 Bó .L a 0 •
118
Capitulo lI. Bsterr.ometrla
EJEMI'LO ~. En una pirámide cuadrangular regular el ángulo entre
dos caras laterales vecinas es igual a 2ct. Hallemos el ángulo formado por la arista lateral de ln pirámide con el plano de su base.
SOLUCJON. Está claro que la figura SABCD (fig. 107) es la representación de la pirámide pre[ijada. Calculemos el número paramétrico de dicha representación. La representación del cuadrado, que
es la base de la pirámide dada, por el paralelogramo arbitrario ABCD
provoca el gnsto de dos parámetros; l a representación de la altura
de la pirámide por el segmento SO, donde
s
O es el punto de intersección de las diagonales del paralelogramo ABCD, conduce al
consumo de dos parámetros más, ya que
se considera que el segmento SO es la
representación de la perpendicular al pll\c no AJJC. Así, pues, p = 4, es decir, parn
rcnlizar las posteriores construcciones métricas, requeridas en el proceso de rcsolnA
ción del problema, a nuestra disposición
q\leda un parámctr.o libre. P ara resolver
Ftg. 107
el problema planteado construyamos en la
representación el ángulo lineal del diedro dado. E ~to puede hacerse, p. ej., del modo siguiente: tomemos al azar en la arista SC el punt.o M y consideremos que el
segmento OM es la representación de la perpendicular levantada
del punto 0 0 a la arista S 0 C0 • Ailora, uniendo el punto M con los
puntos B y.D, obtenemos el ángulo BMD que, como es fácil mostrar,
es la representación del ángulo lineal (el ·ángulo diedro S 0 C0 en el
original).
Aquí, podemos introducir en el dibujo la designación 2a.. Como el
segmento SO es la representación de la altura de la pirámide, el
segmento OD es la representación de la proyección de la arista lateral
en el plano de la base de la pirámide y, pQr consiguiente, el ángulo
SDO. es la .representación del ángulo que buscamos. Es f~cil calc.u lar
que este *ºn gulo es igui-.l a arcsen (ctg et), .doni:l.e ·4~º <p. < 9·0°.
12. Al ;·resql ver ciertós problerµas ·de · estereómétría ·result.an ser
más cómo4as no las representaciones coJllple~as, mébicnmente. definidas, sinó las simpliíicádas.. Por· ejemplo, al reso1ver fos problemas
sobr.c l a ·co¡j¡l>innción de poriedros y cuerpos redondos, eligiendo una
se'cci.6n idónc'a para la construcci.ón de la combinación do los cuerpos,
.e s posible mostrar, en dicha repr<isontación la figura obtenida en la
sección con una precisión hasta la semejanza.
P. ej., en lugilr de la ·roprescnlació~ de una pirámide cuad(angular
regular. y tl.e l it esfera inscrita en ella podemos dar la imagen de la
figura obtenida en la sección de esta cpinbinación de un cuerpq por
un pi.ano que pasa por lq altura de la 'p irámide, paralelo al lado de
la base. Esta sección es un triángulo ísó·scelcs, cuyos lados laterales
§ 8. GMeralidade• sobre la r.onstrucci6n de la representación
119
son las apotemas cJe -la pir.ámide. Como el centrQ de la esfera yace en
la altµra de ·la pirámide, ·la secc.i<'!n de la esfera por dicho plano· se~á
l a circunferencja de ~n gran cfo;u'lo. C)>mo- In esfe~a es ti.ÍQgente a las
.caras de ia pirámide, su centro yace en el ptano biscc.tri~ de los planos de 1as caras OJ!Uestas. Pero el plano bisectriz corta !as otras dos
caras por las apoteµi.as.
·
. .
Así, pues, la circunI.e roncia p'btenjda én la sección e's tangeptQ n
los l ados laterales del ti'iángu.ló isósceles. J?or analogía, dicha circunferenci_a es tambión timgente} a .Ja base del triángulp. Así,, pues,
en la-sección obtenemos. un. triángulo .isósceles con. una circunferencia
inscrita en él.
·
13. En ocasione:?, al construir el dibujo es conveniente realizar
Ja construcción empezando «por el finab.
EJEMPLO 5. En una pirámide cuadrangular regular está inscrito
un cubo de modo que cuntro de sus vérticos se encuentran en la baso
de la pirnmidc y los otros c11ntro,
en sus aristas laterales. ConstruS
yamos la representación de la figura dada.
SOLuc10N. Sea ABCDA,B1 C1 D 1
la imagen del cubo (fig. 108).
Hallemos los centros de las bases
R
del cubo, .es decir, Jos puntos O
y 0 1 • Tracemos l a recta 001 y tomemos el punto S (fuera del cubo).
Segui damcntc, tracemos las rectas
SA 11 SB)t SC 1 , SD 1 y hallemos
los puntos P , Q, R, N. Éstos son P
los vértices de la base de la piráFig. 108
mide. Entonces, SPQRN es la
imagen de la pir~mide prefijada.
14. Como conclusión del presente apartado señnlemos lo s iguiente. Si en el problell)a se trata de la construcción de una figura que
requiere el consumo de no más de cinco parámetros, en el proceso de
la resolución la construcción de la representación no se describe (no
obstante, es preciso construirla). En semejantes casos sólo hay que
cerciorarse de la plenitud de la construcción de la representnción y se
calcula su número paramétrico. Semejante cálculo es necesario, ya
que en el proceso de la resolución pueden ser necesarias construccio·
nes auxiliares de carácter métrico que, al haber parámetros libres
podrían realizarse de modo arbitrario (teniendo en cui::nta las lunHaciones concretas durante la realización de dichas construcciones).
Si en el problema se trata de una f.ígura cuya construcción requiere el consumo de más de cinco parámetros, aguella paf to de la construcción de la representación, pnra la que se requieren cinco parámetros, no se describe; las posteriores construcciones necesarias se
120
Caplttt.lo J1. Estereometría
d!.'scriben obligatoriamente y se efectúan en corresponclenda con
las reglas de la proyección p aralela.
Si en el problema se estipula que hay que construir algunos elementos de la figura dada (p. ej., la sección de la pirámide dada) la
co11strucción de la representación de dichos elementos de la figura
dadél so describe obligatoriamente. Claro está, que durante la constr ucción se lleva a cabo el cálculo del número paramétrico de la
representación que se obtiene (tales ejemplos se estudian en el § 12
del capítulo JI). En alg110os casos, la descripción de esta construcción
se realiza $egún el esquema completo de resolución del problema
:<obre Ja construcción (análisis, construcción, demostración, investigación), en algunos otros casos se limitan a 1n descripción de etapas
por separado de este esquema, p. ej., la descripción de la construcción y Ja dcmost.ración o bien la construcción y la investigación, etc.
!\ !J. CONSTHUCCIONES GEOMf:TRJCAS EN EL ESPACIO
l. Construcciones más scnci llns l'n el espacio
EJEMPLO t. Tracemos por el punto dado A un plano paralelo al
plano dado P.
SOLUCION. A 11álisis. Sea Q el plano que buscamos (fig. 109). Coustruyamos en el plano Q dos líneas diferentes l y m que pasan por el
punto A, en tanto q·ue en plano />
tomamos el punto A 1 • Construyamos los planos L y M que pasan
por el punto A 1 y la recta l y por
el punto A y la recta m, correspo11 dientemente. Como la recta l
yace en el plano Q y Q 11 P, l 11 P.
Pero,' entonces, el plano L cruza el
plano P por la recta l 1 paralela a
l. Por analogía; M cruza el plano
P po.r la recta m 1 11 rn.•
Como Q so determina unívocamente con las rectas l y m , podemos
reducir el prob-lema a la construcción
do lns rectas l y m que pasan
Fig. 109
por ol punto A y que son ¡iarnlclas al plano P.
Co11struccí.ó1~. 1) En el plano P tomamos al azar el punto A 1 •
2) Por el punto A 1 en el plano P trazamos las rectas arbitrarias l 1 y
mi. (l1 it m 1 ). 3) Construimos los planos L y M. 4) Por el punto A
eri e · :i)·Jp.no L traza~ os la recta l il l 1 y en el plano M, m 11 m 1 • 5) Po1·
las rectas l y ni trazamos el plano Q.
§ 9. Co11•trucclones geométricas en el espacio
121
Demostración. Como de acuerdo con la construcción l ll l 1 y la
recta l 1 yac·e en el pleno P, l 11 P. De modo análogo, m lf P. Entonces,
el plano Q, que pasa por las rectas l y m, será paralefo al plano P,
así como pasará por el punto A. Esto signiflca, el plano Q es el
buscado.
/nr;estigaci6n. El problema tiene una sola solución. En ,efecto,
supongamos lo contrario, es decir, que, además, exist,e el p lano Q1
quo no coincide con Q, pero que es paralela al plano ;p y que .Pasa
por el punto A. En tal caso, el _p lano L erutará este plano Q1 por la
recta l, paralelll a l. Pero esto significa que en el pl~no L, por el
punto A, pasarán dos rectas l y l 2 paralelos a l 1, lo que contradice el
axioma acerca de las paralelas. L a contradicción obteñi da nos muestra que el problema tiene la única solución. En particular, si el
pun t o A dado yace en el plano P, los planos Q y P coinciden.
11. Lugnrcs geométricos de pun tos
H allemos el lugar geométrico do loi; puntos del espacio equidistantes de los ¡1lnnos P 1 y P, secantes dados.
SOL1.iCJON . 1) Sea que el punto M es equidistante do los planos
P 1 y P 1 (fig. 110). Construyamos el plano Q que pasa por el punto M
y es perpendicular a la recta a que
es la línea de intersección de los
JllRnos Pi y P 2 •• Para ello, del
punto M bajamos la perpendicular
ML Al punto a y, a continuación,
ca el plano P 1 , por el punto L,
trazamos la recta l 1 .J... a. El plano
determinado con las rectas Lilf
y l 1 (plano Q) será perpendicular a
la recta - a. A continuación, constntimos la recta l, por la que se
intersecan los planos P 2 y Q. Entonces, l 2 .L a. A continuación , del
punto M bajamos 11\s perpendicula1·es MM1 a la recta l 1 y MM 2 a la
recta l 1 • Como a .J... l 1 y a .J... LM,
Fig . 110
a .J... MM1 o bien MM 1 .J... a. Pero
de lo que MM1 .J... a y M/tf1 .J... l 1
so desprende que MM 1 .J... P1 , es decir, que la longitud del segmento
J\fM 1 es la distancia del punto M hasta el plano P 1 . De forma anéloga
Ja longitud del segmento MM 2 es la di.s tancia desde el punto ftf
basta el plano P 2• Entonces, en los triángu los rcctángulos MM 1 L y
MM 2 L MM 1 = MM 2 y en olios la hipot enusa LM es común. Por
c!lo, c!t:::.MM1 L=t:::.MM2 L, lo q11 esignific11 queLMLM1 =LMLM 2 •
Pero, de acuerdo con In construcción, LMLM1 es el ángulo lineal
EJE?>JPLo 2.
122
Capítulo JI. Estereometrfu
del ángulo diedro M 1 aM y, por analogía, LMLM~, el óngulo lineal
del ángulo diedro M 2aM. Así, pues, los ángulos diedros M 1aM y
ltf 2 aM son iguales, es decir, el scmiplano determ"inndo por el pu.nto
Jlf y la recta a. es, ni más ni menos, el semi plano bisector del ángulo
diedro 1lf1 aM2 • De modo que si el punto M está igualmente alejado
de dos planos secantes P 1 y Pt, él pertenece al semi plano bisector del
ángulo diedro formado por dichos planos.
Pero los pla nos secantes P 1 y P 2 forman cuatro ángulos diedros
y, por consiguiente, el lugar geométrico de todos los puntos Mes el
par de planos S 1 y S 2 , donde S1 es la unión de dossemiplanos bisectores de un par de ángulos verticales formados con los planos P 1
y P 2 , mientras que S 2 , la unión de dos semiplanos bi.scctóres con otro
par de l}ngulos verticales formados con P 1 y P 2 •
2) Sea q11e el punto 111 pertenece a cierto semiplano bisector de
los ángulos diedros formados por la intersección de P 1 y Pi· Construyamos el plano Q que pnsn por 11![ y es perpendicular a et, para
lo que del punto 11'1 bajnmos la perpendicular ML a a y, a continuación, por el punto L trazamos 11 .L a.
El ¡>lano Q se deter mina con las rectas 11 y LM. Seguidamente,
construirnos en el plano Q la recta li que pasa por el punto L, con la
particularidad de que
...1.. a y d~l punto M bajamos las perpendicu l11res Mi\1 1 ...1.. l 1 y MM 1 .L l 2 • Examinando los tríángu los rectángulos MLM 1 y MLM 2 en los que LMLM.i. = LMLM 2 • ya que
el rayo LM es la bisectriz. del ángulo· lineal M 1LM 2 y el segmento
L1ff es l a hipotenusa común de dichos °triángulos, llegamos a la conclusión de que MM1 = 1WM,. Así , pues, si el punto M pertenece al
semiplano bisector (le cualesquiera de los cua·tro ángulos diedros
formados por los planos P 1 y P 2 , él está elejndo a una misma distan·
cia de dichos planos.
Así, pues, el lugar geométrico de los puntos buscado es la unión
de los semiplanos bi~ctores de Jos cuatro ángulos diedros formados
por los planos P 1 y P 2 , es decir, l a unión de los planos S 1 y S 2 •
OBSBOV~CIÓN.Sería posiblomostrar que S 1 .LS,. Sea c:l lector el que lo hago
por sí solo.
z,
111. i\¡5Íieaci6n de ciertos l'ugarcs "geomét ricos do pu ntos y rectas
EJEMPLO 3. Construyamos el h1gar geométrico de los puntos pertenecientes al plando dado P.:y equídislantes do los puntos dádos A y
B que no racen en el .p lano F'. .
SOL1J.C.JOÑ. 4,nálisis. Sea que el punto 111 (fig. 111) pertenece al lug~r geometrljc9 de puntos qu,e ·bus<;aino~. Como AM = B¡l1, el punto
M pertenec.e .a l plano Q que pasa. por C, que es el punte medio· del
segmento 4B, y es perpendicular al segment o AB. Como, además
de ol!tO, el punto M yace en el plano P, el lugar geométrico de los
§ 9. Construtclonu ttomltrlca• tn ti espado:
123
puntos es la línea de intersección de l os planos P y Q. Designemos
dicha línea con l. De lo que AB ..1.. Q y que l yace en el plano Q se
desprende que AB ..J.. l. Sea AA 1 ..1.. P, BB1 J.. P y l jn lerseca A 1B 1
en el punto D. Entonces, AB ..l. CD y, por Jo tanto, asimismo A1 B1 ..l.
¡
..l. Lo bien l ..l. A 1 B1•
Así, pues, el problema puede reducfrse a la construceión de la recta
l que pasa en el plano P por el p1,1 nto D y es perpen4icular a la rec'
ta A 1B 1 •
Construcci6n. 1) Construyamos AA 1 ..l. P y BB1 ...L,P. 2) Construyamos 4. 18 1 • ·3 ) Hallemos C que es el punto medio del¡ segmento AB.
4) En el plano que pasn por los
A
. . ,,.,.. ;
punt os A. A1 y B, por el punto e,
.....:~¡K;\W~ ~:
t razamos la recta n J.. AB. 5) Ha.·.' '¡~$::· "'. };'.'{:·;:·
llemos el punto Den el que se cor.. :.._:·:·:':;:~~f ;: .:·-?.:-;
tnn las roetes n y A 1 B 1 • 6) En el
-~~;..;:.;•:.~(:::-.;· -.._:~;t:~:,~.
plnno P constniímos la recta l .l.
.;:}~;:·,:::·;«.::: . :~;::::.~«
..L ..4 1 8 1 que pasa por el punto D.
";;~::.'.,.'· ,:¡.::: (!;:;;;:;'.),
7) Construyamos el plano Q que
'·:r:::'4 f{J/ )
pasa por las rectas l y n.
10 ..-:.. ·::!;:.;;;"'
Demostraci6n. Si el punto M
/ M '~:.
yace en la recta l , él yace también
en el plano P. Unamos el punto
M con Jos puntos A, By C. Como
Fig. 111
AB ..L Q y la rectn CM se encuentra en el plano Q, AB ..1.. CM. Ade- .
más, AC = BC. E ntonces, el t::.ACM = t::.BCM (por dos catetos)
y, por ello, A.M = BM. De este modo, la rectal es el lugar geométrico de puntos buscado.
lnvestigac~~n .. La existencia de la solución depende de la disposición mutua de la recta AD y el plano Q.
1) Si AB ..1.. P,. entonces como AB ..l. Q, resulta quo Q 11 P. es
deci r, en semejante case¡ no hay solución.
2) Si AB no es perpendicular a P. la rect a l, por la que los planos P y Q se intersecan, es el lugar geométrico de puntos buscado.
l V. C•>nSlt'ucclont-s en !ns reprcscntnclonc.'
Para abreviar acordaremos que en las cnunclacioniis de los ejemplos de este apartado y el plant eamiento de los problemas que han de
ser resueltos independientemente por el lector, vamos a omitir por
doquier el vocablo «representación». P. ej., en lugar de escribir: el
t.riñngulu ABC es la representación del triángulo A.J]0 C0 , en el que
A 0 D 0 : B 0 C0 - .2: 3, escr ibiromos do modo más corto: en el triángulo ABC AB: BC = 2 : 3. De la misma manera, en lugar de los
vocablos cconstruir la representación de la bisectriz del ángulo
A 0 B 0 C;11 vamos a escribir: •construir l a hisectri2 del ángul o A BC~.
Capítulo 11. E$ltrtomelrta
l24
Asimismo acordemos ahora que ea este apartado emplearemos
en las designaciones de l a figura-original el subindice -.cero•. Así, al
decir que en el triángulo A J30 C0 se ha construido la mediana AoAf0
so tiene en cuenta que la mencionada construcción so ha realizado
en el original, con ello, en nuestra representación tendremos el triángulo ABC y en él estará construida la mediana AM. Consideremos ejemplos.
EJEMt>LO 4. En el triángulo ABC
8
AB : BC = 2: 3.
Construyamos
D, D M C
L
"
la bisectrit· del ángulo ABC.
SOLuc10N. 1 procedimiento. Si el
segmento BD es la representación
e,
de la bisectriz B 0 D 0 del ángu lo
A 0 BnCo (fig. 112), AD : DC =
f'ig. 112
= AlJ : IJC, es decir, AD : DC =
= 2: 3. Así, puE!s, después de construir el punto D en el lado A C do modo que AD : DC = 2 : 3, obtenemos BD que será la representación buscada de la bisectriz (en la
fig. 112 se muestra la construcción del punto D con ayuda del rayo
auxiliar AC 1 , en el que AD 1 : D 1C1 . . 2: 3).
11 procedimiento. En los lados AB y BC bailamos, respectivamente, los puntos K y L tales que BK = {-AB y BL = ~BC (Hg. 1'12.)
Entonces el t r iángulo BLK es In repl'esentación del triángulo iaóscelcs 1JoL 0 .K 0 y, por lo tanto, la medíana BN del triángulo BLK
es la represontación de la bisectrii riel ángulo A0 B 0 C0 •
Jl/ procedimiento. Como con el segundo procedimiento, hallemos los puntos K ·y L y, a continuación, construyamos KM 11 BC
y LM 11 AB (Iig. 112). Entonces el paralelogramo BLM K es la representación de un rombo y el segmento BM, Ja de la diagonal del rombo que, como sabemos, rlivide por la mil.ad el ángulo de la mencionada figura. Asi, p11cs, la semirrecta B 0 M 0 contiene la bisectriz del ángulo A 0 B0 C0• Al ballar el punto D, que es la intersección de la semirrecta BM con el lado AC del triángulo ABC. ·obtenemos el segmento BJ1 9ue es la representación de la bisectriz buscada.
onsenvACION. P.ara representar el trión¡:ulo AoQ 0 C0 s6lo hem.o s consumido
un parám,e.tf'o ~A ~o : /Joco= 2: ;l). Por ello, ptira realizar l.as conslrucelones
A~K
q¡étrlcas .a6n l énumos ciert.a libertad (en rcsorva 1 ul'I parámetro). No obstante¡
al construir' lá repre\l(lntación de la Ulseotri1 del angulo
.A,A 0 C0 , tomando !)ne
segmento AC el punto atbitrario D, no podemos decir que el segmento BC os
la re_presoniací6n de .la biseetri• del ángulo A 0 8 1 C0 y que al :repres.entar, p. ej.,
la b1aeclrlz del áDgulo B 0 A 0 C 0 , tomando a l elllr en el segmento BC el punto
E, no e.s posible considerar que él segmento AE es la representación de la blseetrii
'B.,A 0 c 0 •
•
En la diagonal BD de la base do un prisma cuadrangular ABCDA 1.81C1D 1 se ha tomado el punto P. Construyamos la sec-
del •naglo
• EJE!lf'PLO ~.
§ 9. Conslruccionts geométricas en el erpacio
i25
ción del prisma con el plano que pasa por la recta CP paralela a la
.
recta A 1B.
SOLuclON. Sea ex. el plano de la sección busc.a da (fig. 113). Como
A 1B 11 a., el plano j}, que pasa por la ~recta A 1 B sin ser paralolQ a a.,
interseca a éste por úqa recta paralela a A 1B. Tracemos el plano
BA1D por la recta A 1B y por cierto punto P de J'a recta CP. Desig·
nemos este plano con j}. A continuación, hallemos la recta por la quo
13 interseca al plano AA 1D 1• Es evidente, que ella ser~ la recta A 1D,
Fig. 113
ya que ambos puntos A 1 y D yacen tanto en el plano j} como en AA 1 D 1 •
Tracemos por el punto P del plano j} la recta p 11 A 1 B y hallemos el
punto Q de i ntersecciqn de Ja recta p con la recta A 1D.
El plan.o a. de la sección se determina ahora con las rectas CM
(M es el punto de intersección de las rectas CP y AD) y PQ. Como
ambos puntos Jlrf y Q yacen tanto en el plano a. como en el plano
AAlDu dicho·s planos se intersecan por la recta MQ. Seguidamente,
hallemos el punto K que es el de intersección de las rectas MO y
A 1D1 . Como los planos ABC y A 1 B 1C1 son paralelos, también lo
son las rectas por las que ellos se intersecan con el plano secante ex..
Así, pues, en el plano A 1 B1 C1 tracemos por el punto K la recta k
paralela a CM y unimos el punto L, que os el de intersección do In
recta k con la B 1 C1 , con el punto C. El cuadrilátero CMKL es la
sección buscada.
Investigación. Como el punto A 1 no yace en el plano ABC, las
rectas AlB y CP se cruzan y, por lo tnnto, el plano secante C' es el
único. En función de la posición del punto P en la diagonal BD y
del tipo del prisma dado, la sección puede tener una forma distinta
i26
Capítulo //. Estereometría
de la mostrada en la fig. 113. En la fig. 114 se muestran algunos tipos de secciones.
EJEMPLO s. En el paralelepípedo rectangulnr ABCDA ,B 1 C1Dt
AB : AD : AA 1 = 1 : 2 : 2. Bajemos una perpendicular del punto
A 1 a la diagonal B 1D.
B
Ftg. 114
s0Luc10N. Advirtamos que la representación del paralelepípedo
dado (fig. 115) es complet.a y está métricamente determinada. En
efecto, considerando el paralelogramo ABCD como la representación
de un rectángulo, para rcpres!lntarlo consumimos un parámetro.
K
Considerando que fos segmentos arbitrarios AB y AD son tales que AB : AD =
= 1 : 2, consumimos un parámetro más.
Suponiendo a ·continuación, que el scgmt>nto AA 1 es la representación de la
perpendicular al p1ano ABC, cons111nimos par.a olio dos· parám.()tros y 1 por {in,
consi(i)lrando que 4D· : AA 1 = "1· : 1, cons1.1mim·o s otro ph'r.il:Piotro. Asf, pues; para
la .reptesento,ción ~t¡l paralelepíp.e!}o dado
A
hemos consumido-· un total Cle cuatro
parámetros\ es. qecir, la r·e présentaci6n
es métricamente determinada. Pero, cntopces, tomando al azar en. B1D el punto L, no podemos considerar
qµe A 1 L ·es~a rep!"esentacíón de la per,pendicutar a B1/).
A'd\icimós dos procedimientos para ·construir A 1 L ..L .B1D.
I procedimiento. H.allemos las depende·ncias que pueden ser utilizadas ·paraº l a ·r!?querida construcció.n, Suponie.n do que A,.8 = a,
obtenemos: AD = 2a, AA 1 """ 2a. Entonces, del triángulo rectángulo
§ 9. Construcclo~J g<ométrico.s
e>t
el espacio
i27
AA 1D obtenemos que A 1 D ·= 2aV2 y del triángulo rectángulo
A¡/31D - B 1 D = ·aa. Ahora calculam·os en qué razón .Ja·:·per-pendicu-
A 1 L clivi:ilírá él..Jado B 1 D del t.r iángulo rectángul.o A 1 B 1 D.
De.la sefücjanza de ·19·s tl'.Íápgulos rectangulos A.1 B 1 L y A 1 BjD, tena.
B,L
A1B1 d d d B L
a
.
mos:
A ;-B, =
B,D,' e on. e
=
Jar btiscac!.a
1
3
Así, pues, B 1 L : B 1D = ~
3· : 3a, . es decir, B 1!., : lJ.I1'D = 1 : 9.
Como durante la proyección paralela l~ .razón de.J!ls longitudes d·e .los
segmentos paralelos se conserva en la representación construida, lo mismo que ~n
original, la base L de la perpendicular
husada debe dividir el segmi,mto B 1D en
una razón B 1L : B 1D
1 : 9.
Ahora, realicemos la construcción. En
cierto rayo, cuyo origen es el punto 8 1 ,
p. ej., en el. r.ayo B1 Cu dispone_mos \) segmentos iguales entre sí y por el punto M,
Ftg. 116
que es el extremo del nrimei: segmento,
trazamos una recta paralela a KD. Entonces, L es el punto de intersección de la recta construida y B 1 D, será la base de la perpendicular buscada.
JI procedimiento. Hagamos uso del llamado dibujo auxiliar. Construyamos en él t riángulo A;B;D' semejante al original (prototipo)
del triángulo A 1 B 1D (fig. 116), es decir, un triángulo con ángulo recto en el vértice A~ y con una razón entre los catetos A ;B~ : A;D' =
= 1 : 2y 2.
(Esta razón se desprende de que según el planteamiento AB :
: AD : AA 1 = i: 2: 2, es decir, si AB =a, AD = AA 1 = 2a y
entonces, A 1D = 2a:V2. Por Jo tanto, AB: A 1D = a: 2aV2 o
bien AB : A,D = 1 : 2lf 2).
En el triángulo A ~B¡D' del vértice A; bajamos la perpendicular
A;L' al lado B;D'. El punto L' divide el segmento B;D' en cierta
razón: B;L' : B¡D'. En esa misma razón dividimos el segmento
B 1D con el punto L.
Como vemos, al construir el punt.o L con ayuda del segundo procedimiento no es preciso calculat· la razón B 1 L: ll1 D.
F..J.EMl'LO 7. En el paralelepípedo rocto ABCDA 1ll1 C,D 1 la base es
un paralelogramo con ángulo agudo de 60° y una razón entre los
fados AB : AD = 3 : 4. Construynmos el plano que pnsn por la
arista AA 1 perpendicular al plano diagonal B 1BDD 1 •
SOLUCJON (fig. 117). Es evidente que sí desde cualquie.r punto de
la recta AA1 se baja la perpendicular mal plan.o [JDD 1 , el plano determino do con los rectas AA 1 y m, será perpendicular al plano
B 1BDD 1 , es decir, es el buscado.
=
Capitulo lI . Eslereomtlrfa
128
Supongamos que trazamos la recln m por el punto A . Como el
paralelepípedo dado es recto, la recta BB1 será perpendicular a
toda recta yaciente en el plano do la base, es decir, es conveniente
trazar la recta m al plnno de la base. Con osto, ya que la rocta m
debe ser perpendicular a l plano diagonol B 1BDD 1 debe tener l ugar
m ..L BD. Así, pues, el problema de la construcción del plano perpendicular al plano B 1BDD 1 se reduce, prácticamente, al problema
de la construcción do In recta m que pasa por el punto A y que es
perpendicular a BD. ¿Es posible que podamos tomar en la recta BD
el punto arbitrario L y considerar que
A L ..L BD? Para responder a esta
pregunta hay que calcular p, es decir,
el ·número paramétrico de la representación de la base del paralelepípedo. S i p = 2, es decir, ln roprcc sentación está m6tricamento defi nida,
el punto L no se puede tomar al
azar. Asi, pues, calculemos los parámet ros cousurnidos para la representación. El cuadrilátero ABCD se conFtg. 117
sid11ra que es l a representación de
un paralelogramo: esta su11osición
no provoca el consumo de parámetros, ya que durante la proyecció n
paralela el paralelismo se conserva. No obstante, la razón de las
longitudes; de los segmentos páralelos con la .proyección paralela no
se conserva, es decir, consideran(to que los segmentos AB y AD,
que hemos tomado al azar, soi:i precisamente tales que AB: AD =
= 3 : 4, consumimos un parámetro. Del mismo modo que el valor
del ángulo no se conserva con l a proyección paralela, el hecho de que
consideremos quo el ángulo BCD es la representación de un ángulo
igual a 60°, acarrea el consumo de un parámetro más.
Así, pues, el número pnramétrico de la representación do la base
del parale)epípedo p = 2, es decir, ningunasotras suposiciones de
carácter i:q~trfco (es decir., suposíción acere!\ de Ja ·pres·irncia de
propiedades .que no se conserYnn con la proyección par.alela) no se
deben totri,ar. De e·sto modo, el punto L, que es Ja baso do ln perpendicular bajada de l punto A a l a recta BD, no se puede tomar al azar.
Su construcción sólo se puede eícctuat después de caléulnr la razón
BL: BD. :
Al re(\lizar dichos cálculos hacemos AB = 3a y, entonces,
AD = 4a. A continuaclón, según el teorema de l os cosenos BD =·
= alf13 y de l.a ecuación AB2 ~ BLi =AD~ - (BD - BL) 2 hallamos qu~. .BL = ;~. En tal cnso, BL : BD = 3 : 13. Después do
1
con·struir el -punto L (co n ayuda del royo auxiliar en el que homos
§. 9. Conrtru~'lonu geom étrica• en el espacio
l20
dispuesto i3 segm.entos iguales) t razamos por él In rec ~a LL1 11 88 1
y, seguidamente, el plano buscado (en In fig. H 7 ést~ -es el plano
AKK 1A 1).
¿
OBSERV·ACfON. Co~o en el anterior -ejemplo, lo conslrucclQn ALiBD so
pod rlo· haber h echo (véase el 1I proceilim!eMo) con ayuda de .u n :dibuio 'i1u1cilinr
(lig. H8). ·construyamos, preclsnm·en.t e, el trhi'i1¡¡ulo A' B' n·· semejan lo ni orlgíilal (prototipo). dél triángulo A BD , es decir,
un trlán glilo con ángulo en el v6rHce_ A' igual n
B
60º y razón entre los lados A' B': .it'D' = 3: 4.
En esto triángulo dol vértice A' bajamos la perL'
pendicular A' L' al lado B' D'. El segmento B'D'
se divide en el punto.L' en clerla raión B' L': B ' O'.
En csl4 misma raión divido el punto L el seg.
mento BD.
ólt .
A'
-
D
s. A BCDA 1 B1C1D 1 es un cubo.
En él se ha construido 111 sección qnc rnsn
por el vér tice B y los puntos P y Q que son
los pun tos medios de las aristas A 1B, y 8 1 C1 • Bnjemos del vó rtice
D una perpendicular a ·dicho plano.
SOLU CION. Corno la representación del cubo (fig. 119) es complotn
y métricamente determi nada (para. dicha representación 5e consumen 5 parámetr os), tomando 111
azar en el plano PQIJ el punto
N, no podemos decir q ue DN es
perpendicu lar al plano PQB.
Con el Iin ele construir la por·
pendicular buscado, construyamos primero el plano diagonal
BB 1D 1 del cu bo. Los planos
BB1D1 y PQB son perpendiculares entre sí. En efecto, PQ es
e l a línea media del triángulo
A 1 B 1C1, por olio PQ 11 A 1C, y,
por lo ta nto, PQ ..L 8 1D 1• A<le·
más, como 88 1 ..LA 1 B 1 y 88 1 _:_
A
..L B,c.. BB¡ ..L PQ o bien
PQ ..L BlJ 1• Así, pues, PQ ..L
Flg. 119
..L B 1D 1 y PQ ..L 881 , o sea,
PQ es per pcndicul11r 111 p lano
B/J1 D1 y , entonces, asimismo el plano PQB, q ue pasa por PQ,
también será per pendicular al plano BB 1D 1 • Pero como los planos
BB1D 1 y PQB son per pendiculares entre sí, lo perpendicular !J11jada
del punto D a la líneo de intersección do los mencionados planos será
la perpendicular buscada. La línea de in tersección de los pl anos
B81D 1 y PQB es fácil de hallar: es la recta BK que ¡)asa por los
pu ntos ~comunes B y K de dichos planos.
EJEMPLO
0- 0290
Capitulo J/. Estereo=lrla
130
Asf, pues, realicemos lns construc.cio~es necesarias en el plano
diagonal BB1D 1 • Como en el anterior ejemplo éstas pueden realizarse
con dos procedimientos. .
l procedimiento. Sen AB =a. Calculemos Jos ladoio del triángulo
2
BK D. BD = a
del Lriángulo BB1K BK = 3n
y del trián-
y
·vz.
gulo rect.ángulo DD,K DK=BDª
~34'.
Si DN ..L BK, DEJZ-BNZ=
= DK 2 -KN2 o bien DBZ-BN 2 = DJ(Z-(BK-BNf·, de donde
1
Bl•r
" = "1a 2
. 3a VÍ
.
BN .. BK = -a '112
3- . - -4- , es dcc1r,
BN: BK = 4: !J. Como con la proye.:ción
paralr.la In razón da las longitudes de
los segmentos paralelos se conserva, en
l a representación, Jo mismo quo en el
original, e l punto N, que es la hose de
la perpendicular que buscamos, dividirá
BK en la razón BN: BK = 4: !l. La
construcción del punto N mediante el
rayo auxiliar BP se muestra en Ja
fJ
Iig. 119.
Fig. 120
JI procedimiento. Dirijámonos al dibujo auxiliar. Construyamos un rectángulo semejante al original del rectángulo B.B1D 1D. es decir, el rectángulo B' B;D;D' con una razón de los lados B' s; :
: B;D~ = 1: y2(Cig. 120). Después, en el lado B~D; construimos el
punto K' de modo que B;K' :
: B;n; = 1 : 4 y unimos los 1>unt os B,
T
e,
B' y K' . A continuación, del punto D' bnjnmos Ja perpendicular
D' N' al lado B' K' del triángulo
Q
B' K' D'. El segmento B' K' se di vide con el pµnto N' e!l cierta razón .
.En el.la m~sma. e·l punto N divide
el ·Ségmentp BJ(.
EJ.EMPLP 9. ·E n las aristas AB y
Cc1 ciel pt,lsma triangular .regular
ABCA 1 B~C1 ; en el qu() AAt: AB
. -1
= 1 : 2; se: toman, · rcsp,MtivamcnFig. 121
te, P .Y Q q~e sgn los puntos medios
de d ichas : aristas. Construyaq¡os
el lugar g~omctri~o de los puní.os que yacen en las superficies del
prisma y equidistantes de los puntos P y Q.
sox;ucJON (fig. 121). La reprcs~ptación del prisma dado es completa
y métricamente definida (en efecto, considerando que el triángulo·
Así,
pues,
=
§). 9. Con$lruccion~s geomitricd~ en el upacio
t3t
arbi.trario ABC es la re,Prese"!l~nció.n del tr.iángulo rcgnl,ar en el oriqP,risumimol' do.'l pal'ál)ietros para la reprcscntac,i ón; co_nsideral)¡.lo q:Uo el .segme1rto.·AA. r e~ la· rcp·resen.tació11 ele la ¡fürpendicular
~a'l. plano,- consu.m!mos dos par.ámeti:os más y, por fin, s_úp0 nierido que
los segmentos AA 1 y AB son las representaciones de ·1~s scg-rilcn~os,
· cuyas . lo.ngitudes c.n el original con·cspondcn a la razó!J 1 : 2, con. sumimos. 4'n pafáiifo~r.<i). Así, pues; pa.rir
,
,
,
··la representación hemo.s consumido ·los
P .C
F T
;
]
. ,C,
cinc.o· .pRrámetros. Por ello, . en la repre'
.
,
sentación ya no pod·emos rea'lizar constru~.ioncs. métrica.s de forma arbitraria.
•t.(
.· Q
Señalemos después que el lugi¡.r geo. . ,·
métrico d!! puntos que buscamos pertep'
! K'
e·
nece a cierto plano que pasa por A1,
punto medio del segmento PQ, y que es
Fig. 122
perpendicular a éste. Así, pues, el problema se reduce a la construcción de. la sección del prisma con el plano,
que pasa· por el punto M y es per_penciicular al segmento PQ. Para
construir el plano sec1,1ñ~e construyamos dos reétas de las cuales, cada una de ellas,. p11sa por el 1Junto M y es perpendicular a PQ.
Construimos una do estas rectas (DE) en el plano ABQ de fol'rna
que sea paralela 11 la recta AB. Como AB ..L PQ, DE ..L PQ.
Con el fin de construir la segunda recta, FK ..L PQ, t racemos
primero en ol plano PCC 1 P 1C1 ll PC. Para calcular la rnzón PK: PC,
en la que el p lano secante divide el segmento PC , íntroducimos un
parámetro auxiliar, p, ej., haciendo AA 1 =;a. E11to11c.es, AB = 2a.
.
PK
PM
S1 FK..LPQ, el t::i.PMKcnt::i.PCQ, y, por lo tanto, PQ = PC•
g_ina~,
dondePC=aV3, PQ=YJ>c2+Qc2=
PM =
V13 .
---ra
Ob tenemos que P K =--¡;ePQ·PM
=
PK:PC= ~31ª_ :a 11 3
8
I'
1/c(i ll3)2+ (; ) 2 = ª -V¡3
3 ..
y
,
13a
VJ . De moclo que
8
o ·bien PK:PC= i.3:24.A&í,pues, constru-
yendo un punto K tal que PK: PC = 1.3: 24, trazamos la recta
MK, perpendicular al segmento PQ. Según Ja construcción FK ..L PQ.
O sea, el ¡1lano secante so determina con dos rect.ns: DE y FK.
Posteriormente, como DE 11 AB, DE es para lela al plano ABC y
de aquí sigue que el plano secante intersecará el plano ABC por unR
recta paralela a DE. Teniendo en cuenta esta consideración trazamos
en el plano ABC la recta GN 11 DE por el punto K. Después, construyamos GE y ND, hallamos los puntos R y Ten l.a s rectas A1 C1
y B 1 C1 y, tras de unir los puntos R y T, obtenemos 1).1,cuadrilátero
NGRT que es la sección del prisma con el plano secante perpendicular al segmento PQ y que pasa por su punt.o medio. De este modo, el
cuadrilátero NGRT es el lugar geométrico de puntos buscados.
n•
132
Cnplt11lo 11 . Esl•rtomtlrCo.
onsP.n VAC ION. La conslruccl6n de la rC!Cta F' K .l.PQ pudo ser renliiadn com o
en los anteriores ejemplos, o sen, con ayuda de un dibujo a\lxlllnr. Es decir,
const.ruimos un rcctñngulo scmcjanto al original del rcclángulo PP 1 C1 C (íig. 122),
o sea, el rcclñngulo P' P\C;c• cnn unn razón do los lados P' P ¡ : P' C' ... t : v'3.
(Si AA 1 =a, en el triángulo equilátero ABC, donde AD = 2a, tendremos
/>C = o. 113. de donde AA, : PC = 1: 113°:)
En dicho rcelÁngulo haUnrcmos Q' que es ol punto medio del segmento C' e;
y, a conti11uncí6n, M', punto medio del segmento P'Q' . Después, por el pun l.Q
Jll' trazamos lo rec ta M' K' j_P'Q' . El segmento P'C' se divide por esta recta
en cic:-tt.a razón />' K' : P'C'. En csln misrnn rn16n dividamos ol segmento PC
con el puulo K. Segu idamcmtc, construimos el pla no secante.
rnoOLEMAS PAHA E L THABAJO I NDIVIO UAL
T. Construcciones mús sencillas en el espacio
'16;1.
Trncr.n por In rcc:t:i clacln 1 el plano Q rnralC'lo a otra rcc:ta dada m.
r,1;t,. C1111~l1,.y:111 •l»s pl:.110;¡ /' y (J pnr:ilnloR clo• 11111tl11 quo /' pnso por In recta
l y (J, por 111 "'· clouclc 1 y m so11 lnM l'C(!t;1s clu1ln11 cruzutlus.
aü:. . Por el vunto A dado Ll'<IC<•ll el pln110 Q 11nrnlelo al plano l' dndo .
!i(iíL Por el punto A 1hulo Lrnccm el plano Q JlCrpcndiculnr o In roctn l prufijadn.
Ml7. Por d 1>111110 11 tlado trnccn In rC!Cla q perpendicular ni plano P dacio.
1;68. Por In recta 1 dada tracen el plano Q perpendicular al plano P dado .
.5fi9. Trn ccn Ja recta q pcr¡>t'ndicular a cado una de las dos rectas l y m
crutnd:is da1las )' que inlcrs<'cn cntln una de C'llas.
11. Lugnrcs 1:1cométricns de puntos
!iíO. Hallen ol lugar geométrico de t-0dos los puntos del espacio equid istan·
tes tic dos puntos A y B distint.os.
571. Hnllcn el lugnr geomútrico de todos los puntos del espacio equidistantes de tl'<'S puntos A. B y C no colincales.
572. liallon el lugar gcomótrico de todos los puolos Jcl espacio equidistantes de •los rcictns '¡ y 12 inlersecnntcs.
573. Na ll on e l ugar gcomótrico rle todos los puntos del espacio equid istantes de c11nt1·0 puntos A, B, C )' [) dndos que no pcnoneccn a un mismo plano.
1i74. U nll en l'I Juga r geomútrlco de lodos los puntos del espncio equidistantes de cuatro punLos it, D , C y D pertcne!Cientos o un mismo plnno.
. 575. Hnllon el lugar geomét rico de tod.os los puntos del espacio equidisl.antcs do. dos recias paralelas 1, y l. dadas.
57.6 . Hallen el Jugar gcomói'rlco de todos los puntos del espacio equidistantes ele ll'cs rccjas 1,. 11 "y 13 dadas que pasan por un punto y no pertonocen a un
mismo J>lnno .•
.5 77. Hnllpn el lugar gcon¡ótrico de todós Jos puntos dol c.spaclo cquidi.stan\'cs do tres rcc\os 1 , l 2 y 1, <lndns que no yacen on un plano ·s 1 !1 1110 , !, 11 !,.
.578. lfallon v1
, lugar ¡feomótrii:o dc· t.Odos los punt.os del espacio que so hallan a la clistll'ncia dnifo del plano P dado.
:>79. Hnllpn ol lugar geomét rico de t.odo~ los punt.os del espacio equidi.s tantes de dos planos raralelos 1'1 y P 2•
:>80. Hallen e lugar geomHrico de todos los punt.os del espacio para los
que la dlf~rcnéfo de los. cuadrados de las" c!istancins desde eUos hasta Jos direrenlts. runtos 11 y 8 es una magnitud CQpstante.
58 •. .Hállen eJ lugar geométrico d.c t 0 dos los p1mtos del espacio desde los
cuales el segmento AB ·dado so ve bajo ángu lo rec to.
13.3
582. Hallen.. cl lugar ,geométrico do todos lgs puntos del j!spacio para los
que la suma de l9s cu11-drnifos de !.os distancias <!esdc el.tos hasta dos ·puntos A,
y B dados es una magnitud constante.
·
583. ·Halleµ el Jugar geométrico de tod<;>s los puntos del ~spoclg .p nr a Jos
" que la .razón de 'las distancias desde ellos hasta· dos pun'tos A y B dn d.os' ·~S ·un¡.i
magnitud constante.
JIL Aplicación <!e ciertos lugares geométricos
c,le punto.s y rectas
'
584. Se dan dos recws ch1zn(ias .11 y' 12 • Pot el punl'o A d'n dot que no pert~ne­
ce ni a 11 , ni a 12 , trace·n la recta que inl~rscéa las. dos rectas <loilas.
585. Por el punto A dado tracen la recta (J perpendicular a idos rccl'a's dadas
cruuidas l 1 y 11 •
586. Se dan t res rectos 11, 12 y 10 que se cru1~~n a ·pa res. 'rracer1 la r ecta q
que intcrseca los .. re<;,las dadas en lates puntos [,1, 1,; y /,0 que /, 1 l, = L 2 f, 3 •
587. Por. el punto A darlo tracen lo rccln q pnratol a a l pl¡¡no f' darlo que
interseca In rectn l prefijada.
588. Trncon h recta q que inlc1·scca lns dos tcct.as l 1 y l 2 dnrlns y pnralela5 a
lo tercera recta l prefijada.
S89. Tracen ia"rectn q, ·qu~ intcrsecn lns dos 1•ecl.as dados cruzadas l y 111,
~rpendicular a l y paralela al plano P dado.
590. Trocen la recta q, que· lntersei;n dos r~ctas 11 y 12 dadas, perpendicular
a la tercera recta 13 y paralela al plano P dado.
S!U. 'Prncen la recta q. perteneciente al plan.o P 1 dado, pnralola a olro p lano
dado P, y que pasa por el punto de intersección de la recta l dada con el pfono
P 1 da<!o.
592. Tracen en el plano P dado la recln q, perpendicu lar a la recta 1 da da , que
no yace en P y que pasa por el pu nto A dado.
IV. Constr ucciones en las represenlaclo11cs
1) Co11structiones en figuras planas situadas en el espado.
593. E n un t.rhíngulo se han traia<lo dos alturas. Const1·uyon el centro do
la circunferencia cirounscJita a dicho triángulo.
5%. La razón de los calotos do ·un triáng ulo rcctñngulo es igua l :i :l: 4.
Construir el centro de In circunferencia inscrifa en este triángulo.
595. Está dado un trián¡¡ulo rectángulo isósceles . Construyan el cuadrado
que yace en el plano del t.riangulo si do lado del cuadrado sirve: a} el cateto
del t riángUlo dado; b) Ja hlpotenusu del triáng11lo d ado.
596. Se dn un hexáiiono rcf!ular. a) Constn\yan In apotema del hexágono;
b) construyan la bisectriz de uuo de sus ángulos exteriores; c) ba¡'cn una pcrp~ndicular desde el ccntroid e del hexágono a una de sus d iagona es menores.
597. El ángulo agud o de un rombo es igual 11 45°. Coni;truyau In al tura
del rombo.
2) S ecci6n dt 1m poliedro con un plnno paralelo a dos rectas.
598. En las aristas AA 1 y DD 1 del paralelepípedo A BCD A ,B1 C,D 1 se hnn
toTI)ado los puntos P y Q, respectivamente. Construya n )3 scc'ción del paralel~p1pcdo con un plano paralelo a las rectas B 11' y A ,Q quo pasa por ol punto K
perteneciente a la arista: a) CC 1 • h) DD 1 ; e) A 1B 1; d) AD .
.
599, En la cnsa AA 1 B1B del prisma trinngul;ir ABCA 1B~C1 se ha t om ado
el punto P y en la a rista ce., el punto Q. Construyan In scccion dd prisma con
134
Capftulo 11. Esttuomtlrfa
un pla'!o patnlelo a las rcclasB 1 /' y A¡Q y que pasa por el punto K pert~necicnlc
o la ari sta: a) AA 1 ; h) DB 1 ; e) AC; d¡ B 1C1 •
000. En lns aristas AA 1 y CC1 del prismn t riangular ABCA 1B C se h:in
tomado los punt os P y Q, res¡1ectivn1ne11 t.c. Construyan la sección
prismu
con un plano quo pasn por KJ. <¡\lo o~ ol punto modio del segmento PQ, y es paralelo n las r~oelns B 1P y A11 .
601. En el plano de In base del prisma cuadrongular A BCDD)C 1 0~ ~o loma
el punto P y en la arisln CC 1 , ol punt.o Q. Construyan la sección <1el pru;m:i con
un pfano ¡>aralelo a las rectas D 1J> y A 1 Q y que pasa por el punto J{ perteneciente a In nrist.n: a) 00 1 ; b) A1 DJ; c) AD; d) B,C1 •
602. En las nristas $A y SC do In pirámide t ri1ml\'ulnr SABC se han tomado
los puntos /' y Q, rcspeclivamcnt-0. Construir la 8ecc16n de la pirñmldo con un
J)lano paralelo n lns rectas BP y A Q y que pnsn por el punto K perto11ecienle n la
11risla: a) SA- b) S B; c) AC; d) BC.
603. En las nristas SA y SC do In pirámide triangulnr SA BC se han tomado los punt.os /' y Q, respc?Ctivamente. Conslruyon Ja sección de la pirámide
con un plano que pasa por K,: que es el punto medio del segmento PQ, y es par.-lelo a las r«tas BI' y AQ.
60>1. En lns nristM SO y SC de In pirflmi1lo cundrnng11lar SAflCO se han
1omn1lo los punLos /' y Q, rrs¡1ccLivnmc11le. Construyn11 In ~ccciún 1lc In pirí1noiil1•
con uu plnuo qut1 pasa por J\, que es t1I punl.I• medio dol scgmcot<i l'Q, y os pnrnldo a lns recia~ A 1' y DO.
605. En bs aristas SA y SC de In pirámide cuodrangular SADCD se hnn
tomndo los punt.os P y O. respectivamente. Construyan fo sccción ele la pirámide con un plono paralelo a l:ls rectas BP y AQ y que pasa por el punto K pertrnecicnle n la ar ista: a) SA; b) AD ; c) DC; d) B C.
606. En las aris~s AA 1 y CC 1 del prism;i cundran¡:ular ABCDA 1B 1C1Di
~ hnn lomado los puntos I' y Q, respectivamente. Construyan In sección del
prisma cou un plano equid istnnlc do uo plano que pasa por 0 11' paralelo a DQ
~·del otro plano que pnsn J>!'1r DQ pnrnlelo a 0 11'.
607 . En el psism:i cuadrnngulor ABCDA 1B 1 C1D, so han trozado las diagonales AC y A,B de las cnrns ABCD y ABB1Aa> rcspccLivamcnte. Construyan Ja
~ccción del prisma con 110 plano ~qnidist:inte de un plnnn que pasa por A C p:irRldo n A 1D y de otro plano que pnsa por A 18 rnrnlclo a AC.
Ael
3) Co11•lriud611 de la perpr11dicular a una rnlo y rk la ptrptttdk11/ar o un
pin no.
608. En In 11rist11 SB 1lcl lutracdro regular SA BC se ho lomudo el punto
m~dio K do Ju nrisln. a) Baj11n desdo el punt o K por¡1cndlculares o las aristas
SA y SC; b) hallen el punto do lnlcrscccion do la altura SO del tetraedro con un
plano que paso por Jos perpendiculares bajadas del punto K 11 las aristas SA
y se do! tcLl'acdro.
6()9. En el p:iralclepípcdo rcclnngular ABCDA 1 B,CJ.D 1 , en el que
AB: AD: A Al= 3: 2: 1, construy?n rectas que pasen por el vértice 8 1 y
que s~an perpendiculares a. )ns rcctns DC1 y DA 1•
610. En In nrlsta CC1 del cubo ADCDA1 B 1C 1D se t omad punto P. Dajen
una- petpon1lletilar ~os<lo el vó1·Licc A 1 n· la rect.D /> ·si: a) CP : PC1
1;
b) C.P : PC1 - 1 : 2; e) CJI : PC 1 = 2 : 3.
.
611. En el tctrncd ro rcgulnr SA BC bajen desclc K, que es el punto medio
de In aristó A C. una p11rpendlculnr al plano de la cosa SBC.
612. Eia la pirámide cuadrnn~ln r regular SABCD la altura SO es igual
al lado de fo bue A B. Desde el vertice D b;tjen una per pendicular al plano de
In cnra SBC.
ffl3. En lll base de un prisma yace el· triángulo equilátero ABC. tas caras
A BQ1 A 1 y A CC 1A 1 <!el prisma son rombós con ángulo oguclo de 60°. J3ojomos una
porpcnaii;ulor dcsrte el puato P, t omado en In arista AA 1 , en I~ diagonal IJC 1
íJ
= \:
§>D. CoMlruulonts gtomilricas ·tn tl tlpncio
135
de la cara BB,c,c si: a) AP: PA, = 1.: 1; b)-.AP: PA, = l: 2¡ e) AP: l'A,=
= 2: 3.
•
614. En el cubo ABCDA-1B 1C1D1 Stl ha truado un plono por los vértices
R, 'C1 y D; ,En.la arl~tá A 1 B~ ..se tom!I el pun't o.P .•. BnJen una· per'pendiculnr del
pun.t o p· .ol plano BC,D si : a) A,I': PB, =. 1: i; b)' A,,P ¡ PB, >= 1: 2;
e) A p : :_PB·, ... 2 : a.
..
.
,
~15. En la arista A 1 B¡ .dol cubo ABCPA 1 B,C 1D 1 se toma •el punt-0 I' y
por los punt.o!i P, B y C1 se· traza un plano. Bajen una porpcndicl!far del vértice
D al plano·PB.C1 si: a) A 1P: PB 1
1 : 1; b) A 1P : PB 1
1 : 2; e¡ A 1 l' : 1'81 = 2: 3.
'
616. -En el parol~lepípe\lo ADCDA 1D1C1D 1, con una razón¡ do las aristas
AD: AD: AA 1 =1:2~1,. por los vértices ,O; C1 y D se trnµ ¡un plano. En
la- arista 4 1 D1 so ,ha tomado el punto· P. Bajen una perp~ndloulor .ilesdo el punto
P al plano
DC1D si: a) A,P: PD 1 = 1 : 1; b) . A,P; ! l'D, = ·t : 2;
c)A 1P:- PD1 =2:3.
:
617. En la pirámide triangular S ABC la base es el l r iilng\Jln r1?Ct6ngulo
ABC con el ángulo rect.o en el vérUce C y razón de los coletos A C : BC
3 : 4.
La al tura de la plrámJde se proyecta 11n el punto C y es igua l a la hipotenusa A 8.
Del punto C. liojcn una perpendicular ni plano SAB.
=
=
=
4) Co111truccl6n de la ucci6n del poliedro con "" plano perptndumlor a la
rtcla. dada qut para .por el punto dado.
618. En los arlslns DD.1 y CD dol cubo ABCDA>O,C D, se hnn loino.do
los puntos P y Q, respectivamcnlc. Conslruyan
secciun de\ cubo con un flano
perpendicular a la recta C,Q que posa por el punto P si: a) DP: PD 1 =- : t ,
DQ: QC = 1: t ; b) DP: l'D, = 3: 1, DQ: QC - t: t ; e} DP: PD 1 - 3: 1
DQ: QC = 3: I ¡ d)rDP; PD 1 = t : 3, DQ: QC = 3: l.
619. En la piramide &riaogular regular SABC la altura es ig1rnl o l lodo
de la base. Construyan la sección de la plrámide con un plano perpendicular n la
arista SC que pasa par la arista AB de la base.
620. En la ntista DD 1 d'o l pnralclcpípcclo rectangular ABCDA,TJ1 C101.
en el que AB: AD : AA = 1 : 2: 1. se loma el runto /'. Construynn la
scc-ción dftl paralelepípedo con un plano, perpendicular a 1:1 recta c,r, quo
pasa por el punt.o K que es el centrolde de Ja caro AA 1 0 1D si: n) DI':
J>D,- 1 : l · b) DP : PD ,
1 : 2: e} DP : J>D 1 '"' 2 : 3.
621.
las aristas B8 1 y DDk riel parnlelepl¡>o1lo recto ABCDA 1B 1C,D,.
en el que AB: AD : AA,= 1 : l ; :1 y el ángulo BAO es igual a 60" se han lomado los puntos P y Q, .respectivamente. Construyan la sección del paralelepípedo con un plano, perl)enclicular a 111 rec\n A I', que pasa 'p or el punto Q.
si: n) 8P: PB,
1: i, DQ: QD 1
1: i ; bJ BI': PJJ 1 = '\: 1, D<J: QD, ~
- 1 : 2; c) BP : PB, "" 1 : 2, D<.! : QD, = 2: l.
ra
=
i:.
=
=
5) Con.<lruecl6n del lugar
g~mélrico
dt pw1to1 equidistonles de lo.t puntos
dtul.0$.
622. En el prisma triangular regular ABCA 11J1C 1 AA,: AC = ·1 : 2. Construyan el lugnr goométrico de los puntos que yacen en Ja suporíicic del prisma
y cc¡uidistnntcs clo los puntos: n) A y C~; h) l' y
donde /> es ol punto 111~dlo
110 la arista AB; e) Q y C1 , dondo (J l'S ul punto meclio de 111 nrista AA,.
623. Construir ol lugnr gcomótrico •le los puntos pcrteuecic.olcs a In superficie de •tn cubo y equldist;intcs de los dos siguicntos puntos: a) J> y Q perte.ncelentes 11 las aristas A B y CD, respectivamente; b) p1 y /J que son los extremos
de In diaguonl B 1D del cubo; e) M y K, que son el centroide de la cara AA 1D;fJ
y el punto medio de In arista BB1 , respeclivnmcnte; ti) 8 1 y O, que son Jos
vértices de cubo y el ccntroide de fo cara A BCD. respec1ivaménle.
624. En las nristnsA 1B., C,C y AD del culm ABCDA,o,c.D, se lo111on los
punt.os medios P, Q y R de cstns n1·istns, respectivamente. Conslruynn los 11un·
e,.
Capitulo I1. Estereometría
136
tos que yacen en lo sup0rficio del cubo y equidisl!\ntes de Jos siguientes puntos:
a)D.D,yl';b)D 1,PyQ;c)P,QyR .
625. En las aristas AB y DD1 del cubo ABCDAJ.B1 C1D 1 sc toman los puntos medios l' y Q de estas oristas, respectivamente. 1.:-0nstruyan un punto equidlstnntc de los siguicn~s cuatro puntos: o) A,. C1 , l' y Q; b) A, C1, P yQ; e)
D,C,.PyQ.
626. En las continuaciones de las aristas AB y DD, del cubo ABCDA 1B 1 C1D 1
se toman tnlcs puntos P y Q, respectivamente, que AP : BP - 3 : 1 y
D 1Q : D? = 3:1. Construyan un punto equidistante de los siguientes cuatro
¡11mtos: a} A,. C 1 , P y O; b) A, C1 P y Q: e) D, C1 • P y Q.
627. En las aristas /JB 1 y CC 1 del prisma triangular regular ABCA 1 8 1 C 1
SCJ toman. respectivamente, P y Q Q\lC son sus puntos medios. LllS caras Jal~rales
del prismn son cuadrados. Construyan el lugnr geométrico <le los
puntos equidistantes de los siguientes tres pu.n \os: a} A, By Q¡ b) A, B1 y Q;
Q.
628. Dos triángulos rcctáng\llos isósceles igua les ABC y ABC, están situados de tal forma que sn cateto AB es común, mientras que CC 1 = BC. En
las rectas BC, A.8 y AC, construyan puntos equidistnnli!S de los puntos C y e,.
GW. En ol cubt> ABCD/t 1 8 1 C1 D, sn t.ra>.an las rlfogonalcs AD, y De, <le
las caras latc.r alcs . Construyan el lugnr geométrico do los ¡motos óquidistant('s de las rectas 11D 1 y DC,.
630. Construyan ~1 lu~ar gcom6trico de los puntos equidistantes de las
rectas que contienen las artstns B,C, y CD del prisma recto ABCDA 18 1C1D.,
cuya busc ('5 el c.uadrado ABCD.
e) A . P y
§ 10. RECTAS CRUZADAS. ÁNG ULO ENTRE UNA
RECTA Y UN PLANO
EJEMPLO 1. En el tetraedro regular SABC el segmento DO une el
punto mcclio D de Ja arista SA con el ccntroide O de la cara ABC,
mientras que E es el punto medio
de la arista SB. Hallemos el ánguS
lo entre las rectas.DO y CE.
SOLUctON. Supongamos que el
cuadrilátero S ABC con sus diagonales (fig. 123) es la representación
del tetraedro dado. Es fácil cerciorarse de que la repreS'entación
·C
A
es completa y _métricamente d efiní~
da. Sea D el punto medio d.e la aris·ta SA, E. el pun.t o medio de la
arista S B y el punto O el centroíde de la cara ABC. Construyamos
B
las rectas OD y CE. (Estas consfig. 123
trucciones no provocan el consumo
de parámetros.)
A continuación, co1)struyamos un áng\1.LO igual al ángulo entre
fas rectas DO y CE. Con este fin, hallemos el punto K que es la iníers.e<;c;ióu qe las rectas CO y AB y unátnoslo con ·el punto E. En el
plano CEK, por el punto O, trazamos la recta OF 11 CE. Entonces,
§
Jp. Rectas cru:adas
13•
LDOF será igual al ~ng:ul.o. entre las rectas DO y CE. fa~¡i abreviar,
hagamos LDOF = x. Para hallar x cal.c ulemos l'os lados. del
t::.FOD y, seguidamente, empleemos. el teorema. de'. los coseno¡¡.
Al realizar los cálculO's introducimos tin -paráme,Jltro atixjliar;
)laciendo AB= a. Entonces, como D es· ~l punlo medio ·de. la arista ...SA y el triángulo SAO es rectáng\l)o, OD =
+
SA = ; , mien-
+,
tras que de. la semejanza de t::. KFO y ó K EC: ~ = .~: =
3
es decir, OF=+CE=ª( . Después, en · el ~DKF .DF 2 =
=DK 2 +KF2-2DK ·KF·cosLDFK. Como D, E y K son los
puntos medios do los lad os del b. SAB regular, DK = ~ SB =
=
~
y LDKF=60º. J\riemús,
KF=flrn=;;.
31
De
t::. DKFobte-
nemos que DF2 = ~ a2 • A continunción de t::. ODF obtenemos: Df2=
~
=0F2 +0D2-2QF.ODcosx,
do
• f
donde cosx=
"!s 3
EJEMPLO 2. E~tádado el cuboABCDAlB1 C1D 1 cuya aristR. es igual
a a. En la arista DC se ha tomado P que es cJ punto medio de ésta.
Hallemos la distancin entre los siguientes
8,
pares de rectas: a) AA 1 y D,P; b) AD y
D ,P; c) BD y D1.,f'·
SOLUCION. Sea la figurn ABCDA1B1C1D1
A,,_....,..,__---.<
la representación de un cubo (fig. 124) y
'' .l
sea ésta completa y mé'tricamente definida,
\ .l'
.
ya que al construirla se hnn consumido los
\"ºfa-- , e
cinco parámetros. Como en estereomctr~a se
Q)¿.- _::..:...~
puede considerar que la repr~entación de
la figurª'- original <Il 0 es cualquier figura c!J.
A
D
semejante a la proyección paralela 11o <l> 0
Fig. 124
en cierto plano, es ·posible suponer que
ABCDA 1 8 1 C1D 1 es la representación de un
cubo con arista igual a a. En In representación obtenida construyamos P, el punto medio del scgmeut.o DC y unamos P con D 1 .
a) Es evidente que como AlD1 es la arista del cubo, A 1 D 1 ..1.
J_ AA¡ y A 1DJ .es perpendicular al plano CDD11 es decir, A 1 D1 ..L
..1. D 1 P. Pero, entonces, A,D1 =a es Ja distancia que buscnmos.
b) Como el punto P yace en el plano A 1D 1P y en él ABC, estos
planos se cortan por una recta que pasa por P. Sea esa recta PQ.
Como A 1 D 1 es paralela ni plano ABC, PQ ll A ,D 1 • Además, construyamos A 1Q. Es fácil mostrar que AD es paraleln ut plano A 1 D 1 P
y, por lo tanto, la d istanc.iR. entre las rectas AD y D,P es igual a In
distancia desde la recta AD hasta el plano A 1D 1 P.
138
Capitulo JI, Eslueomtlría
Como la construcción de la perpendicular del punto D a la recta
D 1 P en el plano D 1DC debe realizarse en la representación métricamente definida, no es posible tomar al azar en Ja recta D 1 P el punto
H y considerar que DH ..L D 1 P. Realicemos la construcción del
modo siguiente. Construyamos el segmento DM, donde Mes el punto
medio de la arista CC1 . Entonces, de la igualdad delos triángu los
e, DD1 P y DCM so despronde que LDPH =
~--::::--~
= LCMD, o sea, LHDP
LDPH = 90º,
lo que significa que en el triángulo DPH
el LDHP = 90º. Así, pues, DH = D 1 P.
Mostremos que D H es perpendicular al
T
plano A 1D1P. En efecto, como AD es una
C
perpendicular al plano DCC., DH ..L AD y
DH ..L PQ. De modo que, DH ..L D 1 P y
DH ..L PQ, por lo tanto, DH es perpendiA
cular ni plano A 1D 1 P. Pero, en tal caso, la
Flg. 125
longitud del segmaoto DH es la distancia
buscada. Expresando con dos procedimientos ol área del l::.DD1 P, obtenemos una ecuación con relación n
+
·¡15
DP·DD .. de donde hallamos que Dií =
c.) En el plano ADC (fig. 125) const.r11yamos la recta EK que pasa
DH: DH·D 1P
T·
=
por el punto P y poi· E, que es el punl•• medio de la arista BC. Entonces, BD 11 EP. Construyamos el plano B1EP. Como BD 11 EP,
BD será paralela al plano B 1 EP, y, de este modo, ln distancia buscada será igual a la distancia desde la 'recta BD hasta el plano B1EP.
Con el fin de' hallar dicha distancia ·i racemos del punto O una perpendicular al plano B 1EP. La base de la perpendicular, es decir, el
punto S, no puede ser tomado al az"ar, ya que la representación es
métricamente de{inida. Realicemos la construcción de S del modo
siguiente. Construyamos F que es el punto de intersección de las
rectas OC y EP, a continuación, el punto 0 1 en el que concurren las
rectas A 1C1 y B 1D 1 y, después, \lnamos los puntos 0 1 y F.
A continuación, tomemos en la arista CC1 tal .p unto T que se
·1 i · .;,, · .ld d er
OF
cump a a
1~ua
a
(J{}"= 001 .
a ·¡/2
Ya queOp .o=<~, .OF=-;¡¡-Y
!
a ·¡f2
a
001 =a, C'f=t;·
Construy:imos -el seg1ncn1 o OT y el. punto S que es el do interst?cción de
OT ,con O,F. '
Como de pcuerdo ·con fa construcción los triángulos, rectángulos
OO¡F· y ocr ~Qll seJilejantes·, LO,FO == LO'I:C =; LCOT y, en
L,COT = \JOº ·y, por.lo~tanto, OT ..L 0 1 F. Aho•tal.caso, LQ'¡FO
ra, desmostrpmos que ·OT es · perpeqdicular al plano B 1EP. Como
B 1D 1 es p!irpeildicul"ar al .plano A-A 1C y OT yace en el plano AA1C,
OT ..L B 1D 1•
+
§ JO. Rtcta.s erw:adiú
139
Así, pues, OT ..L B ,D, y OT ..L O,F. De modo que OT es per pondicular al plano B ,EP. Pero. entonces, la largura del segmento OS
es la distancia· buscada. Expresando con dos procedimientos el área
del triángulo 0 1 0F, obtenomos una ecuación con relncióo a
OS: OS . O,F = OF-00 1 de donde,
3ª 1fl! 1 11
f i , 00 1 =a, O1F " " "
~
ya que OF = -ª ·1-- - , 1a amos que OS ='a·
4
4
La superficie lateral de la pirámide cuad.rangular regular SABCD es dos veces mayor que el área de su base. En las caras SAD. y SDC se han t ra:i;ado
s
las medianas AQ y DP. Hallemos el ángulo entre ellas.
SOLuctON. I procedimiento. Sea
la figura SABCD la pirámide
darla (fig. 126). Esta es una reprcsentnci(m C!lf!!pletn y pnrn rcnliznrln se han gastado los cinco
parámet ros, es decir, está métrie
camente definida. En efecto,
considerando que el paralelogramo A BCD es la representación ele un cuadrado, consumimos dos parámetros; considerando que SO es la representación A
de un segme nto perpendicular al
Flg. 126
plano ABCD consumimos asimismo dos parámetros. Suponiendo
quo S1a 1 = 28 A »CD• de hecho considera mos que la al tura de In
cara lateral es ig\1al al lado de su base. (Tomando SE, es decir, la
altura de la cata lateral, igual a h y el lado do la base de la pirí1mido DC = CJ, de la igualdad s,~, = 28 A BCD• obtenemos: 4·tah =
= 2a2 , de donde se desprende que h = a .)
Asf, pues, para las construcciones auxiliares de carácter métrico
no hay parámetres libres. Señalemos, que con el procedimiento elegido para la resolución no habrá constr ucciones q ue requieran el
posteríor consumo do parámetros.
Con el fin de hallar el ángulo buscado, incluyámoslo en algún
t riá ngulo. P . ej., esto se puede hacer del modo que sigue: uniendo
los puntos Q y C (CQ será la mediana del triángulo SCD) y designemos con F el punto de intersección de las rectas D P y CQ. En el triángulo AQC, por el punto F, t razamos la recta KF 11 A Q y, por fin,
uuimos los puntos K y D.
Como según··.la construcción KF 11 AQ, el á ngulo entre las rcctns
KF y DF es igual al buscado. Para abreviar s upongamos que
LKFD = ::c. Para hallar ::e calculemos los lados del t riángulo DFK.
EJEMPLO 3 .
Capitulo II, Esltr•Omtlrla
140
Introduzcamos un parámetro auxiliar, haciendo para ello DC =
= a. Entonces, como dijimos .a l calcular los parámetros de la representación, SE=-a. Pero SE es la mediana del triágulo SDC, o
sea, FE= ; . Entonces, de l triángulo DEF, donde DE .... ; ,
hallamos que DF =
ª 1~ 13 .
·
.,
l
SCDlCJante
al tnanguo
Másadelanto,yaqueel triánguloCFKos
CQA , KF
CP
p eroCQ=aY•POr
CF
2
Io
AQ = 'CQ·
2
2
tanto. KF= TAQ. Mas AQ=DP y, por ello, KF= "3 DP. Ahora,
tomando asimismo en consideración que DF = : DP, llegamos a la conclusión dequeKF -DF, os decir, KF= 4
~!a.
El tercer Indo del trián2
2
,,-
gulo DFK so halla dol triángulo CDK, donde CK =s· AC =a a y 2,
CD-a, L DCK =4!iº. Oht•,nemos, Df(t = CK2 +DC'l.-2CK-DC x
5a2
x cos 45°, de donde DKZ = - 9- . Así, pues, conocemos los tres Indos
del triángulo DF K. Aplicando a ésto el teorema do los cosenos,
134'
obtenemos: DKZ = KF2+FD2 -2KF-FDcosx o bien Sci•
g- = -a¡¡+
t~
aVH
+ 362 --6-
= nrccos
3
13
aVR cos x, de donde cos x = ;a
3
· - -6
, es
decir, x -=
.
ODSERVACJON. I ndiquemos una vía más pnra inelulr el ángulo buscado en
el tr iángulo. Por el punto Q en el plano SDC trazamos la recta QT 11 DP y unamos el punto T con A (fig. 127). Es cviden\e, que LTQA es igual al busco.do.
e l puede ser hallado en al triángulo TQA. Podemos oporar de otro modo. Hallar
en el triangulo AQL LAQL que completa el Angu lo buscado hast.a 180" y, después, hallar el buseado.
I I procedimiento. Jntroduzc.a mos en e\ espacio la base i:ectungular
cnrtesiana O~fk, cuyos vectores t; y k ·son los "vectores unit arios de
Jos ejes:Ox, C{Y y Oz (fig~'8). A continuación; después dó descomponer los ve«tdres DP y AQ por lo.s vectores de dicha base hallamos
cos L (AQ; EJP). In troducimos uo pruúmco~ro auxiliar haciendo
T
DC=a.
En~ncos,
OD = OC= ª
~i
y del triángulo SOE, d onde
a ¡13
SE=a y 0/f =-:¡-, obtenemos que SO= -z- . Ahora, oxprosemos
los vectores
;
a
PP
y AQ p.or
l, j
y
k.
DP """OP -OD, donde
=+oc+ ~; os= 4! i + a ra k, 075 =
"
r 7.
2
OP =
§ JO. Rtetiu cruzados
EntonCf>.S, DP""
-
-
11
ri rí /+ r
7-
~
-
141
3
11
IJ
-
t -
f
k.
A
a
-
continuación,
¡12 -
i+ - y4 -3 -k,.
AQ = OQ-OA, oonde OQ = -:¡: OD + TOS= - 4 '
.
s
11
:
e
;I
Flg. 127
OA.,., -
ar~ t.
Flg. 128
Así, pues, AQ=
rz
ª
('.Onsig uiente,
cos L (-AQ
- , D-P-)
t +~ f+
ª
rj
k.
Por
= -A=Q,_·-"D=P~ -1AQI1DP1
a
VÍ
nVi
a 112.
a lf i +a l/S
-4- ·---¡----¡-·-¡-
3
= 13
,
-
-
o sea, L(AQ, DP) =nrccos
4 113
-4-· -4-
3
13 .
En la base de la pirámide cuadrang ular SABCD yace
= 1 : 3. Cada una de
las aristas laterales forma un ángulo de 60° con el pla-no de la base.
Hallen el ángulo quo forma la recta DP c.on el plano SCD si P es
el punto medio de la arista SB.
EJEMPLO 4.
un rectángulo con razón de los lados A D : AD
Capitulo JI. Esltreometrla
142
SOLUClON . Supongamos que la figura SABCD (fig. 129) es la representación de la pirámide dada. Ella es completa. Caiculernos su
número paramétríco. Consideremos que el paralelogramo arbitrario ABCD es la representación de un rectángulo (es decir, consumimos un parámetro) con una razón conocida de los lados (por consiguiente, consumimos un parámetro más). Bajemos la perpendicular
SO del vértice S al plano de la base (es decir, para representarla
tonsumimos dos parámetros más) y unirnos el punto O con el vértice A . El ángulo SAO está formado Pº! la arista SA y el plano de
la base. Vamos a considerar que el
s
ángulo SAO es la representación
de :un ángulo que en el. original es
igual a 60º (con ello, consumimos un
parámetro). Así, pues, para la representación hemos consumido un
total de cíuco parámetros, lo quo
e significa que la representación
es métricamente definida, es decir,
que las posteriores construcciones
de carácter métrico no pueden ser
rcalízadas arbitrariamente en esa
A
representación.
Ftg . 129
Seguidamente, unimos el punto
con el vértice
de la base.
Entonces, LSAO = LSCO = 600 y los tr.iángulos rectángulos SAO
y seo, que tienen el cateto común so y un ángulo agudo igual, son
equiva lentes, Esto quiere '1ecír que ·OA .., OC, es decir, O es el punto medio de la diagonal AC y, esto significa, que es el punto de intersección de las diagonales del rectángulo ABCD. De la igualdad
de Jos triángulos SAO y SCO se deduce que SA = SC, o sea, que el
t:. SAC es isósceles. Como LSAO = LSCO = 600, también
LASC = 60º y, por lo tanto, ol t:.SAC Cl\ equilátero. Por analogía,
el t:.SBD es: ai;imismo equilátero.
,
Pasemos 1ahora· a la coni;t'l.'.Ucci6n del ángulo buscado. Como el
ángulo· ~.nl!'efuna recta y· un pl,ano es igual al ángulo formado por la
recta ·c<)n
proyeccion ortogonal sohre di'c\ió plano, bajemos una
perpendicular' de cualquí'~r pu.oto de la rect~ PP al planc;> SCD.
Con este fin,¡tracemos primero fa. ap.o tema SM de la cara ·SCD (Mes
el. punt.o me?io de la allistn CD) y unimos el punto M con el O. En
tal ·caso, con;io ·SM _L CD y OM es la proyección de SM en el plano
ABCD, OM U... CD.. Así', pues, CD _L SM y CD .l._ CM, lo que quiere
·4écir que Ja ~ecla CD será perpendicular a cualquier recta que yuca
en el plano ~OM.
P.or esta tazón, si ·desde el punto K bajamos la perpendicular KL
;da apotemá SM; KL será tambiiírr perpendicular al plano SCD y,
por consiguiente, LKDL será: el ángulo buscado. Hagamos LKDL =
o
s\4
e
143
§ JO. Recios cr11:ada1
= %. Como ya hemos dicho, no podemos realizar al aznr n.nevas construcciones de carácLer métrico en la repre$enLación de la pirámide
dada, es decir, tomando ni azar en la 11p0Lema S,1f el pimto L. no se
puede decir que el segmento KL se va a con$ídcrar com.o. lo representación de la . perpendicular o SM. Parn consLnür ·KL ..L SM,
construyamos primero. ON _¡_ SM. Esto so pued·e hac.cr si cnlculnmos
en qué razón divide el punto N la hipotenusa SM en el b.'SOM.
Haciendo AB =a oblenemos conseculivnment.e: BC """3a, OM =
:la
,,~, "ll ii:i
" ·1130 s
= T· AC=a y 10, SC=a ,, 10, OC=-2- . S0= - . 2- , M=
= "~39 , MN = 3a
es decir, MN: SN = 3: 13. Después ele
construir tal punto N que MN:S/11=3:13, trnzamos Kf,llON.
Hagamos L K DL =- :i:. Para su determinnciún es suficicnlu 1:alc11lnr c1111losquicra dos Indos clol t.ri:\ngulo rcct:íngulo DKL, p.ej. ,
ol cnt.clo KL y In hipolo1111sa DI\. De Ja smnuj11111.:1 do los !::; SJ\f.,
K.f,
SK
O
3a
S
a ll :'fü
y e::, SoM, obtenemos: OM ::.: SM, donde Jl,f = T , M = - --
r3li ,
2
el punLo de inrsección de las medianas SO y DI'
.
SBD
2 SO
.
KL = OM·SK
del triángulo
' l)"K =a
= -a 1130
3 - . :\si' pues,
---sM' =
a 1130
y, como K
tlS
= --;¡g·
Como el /.:::. SBD es cqnilátero, DP = SO. Poro DI( = ; Df' y
2
o 11 iili
SK=-sSO . Por lo lnnLo, DK = SK = -
.
- . Por fin. del t:>DK IJ,
3
KL
tf3ii
donde LDLK =!)O• , lcucmos senx=OK' "" --¡J•
rs 1lccir,
X=
= nrtsen 1' 39
13
EJEMPLO s. En la ~ase de la pirámide c11ndrn11g1ilnr SABCD con
el vértice en el punto S ynce un cuadrado, mientras q\\e su alt11rn se
proyecta• en el punto A. El úrea de la cara SBC es dos veces mayor
que el área de la co?R SAB. Hallemos el tíngu lo entl'e la altura AF
do la c:ira SAJJ y el plnno diagonal SAC.
SOLuc10N. Supongamos que la figura SABCD (fig. 130) es la
representación de la pirámide dada y que elJn es completa. Calculemos ol número de parámetros consumidos ¡>nrn esa reprcscntnción.
Se considera que el paralelogramo ABCD es la rcprcse11tación de un
c11adrado, esto son dos parámetros, consideremos que el segmento
SA es Ja representnción do In per pendicu lnr al plnno de Jn buso, es
decir, dos parámetros. El área de la cnrn SAB se to¡na dos veces
(}-so.
menor que la de la coro SBC, es decir, ( -}SA ·AB):
BK) =
= 1 : 2 o bien que SA : SB = 1 : 2 y, do este modo, consumimos
Capítulo JI. E$lereomtlrfo
u n parám~tro más. Así, pues, p ara la representación hemos consumido 5 parámetros y, por ello, ést a es métricamente definida. Esto
significa •que ahora ya oo podemos tomar a l azar el punto F en la
arista SB y considera r que el segmento AF es la representación de
la altura de la cara SAB. Al mismo tiempo, con el fin de determi nar el ángtdo buscado, l1a de
construirse la a ltura AF (a cont i nuación, será también preciso
construir. la representación de
su proyección en el plano SA C.
Para efectuar laS construcciones necosario.l! y los posteriores cálculos, introducimos un
parámetro auxiliar haciendo, p . c.,
8
AB =a. E n tonces, como en el
triángulo SAB SA=-} SB, ol
L SBA = 30° y, por lo tanto,
f)
si AF _L. SB, en el triángulo
rectángulo ABF tendremos AF =
Fig. 130
= ~ AB, osca, AF= ~. En tal
a i/s
'
2a-113
y S F= - - . Asi, pues, SF : SB=
Cl\SO, ,,"A = -a i/3
- , SB =
3
0
3
= -a ita
6
o b'1en SF: SB = 1 : 4, de donde queda clnra la
:. -2a if3
3
construcción de la al tura AF.
OBSERVACION.La conslrucción de lo altura AF se hubiera podido reali~ar
asimismo mediante un dibujo auxiliar en el quo estuviera representado un triángulo original SAB. Acerca de .scmej.inte procedimiento do construcción se habló
con detalle en los ójcmplos 7, 8, 9 del aparwdo anterior.
Más .ndelaqte, ·e s pi;eciso construir la. pi;oyecci9n de Ja. altura AF
ei) el plano' SAC. Esto se puede. r.e!llizar _d~l modo siguiel).~~ . .Por el
punt o F ¡¡n el plano S'AB 'trl!~árµos '~1)11 ·rec~a paralela a l_a r.é cta. SA.
'El ·puntct de intersección ·de :ésta é,on-.la .r~ta ¡JB se d11signa· con K .
En el. .pl¡nio Al)C.,· por el punt.o. ·K; tra?-amos. una· .:i:\?Cta paralela a
. BD . .Designem,os con M el punto· de .il;ltiirsecci6n de AC con la nienci_o nada recta. Por el pun.t o M en. el plal)O S.AC t.razam.os l;i. recta
m, por la q1,10 -se· intcrse«an lo!.! plal).QS SAO y Fl(M. Ya que t anto
el plano SA{: como éi plano FKM so·n pcr pendiculali'es al plano
A.BC, es evidente que su lí'nea .de intersección, es decir,. la recta m,
será. perpendicul ar al plano 4BC,_o· sea, m, ·11 SA y 1n 11 FK. La intersección de la 'línea m con el plano SBD será designada con L.
A c.ontinuaci"6J;l, unimos los .p untos F y L.
§ 10. Re>la• cruzada•
t45
El segmento FL represen tará la perpend icu l1tr al plano SAC.
(En efecto, FL llMK, pero MK
perpendicular al . J>lano SAC
ya que MK ..L AC y MK ..L SA.)-Pero, entonces, AL c5 la proycción
de AF ori el plano SAC . y, por lo .tanto, L FAL \IS el .án gulo
.busc(ldo1 P.á'ra ;abrevia.r';. hagamos· ,L f, AL = x. 0011 ill f~n de deter minar x ·es ·posi'ble hallar PL· y, seguid amen te, sen x == ~; . Mús
es
arriba, ya hallamos AF= ~ .. P ara .calculor FL ad v.i~ tamos que,
de acuerdo con la construcción , el cuadrilátero ~1KFL ds -un rectángulo, es decir, FL =KM.
A continuación, podemos hallar KIY[ si notamos que L\ AKM en
KM
AK
· C'-)L\ ABC, dondé BC =-¡e;-. donde BC=n, AC=al/2 y AKcc
= ~ (ya que en el triángulo SAB SF: SB = 1 : 4 y, por lo tonto,
nsimismo AK: AB = 1 : ~).
Asi, pues, KM= ; . Ahor11, hnllamos que sen x = ~ :
4 12
411 2
ª
-Vi
y
de este modo, L F AL = arcsen 11 2
:2= -4- '
2 •
PROBLEl\fA'S PARA EL TRABAJO JNDlVIDUAL
GSI. .~oblicua AB forma con el plano P un ángulo de 4l'>º.quo es Igua l ni
á ngulo entre la proycccí6n de dicha oblicoa y la rcct.a A C slluadn en el pl ano
P. Hall en ol ángu~p BA C.
632. En una pirá mide triangular regular la o lturo el! Igual ni lado <le la
baso. Ha.Ueu el ángulo formado por Ja arista la lera 1 con el plnnó de la base.
633. La recta AB yace on el plano P p or un Indo dol cua l, por el puntu D.
se traian las.rectas B C y B D perpondicularC-'I a la recta A.8 y que con el plano
P forman ángulos iguale.!! a 50" y iSº. Hallen el ángu lo CBD. ,
634. La diagonal do UI\ prisma cuadrnngu lar regular forma con el plono
de la hllse· un ángulo de 45°. Hallen el ángulo formado por esta diagonal con Ja
d iagona l de la cara _lateral quo la intersee-0..
635. En. la secc ión axial dol cono el ángulo en el vérlico es ignal a 2.a. l'nr
dicho vértice se trnin un plano de modo que la o.llora !!el cono form11 con él el
á ngulo ~. Hallen el ángulo entro las generatrices por las quo este plano corta
1n su pertlcie del cono.
636. En el cubo ABCDA,B1C,D, se ha tra.tndo oJ plano secante PQC,8 1 ,
donde P y Q son lo.s puntos mccl10.s de las aristas A A 1 y DD,. rcspcctivomentc.
Hallen el ángulo ent-re la recte CQ y el pl;ino secante.
637. En el t.etraedro regular SA B C hallen el ángulo enLre la arista A B y
el plano de la cara SA C.
638. Por la diagonal A C do la pirám ide cund rnn&ulnr regulnr SA BCD se
troza un plano secante perpendicular a la cara SAD. ¿q ué óngulo forma Ja arís·
la SD con el plano !ecan tc?
.
639. En la pirámide cuadrangular regular SABCD la ra-ión entre la a ltura
SO y el lado A~ de In bnse es lgunl a 2 : 3. En In diagonol A C-s~· toma ta l punto
P que AP =PO. Hallon el ángulo formado por la recta SP coi;i el plano S AO .
640. La base do la plrámiilc cuadrangular SA BCD ca el r.uadrado A BCD.
En la cara lateral SA B. pcrpen1licular al ¡)lano de lo b~so y qné es un trióngu lo
f 0-0290
Ca pilulo I J. ¡.;slereomclría
146
regular , ~o ha lrilzadu la mediana A K. El punto K t>St;\ unido con el vértice C.
Hallen los ángulos que forman los lados AK y CK del triángulo ACK con el
¡>lano cln In b;l~l'.
641. La bnscdo lo.¡iirÍlmidc SABCD esunrectángul<icon losl11dosAD =a
AB = b. La altura de la pirámide se proyecta en el punto O, punto de i ntcr:
~ccción de las diagonales de la base. La arista lateral forma 30º con el plano ele
la bnse. Hallen ~• án¡¡ulo entre la recta D P y el plano SA C si I' es el punto medio
ele la allura SO.
642. La base del puralclcplpcdo recto ABCDA 18 1 C1D 1 es un parnlclogramo
con ángulo ngudo igunl a 60° en el vértice A. ~follen el ángulo entre la d iagonal
B 1D del parale(cpipcdo y In cnra lateral CC1 D 1D si AB : AD : AA 1 = 2: 1 : 3.
613. E n el cubo ABCDA,B1 C1D 1 P e.Sel punto medio de la arista AB.
llnllcn el ñngulo entre In rl'cta C 1P y la sección diogoual AA 1C 1C clel cubo.
644. En el cubo ABCDA,B,C1D 1 se han trazado \jl plano secnnte BDC 1
y la diagonal CD 1 di! 111 cara CC 1D 1 D. Hallen el ángulo entre Ja diagonal CD 1
y el plnno S<'Contr..
Má. L11 hase tic Jn piroimi!l~ SA BCD es un rcctáng11lo con razón do los
lndQs AB : BC = 1 : 2. Cadn arisl~ latera l está inclinada con relación al plano
tic> In has" hnjn 1111 :í11¡¿11lo 1l!l fiOº . llullrn Ju" úngulo~ quo íorrnnn lns rccl:i" DI'
>" DQ con t•I rl:o11•1 11i11¡¡<•1111l SA<: si / 1 y Q son los punt.os rncdillS 110 lns al'istas
sA
y
se.
6"6. l>n 11n11 pirnmltlc cwulrangul ar r('¡,'l1lar In arista lntcral forma con l'I
plano de I~ lin sc un ñn¡:ulo ele 45°. Por el vérlioo de la base y el punto medio de
ln nlr11rn <Ir la pinítrtitlc ~e trnza unn rl'Ct.n. Hallen r.I áng11lo entro d ich11 recta
y la s cMas lnt.umlrs dr lu pirámide.>.
6t.7. En el c•.ibo ABCDA,B c,o, se hnn trazado el rlnno C1 DB y la rcct.a
DP, dondr. /le§ ~1 pnnlo medio Je ln nrista BB,. Hallen l\l ángulo entro la ructa
D /> y <•l plano· secante.
M8. En <'l c11bo ABCDA 1 0 1C D 1 se \raza el plano secante BDP, donde P
c>s el punto mc<!io cln 'ln arist.~. CC, . \.iall~n el ángulo entre la recta A 1Q y el plano
Sl'Cant<.' si Q es el punto m~d10 <le In nr1sta DD 1 .
6/ill. La básc do la pirámide SABC es un . triángulo isóscel.es con el áng11ln
C recto. Cndn n~ista latcrnl forma con el plano de In base un ángulo de 45º. Hnllcn
el áng11lo entre la mcdinnn AM d CI la cara S AB y el plano-de la cara SBC.
65-0. Una ¡ecta es tangente n un cono ~· en el p\toto de tangencia forma
con su gcnerntriz el áng11lo ngudo a, mienl.ras que con el plano de la bnso d <i l
cono, el ángulo
Hall~n el ángt1lo entre la generatriz y el pl11no ele la bas~.
651. UnQ. ·c ircunfcrencin de radio R y un triángulo equilátero con lado
Íj?\lal a /1 lt' 3 yacen en planos perpendiculares entre si. El segmonto que une
r,.I Ct'nlro. d11 In .circunferencia coit el ccntroidc del triángulo forma ángu los iguales a 30º con los planos darlos, a(lemiis, uno tic Jos lados del trié,ngulji pertenece
;:il plamr de In !~ircnnferenci.n • .Hn.llen la longitud de aquella parte· d·c.l lado. de l
t.riúnJ.(11!0 QUI) yntc dr.¡¡lro ·t1c la c'ircunfercricia.
65.~.· .E!! 10 p~ismn. <:1>adrnngular ·regular ABCDA 1 B 1 C,D 1 lds diagonales
B 1D y BD 1 son P.crpondiculai;es entre sí. Hallen el ángulo entre lns diagonales
B 1D y ..¡1.1 (.'. i
.
65a. Un ci111drado, en el QIU) se hn trnzndo In dia¡:onal, nstá· enrollado en
Cormn.do:la ~uger.fi¡¡ic latornl de un prisma cuadrangular ·rcgular y, de cst.il forma.
la diagonal de Ucuadrndo se ha i;onvertido. en .una quebrada no plana. Hallen Jos
ángulos en\·l:C los .segmcn\os .d o Ja ,mc.n cionntln que.brltlin .
654. JJ es.!el pu·nto medio de la aristil SA, ·E , el punto medio de Ja altura
SO· del ·t etrncdro rcgl!lar SABf. ffn)len el ;ing ulo entre las rectas OD y CE.
6,5~. 'D~m¡1estrcn qu!l en una p~r~mide ttiang1~lar regular los pares de
arls.t as no mtqrsccnntcs son pel'p(lnl11culares entr~ si.
656. Dcm11estren q11e si l;is arista~ opuestas de una pirámide triangular
son n pares·pcrpcndiculnrcs, todas las alturas de la pirámide concurren en un mismo punto.
'
r..
§ 10. Rectas cruzadas
i47
-657. E n el t.eLr~c.dro- regU:lar S.A'BC .AD es la mediana dc!·:AABC, E,. el
punto. me'dio de la. arista SB. Hallen el ángulo entro las:recLas.AD y CE. ·
~sil'. E:n el tot.rac!l,ro SAB<J: SA cs·la pii.rjlcndlcl,llnr lll plilno A.BC\ ·AB j_AC
y :SA~ ·= A.B . =¿!C. 'Hallen el angulo .e nlre 'las rectas OD y qe· ~1. D· es.el punto
m.edio de.la ari sUI .S.A·, E,, el punto .mQdio do· t·a, Ari~t.a: -S!J y eJ. p~nto O es el cen·troide ·d·e fa .base'ABO ·del t.ettaed.ro. ·
.
·559, La b"se del paral efopípedo.-recto ABCDA 1B. C1D 1 e,s el ·paralelogramo
A'BCD'.c<;ín..el ángulo agudo DAB Igual n:y. ,Las· diagona~es.A Bi,Y 801 ,di!'las cáras
lateral.es. formari con el plano d11· la base. ángulos lgualés n .¡:r, )' ~ .. respecLlva·
' ·
inentc. Hallen el ángulo entre lás mencion·actas diagonales.
·660. El ángylo entre la~. rect.a s cruzadas a y I> ·es ifiltal a· (l()º • .L'a distil!lcia
desde la perpendicular. común de l as rectas o .y b hasta el. punto, ¡.f., que yace en
la -recia a, es igual a la distancia dcsd·c·él hastll el .p 'unw B en la ~y es ·igual a In
dislaocin entre la.s rectas a y b. Hallen el ángulo entre l'a perpendicular común
y l a recta AB.
661. En el cuh<> ABCDA 18 1CtD 1 se han trazado en las, aristas AA; y
DD 1 P y Q, respectivamente, que son sus puntos medios. Hallen el ángulo
entre los rayos DP y QC1 •
•
662. E11 ol tctraodr.o rcgulnr SA BC hallen el Angulo t1ntrc In mt1dinnn A M
do ln .cara .. S'A·B y :el rnY.i> SC.
663. En la pirámide l~i'angular regular SABC torios lo~ ñngulo~ plnno!I
en el vértice ..S son ·r ectós: Hallon el ángulo entre. los rnyos Cl' y SV si f' y D
son los puntos medios de los aristas SA y BC, respcctivamchle,.
664. En Ja. pirámide cuadrangular SABCD In base ABCD es un pnrnlcloi¡ramo entre .cuyos lados la razó1l es ilB : BC ~ t : 2 y con ~l ángulo agudo
igual a 60°. La cara SAB de la pirámide es perpendicular ni nlano do la basn
y es un triángulo r~gular. Hallen el ángulo entre lo media na D M de fo cara
SAD y la apotema S J<. de la cara SA B.
665. En la pirámide cuadrangular SA BCD todas las arista~ laterales l'Stán
inclinad{ls hacía el XJ;ino de la base bajo un ángulo rle 60º. Por. el punto F que
divide 'la diagonal .C do forma que A ¡: : FC = 1 : 3 y í' , que es el punto medio de Ja arist.a SC, se traza una recta. Hallen el ángulo formado entl'e el ray"
PP y · la diagonal AC.
666. E.n el prisma tr iangular (egular ABCA 1 Q1C,, cuyns ca.ras latcrall's son
CUl!drados, se han tra;ai!a las disgonnll'S BA 1 , A C1 y CB 1 • Hallen los ángulos
•
•
!iDll'jl Jos .rayos ,A_C1 y B-1 1 ; AC 1. y CB¡; BA, y CB 1 •
667'• .En ·el rectángulO A!JCD AIJ : B(: = 2: t. En los l'ados AB y CD
se}an ..t9ma:do, respectiva~erile; ·P, y .Q ~_ue son los pu!ltos njedios ele dichos
lai!os, m1ent.ras !!Ue ·por la re.d a. RQ el rccJ,'ángulg s.c . ha doblndo¡dp forma quo el
á:ngulo enfre lqs rayos PB y .PA' es .igual o 60°. Hallen el ángulo entre lo, rayos
DB y AQ:
.
668. En la pirámide cuadra.ngulnr regular SA BCD el ~ngulo entre la
arista lateral y el plano de la base·esigual a 95°. Hallen el ángulo entro Jos rayos
BP y OL, donde el punto B es el vértice rle la hase de la pirámide, i' , es el punto
medio do lo altura de la pirámíde, el punto O, el cenlroide de Ja base y L.
el punto medio de la arist.a se.
.
669. Una placa cuadrada A BCD está doblada por la diagol)al A C d~ forma
quo el plano A BC es porpendic11lar al plano A CD. E l p11nlo K :;e ha elegido NI
lo diagonal A C de modo que C K : KA = t : 3. Hallen el ángulo entre los rayos
KB
y CD.
670. En la pirámide triangular SABC los ángulos planos:ASB v CSB m
el vértice S son rectos, en tanto que el ángulo A se es ig11nl a 45°. Hnflen el óngulo entre Jos rayos CK y SD si J( y D son los puntos medios de l11s aristas SA
y BC, respectivamente. y SA = SB = SC.
,
671. En la pirámide lriangulnr SABC la base ABC os 1m lriiin¡¡ulo reg11lnr.
las caras SAO y SBC son perpendiculares Al plano de In base y la arista SB
es igual al"lado de la base. Hallen el ángulo entro los rayos SO y BD, dondc
10•
148
Capitulo 11. Estereometrla
el punto O cs. el centroidc del triángulo ABC y D, el punto medio de la nris1a
se.
672. Las diagonales no intcrsccantcs de dos cimis laterales ad1accntes dt'
un paralelepipedo rcct.angulnr forman con el plano de la base los angulos a y
~. H allcn e angulo cnl-rc dichas diagonales.
673. En la pirúmido cuadrangul;¡r regular SABCD el lado rle Ja hnso AB =
6 cm, la altura el\ igual n 4 cm. Hallen la distancia desde el vértice A hast¡¡
el plnno de la cara SCD.
674. El segmento AB, cuya longitud es igual a a, es paralelo al plano /'.
Por Jos p1mtos A y B so han trazado rectas perpendicufores al segmento A 11
y que con el plano P forman ángulos iguales, respectivamente, a« y ~. La distancia entre los puntos de intersección de las rectas traiadaS·Y' él plano P es igual u
b. Hallen la distancia entre el segmento A 8 y dicho plano.
675. Un segmento, cuya longitud es igual a a, tiene sus extremos en <los
planos perpeniliculares entro sí y fonna con uno de ellos un ángulo igual a 30°
y con el otro, a 45°. Hallen la distancia entre las proyecciones de los ex Iremos
del segmento dado en la linea 1lc intersección de los planos.
676. En e.l plano P está situado el u·iángulo equilátero ABC en el que AB =
= a. En la perpendicular al -plnno P, que pasa por el punto A, 5e ha trazado
tal ~c¡¡munto A K que A K =a. Hnlfon la distancia entre las recias AB y CK.
677. Pt\ r el extremo superior de Ja gllnc..ntriz d o un cilindro so Lt·(lin a éste
una t .. ngcnlc bajo un •íngulo igual :1 a:. Hallen Ja distancia desde Jos ccnt1·os
de las l>nscs d~! cilindro hast.a dicha tangente si la nJt.ura del cilindrv y el radio de su base son iguales n h y n, respectivamente.
678. En lbs puntos A y B, pertenecientes al plano P, s o elevan las perpendicularns AC y BD a dicho plano por un mismo lado. Demuestren que las rectas
BC y AD se inlersecan y hallen Ja disuncia desde su punto de int-0rsccci6n
hasta el plano P si conocemos que A C = a y BD = .b.
67!1. Ln altura de un prisma cuadrangular regular es igual a h. Hallen
la distanc.in desde el Jado de la bnsc, cuya longitud es igual n a, hasta la diagonal del prisma que no lo interseca.
680. La base del prisma recto ABCD1t1 B,C1D 1 es un rombo con lac!o igual
a n y ángulo ~gmlo q>. Hnllen Ja dist.ancin desde el vértiCll B 1 de la base superior
hnsta Jn dingonal A /J de- In cnra lntc>ral si la arista latero! del prisma e~ igual
a /¡,
681. Hullcn In dislancin entre la diagonal de un cuho ~· Ja al'isla que no
ln inlersc;:.a si la nristn clol cubo es igual a 11.
6!12. H;tllcn la distancia entre las diagonales no intcrsccantes d e las caras
adyar.entes de un rombo si s11 arista os igual a a.
68a. E11 el cilindro cc¡uilt\tern, on el que el rndlo rln los bases es igual a
JI. un punto de la ci rcunferencia de la base superior se une con un punto de la
i:.ircunfereriéia ele la .base inícrior. Ln recto. trazada forma con el plano de la
base· un áng\tlo 'igual a ex. Hallen Ja distoilcin entre la mencionada recta y el eje
de simetrín del· cilindro.
·6lVí1 Entre dos planos par¡ilcilos se han trazado upa perpendicular y 11110
<iblicua que forma con ca(ln uno de los planos el ángulo et. Hallen la distancia
entre Tos ¡iu·n'tos mcilios· el.e los segmentos de Ja 6blic1¡u y la perpendicular, conteniclos entre los· planos dados, .si fa Jo11gitu1l· del segmento de Ja ¡ier.pomliculnr
es igüal a 2a y la dislanc ia entre los extremos de la oblicua y la perp1.mdicular
en éada unp ·de los p~anos es ,igual a ú.
68.'\. En l!na pi(amidc triangular Ja suma de tres ángulos planos en cadn
·uno de 'los vérVic.c s. de la base.es igual a· 180º. Hallen la distancia ent.re las arisl:ls
cruzada's de. la pirámide si. los .lados de In l>asc son iguales a 4, 5 y 6 cm.
686. Demuestren quo si fo recta a forma iguales iingulos con tres rectas
que no son paralolns ~nL1·c sí y que yacen en un mismo plano, la recta a es porperidicular a diclio plnno.
687. Lós ángulos planos de uno de los vértices de una pirámide triangular
=
§ 11. Ángulo$ ditdrM !/ polltdro•
son rec~s. Demuestren que lo allura de la pirámide, t razada desde dicho vértice, posa por el punto de int.ersección de las·alturu ilc lo cara opuesta.
61!8. Demuestren que In altura SO .de lo pir6mlde triangular. SA BC interseca la altura AD de la hose én nquel •y sólo en aquel caso cuando SA .L BC,
()89. ·~cl)'luestren .que si unn de- lo s alturas de una pir'ái:rHde triang11l11.r
pa~a por el ,J?uhto ~!(). mlers~i_ón ~!l las .nhµrns d~ Ja corn. <)puesta, las ctomñs
alLufes do dicha p1ramido. jlosc,un esa m1slll_a prop1odn,d.
§ t t. ÁNGULOS DIEDROS Y POLIEDROS
EJ1'MPLO 1.
U110
de los ángulos planos de un triedro es igual a
60° y cada nno de los dos restantes, a 45º. Hollemos el ángulo diedro
opuesto al plano que contiene los f>Oº.
SOt.uc10N. .Sea la fig. SM NL (fig. 131) In representación del úngulo triedro dado. Ella es completo. Hallemos su 11ú111cro purnmétrir,o. Considerando ol nngulo MSN
/
como In ~cpre5cn t.ac,ión de l úngulo de 60° consu mil}'los un parámetro¡ si consideramos que los ángulos M SL y NSL son l:is representa ciones de los ángulos que en el
original con tienen cada uno 45º se
consumen dos· •parámetros más. N
_·---+s
Asf. pues, hemos consumido un total de t res parámetros,. es decir, nl
efectuar nuevas construcciones de
carácter métrico, q11e pueden ser
precisas al determinar el ángulo
Ftg. 131
diedro buscado, es posible emplear
dos parámetros más.
De modo que LMSI{ = 60º, LMSL -= LNSL = 45º. Hollemos LSL que es un ángulo diedro en la arista SL. ' Con esto rin,
construimos, y, a contínuación, hallamos su ángulo lineal. Realiinmos la construcc ión del modo siguiente.
En. el rayo SL elegimos el punto arbitrario A y en el plano MSL
const ruimos l a i:ccta AD que consideraremos como la tepresentación
de la perpendicúlar a la recta SL (consumimos un parámetro). Por
nnalogía vamos a considerar que AC es ln representación de la perpendicular al rayo SL (consumimos un parámetro más y, ahora, en
111 represen tación hemos consumido Jos cinco p11rámctros). Así, pues.
LBAC es el ángulo lineal buscado del diedro SL.
Realicemos los cálculos necesarios para lo que introducimos un
parámetro auxiliar. Hagamos SA ~ a. Entonces de ·los triángulos
rectángulos SAB y SAC hallamos que AB =a y A.C = <t, SlJ =
= SC = a1/2. Como en el t:;SBC BC = SB = SC
1 BC =
Cerciorémonos de que se cum plo l a igualdRCI A8 2
ACl = IJC2.
En efecto, AB2
AC'
2a2 y BC' = (aVZ) 2
2a2 • Así, pues,
+
=
=
+
aV2.
150
Capitulo !l. Estereomttrla
en el 6.ABC (según el teorema inverso 111 de Pitágoras) LBAC =
= 90Q. Por lo Lanto, también LSL = 90º.
ousr:n v ACIÓN. El problema cJCntninado pudo sor resuello sin Ja iolroducción del paró metro auxiliar. Eil efecto, como es fácil notar, el ó. A BC = ó.A SB.
por lo que LBAC = LSAB = !lOº.
f.:.H:MPLO 2. Ln base ele una pirámide es un triángulo regular. Una
de las caras laterales es perpendicular al plano de la base, las otras
dos .son oblicuas a él bajo un ángulo igual a o:. Hallemos los ángulos
formados po.r las aristas laterales con el plano de la base.
$01.,UClóN . Sea ol cuadrilátero SABC con sus diagonales
y AC
la representación ele la pirámide dada (fig. 132). Tal representación
es completa (cerciórense de esto
s
por sn cuenta, to mando el plano
ABC, p. ej., como el fundamcntnl.) Calculemos el número parnmétrico de la r epresentación . Considerando que el t,riángnlo arbitrario
e ABC es In representación de un triángulo equilátero, consumimos dos
parámetros. Al considerar que el
A
triángulo SAB es la re presentación de la cara perpendicular al
plano de la base ~n el original , consumimos un parámet ro. SuponienB
do que los triángulos SAC y SBC
Fig. 132
son ln.s representaciones de las caras oblicuas al plano de l a baso
bajo ángulos, catla uno de los cuales es igual a cz, consumimos un
parámetro más. Así, pues, el número paramétrico de l a represcntaciqn. quc. ,e~am i nam.os p = 4.
Con el fin dQ resolver el problema i ntroducimos en el dibujo los
d<1~os y lasf ipagnitudcs buscadas, pára lo que realizamos las constri1cciones adicionales necesarias. .
· ;Ant~,.tó~1p, co't1s~ruyamos el segmento SD que es la apotema de
la car¡J.· SA {!·: Ei:itonces,. como los p l anos SAB y ABC son perpendiculares·, el ~e.gménto SD· también es perpendicular a l plano ABC y,
por .ello 1 A)) es l¡i proyección de la arista SA en el plano ABC, es
decir, LSA,O es ·el. ángulo formado por la arista SA con el plano
ABC. Por tjna'log!a, LSIJ.D es el ángulo foi-mado por la arista. SB
con el plQ.no ABC, ;{...SCD, el ángulo formado por la arista SC con
el .plano ABC. Más adelante, sefüdemos que AE, que es la mediana
d'el. t.ri'ángujo ABC, es tambien su altura, por lo que, después de
'CQJ)Slruir DF 11 AE, .llegamos. a la conclusión de que DF _L BC y,
sn
151
§ 11. Á 118 u los 4trdros· 11 pql/edros
por lo tanto,.DF es la·.proyecci.6n.d.el segmento SF en el plano. ABC *).
De este µiodo 1 SF ..L BO y LSFD'- es el ~ngulo ·line·a1 del djedro I)C,
o sea.•.L.,SFIJ = .91¡J)e'forma •nnffe:loga, ni haber cons~~ui~o DK ...L AC
Uegam·o s a la w11cl ~i(m: que L.S KD = .o:. Ad·emás; lo~. triángulos
réctángulos -SD.F y Si).K tieneij". un .catetó común SD é igualeii a_~gu­
los ,agudos, SFD y SI<,!); es decir, esos triárigü!os. liOn igliales. .Ei:itonc·es,· DF .= DK. =E.sto. significa que el punto D es equidiiitai:ite
.de los lado!! del foguló ACIJ; ~~. ·Pu_es, ·el pun\o D yace en la bisectriz del lingulo ACB d~l triángulo regulai: ABC. Pevo la bisec.t rii
del ángufo de un triángulo regular es, al .~nismo tiempo. su..medi~na
y aitura. Así, que CD es la medi¡rna del éf,AB(J~ o sea·, T) ·és el pu~.to
medio del lado AB y', por consiguie·nte, SD, que es la. a,lttira d~l
L:::.SAB, es asimismo. su mediana. Esto quiere decir que." el L .S AB
es isósceles, o sea, LSBD = LSAD.
Ahora, podemos pasar a los cálculos. introducimo.s un parámet.ro aux:iliar, haciendo AB =a y, para abreviar, hacemos L SBD =
= x, L SCD ~y:· Entonces, también L SAD = x. Es evident.c, que
BD = ~ AB = ~ . A continuación, como llE os la mediana del
L:::.ABC y FD, la línea media del .t:.ABE, DF=
x 11 ~ 3 = 11
ra .
! AE=-}><
Del triángulo rect.ái;igulo SDF tenemos: SO.=
11V"3
= D F-tgo:=-tga.
4
,
SD
a lf3 tg a., d e d onde x
As1,
pu~ , tg x = BD =· - 2
l\'1~s adelante, tg y=~~=
T
= arctg ( V3
T
tg a ) .
(y
tg a, de donde y= arctg
tg a) .
El ángulo diedro en la arista latcrnl dé una pirámide
cuadrang1,1lar f'eg\l l~r: es igual a a. ljal.lemos el ángu\o diedro en la
arista de la base de. :<)iéha pirámide.
,
soL OCJON. Sea la figura SABCD Ja representación de la pirúmide
dada (fig. 1·33). Esta es completa (cerciórense de ello .p or su cuenta).
Calculemos su número paramétrico. Considerando qiie el paralelogramo ABCD es la representación do un cuadrnd·o, consumimos
dos parámetros. Sea O el punto de intersección de las diagonales ele
In base. Si' consideramos que el segmento SO es la representación de
la altura de la pirámide, consumimos dos pnrámétros más. Por
fin , suponiendo que el ángulo entre las aristas laterales, p. ej. SBC
y SDC, es la representació11 <lcl á ngulo diedro, cuyo ·v alor en el original es igual a a, consumimos otro parámetro. Así'., ¡rnos, para In
representación de la pirámide dada hemos consumid·o los dnco parámetros.
BJE~i,.o .3.
• ) (Presten atención. Para la construccióu OF .L lJC no es necesario consumir parámetros, aunqúo ella es también métrica}.
J.52
Capílulo lf. eitueomttria
Pnra resolver el problema es preciso efectuar c iertas const rucc iones adicionales que, en pa rti cul:ir, nos permitirán introducir en el
d i bujo Jos ángulos dado y buscado. (En principio, en la represe ntación métricamente definida que tenemos, al azar no se pueden reolízar
construcciones métricas. No obstan5
Lo, en el caso que analizamos al
verificar construcciones métricas son
/
tolerables exclusiones, ya que el
1
ángulo en la arista lateral de la
1 1
/ ,...¡, a
pirámide está prefijado con un
%4...:. ' e parámetro cuyo valor, como es fácil
// ''-'l,.{;-'.-..'.
de advertir, pertenece al intervalo
190º; 18~º(. Nosotros haremos uso
~ --"<·
de esta c ircunstancia. )
A
D
Tomemos en la arista se el punto M y vamos 11 consiclcrnr que el
l-'ig. 133
segmento OM es la reprcsontnciún
do la pe1·pendiculnr a la arista se.
A continuación, construyamos los segmentos DM y BM. Como
IJD ..l.. AC y BD J_ SO, BD ..l.. Se. De modo que Se ..l.. BD y
SC ...!.. OM, es decir, se...!.. DM y SC J_ BM. Entonces, LBMD
es el ónguJo lineal del diedro SC y , por lo tanto, LBMD = e¡, Ade·
más es posible mostrar que el segmento OM, siendo la mediana del
triángulo BMD, es asimismo s11 bisectriz y altura. Por ello.
LBMO = LDMO =
y OM J_ BD. Además, construyamos SK,
111 a potema de la cora SDC y OK, la mediana del triángulo OCD.
Como los triángulos SCD y OCD son isósceles, SK J_ CD y OK .l..
J_ CD, o sea, LSKO es el ángulo lineal del diedro en la arist a CD.
Ahora, pasemos a los cálculos. Introduzcamos un parámetro auxiliar
haciendo CD = a y, pora abreviar , tomando LSKO = x.
l
'/ -----'tfr.::_ '· .
T
Como en el triángulo rectángu lo DOM DO =
Ahora,
caiculcmos SK.
donde Se =
V
SK2
+ ~· ,
2sen
DM =
Como el ó.. SCK i;:,:. ó,, CDM,
12
DM
"1
y CD = a.
viª
Así, pues, SK =
"rz ,
T
2scn
V SK + ~:
2
='V=cOs!i,
2
~~ == ~~ ,
ª
2
'de donde SK
2 -V -cosa ·
Entonces, dél triá ngulo rec~ángulo SOK, l1allamos que
= ~~
ªViª .
2scn
es decir, x=arccosV -cos~ .
COSX =
§ JJ. Augulo• diedros ¡¡ P.oliedros
1,53
PROBl.EMAS PARA EL TRABAJO l·NDiv:IDUA~
690. p9s ángulos plano~ .del, tr.fedro son. iguales a 4.5° cada'. uno, en tanto
que el di~d~o entre eltos·.cs r.e cto., Hállen .el· tercer :ángulo J!lano.[
.
. 69t;· Uno de los. ángulPJ! ·pl11.nos' del triedro es igual a 9Qº y los otros .d os,
a-.60° c;ida uno. I;-lállen e~ ángulo: ~·ntr,e !!l. plano que corta la"s aristas del, ángulo
·triedro en,' Segmentos .ig\Jales. , ,''.
. . .1 .
692. Los· ~rtgufos'j!fonos.del ·triei!to sonjguo,lcs.a 4.5, 45 y '60j. Por su .v értice
se 'lia· t¡:aiado µna ,reéfa: perpendicular a la~ .ar.is'tns del· á!Jgulo. pi"·ºº i'gunl a 45°.
Hallen el ángulo en~re dicha· rec ta y la arista del ángulo •Cliedro ¡· que ·lio yace en
la mencio.nada aristn.
·
'
'
· 693. Las ári.s tas SAB y SBC del án¡¡ulo t'tiedro ~ABC formari un ángule>
i;eclo, en tanto que los dos ángulos diedros son Qadil itno lgunl ¡.a ex. H¡illen .el
ángulo plano ASC.
; ··
694, Por un punto tomado en Ja arista de un ángulo diedi,o igual a ex, se
ha tra:iado a. una do las aristas un rayo que forma con la arista el ángulo 11 y, a
la otra arista, un rayo perpendicular a ella. Hallen el ángulo ent~e los dos rayos.
695. Dentro de un ifogulo triedi:o, on el que t<>dos los ángitlos planos son
iguales a 2.a:, p,o r su vértice, se ha trazado un rnyo quo es igualmento oblicuo a
las aristas del ángulo diedro. Hallen el ángulo de inclinación de este 1•ayo con
relación a cada arista.
696. Los ángulos planos de un triedro son iguales a ex, 11 y y. Hallen sus
ángulos diedros.
.
697. En el rectángulo ABCD AB: BC = a: b. Por el lado AD se traza
el plano l' con el qui'la diagonal ele! rectángulo forma un ángiilo igual a 30°.
Hallen el ángulo diedro entre ni 11lano del rect.á!lgulo y el _pla~o P.
698. Por el punto A dol plano P se traza In ·recta oblicua.,4B que con P
lormn un ángulo ex. Por AB se ha trazado el plano Q que con P ~,onstituye el
ángulo 1\. Hallen el ángulo entre la recta AB y la línea de intersección de los planos P y Q.
699. Por la bisectriz del ángulo recto de un triánfilo r éctángui() se ha
trazado un plano que con el plano del tri:lngulo forma o ángulo ex. Hnllcn los
ángulos que· forman los catetos del triángulo con dicho plano.
700. En el tetraedro reg11lar SABC, por la mcdioni1 AD ele Jo hnse y K,
que es el punto medio de la arista SB, se lin trazado un plnne>. Hnllcn el ángul<>
diedro entre dtcho plano y el de la base.
701. Eñ el prisma triangular regular :ABCA,B 1C, lodos ;1as nristas son
iguales. Por la arfsla .4A 1
que es el pu!llO medio de_la arist.ai8·1C1, se ha trazado un plano. Hallen el a11gulo diedro entre et mencion·odo p,l ano y el plano
A~~
.
702. En la pirámide triangular reg11lnr el lado de In base es tres veces mi?·
nor que .la arista lateral. Hallen el áñgulo di edro en ·1a aristi1 la\eral.
703. En la firámide cuadrangt1lar regular el ángulo de inclinación de la
arista respecto o plan<> de la bn~e es igual 11 ex. Hallen el ángillo diedro en la
arista lateral.
704. La ·base de una pirámid e es un cuadrado. Hallen los ángulos diedros
formados por las aristas Jatorales, entre los qu~ exis te In razón 1: 2: 4: 2,
con el plnno de la base.
705. En 11nn rirámid ll triangulnr ccgulor el án1n1lo 1Hcdro junto a la arista
de la base es igua a a. Hallen el ángulo de inclinación de la arist.n lateral con
rcsp~tr1 a la base,
·
706. El ángulo <lierl ro en Is nr istn lateral de unn pirámide ·triangular rE>glllar es igual a a:. Hallen el ángulo diedre> entre el plano 1lc la bMe y la cara lateral
de Ja pirámide.
. 701. En una pirámMo trinng11lnr l'Cgl1lnr el ángulo diedro entr~ ln cara
lateral y el plano do la base es igual a ex. !fallen el á11golo diedro entre las caras
yp·,,.
la erales.
Capítulo J J.
154
Ester~omdria
708. Dem11estren que si todos los ángulos diedros de una pirámide triangular son iguall)s, asimismo son iguales todos las aristas.
709. En la pirámide triangular SABC las caras SA C y SAB son triángulos rectángulos, isósceles con la hipotenusa SA común. El ángulo diedro de Ja
arista ,S:A es igual a et. Hallen los á ngulo11 diedros de las 11rist11s SB y SC.
7t0. La base de la pirámide SABCD, todas !ns aristas laterales de la cual
están inclinadas do igual modo respecto. al plano de la base, es el rectángulo
ABCD. Por F, X y L, que son fos punto.~ medios.de las aristas Ali, AD y SC,
se ha tratado 1in plano. Hallen el ángulo diedro que esto plano !cirma con el de
la base si AB : AD : SA = 1 : lf3: 2.
71 1. La base tic unn p irámide es un cuadrado., Dos arist.as laterales son
perpendiculare~ al plano de la b¡ise y los otras dos, const.ltuyen con él un ángulo
igual a a. Hallen el ángulo diedro entre las caras laternlos que no son perpendiculaTes al plano de la base.
712. La altura de una firámide 11-angulor regular es dos veces menor que
el Jodo de In base. Hallen e ángulo entre la cara lateral y el plano d'o Ja base.
713. En una piró.mide "-angula~ rogular el ángul.o entro la arista later1\l
y la 11rista de lá base, adyacente·a la primera, es igual a~- Hallen el ángulo diedro formado por la cara lateral con el plano de la base.
714. En Jn p irámide n-n.ngu lnr rc¡fulu el ángulo diodro en la Misio la toral
es igunl n 2et. Hallm el ñngulo de inchnnción de la ¡iristn lateral hacia el plano
de la bnso de la pirámide.
715. En unn pirámido 11-nngulor regular el ángulo diedro en In nristn lateral
es igual a 2a.. (-fallen el án!fUIO il iedro on lo arista de la baso de l a pirámide.
716. Dos rectángulos 1gualqs ABCD y ABC D 1 tienen el l ado común AB
y sus planos forman un ángulo diedro Igual a 6~º; Hallen el ángulo entre las
diagonales AC y BD 1 si AB : AD ·""' 1 : 2. ·
717. Oosrombos iguall'IS ~BCD ..y ABC 1D 1 llenen 1il lado común AB, mientras que sus planos ro~man un ángul~» dict!i'o igu11l a .45º. Halle'! el ángulo entre
Jos lados BC y ;o c 1 de los rombos si el: ángulo *giadci dé cada uno de ellos es igual
1
a 60°.
•
••
•
• •
718. El ¡i~rol°clepipedo ABCD ._ en ·el qlie ' AC es Ja menor ·diagonal y
AB: AC :. BC j= 1· ; 1: i/f." está - doblado por lo diagonal AC de modo que el
ángulo BAD ~s igu'a l a ·so•.. Hnll'e n el ángúfo. diedro formado por los planos
de los triángufos ABC y ADC.
719. En el paralefogramo ABCD el ángulo agudo es igual a 60° y AB : BC =
= 2 : 1.. En el! ¡a do A 8 ·se tomo P }· en el CD, Q que son Jos puntos medios de
estos lados. Después de doblnr el para lelogramo por la recta PQ los planos PQC
y PQD forman un ángulo diedro igual a 45º. Hallen el ángulo agudo entre las
rectas AP y BP .
720. En l* .Pirámide triangular regular SA BC cl ángulo dredi-o en la ar ista
de la bnse es i\íunl a 45º. Hallen cJ ángulo entre la mediana CD de la cara SA C
y el, plano SA!B . . . ··
721'. Ln lip.s e dc·'uno pit.á!"id.e CU<\drnngular .C HIO rombo con á!ll!'1.l o ·agudo
de 60° ...Cada cara lateral ·e sta 1.n cl!nada hacia el'. plano. de la b~se bn.w un ángulo
de 45º. '!1allcnl l9s ángúlos qµe ·!orina Ja altui~. ae_ la cara lateral con las diagonales de I¡¡ b~ll¡se:
·
·
722. Lll base ele la pirá'm idc. SADCD es el cuadrado ABCD, fa cara SAB,
pc~p~·ndicul~r· 'nl plano· do la base, .es un trii\ngulo rsóscelcs, mientras que las
·aristas laterares SC y SD forman , cacla una de ellas, con el plano de la base
un ángulo de ~5º. Hnllen el ángulo· diedro ii!l la ari~ta· latcral SD .
.7l13. E'n .el plano de l? base ~é. un cono' equilátero (fu<)rn d.e ella) s~ ha tomado un puritj> alejado.de la circunferencia do !11 base a una d1stanc1a igual al
ra"ilio de la base . .P.or d icho punto se traz.n n. hacia el cono dos planos tangentes.
~~.!len e~ ángulo diedro entro ellos.
·
.:.
'
§ 12. Seccto11es de poltedros
§ 12. 'SECCIONES DE POLIEDRQS
.
Los .prci.·bleuias relac'i<;>nados \:ºn Ja ·necesidad de reP.re!¡entar las
'secciQnes bah de dLvidirse en dos tipos:
1) fos _p.rohl~mas en. lqs que se dico qµe es preciso constr.uir la .sección;
2) los problemas en los que se dice (o se sobreen.tiende) que la
secci6n ya !lsiá construidf.1..
.
Al resolver los problem.as tlel prime~ tipo ll! construcCiqn 4e la
sección se acon:ipaña de la descripció.n del- proce.so e!~ la ·constr.ucció9
que se realiza 'bien según el e11quema. completó de resolución deJ
problema dado (anális.is, construcción, demostración, investigación), o bien (en casos más sencillos), según nn esquema algo simplificado (p. ej., se omite el análisis, la construcción se simultanen con
la demostración). La .investigación del problema para In construcción de la sección no se debe confnndir con la investigación de Ja
solución .de los problemas para cn1culnr ciertas magnit11de.'l ligadllS
con la presencia de lA sección. P. ej., el problema de la construcción
de la sección diagonal de un cubo o un paralelepípedo rectangular
tiene seis soluciones; en lo que atañe al problema p·a rn calcular el
área de la sección diagonal del cubo, sólo tiene una solución, mientras que cuando se trata dcl parnlelepípedo rectangular, tres.
Al resolver los problemas del segundo tipo se opera de modo algo
diferente: si para la representación del poliedro prefijado junto con
la sección se consume no más de cinco parámetros, Ja construcción
de la sección no se describe (aunque, como es lógico, para resolver el
problema la construcción de la sección ha de realizarse); pero si los
cinco parámetros han sido cons~nnidos antes de construir Ja sección,
la coJtStrucclón necesaria para representarla, ha de ti.escribirse. Semejante descripción se hace de la misma forma que la de cualesquiera otras cons~r11ccfones realizadas con una repr.e sentación métricamente definida.
Al resolver los problemas tanto del primer tipo como del segundo, hay que cerciorarse de la plenitud de la representación en la que
debe construirse la representación de la sección (para los problemas
del primer tipo) o en la qi1e está representada la sección (parn los
problemas del segundo tipo). (Recordemos que on la representación
completa es resoluble cnnlquicr problema de posición.) Además,
como, por .r egla, se sabe de antemano si ~n el proceso ~e la resolución
será posible limitarse sólo a las construcciones de posición o bien
será necesario realizar, asimismo, métricas, hay que calcular también el número paramétrico de la representación.
Pasemos a estudiar los problemas del primer tipo. Primero, dctcngámonos en la construcción de la sección según ~l método de la
traza del plano secante. (Recibe el nombre de traza del plano secante
156
Capitulo /J. E$lereometrla
una recta obtenída en la intersección de dicho plano secante con cierto plano elegido en la representación.
EJEMPLO i. En las aristas del cubo ABCDA 1B1 C1 D 1 so dan tat
t
t
les puntos p , Q y R qucA P = 'fAA¡,B 1Q =-;¡B1C1 yCR ='3 CD.
Construyamos la sección del cubo con el plano PQR.
SOL UCION. Primero aclaremos si el problema planteado es resoluble. Sea la figura ABCDA 1 B1 C1D 1 la representación de un cubo
(fig. 134). .Ella es com pleta. TamB,
Q
bién está. c laro que teniendo en la
representación 'los puntos P, Q, y
R, que son las proyecciones de los
puntos P 0 , Q0 y R 0 , es posible
hállar asimismo las proyecciones
sccundnrias de los puntos P 0 , Q0 Y
Uo•· Coú csLo fin es su[icicnle efectuar en el p\a no de iroageli la
proyección paralela interior, p. ej.,
X
en Ja dirección pa ralela a AA" De
esta forma, hallaremo·s los puntos
Flg. 134
P 1 , Q1 y R 1 y llegaremos a la ·c oncl.usión de que .la representación
del plano secante es la prefii.a~a . E;ntonces, el problema de ballar la
intersección' del plano, prefijado con los puntos P , Q y R, con la
'
superficie del cubo es resoluble.
Pasamo~ , dire.ctament e, a la construcción de la sección (como,
por regla.,. decimos la construcción de la secc.ión .aunque se trata de
construir l ~ represéntación QC la sección). La primera etapa del
esquema ge:n eral de resolución do 11n problema de construcción, es
decir, el an.á lisis, será omitida en el presente ejemplo, mientras que
las etapas !Segund a y tercera, o sea la propia construcción y la demostración; se realizará simultáneamente.
An te t9~9 1 halle..mos. la traza del plano secante, es decir., la línea
de foter~.ec9i9n del .p lano PQR, con. ·ol plano A,BC (lo~ Jl Untos P1, Q1
y· Rl" ~e Jian .tomac\o en e~ plano ABC}.
t') Hallémos el punt o X en el que.. ~e coi:tl)"n las rectas PQ .y PiQ1.
Como· ·el pu lito X ,yace en. lá recta PQ. y· ésta .so encuentra en·>el p lano
secant~ PQ.fi,, X yace en PQR. Por analógía, ya que el' pun.t o X yace
en lt1 rec.t a I?l.Q 1, él yace en el pla_no. ABC . .,Así, pues, X es µn punto
coni.ún .de· l$s planos P.QR y Af3C. Asimismo el punto R es un púnto
común· de )j)"s mencionados-..planos. Entonces, X R es la recta por la
que l()s pla)'los PQR.. ·y ABC se interse.éan.
2) ·c()ns~ruyamos la ret:.ta XR que es la traza del plano ~ce.ante,
3) H'alt~mos el p.unto Sen el que concurren las reétas RX y AD.
4:) Unimos los puntos. P. y S. ..
5) Hall~mos el punto M en ·el que concurren las rectas RX y AB.
157
Como el punto i lf yacll ·en -la· recta RX y ésta· se encuentra en el
plano secan to .PQR, el puf\to M yaccon dicho plano. P 9r a'n alogia ,
ya que· el pul} to M yac~ :(in·.la r·cc.ta ·A B, mientras que ésta se encuentra en el pláno· ABQ·1 , .e~ punto M es un pt~nto común del plano
secante .y ·del plªno (l~ la cara, la t.eral ¡18fJ1 del cubo.•
.6)· C.onstrui mos ·1a .recta M'fJ por la que S!l cortan los 'p lanos PQR
y A,BB1.
.
.
7) Hallemos el punto K en e).quola re<;ta ¡l{P ·cofta ln x:ecta A 1 8 1•
8 ) Unimos los puntos K y Q.
9) A continuación, hallamos el punto N q.u e es ·el punto de intersección de las rectas RX y BC.
10) Construimos la recto. QN.
11) Hallamos el punto L en el que concurren las rectas QN
y CC1 ·
i2) Unimos los puntos L y R .
Como de ac11erdo con In constrncdón los vértices del poliedro
S PKQLR son puntos que yacen también on el plano s~ante PQR
y en las aristas del cubo, el mencíonaclo poliedro c.<; In sección huscnda. Como según el sentido del prob loma los puntos i>, Q y ll no
yacen en esa misma recta, el problema tiene una sola solución.
EJEMPLO 2. Se han dado ol cubo ABCDA 1 B 1C1Dt y tales puntos
1
1
P, Q y R que AP =
3 AA 1 , 8 1Q = 2B1 C1 y el punto R os el centroide de la cara DD1C1C. Construyamos la sección del cubo con el
plano PQR.
SOLUCION. Como en el anterior ejemplo, hallamos Jos puntos PL•
Q1 y R 1 y ·m ostramos quo la represen tación es complota. Hallando la
traza do! pl;ino secante (fig. 135, a) obtenemos la seccióll buscada.
Examinemo!i en esto ej!)mplo otro método de construcción de
las secciones llí'lmado método de proyección interna. Realicemos la
construcción (fig. 135, b).
1) Constroimos l as recLas PR y P1 Rt.
2) Hallamos e l punto M 1 de intersección de P1 R1 y Q,D.
3) Por el punto /111 trazamos la recta m 11 A A 1 . ·
4) Hallamos el punto M de intersección de las rectas m y Pll.
5) Hallamos el punto F de intersección do QM y DD 1 •
El punto F yace en el plano scr,ante PQR. E11 efecto, el punto M
yace en la recta PR , es decir. M yace en el plano PQR.. Pero, asimismo, el punto Q yace en el plano PQll. Esto signi.(ica que la recta MQ yace en ol plano PQR, pero, en tnl caso, el punto F de la
recta .MQ también yace en el plano PQR . Después de hallar el cuarto punto perteneciente tanto al plano secante como• al plano do la
sección del cubo, la constnlccióu se puede realizar del siguiente
0
-
~~.
ü) Unimos los puntos P y F.
7) Hallamoi; el punto
J~
de i ntcrsecció11 de las roctas Ffl y CC, .
e api lulo
158
J/ . Esl<r<Om<lr(a
8) Construyamos la recta QL .
9) Hallamos el punto N de interseccíón de las rectas QL y BlJ1•
10) Ha:Jlamos el punto K de intersección de las rectas N P y
A1B1·
11) Unirnos los puntos K y Q.
El poliedro PKQLF obtenido es In sección buscada.
Los métodos descritos más arriba de la traza del plano secante
y de Ja pro.yección interior también se aplican al construir las seccio-
,'V
nes de una pirámide. En este caso
se efectúa In proyección central.
En la fig. 136, ·a la sección de Ja
pirámide con el plano PQR so ha
construido con ayuda de la traza
XY del pla no· secante, mientras
que en la fig. 136, b, mediante el
método de la p1·oyección interior.
Los procedimientos para prefie jar las secciones de Jos poliedros
p
son los más diversos. P. ej., el plano secante pu.e de ser dado con ayuda de ·dos puntos y. cier-t:a recta con
la .que J.á • seéción dada. es paralela
(b)
.o ·p erpend.foulár., con .dos punto11 y
un plano·cón el que la sección dada · es 'patj\lela o perpendicµla r.
EJEMPLO a. En la pirámide trian.gufar regular S:ABC tracemos
una s~ción pnrn'le1a a la arfsta SB y qi1e pasa por P ·Y Q que son
los puntos medios de l'a aristas AB :YBC, respectivamente.
soi:.uc10N. Sea el cQadr.i látero SABC, con sus diagonales AC y
SB, la repi.'esen.tai:,ión <,le la pir~mide .qada (fig. 137). Ella es. com·pleta. Con dps puntos P y Q y la recta SB se determina con plenitud
el plano se«~nte. De este modo, el problema de la construcción en
esta representació.n es -resoluble.
N
§ 12. Secciones de poltedro1
159
Pasemos a 1a construcción de la sección. Para abr11\·jar designemos el plano secante con ft Como el ptinto P yace en la aristl\ AB
él yace a.simlsm<> en el plono ABC. De la mísma forma; el . punto Q
yace en e1 ·plano ABp. Pero _los ·Puntos P y Q tam.bién y:icen en el
s
C,Q,
Fíg. 136
plano secante j}. Así, pues, los planos j} y AIJC se <",ortn n por 111 recta
PQ. Bueno y, ahorA, rcn liccmos la construcción.
1) Unamos Jos puntos P y Q. Como SD 11 ~. ul plano SA/J, que
1i11sa por la arista. SB, intcrsecará ~ por la rcct.a que pasa por P y es
p11ro.lela a la arista SB.
2) Por esta razón, en el plano SAB. por el punto P , t r11zamos
la recta PK 11 SB.
3) De forma anóloga construimos Q/Vl ti SlJ.
4) Unimos los puntos K y M.
El cuadrilátero PKMQ satisface las condiciones del problema y,
por lo tan to, es Ja sección b11scadn. Tnmbiéu ei; fñcil cerciorarse de
que In sección requcridn existe, con la partic11lari d11d de que ella
es sólo una.
O.llSEllVAclON. L a diversidad de los procctlimionlos para prelijnr el plano
sccanto no permite aplicar para la construcción de fu sección ciert-0 proccdi·
miento universal. P. ej., en los ejemplos t y 2 de eslc párrafo pu•lleron ser
aplicados Jos método~ de Jo trua tlel plano secante y de la proyección interna.
En el ejemplo 3 ninguno de estos mctodos hubicrn sido eficaz.
Supongamos quo con l11s condiciones del ejemplo 3, el lado de In
baso de la pirámide ('S igual o. a y la arista lateral, n b~ Hallemo!I el
área de la sección. Con esto fin, necesitaremos aclnrar t·a forma do la
sección (~ipo del cuadrilátero P KMQ). Ahora p = 5; por lo tanto,
160
Capitulo 11. Eotertomotría
no es posible realizar al azor cualesquiera construcci'ones métricas
en la representación que tenemos. Pero, todas lns posteriores construccjones .son de posición.
Como de acuerdo con la construcción PK 11 SB y QM 11 SB,
PK 11 QM. Además, PK es la línea media del ASAB, es decir,
PK = {sB = {b. Análogamente, QM = {b. Por esta razón,
PK = QM y, entonces, el cuadrilátero PKMQ es un paralelogramo,
con la particularidad de qu~ PK = -}, PQ =
Para calcular el
área del paralelogramo, los das
tos de que disponemos son insuficientes, por ello precisamos
la forma del paralelogramo
PKMQ. Construyamos BL, la
mediana del AABC (somejantc
construcción no requiere el consumo de parámetros).
A
C
El punto O, que es la base de
la altura SO de la pirámide, yace
en la mediana, BL. Como OL J..
J:.. AC y OL es la proyección del
segmento SL en el plano ABC,
B
SL J.. AC. Esto significa que
AC J.. OL y AC J:.. SL. Pero en
tal caso AC J..$8. (Esto demuestra que las aristas cruzadas de la pirámide triangular regular son
perpendiculares entre sí.)
.
Después, ya que PQ U AC y PK 11 SB, PQ J:.. PK, o sea, el paralelogramo PKMQ es un rectángulo. De este modo obtenemos
S />KMQ = ab
EJ.EMPW ., • La base. de la pirámide SABC es eJ triángulo rcctángul9 A.BC . .La arista SA ,es. perpend~cular· !ll plaqo de la base, S A ·=
=Á.B =.BC
Por ~l .puJito M'lamiil!d de:la arista AB, perrendicu)~r a la .a.r!stá
tracemos un plano secante y hallemos· e área de
la· !¡ección -o.bXenida.
5PLuc;10N._Sea e~ cuadrjlátero SABC cop sus diagonales SB y AC
I~ repi:esenta~¡ón _
de :fa ·pfrÓ.m_íde dada (fig. 138). Elll! es completa.
S.u . numero paraniétrico· p = ' 5 (cerciórense de ello por su cuenta).
A.hora, pasemos ~- la con:Si:r-ucción de la sección.
RealÍcémosla ·en dos etapas. Pri-mero construyamos un plano secanie. auxi.Jiar perpeIJ.dicul.ar a la a.rista SC que :pasa por el vértice A.
Después de esto, s.erá fácil, con.s truir también el plano requei;ido
como un p~ano para:ll}lo al auxiliar que pasa por el punto M.
f·
z•
=a.
se,
§ 12. Setcloner d• polltdr01
t6l
1) Construyan la_me._diana AD del triángulo, SAB. Es evidente
que 'como en el triángulo SAB SA = AB, AD -1.. SB.
2) Construyan AE ..L SC. Ya que en Ja pirámide hemos consumido !} par~metros, e~á-cláro_ que al tomar en se el -puqto E al azar
no podemos decir que AE ~Se. Operemos del modo siguiente .. Del
triángulo is6sceles récÚng.ulo A Be hállemos que Ae = aV·.2 y del
triángulo 't'ectángulo ·sAC hallemos que, entonces, se.= aV3. Así
··pué"8, pára que 111 ..~égnie~tl;> AE sea la .representación !le l~ perpendi·
cu lar a .}a arista se debe realizarse la siguiente igliald!ld:
.
S..{"-SE2=Ae2 -CE2 o bien a=-SE= = 2a2 - (a y3-SE)2,
de donde
halla~os
que SE·= o
.y3 • es decir, SE: se:;= 1: 3.
De modo que en la arista Se construimos el punto. E t.al que
SE : SC = t : 3, n continuación, los puntos A y D se unen con el
punto E.
Demostremos que SC es perpendicular al plano ADE. P rimero,
de acuerdo con la coilstr,u.cción AE -1.. se y, segundo, como nos cercioraremos de inmediato, AD -1.. SC.
s
En efecto, como SA es pe;Spendicular
al plano ABC, ¡A# será la .proyección
de SB en el plano ABC. Pero AB -1..
...L BC, es decir tambi.én SB ...L BC.
De forma que Be ..L AB y Be _LSB,
lo que significa: que Be ...L AD o
A
e
bien quo AD ..L Be. " Pero, además,
AD ...L SB, lo que quiere decir que la
recta AD,es perpendicular a cualquier
recia que yazca en el plano SBe y,
en particular, AD ...L se_ Así, pues,
hemos demostrado que se es perpen8
dicular ol plano ADE.
Fig. ISB
Pasemos ahora a la segunda etapa
de construcción.
3) En el plano S.AB, por el punto M, trazamos MN 11 AD.
4) En el plano SBe, por el punto N, trazamos NL 11 DE5) En el plano SAe, trazamos LK 11 AE.
0) Trazamos M K.
Es evidente, que como M.N 11 AD y NL 11 DE y la~ rectas AD y
DE determinan un plano, las rectas MN y NL asimismo determinan
un plano. De modo que el cuadrilátero MNLK es plano. Además, de
acue.rdo con la construcción el plano MNL es paralelo al plano ADE,
o sea, el plano MNL es perpendicular a la arista SC. Así, la sección
MNLK satisface las condiciones del problema.
Ahora podemos pasar a la determinación de S.vNLK• es decir, el
11-0200
162
Capitulo II. E•lereometría
área de la sección. Con el fin de calcularla, para empezar hallemos
el tipo del cuadrilátero MNLK. Del triángulo rectángulo SAB,
donde SA = AB = a, hallamos con facilidad que MN ={AD =
n'J/2
=
-4-·
Examinemos el triángulo SBC. En él L SBC = 90°, SB =
=a
BC =a, se= a V3. A continuación, en el triángulo SNL
3
SN=-¡;SB= -3a 4112
y NLl l DE, pero DE..LSC (ya que SC es
perpendicular al plano ADE), es decir, también NL..LSC. En tal
NL
SN
caso, el !:::. SLN CZ> !:::. SBC, por lo que ne =se, de donde N L =
v2.
o
V (i
= -1, - y
sL sN
S'íf=sc•
d
1 d
e con e
sL =a-va
2-
Más adelante, l:::.SAC en !:::.CLK y, entonces, ~~
~~ , de
6
111
donde LK = { . Y por fin, del triángu lo AMK que es rectángulo,
ya que SC es perpendicular al plano M_NL, o sea,
,l. MK y,
claro está que también AC será la proyección de
en el plano
ABC, perpendicular a MK. Es asimismo evidente que MK = AM
se
se
a
all ?
AM = T En tal caso, MK = T·
Así, pués, hemos calculado todos l,os lados del cuadrilátero M N LK
y obtenido que MN = MK y NL = NK. Entonces, !:::.MNL =
= !:::.MLK y SMNLK = 2St.MKL•
.----- - --. /,, Pe1:0
111K .l. LK,
es
decir,
1
a•11a
SM:."LK =
2· 2 MK-LK = - 8 -.
-Jl.,,,,_,____,,.+,¡C,
Consideremos los problemas de l
segundo tipo .
EJEMI'LO ~. L a base de un prismn
recto, cuya a'ltura es ígual a 1 cm,
C
es un ro:ipbo con lados'igu~les a 2 cm
y ángulo ¡\gildo de 30q. Por un
lad·o de la base se traza un plano
A ·
K B
secante entre el cual y el. plano d()
~a bpse se forma un ángulo igual
139
Ft,.
a 60º. Hallemós el área de la SCC·
ciói:i.
soLuc10~. Sea la figu:ra ABei:JA 1 B1 C1D.1 la representación del
prisma dado (fig. 139). Ella es completa. Su número pararoétríco
p = . 5 · (calculen .e~to por su cuenta).
Como en este ejemplo la <;onsln¡cción de la sección es un problema
métrico y P. = 5, habiendo tomado ni azar en la recta DD 1 el punto M
y
Í63
§ JZ. StcelonéS dt pol!tdroi
y en la reeta CCr, 91 pµnto ¡.,. (claro. est~, 'tales que ML U.AB), no se
pued~ ·affru:¡ar que .e rcv.afil.HáJer.o .AMLB es la reprei;e!ltnción do la
~cci'ó~ prefij:ada, No .<>bs~ail~e·~ S:Lconociérarnos t·a posJc1ón del punto·
.M q·e ·;i:ite'rsección 4el. .pl~no" s.e<;ánt·e con ~a -arista PDi. pod·6amos
const~.utr.
,con focili!lad Ja .. rcP,re~nfación do la .sección.
..
'Si· c:\et·p.u nto p .bi;i.jaJA!J.S l,a pe~pendjcular"l~K a !a ari~ta Ali. Q~
el tri~ngul() . rectángulo ADK obt eni do,' '·t endremos: AD .= 2 cm,
UK.. ~ 1 c~. Y" por lo tanto, Af(· -F 'V3 cm. ·!'\sf, pµcs,, Jlara ooni¡t,ruir
D K ...L AB debemos elegir ·e l ,púnto K de ma nera qqo. se CQl)lj>Ja, la _
. . .. .
.
¡jgi.uµdad AK: AB = y-3: 2,
Se1¡1ejante construcción 11uede realizarse, p. ej., 11-sL Ópnstruyamos
'Un ánguloJ rectánguló auxiliar con la hipotenusa AB y cateto ·~ AB.
Entonces, el segundo cateto scrli igual n
~3
AB. Como
11/' AlJ :
: AB = 113: 2, AK puede tomarse igunl n In lo11git11!'1 dd segundo
cateio deÍ ·triángulo au.x-i:liar.
.
Más aaelante, como la arista DD1 es perpendicular di plano ATJC,
co~ cualquier p~stdón d·~1 punio M en lu nrista DD_1 el segmento D_K
sera la proyección del segmento MK en el plano- ATJC, es decir,
LMKD será el ángulo lineal del diedro en la arista AB.
Si el punto M es tal que LMKD = 60", en ol triángulo rectángulo MKD DK = 1 cm, MK = 2 cm, MD = )13 cm. Así, pues,
al construir el punto M, hay que tomar en consideración Ja igualdad
MD : DD 1 = Y3: 1. Podemos, p. ej.,, r ealizar la co.nstrucción del
punto Medel modo siguiente. Construyamos el triángulo rectáng11lo
aux~liar con un cateto igual a DD 1 y la hipotenusa, a 2DD 1 • Entonces•. Ja lQngitud del cateto será igual a }'3 DD1 • Pero como
Y3D.Di : DP1 = .
1, MD se puede tomar igual a )a longitud del
segundo · cateto del· triángu.Jo auxiliar..
Después d·e construir el punto M, traccmos ML 11 AB, donde el
punto L. yace ·en la arista CC 1 • Habiendo construido la recta AM.
hallemos el puñto Q ·de intersección de las rectes AM y A 1D 1 y,
de forma analoga, hallamos el punto P. El c.uedril~tero AQPB es
la representación de Ja sección dllda. Calculemos S MIP n·
Es evidente que el cuadrilátero AQPB os un paral~logramo. C.Orno
M K ...L AB, NK i;!S la altura del paralelogramo, donde N es el punto
de intersección delasrectasMKy PQ. Asi, pues, S,.,QPn=AB XNK.
Hallemos NK. En el triángulo rcctáuguloM DK, tcncnios: AfK =2cm,
lv!D= JÍ'lforo. Esfácil demostrar que DKllD1N,csrlecir, ~:.·=::.
v·a:
.
2
113
2
va
o bien NK =-1- , de donde NK=-3-
=
11 •
4
ra
cm2.
ero. Así, pues, SAQPa =
C apltulo f 1. Estereometría
16~
EJEMPLO s. En una pirámide triangular regular se ha trazado una
sección por la Hnea media do la cara lateral, paralela a la arista
lateral y por el vértice de la base de la pirámide que no yace en la
mencionada cara. Determinemos el ángulo de inclinación del plano
de la secció·n con relación al plano de la base si sabemos que el ángulo en la arista lateral de la pirámide y el plano de su base es igual
a CtSOLUC!ON. Sea el cuadrilátero SABC con sus diagonales la representación de la pirámide dada (fig. 140). Ella es completa y su número paramétrico p = 5. Construyamos K y E que son los puntos
medios de las aristas BC y SB y, a
s
continuación el t::.AEK, la representación de la sección prefijada de la
pirámide (estas construcciones no
provocan el consumo de parámetros).
Como SO es la altura do la pirác mide, LSCO es igual al ángulo entre
A
la arista lateral y el plano de la base, es dec ir, LSCO = a. Es preciso
hallar el ángulo diedro formado por
el plano secante AEK y el plano de
B
la hase. Denominémoslo LAK.
Ftg. UO
Con el fin de hallar el ángulo
diedro buscado hay que construir
y determinar su ángulo liµeal. Ya que p - 5, en l a representación
que tenemos no se pueden realizar al azar ninguna clase de construcciones métxücas. Es preciso construir la representación do dos perpendiculare¡¡ a la arista del ángulo diedro. Realicemos la construcción necesaria del modo siguiente.
Construyamos OB y, en el plano SOB, EF 11 SO. Entonces, como
SO es perpendicular al plano ABC, asimismo EF será perpendicular a eso mismo plano,. por lo que EF ..L OB. Como AK es la mediana
del triáng'u'l9 equilátero ABC, BC ..L AK. Tracemos FL.·.11 BC y,
e-µtoñces, f¡[, ..;L 4 .K . .Pol' fin, construy.amo.!1 EL. ;f\sí, ·pues, ~·.{( .L
.J.. FL y 4'11'. ..L E.F. Pero, en ' tíll ·s;.as_o,_ AK ..L EL, o· sea, LELF
es el áng-ulÓ' lineJll'- del d'iéd~o qu¡¡ ·buscamos. lb.gamos LELF = x.
Para d,ct:erfuinar x introducimos el parámetro auxiliar AB = a.
Ahora, nall~mos -EF y FL.
··
Como EK es lalín.ea media del /;:>.SBC, BE= SE. Pero de acuerdo con la c9nstrucció·n ]SF 11 SO. Por lo tanto, EF es la línea media
del b.SOB iY• por ello,. $F = ~ SO. Del triángulo rectángulo SOC
2 ,q/3
av3
hallamos. que SO= CO tg a, donde CO = 3 · 3- = - 3- ·
Así,
pue~, EF = ail:
tg a. Más adelanto, hallamos, que FL =
165
§ J2. Secelontt de poliedro•
t .
2 }/3_
HF
l
=z.BK= 4 a . De manera que tgx =n = -rtg~. ·y por 1o
Jnnto, x=·arctg (
~-~ª- tg ~).
OBSERVACIOl'I. En el ejemplo_quo hemos estudiado, a ¡irirnera vista , puede
puecer. que la conalrucción ""del ángulo lincµil ·del diedro buse~do hubiera sido
su cucn14): l) OM U BC; 2) SM, 3)4)·0P. En efecto, coiísemejanle construcci\Sn L..,P(>M ca el ángulo lineal del J ic_dro formado por fós planps AE}(
y_A.J1C-....Per.o los in.tenlos de determinar LPOM ompleand9 tal construcción
lropléza con considerables amcultados.
.
convenie~ie reali~arla osi (hagan' estojo_
r
p es el puqto do inlers~ión de -S/tf y E
En,.cier.tas ocasiones también se t ropieza con la necésidnd de construir lii sec.ci6n al Iesolver problemas, en cuyo planteamiento nada
se dice· aceren de la . sección (véase el
s
ejomplq 2 en ol § 10). .En algunos cnsos,
·l a co·n.strucción .do••ln . sección, en principio, no es nec<¡saria, no· obstante con
su ayuda· la sol ucióll- del · prohlema .¡>uede
ser simplificada.
EJEMPLO 1.
Una arista de la pirá- A
e
mide triangular es igual a a, mientras
que cada una de las restantes, a b. Hallemos el volumen de la pirámide.
SOLUCION. Sin detenernos en la cons8
trucción de la representación (flg. 141)
Ftg. 141
y las otras etapas de la resolución, señalemos que s i resol vemos este ejemplo
empleando la fórmula V= at Sr, 06./{, al determinar la ulturn H
sería necesario real-izar cálculos bastante ~omplicados. Operemos de
otro modo. Construyamos l a sección d!! la pirámide: con un plano
que pasa po.r la arista AB ' perpendicular a SC. Si A!J = a, entonces: V=+s6All~1SM+fs6AIJMCM=fs6ADMSt= ~ s6ABJll·
(Señalemos que la construcción de In sección en ei caso dado es
un problema de posición, ya que los óSAC y LlSBC son equiláteros, es decir, las medianas AM y BM son la represej1tación de las
perpendiculorcs n la arista SC, por lo que el b.ABM ~asimismo la
representación de la sección perpendicular a la nrista :SC.)
Como s6ABM= ~ AB·MD, donde MD es la med~ana del lriángulo isósceles ABM, en el
i
, /
3b2
a2
a
=Ta V -¡;---¡;-= T
X V3bZ-a 2 •
q~e AMº=BM =
v--3b -az.
2
Por
lo
bra,
tanto,
S 6 Au1t1=
V
ab
=Tz
x
Caplt1•Lo lJ. Ester•omttrfa
166
PflOBLEMJ\S PARA EL TRABAJO INOl\'IDUAL
i24. En ~l cubo ABCDA 1 B~C,D 1 , cu:,.:a arista es igual a a, F os el punlo
medio d!l 111 arista D 1 c,. Hallen la distancu1 desde los puntos A 1 , A y C1 basla
~l plano quo pasa por los puntos D, F y D.
725. Por la diagonal de Ja base inferior rle un cubo, c uya arísta es ígual
a a, y por el punto medio de uno de los lados de la base superior so traza un
plano s~ante. Hallen In distancia desde el centro del cubo basta dicho plano.
726. En una pirámide cuadrangular regular tracen un plano por la oia110nnl de la base' paralelo a la nrist.a lotera!. Hallen el área de la sección obtenida
si los Indos de.fa base y lns aristo.s laterales son igua.les a a y b, respectivamente.
727. Cada unn de las arist.as de una pirámide cuadrangular regular es
igual a a. Construyan la sección de la pirámide con un plano que pasa por los
puntos medios do dos lados adyacentes de la base y el 'Pllnlo medio de la alturn.
Hnllcn el área de estn sección.
728. En :Un prisma hexagonal rcgul:\r las caras laterales son cnadrados.
cuyos lo.dos son iguales a a. Construyan Ja sección del prisi;na con un plano que
asa por un 111llo do In baso inferi or y por el lado opuesto a és~e de )a base superior.
Jallcn ~1 hcá <ltl la sección.
729. En :un parnlcl oplpcdo recto ol ángul11 agudo do la baso es igual a a .
La s~cción dql paralelep1pedo , traza<ln por un lado de la base, cuyn longitud
es tgunl s a. y In arista opucs\n a ól Licue un área igunl a S y forma con el
¡1lano de Ja bnse un ángulo igual n 90° - a. Hallen la longiLud del otro lado
de la bnsc.
730. Ln altura de un prisma lriangulnr rcg11lnr es igual a JI. Por una <le
las aristas de la base y el vértice opuesto de la otra base se ha trazad(> un plano. Hallen el área del triángulo obtenido en la sección si su ángulo en el ind ic.ado "érticc d~I prisma es igual a 2«731. En el cubo ADCDA 1B 1 C,D, por I' y Q, que son los pu ni.os med ios d<:
las aristas AB y AD, y el vértice C1 se trazo un plano secante. Hallen la d istancia desde el vértice C basta el plano secanlo si la arista del cubo es igual
a n.
732. El :plano secante pasa por el vértice A d.e la base del cubo
ABCDA 1B 1C,1D, y por P y Q qu e son los puntos medios de las aristas B,C, y
C1 D., rcspcct1vomc11tu. Hallen el árcl\ de la sección si la arista del cubo es igual
f.
a a.
.
733. El ' plano secante pasn por el p1111t.o /í., Lomado en la arista AA~ del
cub<> ABCDA 1 /J 1 C,D,, y por/> y Q quo son Jos puntos medios <l e Ja~ aristas
B 1C1 y C1D"' rcspcctivam~nle. Hnllen el árt!a de la sección si la arista del cubo
e~ lgunl a a =y Ali.: KA,= 1: 2.
734. 'El plnno secante pasa por/> y Q que son los puntos medios de las aristas AA, y CC 1 • r~~pectlvamP.ntc, del prisma triangulnr regular ABCA 1 B 1C 1
Y. el punto D, tomado en la arist.n lntcral B8 1 de modo que B 1D : DB = 3: 2.
l~allcn el área.'·dQ. la ,sección sí cnda arista del nrisma es igua 1 n a.
La! baso , de 1~n)1 pirámide , c_nda· una <le cuyas aristas es igual a
a lf~. es e~ reclnngulo ABCD· .,con lados iguafos a a .y 2a. ~imstruynn la
5cceión· de
pirá~ide criJi un pi aro. qup pasa por Ja dingpnnJ BD de la base
parál~IO a 111 ari§t.o. la_teral S.A. Hallen· el área de la sección.
136, Ln!Q:lsc infotior d!! 1in .¡>ril¡mn es el rombo ADCD, c.n cuyo vértlt:c ol
iíngúl.o, es igtinl a 60º. :E:J'-Vór.li~c de' la base svpcrior A, es equidistante do los
vértices A, y D, 'l:rnri5ta ,'/IA 1 : · j y íomfa con el plano de Ja base el ángu!o a.
ConslruY.an In sección del 'prisma con un plano quo pasa por la diagonal A 1 C
P.ilralelo· a .l~ .diagonal BD. Hnllcn el árcn d\: lo. sección.
737. Lai ba'so de una pirámide es un triángulo isósceles con lado igual a
º " El, ánguloidc lá, baso de la 1>ir~mido es igú~ l a a y cado. arista lateral es oblicua 111 pll.1no,8ll la base bajo el ángulo ~" co·nstruyan la sección do la rirámidu con
un plano 11ue"JJ•sn por la altura de ésta :y el vértice de uno de los ángulos Jgunl
a o.. Haflen :el iíren de 111 sección.
·1.:%.
I*
a
§ 12. Stctlones de polltdro1
167
738. En un tetraedro regular se ºhan tnuido dos socc:ionlll!, cad a una do las
cualllll es parale.l a a lu aristas A B y SC. El área de la par~ de la cara SA C situada entre los planos secantes es S cm• mayor que el área de la parte de la cara
SAB quo.se encuentra entro dicho~ plan9s. ¿Cu!Íotae voces el área de una sección
es mayor que la. de la otra7
739. Ef área de 11_1• base. qCI 'un·· pa¡al~lépipedo re9t11nguler
igual n S .
.~or el 'v9r~lc.e A 1 'de' la ·base supcrior, A 18 1C1D·1 so traza un .plano que lnter~eGá
'las aristas B!J., CC, ·Y /)01 on los.puntos B,, C, y· D 1 , respectivamente'. Hallemos el volumen de la· porte del paralcleplpedo ·quo está situada debajo del plano secante si sabemos que CC 2 - e y l:t altura del paralelepípedo es igual a H.
·
740. La. base de la plri mide SA.BCP es ol rombo ABCD en el quo A C ... a.
BD e: b. La l!~ista -lateral SA, cuya longitud es Igual ne, es perJ)6ndicular al
plano de la base. P or-el punto A y el punto medio do la ar ista SC tracen un plano paralelo a la diagonal BD . Hallon o} área do la sección obten ida .
741. El lado de In baso de una. pirámide cuadrimgulor regular es lgu11I a
4. Por un lado Jle la haJSe y el punto medio do la arlst4 lciteral, que con ella se
cruza, eo traza una sección. Hallen la r!istancia desdo el plano <lo la socci1ín hasta
el vérUco de la pirámide si su altura es igual a JI.
.
742. En un pri~ma. triangular regular, por uno do los lai:los 1lc la base,
se trn1.a 11n plano que con el de In baso !orina el ángulo a.. Hallen el área do ln
.socción triangular obtenido si sabemos q11e el Indo do la base os Igual 11 o.
743. El ángulo entro In c:tro lateral de una pirámide cuadrangular regu lar
y el plano de su base es igual o a.. La apotema de la tara Jaternl es igual a a.
Por uno de los lodos de 111 base se trai.a una sección de la pirámide qu o forma
con ol plano de la base el ángulo ¡5. Hallen el ároa do dicha ~ción .
744. El lado de la base de una pir ámide cuadrangular regular es lgunl n
o. El ángulo entre la nrista lateral y la altura de Ja pirámide es i~unl a 30•.
p9t ol verlice de la
Construir la sección de la piré. mido con un plano que
base per pendicularmente a lo ari sta opuesta. Hellnn e área do la sección.
145. La base.do .un prisma os el cuadrado A BCD, cu yos vérlices se encuen·
tran a la misma dista11C:ia dol vértice A \ de 111 baso superior, AA ,= a, el ángulo
entre la arista lateral AA 1 y ol plano de la base es· i11ual 11 60°. Construyan In
sección dol prisma con un plano perpendicular a la arista AA 1 y quo 11osa por ol
vénice c. Hallen el área do Ja sección.
74G. En ua·a pirámide triangular regular ol lado do la baso es igual a a.
El &n~ulo e¡ltre las aristas adyacentes. es igual a 2et. Construyan 1:1 aección do la
pirllm1de con un plano. perpendicular a la ar is la lateral npucsta, que paea por
uno de los lados do la baso. Hallen el. área de la sección.
747. Ep uqa pir6~1do triangular regular, por In oristo de la hase de longi·
tud a , so traza una sección perpendicular 11 la arista lntcr:il opuesta. Hallen el
llren de la superlicio do la pirámide si el plano secante dh·i<le la aristn lotcrol
en la razón m : n.
748. E n la pirámide ·triangular SABC la arista SA es P•l':iemllcular al
plano A BC, AC = BC
a, AS - A B
4lf2,° P(lr el punto Il)edio de la arlst.a
A e se ha tratado un plono porpendiculnt a Ía arista SB . Hallen la distancia
desde el vórtice A haala dicho plano.
749. En el cubo ABCDA 1H1 C1D , construyan la ttección ,quo pasa por el
punto B , por M y K quo son los puntos medios de loa nrlslns CC1 y AD, rcspocl i·
vnmcntc. Hallen el ángulo diedro entro el pin no do la sección y el plano A Ofº[J.
75-0. Construyan la sección del paralelepípedo rectangular A B CD A 1C1D 1D¡
con un pi ono que pasa por el v6rtice A , ol punto modio de la arista lºD )' e
centroldo de lo cara s cc,01 • Hallen ol ángulo <llcdro entre el plano de In SC'C·
clún y el pl3no ABCD si A O: AD : AA 1
2: 3: 4.
75l. Construyan la sección dol prisma rectangular reg11lor ABCA,BtC,
con un pi ono que paso por el vértice A , por K y M quo son los puntos mcd los
1: 2 y AB: 80L de la~ nrisi.as 88 1 y cc, respectivamente. si CM: C1 /lf
... t : 3. Hallen el ángu 1o dlctlro entre el plano do In sección )• el plano 11 BC.
es
fº'ª
=
=
=
=
168
Capltufo 11.
P.st~rtomtlrla
752. Construyan 1;1 SC<'ción de la piriímide cuadr11ngular regular SABCD
(S e~ el v,;r~icc) cnn un plano que pasa por el punto A , por J>, quo es ol punto
med io de In nllurn SO. y el punlo K de In oristn SD si SK : KD ~ 2: i
y ~·o = BD. Hnll~n el áugulo diedrn entre el plano de Ja sc<:ción y el plnno
de In base.
75ll. El ;\ngu lo cnlro In nrisla la teral y el plnno J ~ la bas.i do la pl r{1mltlo
tria ngular r<•gular S AJJC (S l?S el vértice) es igual • o:. Construynn In $ecclón.
<le esta plr áu1iclc cC>n un pl11uo qu e paso por el punt o A, P , que es el punt o
1uc1lic1 de la al tura SO y d ¡iunlo K en la 11potemn SD de la car11 S A C si
SK : K D = 2 : l. ll•llen el áogulo diedro entre el plano de la sección y el de
la base de fa pirámide.
754. Un prisma cuadrangular regular cst~ corlado con un plano do formll
que en la sccc;ión so ho obtenido un rombo, cuyo ~ngulo es igual a 2o:. Hallen
<'I úngulo dícclrn ('nlrc ol plnno secante y el de la base del pr isma .
755. La base ele unn pirñmidc es un trillngulo isóS<:elcs w el que el ángulo
entre lo$ lados lgun lcs l'S igual a o:. Cada arista latera l de la pirámide es obl icua con relación a l p la no de la base bujo un ángu lo igual a ~- Eo csl11 plri!.miclo se tra7.a llll pl nno sccnn to q11 c pasn por In nltura du la pirám ícl o y por el
v(?r lic1: dd 1í ngulo igual n et. lfoll ~J• In ra7.ci n entre el ¡íre:1 du Ja succión ubl.t1·
nidu y 1•1 árc:t 1lc la linsc de In piními d1:.
i5G. La ha~" rlc un pnra lclcpipcdo oblir.uo es el 1·01nbo AJJCD en el c¡111.•
LIJA D = llOº. Cncla uristn lutcrnl 1lel p11ralclc•pípedv forma c:on el plnno cfo
la base• un ár1gu l11 igual a 60°. mientras que el plano AA ,C1 C es pcrpcndio:mlttr
a ~l. !fallen la ra7.ór1 entr e rl úrea de lo sección JJ8 1D 1D y de la s~cción AA 1 C1l'.
757. La bnsc de un parolt>lepípedo recta es un paralelogramo, enlre cuyos
lados IR razón es AlJ : Dl' - 1 : 2 y el ángulo tn el vértice B e.s igua l n 120°.
Por c•I punto D y el v~rtice opur.sLo de )1\ hose superior hn de trazarse un planc •
sc.:ante para lelo 11 la dingouo l AC. Hall en el ángu lo form ado por rste plano
con el plano de In base si la rinón enlre la a lturn del paralele pi pedo y el lado
mrnor ol e Ja bhse es iguo1 a 1/ 3: 1.
758. La base cl o unn pirám ide es r.l r~ctán¡pilo A BCD, en lant o que In
altura de la p,irám idc se proyecta en ~J vér t ico B do la baso. AB : A l> : SB
~ 2 : 3 : S. Por Ja diag!lno l Bl> se t raza un plano paralelo n unn do los aristas
do la pirámide que no cort:i In diagonal BD . Ha llen el ángulo do inclim1ciún
cntrr el plano secante y el plano de la bnsc de la pirámide.
759. Uno pirámide lcíangular regular está coruda por uo pfono que pasa
por d vértice de In ba!c y los puntos meditls de dos acistas laterales. llallcn
la rnión entre las óreas de la superficie lalcral de la pirámide y del pla no do
111 base si sabemos que el pleno secante es perpcndiculnr a una do las car as l:i·
ternlcs.
760. En un prisma cuadrangular r cgu lnr se hon traz:ido dos aecciones pn ralelas entre sí: unn pnea por los puntos medios do dos lados ad yecentP.s de la
base y el pun~o med io <l cl eje do simetría del prlsmo , la otro cllvi do el e je en In
rai6!1 1 : '3 . Ha llen ·1a razón entré Jas árens do la primero y segunda seccione~.
761; La baso de uon pirámi<!e e~ uri triángulo regular con "lodo a. Una
de las e.aras de la pirámide es per pcodicu lar al pleno de la base -y ello es un
tr ióngulo igóscelC!l! con lado latera l igual o b. Bollen el. área de la sección de
la pirám ide. que es un cuadrado.
762. Un plano secante divide por Ja mitad el ángulo diedro de la base
de una pirámide cundrangular . Hatlen el área de la sección si el lado de la base
de la pirámide es igual o a y el ángulo d iedro en la b:.so es igual a 2o:.
=
169
§ 13. Á re111
§ 13. ÁREAS
EJ:&MPLO t. La base del prisma ABCA 18 1 C1 es un t riángulo en el
que AB = AC y LÁCB e:: a.. T a mbién· sabemos qué D es el ru nto
medio de '. la. arist'ii AA 1 , LD(lA = ·~ y CD ""' b. H al lemos e órea
lateral d!il prisma.
so1.uc10N. Sea la figura: ABCA 1 8 1C1 •(fig. 142) lil representl\ción
del prisma dado. Ella es completa. Calculemos su núm~o par.a métrico. Suponiendo que AB y AC son las r epresentaciones.de segmentos
iguales, consumimos un parámetro; consid eran do LACB .Ja representación de un ángulo igual en el original a a., consum~mos un parámetro más; si c9~ideramos que el segmento AA 1 es la ·representación de la perpendicular al pl ano ABC, consumimos dos parámetros
y, por fin , suponiendo qye LDCA es la represen tación de un ángulo
que en el original es igual a ~ . consumimos otro parámetro. Así,
pues, el número p oramó t1·ico de la ropresentación q ue an nliznmos
p = 5.
Vamos n considera r q ue CD = b (con ello, no cons umimos nuevos
pnrámctros). Ahora podemos efectuar los cálculos necesarios. Como
ABCA 1 8 1C1 es un prisma recto, S 1a 1 = P·ll, donde P = 2AC
DC, H = AA 1 y el 6-ACD es rectángulo. AC = b cos f>, AD =
= b sen fl, ya que AA 1 = 2AD1t .entonces AA 1 = 2/> sen fl.
Comstruyamos AE, es decir, l a mediana de l 6-ABC, es evidente
que AE ..L B C. Del 6-ACE rectángulo, hallamos: CE = AC cos <X -=
= b cos tl cos a.. · Entonces BC = 2b cos cr. cos fl.
Así, pues, S1 n 1. = (2b cos f>
2b cosa. cos fl) 2b sen ~ =
= 4b' cos tl (i
cos a.) sen tl = 4b~ sen 2tl cos'
+
+
+
+
2
1
T·
%·
De modo que S 191 = 4b sen 2fl cos
EJEMPLO 2. La base do una p irám ide es un triángulo equilátero
con lado igual a a. Una de las caras l at erales, perpendiCu lar a l plaoo
de la bas~, también es un triángulo eq uilátero. Hallen el área l ateral
do la pirám ide.
SOLUcJON. Sea el cuadrilátero SABC con sus diagonales la representación de la pirámide dada (fig. 143). Ella es compléta, su número
paramétrico p = 5 (el 6-ABC es la representación do un triá ngulo
equilátero, es decir, dos parámetros, el 6-SBC, l a representación de
un triángulo equilátero, es decir, dos parámetros, el ángul o died ro
de la al'ista BC, la representación de un á ngulo r ecto diedro, o sea,
un parámetro).
Como la pirámide no es regular, hallemos el área de su superficie
lateral como l a suma de las áreas de las caras laterales S 10t =
= SA8AD + sb8A C + sil.SBC· Pero el 6-SAB = t::..'SAC (por tres
lados), es decir, S 1n1 = 2St.sAs
SABB c · Mñs adelante, si SK
t
es l a altura del 6-SAB, S 6 8 A 8 = 2 AB· SK.
+
170
Capitulo Il. Estueometrta
Como sabemos, la construcción de una perpendicular a una recta,
en el caso general, es métrica. En ol ejemplo qu9 estudiamos esta
construcción ha de realizarse en una representación definida métricamente, es dec ir, habiendo tomado al azar en el lado AB el punto K
s
e
A
8
Fi¡¡. 142
Fig. 143
no podemos considerar que SK es 18' representación de la perpendicular AB.
ConstruyP,mos S K del modo siguiente:
1) Construyamos sv· que es la me(liana del í:::.SBC.
2) Construyamos CM que es mediana del t:::.ABC.
3) Por e! punto D tracemos DK 11 CM.
4) Unamos Jos puntos S y' K.
Como SD es la mediana del triángulo equilátero SBC, SD J. BC.
Pero el plano SBC es perpendicular al plano ABC. Entonces, SD es
perpendiculár al plano ABC y, por lo tanto, DK será la proyección
del segme11tc? $K .en el pl~no ~OC. PeroJJK JI CM y -C~f 9~'.la_.¡nedia­
na .del t:::.ABC· equUiítero, es·~decir, CM .l. AB y, por ·elló,. DK J..
J.....4.Q; e~ ·l).ccfr,. y SK J. Ali. Dei 8SDK ~ctár¡.gulo, :tenemos:
ii.~.· 1í;D{+oK2' cfo~d~ SD=ªYª ' DK,,:;-}CM=ªf3 .
• .
·, . ; .
,q/ ts
at
As1, P\JC~, SK=·- r · Obtenemos: SAsA0=3
at 113
T
.
at V ·= - 4-·-· ",
esta :razón S 1nt·=T ( 15+ 3).
por
v-
·v15-·
y
SASoo =
~· La base de uná ,p irámide es un trapecio isósccios, cuyos
lados paralelós spn iguales ·a a 'Yb (a > b). Cada cara lateral es o blicua a la liase bajo un ángulo igún.l a a.. Hallen el área total de la
pirámide. 1
'EJEMPLO
§ 13.
Área$
171
SOLUCJON. Sea la figura SAflCD (fig. 144) Ja representación de la
pirámide dada. Ella es completa. Hallemos su núméro paramétrico.
El cuadrilátero ABCD es Ja representación del trapecio isósceles
prefi jado (dos _paráme\ros), ~O •.la representación de la perpendicular
al plano ABC (dos parámetros). Consideremos que Lr!.B; LBC, LC.D
y LAD son· las represeQtaciones
los ángulos dfodros, tod.os Jos
cuaJes son iguales a a en el original ,
consumimos sólo un pa.r ámetro.
s
En efecto, supongamos que los
segmentos OM y OL son las representaciones de las perpendiculares a los lados AD y DC del
trapecio; respectivamente. Entonces estará claro que los segmentos
SM y SL serán las ropresentaciollN! de las perpendiculnres a esos
A
p
mismos lados AD y DC, respcct.ivamente. Por esta razón, los ángulos
Ftg. 1'1
SMO y SLO serán, respec;tivarnento, las representaciones de los
ángulos lineales de los diedros AD y DC. Pero, entonc'es, los triángulos SMO y SLO serán las representaciones de igu~les triángulos,
mientras que los segmentos OM y OL, las de iguales Segmentos en el
o riginal. Del mismo modo ~i ON y OK son las representaciones de
las perpendiculnres a los lados BC y A B, entonces ON y OK serán
las de segmentos, cuyos originales tienen la misma lnrgura que el
segmento Ol'tf en el original.
Asi, pues, O es la representación de un punto equidistante de
todos los lados del trapecio. De otra forma: el punt.o Q es la representación rlel centro de la circunferencia inscrita en el trapecio ABCD.
Esto significa, Jlrimero, que M y N son los puntos medios de las
bases AD y BC del trapocio y, segundo, que DM Y. DL son la representación do segmentos iguales en el original Y: que CN y CL
también son segmentos iguales. Pero, entonce.s , ·DM: CN =
os dcci'r, DM: CN =
= DL: CL. Y como DM; CN =
= a: b, tendremos que, asimismo, DL: CL - a: b. Por analogía,
AK : JJK = a: b.
De modo que para la construcción de las perpendiculares OM,
OL. ON y OK a los lados del trapecio no se consuJ!ien parámetros.
Esto significa, p. oj., que considerando que LSMO ~la representación do un á ngulo que en el original os igual a a, cpnsumimos sólo
un parámetro y, a continuación, suponiendo que lo·s ángulos SW,
SNO y SKO también son iguales a a, yn no consumimos parámetros.
Por consiguiente, para repreS-Outar la pirámide dada se han consumido cinco parámetros.
de
,,
-i-: T•
Capitulo !l .
i72
E•tcr~omctrla
Ahora calculemos S 1, es decir, el área total do la pirámide. Tenemos: S 1 = S 1a 1 + S ABcD· De la igualdad de los tríángulos rectángulos SMO, Sf,,0, SNO y SKO se desprende que SM = SL = SN =
= SK.
Entonces, S 10 t = { (AD + CD + BC
AB) SM =
1
a
= 2 (a+ b + 2CD) SM. Pero DL = DM = T y CL = CN =
+
=
b
.
a-~b
2 , es decir, CD .,, - 2- .
Construyamos CP fl MN. Entonces CP ..LAD. Del triángulo
rectángulo CDP, donde CD=
y fJp,..
hallamos:
CP =
i/ ( átb
r-( ª-;• r
ª1b
=
Vab.
ª-;• ,
Pero, MN=CP y OM=ON,
es decir, OM = ~ CP. Por lo tanto, 0 1l,f = ~
rectángulo SOM,
8101 ~·• .'~
tenemos:
SM =
~':a
=
y ah.
2 ·~;cr.
Del triángulo
.
Así ,
pues,
(a+ó+(a+b)) 2J!ii& = (atb)ll~.
Más
adelante,
cosa
cosa
1 (a+b) ,¡ah De forma que S
S
(a-t-b) ·v~+<a+b) V~
ABCD""'2
"
.
t =
2coscr.
2
(a+b) V~cosi-T
=
cosa
Un triángulo regular cuyo lado es igu.a l a a gira en
torno a un eje exterior paralelo a su altura y distan.c iado de él a { a.
Hallemos el área de la superficie del sólido de revolución obten.ido.
s0Luc10N. Como el eje de ro,tación l 0 es paralelo a Ja alturo. del
triángulo, él es también paralelo al plano de la figura y, por ello,
la sección del sóli'do de revolución prefijado con el plano que pasa
por l 0 será la figura 1%> 0 constituida por un par de triángulos regulares simétricos con .r elación a l 0 •
Esto ·qu'iere · ~ecfr_, que en el ejemP.lo que· consideramos podeinos
Jimitarn9~ a ~a.. reJJresenta'ci'ón 4e una figu_ra ·plana en lugar de representar ·ull' sólido .de revoluci(>n basta'n te complicad.o, e~ decir, es
po_si!He consld'tilr J·a reprcsen~ac}lin .de la indicada figura <D 0 (con
pr'ecisjón· hasta .la s~mejanza) . ,Además, para resolver el ·problema
. incluso ·p.odemos limitarnos a. l.a represen~a'ci6n del eje lq y sólo de
uno .d.e 'los triángulos. obténidqs en la seccfón del sólido de revolucíón
dtido en· el pfano que pasa por l 0 (fig. 145): Así, pues, el ~ABC es
regular,. AS= a, Al) .es I'a altura del C:.ABC, l lf AD, DP .l.. l,
-DP ~ ~ y es preciso hallar el áre.a de la superficie del sólido de
r.e voiución. De$ignemos el 3rea buscada con S ABC·
EJ EMPLO '·
§
/.3. Área$
'73
Designemos ·tarp.bién las. áreas _de las ~uperficies formada.s por la
rotación de los ~gilientos !A~. A C y BC .alrededor del. eje l con
S·A. »• S AC y S se ,i::espeétÍvám.onte. ~ntonces, S,,, oc = 'S,,, s +
+ S,,,r; + Ssc·
+
+
Tenemos: S A 8 = n (BP
AQ). Al). ~ero BE = BD
DP =
''i , pues, ,,, 8 .-'P"" 27 11a.2• Por
= 2a, AQ ·= .DP = .23 a, Al! = a. As
s
analogía, S AC = n (CP + AQ) AC = {-. na~· .M'.ás lidolante, S'.ai; =
= nBP 2 - nCP 2 = n (4at - a 2 ) = 3nat. De ~odQ que S,,, se =
·
= 9na 2 •
OBSERVAC!ON. El área buscada so hubiera podído c¡¡lcular il.q íorma algo más
sencilla si hubiéramos hecho uso del primer teorema do Gul~in, ·de acuerdo
con el que S = P2nR, donde P es el perímetro do la llgura q~o gira alrededor
del eje, R, la clistancla desdo el centroido do dicha figura hasta el eje de rota-
ción. En
c.l ejemplo
que hoqios considoradn /> =
:Jo,
11
=-}a.
EJEMPLO s. En una pirámide cuadrangular regu;l ar el ángulo
entre las caras laterales adyaceilt.e s es igual a 2ci;. Hallemos la razón
entre el área de la sección diaglona'l de l.a pirámide y sµ área lateral.
SOLtJCJON. Sea la figura SABCD la representación de la pirámide
dada (fig. 146). Ell.a es completa, su número paran;iétrico p = 5
s
Q
p
8
D
A
Flg. 145
Fig. 146
(cerciórense de esto por su cuenta). Así, pues, o! ángulo diedro en la
arista se, es decir, LSC = 2a. Hay que hallar la .r azón St;.SAC:
: S1n1·
Con el fin de realizar los cálculos necesarios construyamos, primero, la representación del ángulo lill'eal , p. ej., en la arista lateral se.
Para ello, es preciso bajar una perpendicular del punto D a la arista
se.
Sea D K la representación de la perpendicular a In arista SC.
De esto modo, en la representacíón se ha consumido un parámetro
más y, ahora, tropezamos con la llamada representación definida
Capitulo 11. Estereom<trla
174
con exceso (pero cierta). A coustinuación, construyamos BK y OK.
Entonces, LBKD será el ángulo lineal del diedro SC y, por consiguiente, LBKD ... 2a:. Es fácil demostrar que BK = DK y, por
ello, OK , que es la mediana del triángulo BDK, será asimismo su
bisectriz y altura. Así, pues, LDKO = a.
Ahorn, señalemos que S 6 sAc=2St;soc=OK·SC Y S1nt=
= 4S6
scD=2DK-SC. De esta manera,
s~~~c
=
~. ~~
.
Pero
del triángulo rectángulo ODK tenemos: ~~ =Co5cr. y, entonces,
obtenemos:
s6
SAC
S1at
cosa.
2
E n una pirámide cuadrangular reg,1lar el plano secanle, trazado por un lado de la base, divide por la mitad la superficie lnternl y el ángulo diedro en
s
la ar ista du Ja base. Hallemos el
ángulo entre la cara lateral y ul
plano de la liase.
SOLC'CION. Sea la figura SABCD
(fig. 147) la representación de la
pirámide dada. Ella es completa.
e Su número paramétrico p = 4
E F
(calcúlenlo por· su cuenta). En la
pirámide se da una sección que
D
satisface las condiciones métricas.
Calculemos cómo se consumen los
Flg. 147
parámetros al representar dicha
sección. Ante todo, señalemos quo
como la sección pasa por una arista de la base (sea por la arista AD),
debido a ¡¡ue AD 11 BC, AD será también paralela al plano SBC.
Entonces, el plano secante corta ln cara SBC por PQ (PQ 11 AD).
A continuacíón, realicemos algunas construcciones más, tales como:
construyaµios M que es el punto medio del segme!}to PQ, tracemos
la recta 81.M y marquemos el punto· M de· intersección de las réc.tas
S M. y BC.. ' Es e:v,ident e·:.q1,1e BK =. C:K. Gonstruya,m.os, ¡\dem·ás, la
r~c~a f(O."'N.· el pu!ilo N de l.nt.~rseccion· .de I¡¡s:rectas OK y AD. Como
OK 11 ABI N ..es el p\lnto medio de .lá arista AD. Ahora, .constr.u imos·
SN y !f.M,. Como N es el punto medio do la arist.a AD, ON .L AD•
.Pero ON ~s. la proyecCióil del segmento· Sf{ en- el pla'no ABC. Por
'cónsfg\lteit~e 1 . ~'N .L AD.. Pero·, entonoe.s, LSN K es el ángulo lineal
de aquel' ángulo diedro en la arista AD, un a de cuyas arist as pasa
por el puhto' S y 1a otr¡1, por K . Designefl}os este ángulo d.iedro de
la s'iguied,te forma: LSADK. Así , p-µes, .LSN K es el ángulo lineal
del. dfodrQ S-ADK. Considerando N M la representación de la bisectriz del fogulo SN K; suponemos simultáneamente que el p lano
EJEMPLO 6.
§ 19. Ár~as
APQD divide v.or la mitad el ángulo ·S.4DK, lo que correspo·nc\e al
planteamiento del:ptobl~~.a. Tomando N M como la repr.eseritación
de la biseqtriz de LStf.l(, ..para la repres!).nta~ión consuinim9s un
l'al'ámetr.o má~. -Est;<r ·s~g~Hica qúe ah.ora ya ·hemos cónsu1Jlido los
cinc~>- par~.i:Qetros .. Adem~s. todavía i:lo hemos t er:iido en cue11ta ·que
el plano seca.nt~ divide .p or la:mi"te:d l¡i. S'1perficie lá.~eral de la pirámide: Com,.o veremos ~ás .ailelantc, la suMsición d·e qu~ el •plano
bisectriz AP.QD es la. reprc'Sentación c:lel plan<> que •. diviq~ pQr la
mitad el área de la superficie lateral de ia pirámide, conduce !J. ·.1a
suposición de que AD : SM = V2: 1. Esto significa. que.. en la
representación se consume un parámetro más (el. sexto:). ·Cl aro iist6.
que la revelaciqn _de este hecho podría no ha:berse aplazado, sino
realizarlo de inmediato al construir la representación. Sin embargo,
por razonamientos metp.do~ógicos sólo lo haremos en el proceso de la
resolución y vamos a resolver este ejemplo lo mismo que el anterior,
en una representación definida con exceso. Pero, a qiferencin del
anterior ·ejemplo, aqui la representación puede ser tomada al principio caigo incierta», lo que no· se ·refleja en los posti¡riores cálculos
y, después de descubrir la depende,ncia ~D : SN =Vi: 1, dicha
representación puede sei: ya reaffanda de tal -forma que para ella se
consumirán exactamente cinco parám.e tros.)
Así, pues, vamos a resolver un ejempló en una representación para
la que p = 6. Al" pasar a los cálculos necesarios señalemos que como
la pirámide es regular, los ángulos diedro·s en. las arista.s son Iguales.
Por esta razón, es indiferente cuál de e:;tos ángulos halramos. Es
evidente, que. es 'conveniente buscar el ángulo diedro en la arista
AD, ya que por ésta pasa el plano secante. Ya hemos demostrado
que LSNK es el ángulo lineal del diedro S.ADK buscado. Hagamos
LSNK = x. En ·la arista 4.D se han formado dos ángulos diedros
más: las aristas de uno de ellos pasan por los puntos S y M, del
otro, por K y M. El primero de ellos es LSADM, el segundo,
LMADK. ·E l ángulo lineal del primero es LSNM y del segundo,
LMNK :·
Como el plano secante di.vide por Ja mitad LSADK, LSADM =
= LMADK y, entonces, asimismo, LSNM = LMNK, con la
particularidad de que cada uno de .estos ángulos es igual a
Ahora, introduzcamos un parámetro auxiliar, haciendo el lado
de la base igual n a. Del planteamiento se desprende que
f.
.o-+
St:. s ...
2S1:. SQD +286 BPQ = 2S1:. DQC +sBPQC·
(i)
Calculemos todas las•áreas que entran en Ja igualdad (1). Para
abreviar, hagamos l a longitud de la apotema de la cara lateral igual
a l.
i
1) SAsAv=-y al.
Capltulo 11. Esttrtometrla
176
2) Del 1::. SON rectángulo tenemos: cos x = ~
SM+MK=l,
{ s. M _ SN -..!...
MK -- NK -
Del
t:.. MNK
.
Más ade lante,
¡
¡2
dcdondeMK=a~L y SM=a+i ·
a '
(según el teorema de los seno&) timemos: ~
=
%
sen 2
NK
de donde
sen (1so•-(x+ ~)) '
1
- sen2:..=sen.::....
a+l
2
2 •
(2)
-
3) Construyamos QF, que es la altura del t:.. DQC. Entonees,
aQF. Construyamos SE, que es la apotema de la cara
S 6 z:>Q c =
f
•
QF
QC
QC
MK
SDC. H.a lletnos QF de la proporción SF=sc· Pero sc=SK,
. QF
MK
F
al
es d ec1r, se= SK, de donde Q =MK =-;+T ·
•
.
As1, pu~s,
SA oqc =
112 t
2 (a+1)
i
a•l
4) So. s<iD=So.sco - SA Dqc=2al - 2ca+1f =
1 S
5) S¡;,, 8 pq=T
J11 . p Q.
PQ
SM
Pero •.B'C=S'K,
de
al~
2(a+t} ·
donde
PQ=
al
=a+I'
• S
D
e aqu1,
6\ S
o. sPQ =
S
a!3
2 (~+ii•
S
.
1
A .s sc:- ASPQ=2al
a13
a•l(a+21)
·2(11+..ll'
. 2(a+i>.' .
':'/). Ponil)i:tdo en (1) los valo.res 1h aU!).dos, de las .áreas, ca'lculamos
·I
PPQ.c=
qu~
(3)
(I.>e aquí s~ .dedu~e lá. depen,dencia AD i SN = Y2: 1, <le la que
hali)amos a) ·calcular ·el ·n úmero parartié'tricó de 1a repre.~entaci ón.)
Ex¡fresand:oj el vdor de a de la ecuación (2), oht!mei;nos a=
1·sen~
2
l. Poniendo este valor de a en la igualdad (3), lle%
.sen-r
§ 13. Ártu
177
84!01=..
gar.g;os a la ecuacfón
--!- -1 = lf2,
de
donde
obtenemo!':
sen2
3z
"'
sen T-sen
2 =
¿sen 2·:z: .o b'1en 2 sen 2"' cosz= ,I'¡ .-2sen 2z .
i /7i:
I'
Como sen ; #=O, de la l'ilti·ma ecul\ción cos .x =.' ~í ,· de dol.lde
45º,
Z;;:=
PROBLEMAS PARA EL T.Q ABAJO INDIVIDUAL
763. En un pr.isma cuadrangular regular la diagonal es igual a d y está
Inclinada respecto al plano de la cara lateral bajo un l\ogulo lg'1al a Cl. Hallen
el área latorol del prisma.
764. Loa ángulos formados por la diagonal de In baso de un paraleleplpedo
roctangulnr con el lado de lo base y con la diagonal del paralelepípeélo aon iguales a a y 8, respoctivamonte. Hallen el área latéral del paralelepípedo si su iliagonal es lgual a e!.
·
.
765. La altura de uti prisma cuadrangular regular es igual a H y el ángulo
entre las diagona'le8 trazadas dosc;le..uilo de los· vértices do Ja baso a dos caras
laterales adyacenl-es es iguat a a. Hallen el irea laleral del prisma.
766. La altura de un prisma triangular regular es Igual a H. La roela que
pasa por el ccntroide de la base superior y ol punto medio del lado de la base
mferior forma co1_1 el. plano do la base el 6.ngulo a. Hallen el área total del prisma.
761. El ár1ia total de una pirámide cuadrangular regular e.s Igual a S. El
ángulo diedro on la arista de la l>ase es Igual a a. Hallen el lado de 111 base.
768. Hallen el área total de una pirámide cuadrangular regular si su altura
es igual a H y el área de la cara lateral es Igual a la de la bue.
769. La base de una pirámide es un tfl4nRlllo reet.ángul 0 cuy03 catet03 son
Iguales a 6 y 8 cm; reapectivamen~. El 'ogulo de lnclinaci6n de cada una de
las caras lateral~s de la pir,mide hacia el plano de la base es Igual a 60°.
Hallen· el árn lateral de la figura.
770. El área lateral de una· pirámide cuadranfulor regular es Igual a s
y el ángulo diedro en la arista de la hase es igua a a. Hallen la distancia
entre el centrolde de la hase y la cara lateral de la pir6.mlde.
771. El úea total de•una \)irámlde triangular regular os igual a S, el ángulo
plano en el v~rUce de la pirámide es Igual a a. Hallen el radio de h . circunferencia circuDBCrlta a la baso.
772. La base de una pirámide es un cuadrado con lado igual a "· Oos
carns laterales son perpendiculares al plano de .la base y cada una do las otras
dos, forman con ella un ángulo igual 11 a. Hallen el área total de la _pirámide
773. La baae de una piré.mide es un triángulo. Dos caras laterales adyacentes son perpendiculares al plano de la base y las otras dos forman con él
ángulos que son igunles a et y ~. respectlvamonte. La altura dé la pir ámide es
H. HaUen ol &roa lateral de la pirámide.
774. La base do una pfr6.mido es un triángulo, ontre cuyos lados la ra·
z6n es 13 : 14 : iS y cada uno de los 'ngulos diedros en las aristas de Ja base
es igual a 45°. Hallen la razón entre el á rea total de la pirámide y el área de SU!
bases.
775. La base de un prisma recl.o es un trián~lo Isósceles en que el 'ogulo
entre los lados iguales equivale a 2a. De los vértices de In base superior se trazan dos diagonales de las caras laterales Iguales. El ángulo entre las dlagona12-0290
Capitulo II. Eitereometrla
178
les es igual A 2~. Hallen Ja razón entre el área lateral del prisma y el área de
sus bases.
i 76. El ccntroi<lo do una de las caras de uo cubo es unido con los vértices
de ln cara opuesta . Hallen la razón entro el área total de la pirámide que se forma
y el área tot.al del cubo.
777. Por ol lado d<> la base inferior de un prisma triangular regu lnr y por
el punto medio de la ar ista lateral, que no se interseca con este lac!o, se ha trn·
zado un plano que con el de la base forma un ángulo igual a o;, Hallen la 1azón
CJmc el área lateral de la pirámide que con el lo se forma y el área lat-0ral d~I
prismu dnd<> .
778. El, ángulo entre las generatrices en la sección axial de un cono es igual
a 2cz. Hallen la razón entre el área lateral del cono y el área de su ~cción axial.
77!1. El ángulo máximo entre las generatrices do un cono es igual a 120°.
Demucs~rlln que el árcn lateral del semejante cono es igual al área lateral do
un cilindro que tenga la base y Ja altura iguales que el cono.
780. El' área lateral de un cono e,¡¡ un cuarto de 110 círculo enrollado en forma
de una super:ficie cónica. Hallen la razón entre el área total de la superficie del
wno y el úrea de su sección axinl.
781. BI úrcn lateral do un cono truncado es igual a la suma de las árrns
de la~ bases . .en tnnto que la razón entre los rudio~ de lns bases el! 1 : 3. Hallen
el ángulo de inclinaci'óu de In generatrh hacia el plano de las- bases.
782. Un triángulo regular cuyo lado es igual a a, grra en torno a un eje
paralelo al lado del triángulo y pasa por el vértice opuesto a dicho lado. Hallen
el área del sólido do revolución hallado.
783. Uri triángulo rectángulo, cuyos catetos son iguales a 5 y i2 cm, gira
alrededor de: un eje exterior paralelo al cateto mayor. y alojado de él a una distuncin de 3 cm. Hallen el área del sólido de rev.olucióli obtenido.
rectángulo, cuyos la!fos son iguales a a y b, gira en torno a un eje
784.
perpendiculnr n su dia~onal y que pasa pQr uno de sus extremos.Hallen el ár~a
del sólido de revolucion obt.ení!lo.
785. Uri triángulo isósceles c·o n la baso igual a a y ángulo en la base igunl
n o; , gira en ,torno a un eje que pasa P.<>r uno do los extremos de Ja base porpen·
di cu lar a elfo. Hallen el área del sólido. de revoluclón obtenido.
786. Eri el trapeci<> rectangular, circunscrito a la circunferencia de radio
R, el ángulo' agudo es igual a a. Hallen el área del sólido obtenido al girar el
mencionado .trapecio alrededor del menor de sus lados paralelos.
787. Un rombo con ángulo agudo o; gira alrededor de su lado. Hallen la
razón entro el área del sólido de revolución obtenido y la del rombo.
788. Una recta corta do los Indos de un triángulo rectángulo, entro los que
el ángulo es igµal a 60º, segmentos cuyas long.itudes constituyen 0,25 de la largura
d.e Jo hipotetiusn. Hn:l\cn lá razón. el)tre,el área del -tritípgulo dado y 1'a del sólido
-o_b~eni.<!q al girar dicho triúngu·JQ· en: torn·o. a Ja méncionada -recta.
.
· 7.89. Uri trapecio isóscoles· con .ángúl.Q en J.a. hose de. 60~ gira· en torno a In
·bisectriz de ~icho .á ngúlo. Hal len lá ra;ón ·cntto ol:.~r·eá..drl sól_iilo de ~evolución
y ia· del tr,e,Jioc¡o si 1n a"itura d~ é_ste es· J/3menor quo 1.a s_o~is!1ma ~e. sus bases.
790. Un plano ~ecante; pa~alelo a l.a base de µna. p1ram1de 'r1angular re·
gulnr • .éfivide··pbr "tá .mitad 's u supe~!icic lateral. Hallen en qué raz6n se divi'de
pór esto co~te. !a nltura.
.
.
.
. 79.1. E·l lila.no secante·-¡iei'pendiculli.r a la cnra. $BC trazado por el lado AD
.do ·1a base de la _piráliiido cuailrangular, rogülar SABCD, divide dicha cara en
dos· figuras . de ú~ciis iguales. 1-l~llen el 'áfea total ·de Ja pirágildc si AD = a.
~92, En· una plráo¡ide cu:iorángu·lar régúlar ;el plano trr.zaao por 110 Indo
<le la base, <liv.ido por· hr mitad la superficie late"tal y .el ái:igulo diedro en In
arista de la base. Hallen el ñngulp dlcilro lateral de la p1~ám1de.
793. U'1{ plnno secante' vasa por fa arista de la base de una pirámide cuad"r angular regula~ y corta en Ja' cara opuesta un triángulo, cuya área es igual a
Uo
§ 14. Vo.ldnunu
S1 • Halliµt el ál:ea lai.eral ,de.la. ¡>.ir~.npd·e 11.ue eslá separada de la pirámide dada
por el plál')o ·seeante, si· el á·~ea. Jat·eral' do esta es· Igual n s,,
794'. T:.a base ile. una p.f'rám!<!é es ·un romb~, cuyo !ad~ es lgua) a a· y e~. ón•
gu~o .agu~tj e';ltre s~s la!los es·a,. C!!da uno \Ít) los ári.gulo~ d1c.dros en.Las aristas
de'la·base·es 1g1i.al a q>. }fallen el.área lateral do ·la pirám\de,
7.95. Ca hMe de una. ,pirámí'dé es un trapecio' isóscéJcs, tuyo diagól\ál ·es
igual a d ·y el ·á ngulo entré diól)a .dlagon('l y ·la base ma,yor del trape<;io '.cs igua•l
a a,. Cidá un·a .. de las· caras.laterales.de la .pirámide ·está inclinada hacia et pl'ano
de la· base bajó un ' ángulo •q> . .l-4'1len .el área tOtal de la pirámide.
.
196. LaJopgitud (le cadá ,1.!ó.o·de los .lados de un prisma ·triangular es igüoi
a', a.1.a· P.roye~ión de uno de loa vértices de 'la base"snperior es e\.óentrolde "do)a
ba5e. infer ior, Las arlst.n s. lateral~s e·s~n iñclin:i.das ni plano do a base bajo angulos. iguales, cnd11 un9 de·ellc;is, 8' et'. .Halle'n e.l á.r.e a la~ei:al del p,rism3. _
797, La basé de un paralelepf¡>edo, cuya arista ln.teral és igi1al a b, es. un
cuadrado con .lado a. Uno· de Ios.'v órtices de la ba~e ~.uperíor es equidi!tante a
todos. los vértices de la base inferior. Hallen ol área to.tll.1 del. paralelepípedo.
798. La base de un prisma es un triángulo. regular, cu.yo Indo es igual e, a.
Cada arista lateral de In figura es igual 11 b, el ángulo entre una ele las aristas
lotera les y los lados de la bnso ndyaccntcs a ella, os lgnnl 11 45º. Hnll~n l'I ñrcn
Jntéral dél prisiµa
799. ¿Cuántas veces es mayor la distancia desclo el punto l uruino~o hasta
el centtó de una esfera.que el radio de ésta sí el área de la parte iluminada de la
esfera es 2 veces menor que lá que está en sombra?
§ t4. VOLÚMENES
EJEMPLO l. La base de un paralelepípedo ·recto es un paralelogramo, cuyos lados son iguales a a y b, on tanto que el !ángulo obt11so
es igual a cp. Hallemos el volumen del pnralelepípedo isi su diagonal
,
menor es igual a la diagonal mayor de la base.
SOLUCION . Sea la figura ABCf?A1 B 1C1 D 1 (fig. 148),'la representación del paralelepípedo dado. Ella es completa. Su número paramét rico .P = 5.
En efecto, considera11do que AA 1 es la· representacij)n del segmento perpendicl\l¡n '"~l plano de la ~ase, i,onsutnimos d9s parámetros.
Suponiendo que AD : CD = :a: b, . consumimos un 'p~rámetro. Si
consideramos que, p. ej., LABC es la representación fiel úngulo que
en el original es igual a cp, consumimos un parámetro;más. Con ello,
ya que consideramos que LABC es la representaciól']. de un áng11lo
obtuso, debemos suponer que los segmentos B.D y AC son las representaciones de las diagonales menor y mayor del paralelogramo
ABCD. Por esta razón, consideremos que B 1 D : A"C = 1 : 1. Así,
pues, consumimos un parámetro m.ás.
.
Hallemos ahora V, el volumen del paralelepípedo. Del L::..ACD
tenemos: ACª = AD2 + CD 2 - 2AD·CD cos cp. Pero AC = B 1D.
De este modo, B 1 D 2 = a 2 + b2 - 2ab cos cp. Del L::..ABD obtenemos: BDª = az + b2 - 2ab cos (180º - <p) = a 2 + b' + 2!1b cos ip.
Del triángulo rectángulo BIJ1D tenemos: BBJ
B1D 2 -BD 2 ,
es decir, BB~ = (a2 + b2 - 2ab cos cp) - (a2 + b
2ab cos <p) =
= -4abcos CJl y, por lo tanto, BB 1 = 2 V-ab cos q>.
=
+
12•
180
Capitulo 11. Etltreomtlrla
Como SAnco=absenqi, obtenemos: V=2absenq>V-abcosqi.
De acuerdo con el sentido del problema q> es un ángulo obtuso.
Por ello, -1 < cos q> <O y, entonces, (-cos q>) es un número
positivo . .Como sen <p >O, el valol' hallado de V es positivo.
Por esto, V = 2ab sen cp V - ab cos cp.
EJEMPLO 2. Las caras laterales de una pirámide triangulnr son
perpendiculares a pares y sus áreas son iguales a Q1, O: y Q3• Hallemos el volumen de la pirámide.
so1.uc10N. Seo ol cuadrilátero SABC con sus diagonales la re presentación de la pirámide dada (fig. 149). Ella es completa. CalculeA
B
a
Ftg. U8
e
Flg . 149
mos su n(ímero paramétri~o p. Como las caras laterales de la pirámide prefijada son perpendiculares a vares, asimismo, sus aristas
laterales son también perpen<liculares a pares entre i;í. Considerando
SA la representación de la arista de la pirámide perpendicular a sus
aristas SB y SC, consumimos un parámetro. Considerando SB la
representación de la arista perpendicular a la arista se, consumimos un parámetro más. De este modo, habiendo consumido tres
parámetros, hemos asegurado la perpendicularidad mutua de l as
cnras laterales de la pirámide. Suponiendo, a continuación, que los
í::.SAB y !L::.SBC so n las representaciones de las caras lattirales, entre
CU~as. ár.~as la razón es igual a Q.¡ : Q~, es decir, SUpQniend·o ·que
(tSA·SB). ': ({sn.sc)
~Qt _:Q2 o .IH~nSA
:SC=Q.:Q2.
cónsumíJos u n parámetro, Por. analogf'a, considerando que L::.SBC
y L::.SAC{son 1as representaciones de las caras late1·ales en las que
la razón entre las í1rcas es igual _a Q, : Q3 , también consumimos un
parámetro. Así, pues, para ·Ja 11lpresentación C!mstruida p ... 5.
A}).ora\ calculamos V, o sea, el volumen de la pirámide. Señalemos que :s i por tradici6.h consideramos el í::.ABC como la base de
I.a pirái:ni"de, primero debí11.b)os haber calculado So.A ac y la altura
de la. pir~mide trazada del vértice S al plano ABC. Sin embargo, en
el ejercicfo que est udiamos es posible realizar los cálculos ruicesarios
de for.nia más sencilla si se nos ocurre •poner• la pirámide sobre cual-
§
U. Voldmenu
181
quiera de sus caras latera les. A~í. pues, a l advertir que l a arista SA
es perpendicu)ar a la cara SBC; podemos tomar como base de la pirámide el. .ASBC. Entqnces, V;;::
SA·S.iABC = {. SA·SB·SC.
i-Iiu;iendo, , p¡¡ra .sbréyiar, SA = a, SB = b. y .SC ·= e, tenemos:
f
' V -....
~:ab.c .
·
·
De· los triángulos rectángulos SAB, SBC' y· SAC obtenemos:
ab = 2Qi. be = 2Q 2 , ac = 2Q 3 • "Multiplicando, término por térmi·
no, estas tres igualdades, hallamos (abc)2 = 8Q1Q2Q3 , de donde
abe = 2
2Q,Q,Q3.
V
!
Asi, pues, V= V2Q 1Q2Q3.
EJEMPLO s. El área lateral de un cono, en el que el radio de 111
base es R, es igual a la s uma de las áreas do la base. y .do In sección
n:xiol. Hallen ol volumen del cono.
SOLuc10N. Sen la elipse (1) junto cun un por do tangentes trnindas
a ·ella desde el punto exterior S la representación del cono da(lo
(íig. 150). Ella es completa. Al considerar la elipse w como la roprosentación de una circunferencia, consumimos dos pará!uetros. Considerando ol segmento SO como l a representación do la altura del
cono, consumimos dos parámetros. Construyamos la r~presen tació n
del AB que es giámetro de la circunferencia, así como SA y SB,
es decir, l ns rep·resentaciones de las generatrices deU cono. Para
e llo, no se consumen parámetros. Por fin, suponienqo que se hu
representado un cono tal en e] que S l•t = S buo
S ABA B• consumimos un parámetro más. (En efecto, de esta igualdad. es fácil most rar que la raz6n SO : AO está plenamente defin ida.) De modo que
para la representación construida p = 5.
Pasemos a determi nar V , o sea, el volumen del cooo. V =
=
Sbar~·SO, donde Sbatc = nR,. Hacemos para abreviar
SO = x. Así, pues, para calcular V hay que hallar x.
Como slbt"'" nR·SA, sb.s• = nR 2 y si>S AO = Rx, componemos la ecuación nR·SA = nR2
Rx o bien nSA = nR
x.
Pero, del triángulo rectángulo SAO, tenemos: SA ,,,;,, Y x• +a:.
De modo qué obtenemos: n V z2 R 2 = nR x.
Elevando al cuad rado ambos m iembros de esa ecuación, después
de las simplificaciones necesarias, obtenemos (n2 - 1) z%- 2nRx =O,
bR
O
de donde x1 - 111
_
1 , x,= ·
Está claro que la segunda solución no satisface el planteamiento.
. .
zn•n~
Por cons1gu1ente, V = 3 (n'-t)
IUEMl'LO 4. Un rectángulo con lados a y b gira en t9rno a un eje
que pasa por su vértice y que es paralelo a la diagonal que no pasa
+
F
+
+
+
+
Capitulo Il , Bstereometrí:i
182
por dicho vértice. Hallemos el volumen del sólido de revolución
obtenido.
SOLUCJON. Como en el ejemplo 4 d el § 13 , limitémonos a la re proscnt ación no del sólido de revolución, sino sólo de la figura que se
obtiene en la sección de dicho sólido con el semiplano, cuyo límite
es el eje de rotación. Somejant.o sección, que es un rectángulo ABCD
con la diagonal BD. con la recta l paralela a l a diagonal BD y que
pasa por el punto A , es la representnción de dicha sección (fig. 151)
s
Fig. 151
Flg. 150
(En t.111 caso la representación se efectúa con una. precisión hasta 1a
semejanza, es decir, aquí no hay ni g,u é hablar del consumo de parúmetros. )
·
Asl, pues, AB = a, AD - b. Calculemos V ABCD• es decir, el
volumen de~ sólido de revolución. Dnjomos de los puntos B, C y D
las perpendiculares 881, CC1 y DDt a la rectal y desde el punto A ,
la perpendicul ar AA 1 a la recta BD.
Ent onces, VADcn=(V1 +l'2)-(V3 + l' ,), donde V 1 =Vc1s 1sc =
nB 1 CdCC~+cc,.BB , + BB!)· Es fácil cerciorarse de quo
=4
CC,=2881 •y, entonces, V1 = ~ nB,(:,.788!. Acontinuación , V2 =
... VD1C1éo..,; ~- nC,D1 (Ce;
=
~ nAB1
· Poro co~o l
·PBi, V =V
1
4
+ ce, . DD¡ + DD~).
ADi D=
.....--
hallamos qiie BD= 11
y a.· + 1>2 Y AA, =
'
= V AO¡ D =
! nAD1 ·DD~ .
Ít BD , .BB 1 = DD 1 == AA,. Del
.
V3
t:;,, ABD rectángulo
ab
'Va2+b•
Más adelante, hallRmos que CC, = 2AA 1 =
•
~b
s . Ent onces,
a• +ó
t
( 4o 2ó'
V, + V 2 = : n (CC:+cc,.AA 1+ AA~)(B 1 C 1 +C1D1)=3 n a•+b• +
3
2aó
•
aó
a'ó' } , ; 2 + {>Z _ _!_ :rt
1a'b'
+ 11;;r.:¡::¡;o ·• ll a•+b' + a•+b• Y a
- 3 Ya•+b' ·
,
1
')/
.183'
OBSERVAC.Í óN. El volumenbuscadQ del ·s61ido do revolución.. se hubiera po~
dido calcular. haciendo ilso del segundo.teore.ma d.e Gul¡Hn, do acuerdo CQn el
.cual V= s ...2nR, donde Ses el área· de1a figtira quo gira en ·torno. al !ije y" R.
la distancia d.eado el éentroi¡:le de esa flgu r{I b1,18ta el 11ie do rotacíón .· ~s. fácil
hallar que en el ejemplo que heI.llOS considerado S
=
ab, R· =A.Al
·v :b+ :b~· .
0
EJE.!llPLO s. El lado de la pase· de una pirámide, trlaQgill~r regu.lar
es igual a a, la altura bajada desde el vértice do la base hasta la
cara lateral opuesta es igual a b. Hallen el volumep rio la pirámide.
soLuc10N. Sea el cualdrilátero SABC con s us diagon¡¡les la representación de la pirámide dada (fig. 152). Elln es complota. Considerando que AABC es la r!!presentnción de un
s
triángulo regular, consumimos. dos parámetros; suponiendo que SO es la ropresontación de la altur.Jl .de la pirámide, también
consumimos dos parámetros. Construyamos SD, que es una mediana del ASBC y e
la representación de la apotema de la cara
lateral de la pirámide. Sea AK la representación de la perpendicular bajada del vér8
tice A de la pirámide dada a la apotema
de la cara .. lateral opuesta. Como es fácil
Ftg. 152
mostrar, el segmento AK será también,
en tal c~so, la representación de la altura
bajada desd" el vértice de la base hasta la cara lateral. Esta
condición métrica impuesta a la .representa.ción provoca el consumo
de un parámetro más. De este modo, hemos consumido en la representación loR· cinco parámetros. Pero, no se ha tenido en cuenta una
condición de acuer.do con la que en el original A 0 8 0 = · a y A 0 K 0 =
= b. La circunstancia de quo estos datos son literales nos ofrece
cierta libertad; vamos a considerar que en la representación construida AB : AK = a : b. Así, pues, imponemos a la representación una
condición métrica mál! y, por lo tanto, consumimos otto parámetro.
Pero en semejante caso, la representación será métricamente definida
con exceso. A pesar de todo, al realizarse cierta dependencia entre
a y b (más abajo ella será obtenida) la representación construida es
cierta.
Mostremos ci.uo lu1biendo tomado al azar en la apotema SD ol punto K,
no se puede considerar que A K es la representación del segmento A 0 K, que eo
el original es perpendicular al segmento S0 D 0 •
C4pltul<> / 1. Estereomtlrfa
184
0
Como según el senLido del problema la altura A0 K0 existe del D. S¡y40 D, ,
a i/ 3
.
'
don de A, K0 =b y AoD0 = -- , es posible calcular quo D 0 K 0 : S0 K0 =
2
&•-4b 1
.
4b•-2a• • Eligiendo diversos vo.lores (tolerables) de a y b o distintos valores de la razón a : 11, obtendremos diferentes valores de la razón D K0 : S 0 K0 •
es decir. distintas pósiciones del punto K 0.P. eí., prelijantlo
obtenb-8,
2
3a -4b•
H
dríamos que 4.b' _ 2 , =-¡¡-. Como vemos, prefijando de modo t.an concreto
4
los valores de a y b la posición del punto K0 se determina con plenitud, ya
que D0 K 0 : S 0 K0 = 11 : 14. Esto quiero decir, que ya no podemos elegir al azar
el pumo K 0 •
{a= fo,
Así, pues, sea SABC la pirámide regular dada, en la que AB =
= a, AK = b. Hallemos V, es decir, el volumen de la figura.
1
a•i/3
ComoV = 3SAAoc·SO, donde Sl:l.,uc = - 4 - , para determinar el volumen es suficiente hallar la altura SO. De la semejanza de los triángulos rectángulos SOD y ADK, obtenemos: ~~ =
OD
= DJ(• de doncle
=
ª~ª,
SO=
AK · OD
DK
,
DK=f AD2 -AKi
De esta manera" SO
ab
1
donde AK=b, OD=aAD =
· V~.
Vii
V34' -4b'
. Entonces, V
a•b
•
12 i/3a,-4b•
Según el sentido del problema AK es la altura bajada a la cara SBC
(!precisamente a ella y no al plano de la cara!). Es evidente, que
si la pi rámide dada es tan .-hajitu que LASD = 90º, los puntos
K y S coinciden, lo que significa que los segmentos AK y SA coinci·
den también. Si, poT lo contrario, la pirámide es aún más •baja»,
la altura bajada a lo. cara SBC ya no existirá por completo.
Haciendo LA.SD=90°, de t::; MD rectá'ngulo, ·donde OD =
a·ra'
ªYª '
V ar.a·.
"rª
ªr6 .
A0=
hallamos: SO=
=
Así, pues_, para Ja ·existencia de la altUl'a AK es necesario que
1os valores de a .y b satisfagan ia desigualdad
s·v~X~ 4 b, ~ ª
r
5
o bien el sistema ·de desigualdades equivalente a la anterior desiallZ
a'J/3
gua1dad; - 2-..:s;; b~ --z .
. Al mismo ~ie.mpo, es evidente que de acuerdo· con el sentido
del prol¡lema AK <AD, ya que AK es perpendicular a la Clll'a
SBC y AD, oblicua a ella. Entonces, b..:::: ª
Esta dependencia
ra .
§ U. V.olúr¡ienes
entre · a y b ·y a 'fue ·advertida más
ra .
¡¡~riba·.
t85
De tat i:nanéra;
si ·~~
~b< .ª
Ja pl1:ámide que satisface las ·condiciones del próble.ma existe.
A-i
""' " pues,
V=
'ct3li
d nd a')Í2 :;:::.b.<. a"V"3
t2 V3a'-4b' • o e 2 """'
2 •
EJEMPLO e. Demostrenws que e~ ·volumen de un s6lidó ·ohte11ido
duraµ'te la rotación· de un segmento circular cetra.do eón lá cuerda;
cuya longitud es 'i gual a a, en torno al diámeiro de este círculo-'paralelo a la cuerda,. .no depende del radio del cir.A
culo.
SOLlJCION. Limitémonos a la representación o,,_..-~~·
no del sólido do revolución, sinO' sólo de la
figura que se obtiene en la sectión del sólido
con un semiplano, cuyo límite es el eje de
rotación.
Sea el segmento CmD del círculo (1) la
representación de dicha sección, mientras
que el diámetro AB del circulo, la representación del eje de rotación (fig. 153). Toda la
representación ·es una figura plana, por lo
que acerca de la plenitud de la representación y el cálculo de los parámetros aquí no e,
se puede ni hablar: la representación ha de 8
ser realizada con una precisión hasta la seFlg. 163
mejanza.
Así, pues, sea la cuerda CD igual a a y paralela nl eje de rotación
AB. Demostremos que V, que es el volumen del sólido de revolución
del segmento CmD· en torno al eje AB, no depende de OA. Para la
demostración construyamos: CC1 ..L AB y DD1 ..L AB. Entonces,
V = V1 - V~ - 2V 3 , donde V1 es el volumen de la esfora, cuyo
radio es igual a OA; V 2 , el volumen del cilindro, cuya baso es la
circunferencia de radio DD 1 y cuya altura es igual a C1D1 ;
V 3 , el volumen del segmento esférico, cuya base es la circunferencia
de radio DD1 y la altura es igual a OA - OD1 •
Para abreviar, hagamos OA=R. Entonces, V1 = 34 nR3, V2 =
=stDD~C,D 1 =n( 11 ~
2
) a,
V3 =stAD~(OA--}AD 1 )
=n(R-;) (R- ~ (R-;))=n(R-;) (~ R+:).
2
2
4
De este modo, V= 3 nns-
4nR'a-nas
4
-
Capitulo Ir. Est.ertometrta
186
X (
2R +
; ).
Después de las correspondien~s simplificaciones,
nas
obtenemos: V= -
6
-
na3
Como la expresión - 6 - no contiene R , hen1os demostrado lo
que era necesario.
EJEMPLO 1. Una pirámide triangular se corta con un plano en
dos poliedros. Hallemos la razón entre los volúmenes de estoii polis
edros si sabemos que el plano secante divide las aristas que convergen en un vértice de la pirámide en las razones 1 : 2,
e 1 : 2, 2 : 1, contando desde el mencionado
vértice.
A
SOLUCION. Sea el cuadrilátero SABC con
sus diagonales la representación de la pirámide dada (fíg. 154), el APQR, Ja represen8
tación de la sección dada. Esta es comFig. 154
pleta y para construirla no se ha consumido ni un parámetro.
Supongamos que V es el volumen de la pirámide SABC y V 11 el
de la pirámide PAQR, con la parti.cularidad de que AQ : QB = 1 : 2,
AR : RC = 1 : 2 y AP : PS = 2 : 1. Hallemos la razón buscada
V-Vi
--v.-.
Tomemos· al azar en el plano ABC el punto O y, uniéndolo con
el vértice S,' vamos a considerar que SO es la representación de la
nltura do la pirámide SABC (con ellos se consumen dos parámetros).
Construyamo's AO y PM 11 SO. Entonces ol punto M yacerá en la
recta AO. Para que la forma de los cálculos sea más sencilla, hagamos
AB = a, AC = b, AS =e, SO = H, LBAC =ex.
Entonces, AQ
a, AR= b, AP = ~ e y de la semejanza
T
=;
2
de los trián~los APM y ASO, tendremos: PM=3H. Ahora,
~alcu·lemo!? V y V 1 • ·
- f
.
1
Tepeillos: ·V=~ S 6 .i.sc.H = 6 abH s~n ex~
V1 =
Entone.es, ·V = V1 .-
·! Stf. ~ 9.n ;
-
H = ; 1 abH sen cz.
2
; 2 abH sen ex. Así, pues,
1
V - Vi
y;-;=
25
162 abHsena;
1
ar
abll sen
Ct
25
=T·
§ U. Voldmcncs
187
· PROBLEM~S ('ARA EL TRABA10 IN,DiVIDUA~
800. La base ile un p!lraTelepfl?edo 'Qs .el rombo ABCD,. <;UY.O 'lado es i'g ual"
a ·a,.:J!.l áügulo agudo cs.::igu.o.l· a •60º... Hallen el volum,en del j>{lf1',lelepip1¡do .si la
a,riS.t'\l l'ii.teral es igj¡al a á, L~JAB.
L A·1AD
45°.. · ·
=
=
'801. •eada una do lai¡ arista~ dl'.l· un paraleleplpedo es lguál a o. Ci:nd_n -uno
\le. los tres ángulos pfanós 'eil uno .de l'os v'érUccs del' paraJeleplpedo es igual a
2Cl •. Hallo'h el' volumen· de· la figura .
· ·
. . ,
.· · so~.' Las ari.sto.S de ·U!l. 'paralele~ípedo, iguales ·a a y b, son perp~ndiculares
.entre si,. j¡ri ~anto que.•la arista' igual a· e formá· con cadB' una: de. las el.os ,Primeras áristas. el ángulo <:l. Hallen el volumen. del paral elepíp'edo.
,
803. El área· de .-u na de las caras "laterales de un prisma triangular ·e~ igue,l
a m•: .Hallen el vo!unien del ·prisma si la •dista'ncia de.Ja. i!.~ista opu.esta al plano
.
do la 'mehciohada cer'a.:es jgual a 211.
80.'.i, La bese de \Iº prism.a triañgular es un triángúlo i~pscelés ' en 1ll rn1e
los )ados iguales son de u.na lQf:!gitü d ·ii y, enke·aí; formán .un ángµlo a, ~ diagonal de la cara·opu,esta a dich.o áilgillo' f,orma c;on la ,otra cara1:lateral un ángulo
Igual. 11, g>. He,11.en ·el v.olumen dol' prisma.
·
,
805: Un pfi!ima cuaBrírnil'\Jle,r 'regular, en c'l!YIÍ base ollado es .Igual a a,
cst~ tnfncndo il.o mllnora qiie cadn una do dos aristás lnteralcs 11dyacentcs es
igual n b ·y cada una de ·¡ns ·otras dos aristas es igu:rl a c. Hllllcn ol volumen de
este prism·a trunco.do.
806, Un plano ~ant,e pasa ·por· el fado 'd e la base de un ·prisma tr\angular
regular bajo ol i\tigulo a lil 'plQ.no '.d e l;i base. y corta del prism'a una pirámide,
cuyo volum.~n es igual e, ·v. Hallen el lí~oa de)a sección.
807. En una pirámide cuadrangular .regular un pláno que pasa por el la'do
de la base y la linea media de ·l a cara lateral opuesta for.ma •COn el plano de la
base un ángulo do 60°. Hallen ·et volumen de lll pirámide si: el l'ado de Ja base
es igual a a.
.
808. La base de una pirámldo es un tr\á.n·gulo ~eles con lados latera los
iguales a a y con ángulo a en el vértice. Los ceras laterales do la pir~mide forman con el plano .de la base ángulos iguales a 45°. Hallen el volumen de In pirámide.
Bó!l. En una '¡>irámide triangular todas las aristas later;i.les y dos áris\as
de base son iguales a a. El ángulo entre los lados iguales de lll base es ígual a
2a. Hallen el volumen de l;i. pirámide.
Sto. El área ·de la seccion diagonal de una pirámide cuadrangular regular
es Igual .a· S. La arista lateral forma con el plano de la bese el ángulo a. Hallen
el volumen de 'la piramide.
Sl:I. La altura de»una pirámide trla'n gulor regular es igual a !f. El ángulo
diedro cnire las ·c aras laterales·es igual a <p. Hallen el volumen de la pirámid.e .
812. La arista lateral de una pirámide cuadrangular regular es igual a b.
El ángulo "di~ro entre dos caros 'laterales adyacentes es igual a <p. Hal)en el volumen de la pirámide.
813. Hallen el área de Ja superficie de une pirároide triangular regular,
cuy<> <Jolurocn es igual a V, el ángul o entre la cara lateral y el plano de la base
es igual a a.
81~: La base de una pirámide es un triángulo recUlngulo , cuya hipotenusa
es igi1al a e y el ángulo agudo es igual a a . Cada arista lateral ~e la pirámide está
inclinada hacia el plano do In base bajo un ángulo ll· Hallen el volumen de ln
pirámide.
.
8 15. La perpendicular bajada desde el centroide de lo base de una pirámide trlanipililr regular a su arista lateral es igual a L. Hallen el volumen de la
pirámide s1 el ángu lo diedro entro In cara latera l y el plano de la base de la. fi'·
gura es igual a (l.
_
8 16. Ln perpendicular bajada del ccn~roide de la base do uno pirámide-triangular regular a la arista lateral os igual a l. Hallen el volumen de la pirámide
si el ángulo entre Ja ari11ta lateral y el plano de la lmse es ígua 1 a ~.
Capftulo lI. Eittreomtlrla
f88
817. En una pirámide triangular, en la que cada una de las arist.as laterales es igual a b, uno de los ángulos planos en el vértice es recto, mientras que
cada uno de los otros dos es igual ') 60°. Hallen el volumen de la pirámide.
818. En una plrb1ido triangular el área de dos caras, perpondlculares
entre sí, son iguales a P y Q, en t11nto que la longitud de la común entre ellas,
igual a b. Halle!l el volumen de la piramide.
819. La altura de una pirámide, cuya base es 11n cuadrado, se encuentra
tuera de la figura y es igual a H . Dos caras laterales opuestas de la pirámide
son triángulos is6sceles que eon el plano de la baso forman los ángulos et y p.
Hallen el volumen de la pirámide.
820. Un pl~no secante traiado por el lado A C de la bru;e de la pirámide
tllangular regulo.r SABC es perpendicular a la arista SB, corta la pirámide
DABC, cuyo volumen es 1.,5 vecos menor qu e el de Ja pirámide SABC. Hallen
el área lateral do la pirámide SABC ai AC ~ a.
821. La base de la pirámide cuadrangular SABCD es el poralelogramo
ABCD. Las caras laterales SAB y SBC son perpendiculares al plano de Ja base.
Por los puntos medios de las aristas AD y CD se trua un plano secante paralelo a la arista SB. Hnllen la raión entro los volúmenes de los sólidos obteuidos.
822. Ln baso de una pirámide os un rectángulo. Dos do los corns laterales
son porpendicul~res ni plano do In bu~o y !ns otros dos forman con él los tlngolos
et y ~. Hallen el i\rea de la boso do la pil·ám.ide si su volumen en igua l a V.
823 •. La baso do una pirámide os un trarecio en el qua los lados lotarales
y Ja base menor es igual a a, en tanto que e ángu lo entre el.lado ll\tora l y la
bese es igual a et. Cada una de las aristas laterales está Inclinada hacia el plano
do le base baio un ángulo igual a ~- Hallen el volumen da la pirámide.
824. La base de una pirámide es un trapecio isósceles, cuyo ángulo agudo
es igual a a y su área igual a S . Cada unn de las caras laterales de la pirámide
forma con el plano de Ja baso un ángulo ~- Hallen el volumen de la pirámide.
825. El triángulo ABC, on el que AC = b1 AB = e y LBAC - et, gira
alrededor di' un oJo que pasa por el vértice .il fuoro del triánirolo y forma iguales
vQlumQn dcJ' sólia·o ~e revolµclón obángulos con los lados AC y· A.B. Hallen
tenido.
826. Un trnpecio isósceles, el\ el que el ángulo agudo es igual a 45° y los
lados laterales son iguales o la base menor, gira en torno al Indo lateral Hallen
el volumen del e611do de revolución obtenido si el lado lateral del trapecio es
iguaJ a b.
827. Un triángulo recUln¡¡ulo gira en torno a un eio paralelo ti la hipote·
nu.sa que pasa por el vórtice del ángulo recio. Hallen ef volumen del sólldo..de
revolución obteni do ai sabemos quo el ó.rea del triángulo es igual o S, ~n tan.t o
que la ~rpcndiculnr bajada del \'érUce del ángulo recto a '!.!ol tíipotenusa cs. is¡ual
..
.
. ,
a la m1t11d do uno qe l os catetos.
828. Hallen la razón éntre los volúmenes do los aólidos obtenidos durante.
la rotación de un triángulo alrededor de la· base y do uná recta paralela a ella
y que pasa.por el vérti.ce del trl,n¡¡ulo.
~
,
·
8~9. Demuestren qüe los volumenes~ de. los ~!idos óbtenidos óurante la
rotación de 11n paralelogramo en torno .ÍI su~ Indos adyacentes iion inversamente
propo!C:ionales 11 ellos.
.
,
·
830. Un tr.lángulo en el quo Ja. ra,ióri¡ ei:iLr.e su.o ladós .e s Igual á a : b : e
primero gira al rededor de 11no de los lados, a continuación en torno a ot ro y,
por fin. alrededor del tercero. Hallen la ratón entre los' volúmenes de los sólidos de revolución obfenidos.
831. Durante la rotación de un triángulo rectángulo alrededor de Jos catetos y en torno a la hipotenusa ge forman sólidos de nvolución, cuyos volúme·
el
nes soq iguales o V1 ,
1.
+y¡·1
Y.
y V 3 , rospeclivamonte. Demuestren que
1
V?;=
v:1 +
§ 14. Volt!11Uno3
.189
832. Los lados de la base de ·una plr'ámldo o;:11>a:drañgular regular truncada
son igu_ales a a · y. b (a·> : b) . .El ;ángul 0 .formado. por el plano de, la, cara -lateral
con el plano de la base el!Jguak o· ª .: .H~llen el volumen ~e· la piramide:
833. En .dos recta:s .cruz,!l!!!I~ se l;ian to!llado segmentos, cuyas. lo.ngiludes
.son: lguales a a y b. pemi¡estten' Cí.\!O ol volumen del ·¡>aralelepipedo, cüyas:ªrista8· son Jos niencionados segmentos; no · d~pend~ .de la. dispó~ición de .los seg.
'
mentos en füchits·:rectas. ·
~34. El radio de la.·base de \!.tHono es Jgual .a R. Dos geileratrices.perpendiculares entre st div~den -el á·r'e;i de Ja l!UJiorfic.ie lateral d~l eolio en .la razón.J : 2,
Hallen el Yolum·en del .cono.
835. En uri círculo, tuyo radio ·es ig1,1al a 8, se ha cortado. un sector con
ángulo central a. Este se ba enrollado en 'forma do un emb.u do cónfoo. Hallen
su volumen.
.
836. La base de una ..p¡r&mide es -un triáµg11lo Tegulár, c11ro lado es .Igual
a a. La perpendicular bajada del punto medio·de la arista latera ínenor·al plano
de la cara opue.sta es igual a a . La' base de la altura, que se encuentra fuera
4
de la pirámldo, es equidistante a dos v~rtices del triángulo, os decir, de In base,
mientras quo !lel tercero se cncilontra a una distancia dos veces menor que do los
dos primeros. Halhm el volumen de ln pírámide.
837. En un cilindro se traui un plano paralelo a su eje, 11 una distancia a
de éste, que ~n la circunferencia de la base corta un ore() a. El área ele la sr.c·
ción es iguol a S. H11llon el volumen del ~ilindro.
838. El área .de .Ja sección, axial de un sector esférico es tres veces menor
que la del círculo mayor de un11 esfera. Hallen la razón entro el volumen del
mencionado sector y el de Ja esfera.
839. En ·una pirámide cuadrangular regular el área do la sección paraleln
a la base es tres veces menor que la de la base. Hallen en qué razón se divido el
volumen de 111 pirámide con dicha sccci6n.
840. Está dado el cubo ABCDA 1 8 1C1D 1 • El punto M es.o! centroido de
la cara A A ._B1 8, N , el punto medio do la arista CC 1, el punto K yace en la arista DC y DK = { DC. El plano trauido por los puntos M, N y K div!<lo el
cubo en dos poliedros. Hallen la razón entra sus volúmenes.
.
84f. Está dado el cubo A8CDA 1B 1C1Di:. Mes el punto modio do la arista
AA 1 , N, el punto medio de la arista A 1B1 • !!:1 plano traiado po_r los puntos M ,
N y C divide el cubo en do~ poliedros. Hallen la razón entre su¡¡ volumenes.
842. El- plano secante pasa por los¡un tos P, Q y R que yacen en la continuación.do lascorres_pondiente.s aristas B, AA 1 y AD del .;ubo )\ BCD.A 1B 1 C\D 1 ,
con 111 particularidad de que A P : B P = AQ : A 10
AR : DR
5 : 3. Hallen
la razón entre los volúmenes de los cuerpos obten idos durante, el; corte del cubo.
843. Está dado el prisma triangular ABCA,81C 1 • Halleh en qué rnz6n
divido el volumel). del prisma un plano sec~nte que corta las aristas A ,B,,
B,C, y BC por los puntos M. N y K. respccl1vamente, si D 1 M: A 18 1
f: 2,
8 1 N: 8 1C 1
2: 3, BK : CD= t : 3.
844. EsládadllelprismatTinngularrectoABCA 1B 1C1 enelqueAC: AA 1 =
= 3: 4. Hallen en qué razón divide el volumen do 1m prisma un plano trazad()
por el vértice A y que interseca las aristas laterales BB 1 y CC 1 por los puntos .M
y N, respectivamente, si BM
MB 1 y AN es la bisectriz dól ángulo C..4C 1 •
845. Un plano secante pasa por el punto M, que yace en b continuación
do la arista AB del prisma triangular rcgulnr ABCA 1B 1Ci. por el vértice 8 1
y el punto moaio de la arista A C. Hallen la raióo de los volúmenes de los cuer.
pos obtenidos si AM: BM = 2: L
846. Un plano secante pasa por los puntos M, N y P de las aris tas A 18 h
B 1CJ y B C. respectivamente, del prisma triangular ABCA 1B 1C 1 , con la particulari ad de que A 1 M = MB1o B 1 N: NC 1 = 2: t y BP: PC i= 1: 2. Hallen
en qué razón divide el plano secante el volumen del prisma.
=
=
=
=
=
190
Captlulo Ji. Estereometría
847. Un pl.:ino sec:inte pasa por los puntos K, L y M en bs nristas SA, SB
y se, respectiYamcnto, do una pirámide lriangular, con la particularidad de que
SK: KA = SJ; : LB= 2: l y la mediana SN de la cara SBC se divide por la
mitad con el in~icado plano. Hallen en qué ra1.ón divide el volumen de la pirámide el plano ,secante.
848. Un plano secante pasa por el vértice A de la base de la pirámide triangular SA Be, por D, que es el punto medio de In mediana S K de la cara SA 8
y el punto E de' la mediana S L de la cnra SAe que es tal que SE : E/, = i : 2.
Hallen en qué razón diYide el volumen de la pirámide el ph1110 secante.
849. Está dada la pirámi de cuadrangular regular SABCD. Hallen en qué
razón divido el; volumen de la pirámide un ·plano trazado por Jos puntos A y B
y por el punto' medio do Ja arist.a se.
850. Un plano que pasa por una de las aristas de un tetraedro regular
divide su volumen en la razón 3 : f>. Hallen las tangentes de los ángulos en los
que diclio planp diYide el ángulo 'diedro ·del· tetraedro.
851, Por cada una de las aristas del t.etracdro se ha trnzado un plano paralelo a la arista :Opuesta. Hallen la razón enire los vól\tmenes del paralelep!pedo
obtenid o y del 'tetraedro.
8.~2. Esta :dado lll cul>o AJJCDA,JJ,c,p,. E es cil punto medio <lo In aristn
DC. F. el puntó mcrlio de 111 nriRln IJ/J 1. ¿11ué pa1·te del v(l(umen del cubo ocupa
el volumen de .f a pirámide -AFED 1 ?
§ 15. COMBJNACIÓ.N DE POLIEDROS Y CUERPOS
R EDONDOS
Antes de pasar a estudiar los ejemplos, hemos de indicar que al
resolver los problemas en los q110 tratan combinaciones de figuras
tridimensionales, ·debido a la dificultad
s
de realización de los dibujos en ciertos
casos es ,preciso simplificarlos. En unos
casos es suficiente tener sólo la represontación de la sección· de las figuras que
participan en la combinación (tal es la
A
e mayoría de los problemas sobre la combinación de cuerpos redondos), en otros,
sólo la representación de una de las figuras de la combimición; c.n ocasiones e.s
B
preciso representar mw de :lás figuras
Eig. J55
pfonamente, on: ta1ito que :l'a otra, sólo
pa'fcialoicnte_. 'AJ te~o~ver alg unos pro...
blemo.s s~ considera convenient,!l ha~er. \ts.o dc1 diagrama de..lo.s proy.ecéi¡>nes canónicas de las figuras combtnadas.
'ErnMl?Lo 1. La esfera Q es tangente a la ·h ase de una pirámide
trif!.n gular regular· $ABC e.n el punto B y a su ·arista lateral SA.
Hallemos el radio de' la e.sfera si AB = a y SA = b.
5o~uc16N,. -Sea ·el cua-drilá~ero SABC y sus diagonales (fig. 155)
la repre5en.to.ción de la pitáiilide dada. Ella es completa y métrica·I'nerite ·definida .(cer.ciórense de esto por su cuenta). La construcción
de Ja representación de la esfera es dificultosa, ya que su radio es
desc 0 n·ocido (resulta una especie de «círculo vici'oso»: para construir
§ 15. Comblnact6n dt p'Olltdrw 11 cutrpos redondos
.19t
la representación .de fa osfera..hl\Y que· conocer su, radio, en:tanto que
para hallar su radio es.·d.alleable .te.ner su reP.resentación). Intenter,no~
reJ¡olver el problemJi con ayuda. de; .u na .representación en la- que ·el
centro de 1a .esfera e.s ie consttu'ido mientras que el!a misma, no.
Construyamos el centro de la esfera .. Ante to'do, .$efialenios .q ue si
tuviéra mos como repr~sentación del centro. qe la esfera el .pu.nto P,
la distan.c ia d:esde P l:iast.a· B, e.ñ. dQ.Me la esfera Q es ·tal)genie al
plano de la base, será lgµaJ al radio d'e dicha esfera·: Así, pues, como
vemos, rio es obligatorio ten·er la.. repI'jjsentación de la propia· esfera
para calcular su radio. La construcción del punt·o IJ será efüctundn
partiendo de los siguientes raz·onam!e~tos.
, Como la esfera Q e.s tangente.al plano ABG en el punto B, ·el Jiuilto
P yace en la perpendicular al plano ABC levantada en el punto B.
En el plano ya es~á trazada la representación de la altura SO de la
pirámide.: Por el punto B trazamos la recta m 11 SO. Entonces, como
SO es perpondiculm· al plano ABC, la rcctn m. será asimismo perpendicular al plano ABC. Así, pues, el punto P yace on la recta m y el
segmento PB es el radio de la esfera Q.
Si la esfera es· tangente a la arista SA en cierto punto D, está
claro que AB = AD (como la longitud de los segmentos tangentes a
la esfera trazados desde un mismo punto). Pero AB = a, por lo que
para la constTUccfón del punto D hay que tomar en el segmento S A,
a partir del punto A., tal segmento AD que AD = a. Haciendo uso
de que las longitudes de las aristas AB y SA están ·prefijadas en
forma general, tomemos al azar el punto D en la recta SA y consideremos que AD = AB. (SI estipulamos a a y b valores numéricos
concretos, el punto D no puede tomarse al azar. P. e]., si a ~ 15,
b = 20, 4.D:SA = 15:20 bien AD:SA - 3:4 de don do queda clara la
construcción del pun~o D.)
SeguidaCQente, c.o nstruyamos el segmento PD. Como la arista SA
es tangente a la esfera Q, PD .l. SA y PD es el radio de la esfera Q,
es decir, PD = PB. Córi. él fin de calcular la longitud 'del segmento
PD hetl)os de realizar las siguientes construcciones adicionales:
i) Unir los puntos S y P.
2) Trazar OB, la mediana del l:l.ABC.
3) Trazar PK UOB.
Hagamos PB = x y compongamos una ecuac1on.
Como SA = b y AD=a, SD=b-a. Del 6 SDf rectángulo:
tenemos: SP2 = (b-a}2 + x2 • Del 6 SBO rectángulo, tenemos,
BO-=
ª
rs
y SO=
l/
b2 -
ª;.
Como m 11 SO y PK 11 OB, OB=PK
Entonces, SK
=Y lJZ -
~·
- x.
=ara
y
PB=KO=x.
Capílulo II. Estereometría
192
Ahora, d_el 6 SPK rectángulo obtenemos: SP 2 =SK 2 +PK? o
bien
(b-a)Z+xz ~ (V bl- 0;
-
af
+ (a
r3 )2.
Después de resolver esta ecuación, hallamos: x
Vta (
2
2
b-a)
3b11 -a•
Como hemos señalado más arriba, AD = AB, es decir, AD =a.
Pero SA >AD. De modo que, según el sentido del problema,
b>a.
,
PB
vsa (2b -a)
d d b>
A s1,
pues,
= 2 VJb•-a'i , on e
a.
oDSERV ACION. Llamamos la atención del lector a que. la desigualdad b >a
fue obtenida a base do razonamientos que se desprenden del sentido del problema
y no de la fórmula .que obtuv.i mos para la magnitud buscada. De la fórmula
para P B hemos hallado q_uo a y b deben satisfae-0r Jns desigualdades del eistcmn
{
2b-ci >u
.
Sl>'-a' >o', (las Jcs1gualdadcs a> O y b > O so han om\LIJo
ya quo son
"fª.
evidentes). f>el sistema ofrecido ohtondrínmos quo b >
Sin embar¡¡o, resulta que este dependencia sólo es-suficiente para la existencia de la propia pirámide: en el triangulo rectángulo .SAO la hipqtenusa seró.
mayor que el cateto. Para Ja existencia da la. esfera tangente a SA, 111 dependen0
cia b > ~-3 es insuficiente. En efecto, sea b = _0,8a (en este caso b > ª~ª)
.
Como dijimos más arriba A·D = "· De este modo, AD > SA, o s ea, la esfera
será tangente no nl propio segmento S A, ·sino a su continuación .
EJEMJ'Ló 2. Está dado el c ubo ABCDA 1 B. 1C1 D 1 en el que la arista
es igual a a. Por los vértices A y (: y por los puntos medios de las
aristas B 1C1 y C1D 1 se traza la esfera Q.
F,
Hallemos su radio.
SOLUCION. Sea la figura ABCDA 1B 1C1D1
.A,
(fig. 156) la representación del cubo dado
y F 1 y E 1 , los· pu~tos m~dios de sµ.s aristas
B19 1 .y C1I),1; ,respec~iN~mente. L.a indicada
repf.e~en~adón. es completa y Jllétf~!)amente
dehnida'- (cetciórense de esto por su cuenta).
Com<? en el a.nterio:r· ejemplo-,. es· muy difiA
·D
cultosa la coristrucción de la representa.
cióµ de ·l a ·esfera" ·dada, ya que no conoce·Flg. 156
mos la longitud ·de su radio. No obstante,
la.representación de la esfera no es muy
necesaria: si en eila l!Stuviera ind.icado el centro de la 03fera y
alg\lno de sus puntos (a.qui se muest¡:an cuatro puntos), sería posible calcular tapibién la longitud del radio.
Ante todo, busquemos el punto M, centro de la esfera Q. Como
.los puntos. A y C pertenecen a la esfera O, el punto M pertenece al
§ 15. Combinacitin dt pol(tdros y t11erpof redopdos.
.193
lugar geomé~rico di! lq!¡ pu.!lt<>s é.quiclistantps de A. y C. Els fácil adivinar que·ese·lugar georriét.ríco de puntos· es el plano:'dingo 0 al BliyD¡D.
Por ariaJogía,, el- pupto .J.{ pér.timece. ar pl_aµo diagonai AA:i.CiC• Así,
pues,· M j!'E!riene¡;e a la ,r.ectn· 001,· po.i: 'Ja q.ue se inter'se.c¡ui l.os planos
ACC1 ·y .J;JDD,..
.
'
.
· Por .fin,. el ·punto·· M :per~!m~.c(l .~1 nlan9 a que cori~ el se~mento·
C.E1 en L que ·es el ptinto·medfo :a·e CE1 , con la part.ieuh1ridaq de que
a ...l. QE1 • Como ·1os: ·planos ét· y D.CC 1 t.ienei:i el punto .común l,
eHos se intersecan· por la recta /;:que pasa -.p or el punto L, pero. como
CE 1 ..l. a, CE1 .L l.
,
~
.
Construyamos E, q~e e~ el punto m!)dió· del segr'nejlto CD, y el
segm!mto EE 1.. Como fa repre~entación en· ta q.uii se' real.izan .las
construcciones adicionales· está métricamente definida,, no pódemos
trazar·a1 azió' por el punto L la rectal y decir que l ...1.. CE 1 • Además,
la recta l corta .el segmento EE 1 • Designemos con K su punto de
intersección. Entórices, el 6KLT!;1 será rectángulo y :semejante al
ACEE,. De la semejanza de estos triángulos se desprende que
KE1 : C~ 1 = LE1 : EE 1 y como EE 1 =a, CE =
CE 1 =
T•
=
a-V5
LE
-2- y
1
alis
= · - 4- ·
fa.
Asi, pues, hallamos que sil ..1. CE 1 , KE1 =
Empleando esta
igualdad construimos e1 punto K. Más adelante construimos OE
y por el punto k. trazamos la recta k 11 OE.
La recta k yace en el plano OEK y corta 00 1 • Designemos su punto de intersección con M. Entonces, ya que M K 11 OE y OE es perpendicular al plano DCC1 , MK es perpendicular al plano DCC1•
De modo que CE1 ..l. LK y CE¡ .l. M K. Entonces, CE1 es perpendicular al plano LKM. Con otras palabras, el plano LKM coincide con el plano a y, por lo tanto, el punto M yace en el plano ex
y, por ello; el punto Mes el centro de la esfera n.
Hallemos ME 1, es decir, el radio de la esfera Q. Del t;,MKE ,
rectángulo, obtenemos: ME,=
y M K 2 + KE: = ª 1~41
.
(Hubiera sido posible buscar MC, o sea, el radio do Ja esfera
r .)
del triángulo rectángulo MCO, dondo OM = : a, OC= ª
E.JEMPLO a. "Un ángu}Q triedr o está formado por los planos a, f1 y i'·
con la particulari<\ad de que ex ..l. v. fl ..l. '\' y Letfl = 2q>. La esfera
l: es tangente al plano'\' en el punto B, en tanto que lo~ planos et y fl
son interseeados por las circunferencias ro 1 y w2 , cu;y:o~ radios son
iguales a r. La distancia desde en punto O, que es ef centro de la
esfera, al plinto A, que es el vértice del ángulo triedró, es igual a l.
H allemo~ el radio de la esfera.
Q
13-0290
194
Capitulo JI . Estere11111etrla
so1,l1c10N. La representación tridimensional de la combinación
de !n esfera .I: y el ángulo triedro apy es muy complicada. Para constnur la re¡1rescntación apliquemos el método ele la proyección ortogonal s_obrc un p_ar de ~lanos de proyec,ción H y V, perpendiculares
entre s1, que scran elegidos partiendo de las siguientes consideraciones. Como la esfera .I: es tangente al plano y en el punto .B, OB ...L y.
Ya que cz. --1.. Y y ~ ...L y, la línea de intersección de los planos cz. v p
es también perpendicular al plano y. Sea que los planos cz. y ¡.i se
cortan por la recta AG. Entonces, AC --1.. y y, por lo tanto, OB 11 AC.
Con las rectas OB y A e Se define plonnmente cierto plano <r, con la particularidad
,..
de que como La 1·ccta OB yace en el plano
cr, cr ...L y. Tomemos y como el plano horizontal de proyecciones y, s iguiendo las anota.:.
ciones 11dopt11das en geometría descriptiva,
.....___..:;:,,..-+-.:::--).:..:.'•·
llnmúmoslo lllano /i. Tomamos o como l'I
u· rplano vertical de proyecciones y lo designnmos con V.
Señalemos, que como las circunCcrencias
w1 y <i>i son iguales, el punto O es equidistant.c a los planos cz. y P y, p04' consiguienl4.!, V es el plano bisectriz del ángulo diedro Ioqnado por los planos cz. y ~ y el plano
V int.ersecn la esfera L por la circunferen·
cia del circulo mayor. Los diagramas de
las proyecc'ii::mes ('.anónicas de la combinoFi¡:. 157
ción ele la esfera .I: y el ángulo triedro a.~y
se ofrecen en 1n fig. 157.
(Seguramente, el lector prestará atención al hecho de que en (?!
11lano vertica l de proyección no se muestran elipses que son In~
proyecciones de las circunferencias <i>1 y w 2 cu el plano V. La construcción de es11s elipses es una tarea no muy complicada, pero en adelanto
no sólo son iiínecesarins, sino que superfluomentc recargarían el
dilrnjo ..)
AsJ:, pues, ·en el diagrama vemos en el plano V la sección de la
esfera 1: co1,1 el plano CJ> que pa_s a por e l ccn.t-ro de la esfera, es decir, l'l
punto O .Y por el vértice del ángulo t riedro, por ello a' o' = l. En el
plano H ' vemos la representación del ángulo diedro a.~ a tamaño
natural, por lo que· Letu~tt=2q> y L<:t11<r 11 =<p, en ta nto. qu~ el segment9 de -es la repre·s e)\tación dll la proyección horizonLa.l de la circunferencia (l)t• es decir, de = 2r y, por !in, el radio b uscndo do Ja esfera
·¡: es igual al radio de -las circ\mfercncias ¡;y que son las proyeccione,s horizontal y vertical de la esfera .I:, respectivamente.
Bajemos desil"e el punto o en el pl'ano H In perpendicular of a a11
(el segmento o/ es la representación del segmento of ...L cz.). Entonces,
df = r. Pnj·a abreviar, hagamos el radio do la esfera i: igual a x.
s'
195
§ 15 . .Combinoci6n· ·de poliedros y cuerpos redondos
En tal .()aso·, los r¡¡.dios.de-'las c.i.rcu11ferel!cias ~y ~· .serán asimisqio.
iguales. a-x, o sea,, qd .:;= x.'j. .p'p' =:''x.
; De· .es.Le. m9do, ; d~t~ e:, o'a'll ree.t ángµJo, obtenemos: . a'o' =
=V· l~...:;,.xz. Pero, ·c.oiifo~ el!, ..fácil adver1.ir, ao=a'..b'. 'Asi.1 pues,
, tám.b iérl '·.ao= l?. - ·xa .y·, ... e.iit.o nces,. yá que L aof = <P, ¡lei. triií.ngufo
·rectá:iígu lc:i ·ªof, telitimos: o/;.,aosén<p=Vl 2 _::.i;•sen· q, y, por consigufente, del ·:t riángulo r~c~áng.u lo odf:
·
xª = ,2 +(l2 - x2) sen~ q>:
(1)
Y
0
Al resol ver esta ecuación con rel ación a ;r,z hallamQs:
_ ri+ ¡i sen' cp
-
x 2 ;;:=
t+sen•q¡
Rechazando de inmediato los valort!s negativos de x , que a
11
·
" c1cr
· t n son iixt r:rnos,
:ícnicp q> •
c1cnc1.,
o bt en cm os.· x = J/ / r•+
l +sen•
Aclaremos ahora, según el sentido del problema, a qué restricciones debe n satisfacer r, l y q> para que el valor hallado de x (nna
de las soluciones de. la ecuación cuadrática (1)) sea la longitud del
radío de la esfera .:i:.
P rimero, como 2cp es el ángulo entre los planos (a y ~). Oº <
< 2q¡ < 90º, o sea, Oº < cp < 45º.
Segundo,
{
of <
X<
ªº
x>df
. { V z2 -
o bien
xZ sen cp <
X
<V l 2 -
x2,
x>r.
Poniendo ·en este sistema de deslgualrltidlls el valor de x y resolviendo el sis~ema obtenido de desigualdades, hallamos que r<
<
vz
-2- l cos q>.
,
, /
As1, pues, OB= V
,
,~+t•scn•cp
t+ i
scm q¡
,
donde
{
r< l~Í lcos1p,
Oº < q¡ < 45º.
ODSERVACION. Podría parecer que en In expresión hallada pa;a el vaJor de"'·
los parámclros r, l y cp puoden to niar cualesquiera valores reales (el radicando
será positivo}, pero, como hn mostrado l a investigación, realiiada seg\m el
s~ntido del problema, ello cslá muy lejos de tu realidad.
EJEMPLO 4. El centro de una esfer a, i nscrita en una pirámide
cuaclrangn1ar regular, coincide con el centro de una esfera circunscrita a dicha pirám ide. H allemos el ángulo diedro en la arista de
l a base de la figura .
.
SOhUCI ON, Sea la figura SABCD la representación de µna pirámide
cuadrangular regular (fig. 158, a). Esta es completa· y para ella
p = 4. Const ruyamos SP, es decir , la altura de la cara' lateral SAB
13•
Capítulo 11. Estereometría
19fi
y OP, su proyección en el plano ABC. Vamos a considerar que PK
es la representación de la bisectriz del ángulo SPO, es decir, que el
punto K ·es la representación dol centro de la esfera inscrita Q. Do
este modo, para la representación se consume un parámetro más y
ella se convierte en métricamente definida.
Como ele acuerdo con el planteamien to el punto K también es
el centro de la esfera circunscrita í!, debemos considerar que AK
.Y SK son las representaciones de diversos segmentos en el original,
s
s
8
D
A
(b)
(a )
Fíg. 158
decir, abreviando, A K = SK. Como vemos, no es preciso mostrar
las representaciones do las esferas Q y Z.
Así, pues, OK es el radio de la esfera inscrita, AK, el radio de la
esfera circunscrita y se requiere hallar el ángulo diedro en la arista
AB de la pirá mide, o sea, LSABO.
Como SP ..L AIJ y OP es la proyección de SP en el plano A/:JC,
OP :..i_ Afl. Por lo tanto, LSPO es el ángulo lineal del diedro SABO
que buscamos. Para abreviar, hagamos LSPO = x y, para realizar
'los«:<i'lcul~;;; in troduzcaJnps .un paró.metro auxiliar haciendo A B = a.
·C0 nsiaeremos e~ triangulo rectáng.ulo S.OP (fig. 158, b) . .Bajemos
del -p¡rntd K !'a perpendicular KL a lª a·potema S P·. Es ovidente que
en el. oriki nal tendremos K 0 L 0 perpendicular al plan"o S 0 A 0 Bo. es
d·ccir., Kk es In representación del radio de la esfera inscrita en la
pirómide. Esto signHíca que KL ::::. .OK. Así, pues, en el triángulo
SKL S.K es el ,radio tle Ja esfera circunscrita a In pirámide, en tanto
qµ e· KL, el radio de l¡i. esfera inscri·ta en ella. Pero, en el t riángulo
rectángulo OAK (fig. 158. a) AK es el radio de la esfera circunscrita
y·OK, el de la inscrita. Entonces, en estos triángulos también SL =
2
= OA. Pero , es fácil calc ular que OA =
. De modo que SP =
l!S
ªr
~·
§ J5. Combinac16n dt pol(edros
·
SL
=:,·~!
a'Ví ·.+ a
+ L P = ----z:2
v
~uerpos
rtdondps
197
-:9', a cont.inuación, obtenemos: cos x =
'='f: (-ªr2·.+i) ·=-Y.i'-1,
de
dondo
x =
arccos'{Y 2 ~ 1).
·P.ROBLEMAS PARA EL TRABAJQ I NDIVlÜUAL
853. 'El Indo de un ·rombo ·es igual n a. Una c~íer.a· 110 racfio n es langento
a todos los lados del TOf!lbo. La -~islnncia d.c~d·e el centro de la esíeru -hasta d
,..plano ,del rombo eft. ij¡u11l a a. Hallen ·el !Írea del -rombo.
.
'.8_54··. En ln superficie de µna e_s{Qra, cuyo·radlo ·eS' /i. e~lár\ $.Indas dos éircunferenc;ias di.f orentes .e n.la.s qui! la cuerda com·ú n es a. Hallen l os rn<)ios cle ·dichas
circunferencias si SUS planos·soh ·perpendiculares enlre .SÍ.
855. Una esfera de radio R cst á inscrita en \1n cono, cuya ~nerHri>. se ve
desdo CI centro de la cisfera bajo el áng1ilo o.. Hollon el volumen del cono.
Ss6. EJ\. .unn semiesfera· de radio /l está inscrit o 1111 cono ~l'Uncndo de 1.HI
lorma que su ba54l .mayor-·coincide .c on In base dc 'la semies'fcra y la genr.ralri7.
está inclin!lda hacia. el plano de. la base Jiajo el. ángulo a. Hnllcn el lirea total
del co·no.
.
857. E11 ·un conp circular recio se dan .cliíren de l a base S 1 y el área lateral S •· Hallen el radio. de .lo esfera inscrit.i en. el cono.
·
8fi8. En un cono está ·inscrita una esfera de f!1rma que el radio du In r.ir·
cuníercncia rtc su tangencia con el cono es 'igunl a R'. Un;i recio que pnsa ¡l'or od
centro de Ja esfera y por un punto que yace en lo clrcunCerencia do lnngeiJclu,
forma con el plano do la base del cono el ángulo e¡.. Hallen él.volumen del cono.
859. Hallen el ángulo en el vértice en la sección axfal de un cono si sobemos
que en su superli<;ie.• se pueden trnar lfl'S gcnératrices perpcodiculhrcs en sí
a pares.
860. Dos conos iguales con ángulo a. en el vértice en la se<:ción axial. están
dispuestos de modo que el ··eje" de cada uno de e.llo~ es la generatriz del ol.ro.
Hallen el ángulo entre las dos generatrices por las que dichos conos se intcrsecan.
861. Un pl.an<• paralelo a la base de un cono y que pasn por el centro de 111
esfera inscrita en diého cono, div.ide a éste en <los partes, cuy<ls -volúmenes.son
iguales. Hallen el án!l'1lo ontrc la gcnaratriz ~el cono y el plano de su baso.
862. En un cono equilátero est.á in scrita una scmios!era 110 mono qui¡ su
clx'Culo mnyor se encue.ntrn en el plano de la base del r,ono. Hallen la ra zón en
la que la circunferencia de langcncin divido In superficie lateral de la semiesfera y la superficie lateral del cono.
863. En un cono est.á·n dispuestas dos esferas de modo (}t\e ellas son llrn·
genlcs entre si y a In superficie d el conn. Ln razón cnlrr. Jos radios clo dicha~
esferas es igual o m : n (m > 11). Hollcn el ángulo cu el vórtice do In sr.r.r..ii111
axial del cono.
·
864. Domucs~rcn que la razón entro el volumen dd cono y ~l volumen 11\•
la esfc:>ra inscrita en él es igual n In razón ont.r·<l el árcn l11lal del cono y el án•a
de la eslern .
865. En un cono truncado está inscrita una esfera, cuyo volumen c<mstiht)'•)
~ del volumen del cono. Hallen el ángulo entre 13 gcncrnlri.z del cono y el
plan.o de su base inCerlor.
866. En una semiesfera está inscrito \lo cono, cuyo vértice coi ncide con ~l
centro de la circunferencia que sirve de base a la semiesfera. ¡El plano de la
base del coño es paralelo al plano de la bnsc de la sem iesfera; la recta que une
l!l8
Capítulo Il. Estereom•lria
el centro de In base del cono con un ponLo, tomado nl azar, en la circunler('ncia
del círculo mayor de In s-0ruicsfcrn constituye con el plano de la base de l con ~I
ángulo a. Hn lfon Ja razón entre los volúmenes de la somicsíera y del cono.
867. En nn conn está inscrit.n una esfera. Su línea do tnngoncia divide el
área dc la csl('rn en la rnión "': 11 • .Hallen el Ángu lo entre la ¡:cncrntriz d <'I
cono y su cje.
868. En un cuho !\st.á inscrita una piriimicle. Uno de sus vól'Li ccs es el cen·
~roide de Ja cara del cuho, los otros cuatro son los vértices de lns e.aras opueslns
de l cubo. En 1¡1 pirámide está ínscritn una (lsfera. ¿k:n qué razón divide 1>1 volu1m11 dol cubol el plano pnralolo n la base do la pidmide, 1¡ue pasa por el centro
de la eslera?
86!>. Una r.sfora es tangtmto a In superficie lateral de 1111 cono por la circunferencia de la base. C.on ello. el área de la superficie de la oslern se divide en
partes do las cuales una de ellas es 11 veces mayor que la otra. Hallen el ángulo
entre la gencratri7. del cono y el p lano de su base.
·
870. Una pirámide triangular r.slá circunscrita a un cono. La supor(icio
late?ral 110! cono se divirlc por las líneas de tangoncln en partes, entre c11yas ám:is
nx islr. la r:12óti 5: ll: 7. i.r·:n qué 1·117.ón tli,·iit~n 11ichns lineas el área laternl rk
b p íril111i1l o?
1!71. l>n ~m conn csL•Í inscrilo 11n cílintlrn, c uyti lirl'll 1<11.11 1 es igu:1I al :ín•n
l11t<>ral 11<•1 cono. El ;\ng11lo ontrn !ns geucrntrícr.s del cono nn su sección ax i:tl
rs igual n !)0º. Domuestt·en q ue l:i tlislancia desde el vértice dol cono hasta la
bnse superior dd cilindro es igunl n Ja mitn<l de In gcnerntrit del cono.
872. Dns conos tienen bnsc común. En l a secci611 axial común lu ¡:cncrntriz
ele uno de ellos es pcrpendiculnr a la generatriz opuesta del otro. El volumen
do uno do los conos es dos v~c~s menor que el del otro. !fallen el ángulo entre la
generatriz cll' I cono mayor y el plann de !ns bnses de lo~ conos.
873. Dr un punto t.omndo en la superficie de un esfera se han trazado tres
cuerclns iguales . El ángulo enfrc cada pnr de ellns es igual a a. Hallen In longitwl
de las cuerdas ~¡ el radio do In cslcrá es igunl n fl.
874. La base de una pi rámide es un tri~ngul n ísóscolcs, en"' que cadn uno
de los Indos laterales es igual a a, en lnnto que el :íni:ulo cmtrc ellos l)S igual a 'Z.
Dos caras laLcrolos ~ .. n pcrpe11di<,11lnr1>s al pl:ino <lo l:i lmso. mientras quo la
l<'rccr.1 fomin con <') '" ('] 6ngulo ~· Hallen ol rndi<' ll<• In l•SÍ<'rn insci·l ta en l;i
pirámi<lc.
875. Por los puntos medi os el~ las nríslas de un cubo pasa una csforn lang1>nl<1 a unn de lns hnsl\s del c.ubo. ¿Qué pal'tc dol volumen del cubo so encuentra
en el int~rior de In csfora ?
8i6 . En un cubo con arisln tt está ín~críln un a csf1•ra de moclo que sn s11pcrfici l\ es tangente n todas las aríst..ns del cubo. Hallen d volumen de la parto d u
la esfera comprendida dentro do! cubo.
.
..
877. Ln. arista ne un cubo es 1gunl n a. Hallen el rnd10 de rlos csfcrns 1g11alus
que pµcden ser inlrl>Cl ucidns en el cubo do fo,rmn qu() cllns no' p1Jctl~n despltiiarse
en .e.l intcriQr del cubo cuando ~slc. se ponó .c11 movlmicnt.o.
87$. · En un ·cubo con :ir.i$tn a cs!.á inscri~n una es lera. A co¡itimrnción , en
uno dc. los .ángul'o s triedros en el vér~ice del cubo está insi;rita la segunda esfera
t.ang~nte.n Ja: .prinicrn. Hallen ol. r:idi11 de la sc~unda oslera .
.8.79. ·Un¡i ·eslcrn. pa~ por los yérliccs A,, B D uel cuho A8CDA 10 ,C,1D,
y por ol ,p unto me.dio de.fa arista A 1 R 1 • Hallen e rn{fio· de In eslcra si la arista
del cubo cs. igual a a.
880. Unn csforn; cs tnngentc n tres caras tic un cubo qucco11licn1m un vér tkl•
>' tres ori~tns tic d ích1> cubn. que contienen ol vértice opuesto. Hallen la arista
1lel: c11.bo si el .radio de In csfcrn es igua l o n.
88.l . U.na cslcr:i os tangente a trcscnrns do nn cubo que contienen un vértice
y .pasa por el vhticc del cubo opuest-0 al priin~ro. Hallen ol radío tle la es fera
~i la aristn dlll f·Ubo es igual n a.
.
882. Una esfera es ~a:ngento a tres nristns de un cubo que contienen un
f
§ 15, Comblnacl61t lk poliedros y cutrpo1 redondos
199
vértice y pasa por .el vérilco del c.ubo opuesto al primero. Hallen el radio de la
osfcra si lo nrlsta del cubo es igual a a.
883. Una eªfera pasa por. los. p11n ~o.s modl"os de lros aristas ~ e un C\lbo que
contienen un vértice y 'por ol vértico del' cubo opuesto al primero. .Hallen la arista de1 cubo si el radio do !ti esfera es lgua l a R.
884. Una esfera es tangente.a cuatro ar istas do un cubopertcneciontosa una
do sus caras y a la catll opuésta. Hallen la rat.ón entre el volumen de aquella
parte do la esfera, .s.ituada fuera ·del cúbo, y el volumen de Ja esfera.
885. Una esfera pasa por los. v,é.rtiees do Ja_ baso inferior de un cubo y es
tangente a las aristas de su bose siiperior. .l;{a llen la rni6n entro la atlsll\ del
cubo y el radio de la ~sfera.
886. Una esfern es tangente a todas las aristas do un prisma cu¡¡.drangul3r
regular y a su base. Hallen la raión entre el área do In supét'ficie do la esfera,
situada fuera del prisma, y el área total de éste.
887. Un11 esfera está inscrita en un prisi:na recto, cuya liase es un triángulo,
en el cual la perpendicular bojada del v6rllcc ·del án¡:,ulo recto 1\ la hipotenusa
es igual a h y forma con uno de Jos cntetos el ángulo CJ.. Hallen el volumen del
prisma.
888. En el prisma lrínn¡:u lar rcgnlnr ABC.A 1 B 1C 1 por el lntlo AB do h base
se traM un plano que asimismo p:isn por el vór tic<> C1 de otrn base. ~11 In pirámide CfA 88 1A 1 (C1 es el vórtice) osUí inscrita unn csfora. Hallen el án¡rolo
entre e plano ABC1 y el do la base del pris1nn.
889. /\ una esfera está circunscrito un parnlelcpipodo recto, cuyo volumen
es m. vocos mnyor que ol do la esfera. Hallen Jos ángulos en las bllScs del paralelepípedo.
890. La arista de uu tetraedro regular es igual a a. Hallen el radio de la
esfera tangente a las caras laterales dol tetraedro en los vórtices de .la base.
891. La aris\n de un tetraedro regular os igual a a. Hallen el radio do la
esfera tangente a las caras lateral es del tetraedro en los puntos que yacen en los
lados de lo base.
892. Domuestrim que si los vértices de la base inferior de un prisma trian·
guiar recto yacen en la superficie (lo una esfera, on tanto que los Indos de la
base sµperior son tangentes a dicha es!ern, el prisma es regular.
893. Una eslora es tongete u lo.dos ln s coros laternles de una pirámide trino·
guiar en los centros da lns circun re.r encias circunscritas a ellas. Coda ángulo
plano en el vértice de la pirámide e~ igunl a 2c:i. en tanto que 13 suma ele las
arist-t1s laterales es igual o So·. Hallen el radio de In esfera.
894. La altura de· 11110 pirám!ae trian11ulnr es igua l n 11 , la sumn de los
nuc''º ángulos planos en loe vértices de la baso es i(!ut1l a CJ.. Hallen ol radio de
lo estera que es tangente Q ~odas las corns laterales en los puntos de inlorsecci6n
de sus medianas.
895. El área lateral de una pirámide tringular es igunl a S, en tanto que
el lado de la bnse, a a. La esfern es tangente a tres nrlstas de la bnse en sus
punLos medios y corta las a ristas latcra!Qs por sus puntos ruerlios. !follen el radio
(le ln eslora.
896. Todos los ángulos planos en el vórtice S de unn pirám ide son recto!!.
Demuestren quo cJ vérllco S, el centro do la esfera circu nscrita a la pirámi<lo
y el p'1nto de intersección de las medianas de Ju baso ADC yacen on unn mismn recto.
897. Una ~sfora de radio R está inscrlt..' en una pirámide en la q11e el &nri;ulo
rle inGlinación do cada unn do las cnr(l8 os lgunl a a. Hallen el vol umon de In
pirámlclo si su baso es un rombo, cuyo ángulo agudo os igunl a ~898. En la pirámide regular SABCD, con el vórlico en S, el Indo de la
bnse es ígunl a a, la arista lateral. a b. La primera cs[~ro con centro en el punto
0 1 es lnn¡¡onte a los planos S.AD y SBC en los puntos A y 8, rcspectivnmentc,
m1en\rns quo la segunda esfera, con centro en el punto O~. ~s tangente :i los
200
Capitulo l 1 . est•rtomtrrla
planos SAB y SCD en los punl<ls B y C , correspondientemente. Hallen d
volumen de la pi rnmido A H0 1 0,.
899. Hallen el ta ello do la esfera inscri la en una p irámide cunclrangular
regular si el volumen de ésta es igual a V y el ángulo enLre dos de sus c11ras
opuestos es igual a ci.
900. La base de unn pirámide es un trifogu lo Isósceles en el que coda uno
do Jo5 ángulos igunlosesiguol n ci y el lado común ele cs\os ángulos, a a. Cnd a
u na de las caras.laLcrales Clo la pirámide estñ inclinada hacia el plano do In base
bajo el ángulo p. Hnll en el rodio de la esCcra inscrita en la pirámide.
901. En una esfera de radio R está inscrito unn pirámide cuya base es un
cuadrado. Una de los ui stas laterales es perpondicUlar a l plano de la base.
mientras que la mayor arista forma con ~I un án gulo a. Hallen el área lateral
do la pirámide.
902. En una pirámide cuad ra11gular regular el nngulo plano en el vértice
PS Igual a ci, la altura de In pirámide es i¡¡ual a JI y sirve de diámetro o una
esfera. Hallen la longitud de la línea de intersección de las supcríicies de In
pir~midc y la esfera.
!IOa. En una piró.mido cundrnn¡?ul11r regular so encuen t1·n11 tlos csforns,
longontes entre sí y 11 t.odns lu s cnras de In pirámldci. Ln oslcrn iníc'rior también
es tnngcntc a fo bnse clo la pirúmidc. La rnión entre lo!l radios dn las cslcrns
grnndo y pequefin es lguol 11 11. Hnllen los ángulos diedros ele In pirámide.
904. En una pinsmirlo trianglllnr rcg\1lnr el ñngulo plano en el vórtlc:c ~s
i¡¡ual a ci. En ella está inscrita una esrera. ¿En quó partes divide ol área rlo In
cslt>rn un plano trazado por los puntos de tangoncia de ésto con las coros laterales de la pirámide?
90:;. La arista lateral de unn pirámide cuadra ngular regular es iguaJ a b
y el ángulo formado por la arista lnteral y el plano do la bllsc, igu11l a a. En
l'Sta pirámide está inscrit<> un cilindro equilátero do forma que una de sus geul'rntrites ~stá situada en In diagonal de la bose de lo p irámide, en tanto que In
círcunforeneia de In baso es tangente a do~ de !ns cllras adyllcontes latera les ele
la pirámide. Hallen l'l rodio de .la i)ase de.I c ilindro.
906. En un cilind ro, cuya olturn es igual a]¡ está inscrita uno p irámide.
Dos caras de ésta son perpendiculares al pi,ano de su base, mientras que dos aristas latera les forman con el plano do la base ángulos. cada uno de los cuolcs es
igual a ci. El ángulo entro eatRs aristas es igllnl a p. H:11Jen el área lat.ernl de In
pirámide.
90i. La arista do un tetraedro rei.¡11lar os i11u11l a a. Unn superílcio c.ilinclrlca pa~ Rºr una do sus aristas y por todos sus ,·~rticcs. Hallen el radio do l:l
bose del ctlindro.
908. Lns bases do unG copa esférica y un c Ulndro coinciden. El volumen
del cuerpo comP.rendld<i entre sus superficies la terales es igual a 3Gn cm>. Hallen
lo alt·ura .d~I c1lindr9 igual a lo ~lt';Lta de la capo esfédc.a. • .
.
.
909. ·En .un cono con. un radio i¡¡u<1,l a R pn 11\ !¡ase esta rnscri to un prcsmo
triangular con aristas Iguales de for!11a que· su bnso Y.'1i:e·en el pli!nó'de la base
del cono. H¡¡llen el volumen d el prislllll si el liogulo ent re l..s generatrices elel
cono y el plano <le su base es l_gual a ·et.
910. En un cubo, cuya arista e·s igual a a, está inscrito un cono recto circular con ángulo entre las gencrntriccs en In sección axial igual a a. Hnll cn In
longitud de la gencrntrlz y el rndio de la bnsn d el cono si su alturn yaco en In
diagonal del c1,1bo.
911. El cubo ABCDA 1 8 1 C1 D, está inscrito en un cono en el qun ~l 1·adio <fo
la base e.s igual a R y su ni tura, n R 11.2. Ln hose A BCD dol cubo está situado
en la baso d'e l cono, en tanto que los puntos A 1 , B ,, C1 y D 1 • en su superficie
lnt~¡nl.' Hnllen e1 área del trl~n¡¡ulo·A 1 C 1 ·M, dond e M es·el punto de intersección
de In ~~eta JJD con In circun forencia' ~e la boso.
912. Dos conos tieMn bases concéntricas y altura común igual a !f. La
diforoncia entre los ángulos que forman las gcncratríte~ con el eje es Igual a !}
§
16. Valorr.s máximos
!/
mínimos
201
el ángulo entre la generatrí z del cono inl'e rior y el ,plano de la .hase es igual na.
Hallen el vólumen éle la parle del espacio comprendido ent.ro las superficies
de tos conos.
.
913. LOS· lados de .u n trap.ecio isósceles son . tnngcnt.cs n un cilindr11, cuyo '
ele.e.s· p.erpendiculer a lp.s fados Pil.rol elo~ ·d el lrapi:>clo. Ha.l lcn el ángJJ!O c.ntr\l
el plano· del- t rapecio· y ·e t eje del cilindro si las bnses d~I tra·pecio son. .ig¡1nlcs
a a y ' b (a > · b) y s11 altura, a h.
·
.
.
·9¡ 4, E 0 una hoja reclangúlar <!e papel, ('D el que u~1 lado es igual a a,
se han-construido dnco· circunf«lreilci.as, una de lns C'u alcs liene 1111 rnd\1) jgunl
a ~ y la otra, a
La disiancia entre los centros de las circunfcrenc;ias ~s igual,
T.
a 2; , en tanto que la linea
d~ los tenlros. cs. pnrnlela _a
ia· base, del redángu(o.
A lns circunferencias se h!l trazado· uná tangen.t e jnforíor com(m. '!hil en la
d isLo'ncia en tre .los. iiuntos do tangencia dcsp~1(\s de enrol_lar la bojo en.. forma
de una superficie cihndrica circular, cuyo eje es pcrpendic.1ilé1.r 'a In línea tic l<>s
ccntr<>s ele las circun(eroncins.
·
91á. Est án dad.os un c11bo y una pirámide ~unrlrnn1tulnr rcgulnr, cuyn
nristn ln tcrnl es igual a b.. Los vértices dc.uncl'< lo las ciu:as ifol r.ubo son los puntos medios tln lns arislns de 'Ja base de I n pirnmi1l.,, mienl.rns qúu cncln 1111n ch• bs
aristas de In cara opuesta del. cubp·corta ·.una de las aristas ln:lóral~s do lo pi rámide. Hallen el voluip'Cn de In porte cl el r,ubo situncfü [ucra ele la pirámide.
9'16. En un cúbo, cuya nrisln es·igual. o a, so ha Lrnz.ndo 11nil dingonnl. Los
aristas del cu bo q·uc convergen en ti11~ de los eilrcmos dii )n diagon al está n
d ivididas ror la mitnd. Lns puntos de división obtenidos y el o\ro extremo de
In diogona so han tomado como los vértices de una pirá ri1idc. Haltm el vol11 men
de est.a Cigura.
917. Las aristas dt' 11na-.Pirámide l,riang11lnr, que. snleñ dr,I VÓfticc A, son
perpcndicula1·cs a pares e igu áles" a, by c. Hall en el volumen del cubc> inserí lo
en 111 pirámide de forma que uno <le sus vértices coincide con el vértice A.
918. TJa arista later al de una pirámide triangular regular es igual a by forma con el p lano de la baso el ángulo a . En In pirámide está in.scrito 11n cilindro
oquilálero de forma que s11 base inferior yace en el plano ele la bnse de Jo pirámide. Hallen la altura de la p irámide.
919. En unn h oja de papel, que es ol cuadrado f'QM L, SD ha hcch(l un agujero en forma del triángltln <'quillttcro A BC de man era que :A B 1l I' /, y A 8 :
: /' {, = 1 : 2. A continuación, el cuadrndo se hn enrollado en 111111 supi:rflcíc
cilin'dricn circular, cuyo l\'j e es perpendiculnr ni segmento A B.. Mallen In razón
l\ntre el árfla del cuadrado y el área del triángulo ABC. cuyos vérLiC('S yncen en
In superficie cilindrica.
!i 16. VALORES MÁXIMOS Y MÍNIMOS
Al resolver problemas estereométricos pnrn b11sc1u los vnlores
máximos y mínimos vamos a observar el mismo plan que e mpleábamos al resolver problemas planimétricos dedicados n determinar los
valores máximos y mínimos {cap. 1, § 7).
EJEMl'.LO 1. Un culJo se corta con un plano que pasa por una de
sus diagonales. Demostremos q uc tiene la menor área ,aq\iclla sección
q ue forma con e l pl ano do la base el ángulo a rceas 1/ i'
--i·
Capítulo 11. Estertometrla
202
SOLUCJON. La magnitud que vamos a optimizar es S, os decir.
el área de la scccíón. Sea la arista del cubo igual a a (magnitud conocida) y B,KDL, cierta sección (fig. 159). Jntrodm:camos una variable
índependiento: C1K
x. Do acuordo con el sentido del problema,
os evidente qne O~ x ~ a son los límites reales de la variación
de X.
Expresemos el área S do la sección con :e y a. AJite todo, señalemos que en la sección hemos obienido un paralelogramo, ya que las
líneas de intersección do dos planos paralelos con tercer plano son
paralelas en~l'e si. El área del paralelogramo se puedo hallar con
la fórmula B 1 K· KD· san a, donde a= LB1 KD . Del triángulo
B 1 C1K hallamos: B 1 K = V a~
x 2 y del triángulo DKC, DK =
= a2 (a - x) 2 • Del triángulo
B 1 KD , según el teorema do los
cosenos, obtenemos: B1 D 2 ... 8 1 fCl
KD' - 2B 1 K · KD · cos a, es
decir,
(a V3>1 = (a: + x 2) + (a' + (a - x)') - 2 va:
X: X
X )1 a•
(a - x)~ cos et, do dondo después do una serio 110 transformaciones obtenemos:
=
+
V +
+
+
+
x• - a.z
COSct=
I
~/
.
l 11•+::'· y a•+(a-:r)•
Vr
Entonces, s en et= ~yr '1 -cosa
• =
1 - (a2+r')(o'+(a-z)')
<r'-=>
a ll 2a• + 2..,• - 2<ir
l/ o•+..,1 'Vo•+(a-%):
Como resultado, obtenemos: S=B 1 K·KDsona=V~X
xVa2+<a-x>2 ·
a 1tio2+2.z' -2a% =aV2a2+iz:-2ax.
Es
=a
ya:+x: Va.. +(<• -
2
:r)'
preciso hallar el valor mínimo de ll\ función S (x) =
v 2a2 + 2x2-2a:i;
,f
en ¡O; a]. S' =a·
V
2z-a
2a•
+ 2x' + 2a% .
En el CASO dado no hay puntos donde S' no exista, ya que el
denominador de la derivada no se reduce a cero en -ningún lugar (da
acuerdo con el planteamiento a;;;;;.: x , entonces a~ ;;;;. a:c y 2at
2;i:~ - 2a:c >O). S' =O i;i 2x - a= O, es decir, con x =
Con el (irt de hallnr el valor menor de la función sólo nos
queda éalcul11r los valores ele ln función •S (x) ·en los extremos del
segmento [O¡ a] y compararlos con el valor de la función en el
+
i.
+
punto x= ; .
=
a•
r.
Tenemos:
S (0)=a 2 V2, S (a) =a2 V2. S ( ~)
El valor mínimo de la función
Este so alca.nzn con x = ~ .
s (x) es igual
A
a•
=
rr. .
§ 16. ValoreJ md;r.imo~ ¡¡ mf11lm0$
203
En el problema l.1ay que .demostrar que cf áng,u lo min1mo que
por el área de la. ~ección ,forma con, el plano (le la base, es igual
n arccos
·~f,
. Para
de~ostrnr _ei;te
hecho Jrng¡unos uso ele
la
fór-
mula S¡,"-••=Sscccoscp. donde <pes el 6.ngul9 ·entre IÓs planos de la
-a 2
•
1/ o
sección y rle Ja base. TMemo~: a2.= - -- 2
mos: cos <p =
2
Jl6
=
Vñ
-3- • es
<l
.
·
·
(;os q¡, de don d.e )lal)a-
•
CCIC ' (j) = H!'CCOS
-Vil .
-r
..
Un tronco de 20 dm d.e longitud tiene ·Ja forma d!i ·un
<:ono truncado con diámetros de l as bases 2 .y 1 dm. Es necesario
cortar del tronco una vi.gueta con se~ión t¡:ansvei;saf cuadrail.a, .cuyo
eje coincida con el del tronco y cuyo volumen se·a el má:dmci:.
so1,unON. La magnitud que vamos a optimizar V es el volumen de
In viguctn, o sen, el v()lnrnen do un pnralclcpípedo rcctangnlnr con
b11sc cuadnit.ln.
IO:n l a sección axial del cono, que al mismo tiempo es la sección
diagonal de un paralelepípodo rectangnlnr, obt-0nemos (fig. HiO)
EJEMJ?LO 2 .
\\n trapecio isósceles (sección axial del cono truncado) , en el que
está inscrito un rectángulo (la sección diagonal del paralelepípedo
rectangular). Designemos c on x la altura del paralelepfpedo, es decir,
In altura del rectángu lo en la sección axial: KM= x. Los límites
reales de variación de x : O < x ~ 20.
Hallemos el volumen V del paralolepípado rectangular. El segmento FK es la diagonal de la base del paralelepípedo. Hallemos
FK. Tenemos: FK = EM =AD - 2MD = 2 - 2MD. Tracemos
CL _¡_AD. Entonces, LD =AD - AL= 1 - 0.5 = 0,5 dm. Como
los triángulos KMD y CLD son semejantes, ~~ = ~Z
o sen,
,
"' = MD
~, d e don d e J11111}lmos:
20
FK
=
2 - 2MD
=2-
...::_
2D .
MD =
"' y, por ·cons1gu1entc,
. ,
40
C apftulo II. Estereometría.
204
El área del cuadrado, que es la base del paralelepípedo rectan·
gular, puedo ser hallada con la fórmula S=f d2 , donde <f' es la
diagonal de Ja base, es decir, d= FK. Así, pues, Sbosc=· ~.' (.2; ) • Como la all.ura del paralelepípedo es igual a. ~_,. __para -el
0
Yolumen obtenemos el siguiente resulL1.<do: V=
2 - :0.)
-
2
i- (
=i- ( :o )
Para la función l'
2ximo en el intervalo ]O; 20].
Ten~mos V' = 2 {2 - ; 0 ) ( --
x(z-~~)·
2
X
hay que hallar el
2~ ) x + {2 -
;0
\t.
~a:lor
)2 = (2 -
má,
~) x
';º
V'= Ocon x = liO o bien con x =
El valor de x = 40 no pertenece al intervalo que consideramos. El valor máximo de la iunción es ig,1al a
Hallemos la interpretación del resultado de este problema. Con el Fin de
cortar del tronc-o la vigueta de mayor
vol·umen, hay que separar la parte más
delgada del tronco dºe forma que quede
3
' 1
un tronco de 13
V 1 O 1
1 10
3 dm de longitud y, a
continuación, del tronco obtenido col'tar
una vigueta con sección transversal cuadraua (ésto se determina con el cuadrado inscrito on la base menor del tronco de 1.3
dm de longitud).
EJEMP LO 3. A una esfera de radio r está circunscrita una pirámide
cuadrangula~ regula:i:. H_
a llen el valor mínimo de su área lateral.
sol:. 1.ic10N i L n .magnitu!l a o.ptimi~a~. es S, es ·decir, el área lateral.
Iñtroduzcamps. la :variable· indepenClJento. ~ecordemos q.u~ el éentro
O de la esfeta i.nscrita yace cn-.:.\a altu.ra <)~ la -P.irámid.e regular y,
prec(sainentc, en ·el puJ1to ·d;; ·intersección 4e la altur.a con la bisectriz
OK del ~ngu['.o entre la nltu1·a MÍ( do la: c·ar.~· lateral y la proyección
H K d.e dich~ ..11,)tura en el' ·P.lano, de ia base (Cig. 161)". OH = r es el
radio· de la eFfera: inscrita. 1-ll!garnos L0l(JJ.. = x. Los límites reales
de variacfón-!de. x: O < x < ~-
3;º
1~
f
,J!:xpreser4os S con r y .3<. Del triángulo Ol(H hallamos: H K =
=rctg x; del triángulo IiKD ha.liamos: KD =HK =rctgx; del
triá:nguló llÍH K hallamos:· M K = JI K2 = r Clg2:z: • Entonces, s =
·
cos :z:
cos r
§ 16. Valoru m4:etmos y mlnlm~s
s
• ctg• :e
r ctg :e
,_
2Ú5
4KD ·M K =~rc~gx cos2x = ·4 r· co~2.r ·
Hallemos el valor. mínJ-mo de Ja función $ = 4r2 .cl"g, "
= 4 c.
MCD.=
iniervaio
en el
COS 2X
Jo; :[.
.
1
2 ctg z (
Tenemos: S ' = 4r2
--.-~·- )
co's 2.r
sen "cos~ 2z
+ 2 S:Cn 2:e ctg• z
·
·
8r•
cos 2x
·-2- X
·' '
cos 2:r )
X ctg x ( ctg x sen 2x- sonilx .
S' no existe si cos2x=O o ·bien senx= .0 , Jo q.uo· no
en
~J intervalo
Jo; ~ [.
se ·cumple.
S'=O si ctgx=O, Jo que no se ctfmple en el intervalo
.
Jo; ~ [
o bien sr clg x sen 2x- ens;.- ::." O. Hcsol vamos estn cc11nció11 trigoscn .r.
nométrica.
Tonemos con~ccutiv11mentc: cos"' Sún 2"
cos 2-.
1
S<?n X
SCl\'Z X:
_o
-
sen x cos x sen 2x- cos 2-i; =O,
scni 2x- 2cos 2x = O,
1 - cosi 2x- 2 cos 2x =O,
cos2 2x+2 cos2x - 1 =0,
(cos2x) ,.2= - 1 ±
v2.
Sólo nos sirve el valor cos2x = YZ-1, de donde se desprende que
x = +arccos <V2-1).
Con el fin de cerciorarse de que con el valor hallado de x la función
S (x) adquiere el val or mínimo, calculemos los Hmltes unilaterales
de la íunción en los extremos del intervalo ] O;
i[ :
lím S (x) = 4r2 Jím ~tgt. x
"-o
x- o cos 2z
lím
x-f~·O
= + oo,
S(x) = ·'ir2 lím ctgt.;~
n
r.o~GJ.
x--¡-+O
=
+oo.
Esto significa, que en efecto en el pnnto obtenido la función S (x)
tiene el valor mínimo. Calculemos dicho valor.
Tenemos: S = 4r2 cos...,.
cig~.~ , con la parLicnlarirlad ele que cos 2x =
,¡--
=y
1
2-1. Como i+ctgix=--,=
sc11 :t
2
:!. ,
1 - co~ x
2
cl.g 2 x= 1 -cos 2z
206
Capítulo JI.
-1 =
Ester~ometr!a
2
,,.. 1' 2+t
( .) -1 =r 2-/-1. Así, pues, Smin = 4r- - - - - =
t -V2 - I
'1'2-1
= 4r2(Y2+1 )2.
Retornando al problema geométrico inicial, llegamos a la couclusión de que el 'olalor mínimo del área lateral de la pirámide cuadrangular ri;>gular, circunscrita a la esfera de radio r, es igual a
4r2 (V' 2 + 1)2.
EJEMPLO 4. En una esfera está inscrita una pirámide n-gular.
¿Con qué ángulo diedro entre Ja cara lateral y el plano de la base de
la pirámide el volumen de ésta será el máximo?
s0Luc10N. La magnitud que vamos a opiimizar es V, o sea, el volumen de la pirámide. Introduzcamos una variable independiente.
M
.4
Fig. 161
Fig. 162
Representemos en la fig. 162 la n-ésima parto ele Ja pirámide: MAB
es la cara lateral, MO, la altura. Designcm·os el radio de la esfera
con la letra H. (parámetro auxiliar). Hagamos MB = x. Los límites
reales de variación de x: O < x < 2R.
Expresemos V con x y R. H agamos uso de la conocida fórmula
para calcular el radfo de Ja esfera inscrita en el caso de una pirámide regul11r.: R = ~;~ . ·<10.nd.~ b es. la arista lnteral, H, la nlturn
.
;t
w.
·
de l a pirá:míde. Enionces, R = 2il, de donde JI=
r
ángulo MEO h.nllamos OB
MB2:_M02= V x 2-
=V
2
AB es el lado de un n·gono r.egular, LAOB= n
n
·
i. .
. 2n
1 ( 2
;z;•
)
zón, S¡:; ADO =2AO.OBsen7=y x
1
1
1 (
De modo que V= 3 .S:bns.R= 3 n 2
2n
nsent2R "
-
x"R,
2
.
Del tri-
•
Como
y, por esa ra-
2n
sen-n-.
4R,
x2 -
xt
;z;•
.\R•
·)
2n
x2
sen-;;-· ZR =
207
§ 16. Valor11 nulz;/mo1 IJ ml11imo1
Hallemos el valor máximo de ll\ función V= k ( x\- ; 1:,
el intervalo )Q¡.. 2R( (para abrevi'ar hagamos k =
nemos V' =k ( 4z3-
t;, ).
n
~;!
)
).
en
Te-
!,
x3 =
Tenemos: lím V (x) = lím V (x) =O. Es decir, con x = R
V~
De la ecuación V'= O hallamos: x 1 =O, x 2 = R
- R
J/
V ~ . De eslos tres valores al .intorvnlo ](); 2R( sólo per-
tenece :r=R
V:.
,._O
.Y-2R
'16R
3 11
sen~
la función V (x) a lean"µ\ el mn yor vnlor. El es igual a
"
27
En el p roblemn es preciso h nllar el ángulo MDO pnra In pirámide de volumen máximo, donde M D es In altura de In cara lateral. Hagnmos L M DO= q>. Tenemos: MO = ;~
(poniendo en
= ':·
lugar de x el valor hallnclo más nrriba R
n
BDO ha llamos: OD=OBcos-u= v
/
x2 -
Vf) . Del
triángulo
r•
n
2R 112
-x
411 , cos-;;- = - 3
x cos".
1l
MO
V2
En t.'11 c11.c;o, tg '.f/=150 =--n-.
cos-¡¡-
"'" "rclg
De
m;tnern
que
1p=
(4).
cos-;;-
PROBLEMAS PARA E l , TR ABAJO
J NOlVlOU ~L
!120. De muoslren que ele todns )M pirám idcR cun<lrnngularcs rogul nr~s. en
los que In snmn rto In alturn y l os Indos de fo bnsr t•s conMnnt n, el "olunw n
múximo l o tirnc l u pi1·:ímido en Ja quo la cnra li1tcr11I rormn con 1•1 plnno 1lc la
linsc un ángulo do 45°.
921. El lÍrt'a de 111 bnse dr, un parnlclcpipedo nu:tangula r es iguol a t cm•
y la JongiLud de su diagonal, a 2 cm. Hallen: a) el volumen móximo: h) el :irca
lolcral máxima.
922. La suma de los cuadrndns <lo las longit udes do todas las nrlstns de una
pirámide triangu.lnr. regula r es ig11nl n I'. Hall en el "nlor máximo clrl árca lutcrnl do la pirámi1lc.
Capítulo II. Estereometrfo
208
923. La base de la pirámide M ABCD es un cuadrado. M 8, la altura de
la pirámide. Hallen el valor mínimo de la longitud de la arista MD si l?l volu·
meo de la pir~mide os igual a !.l cm3 •
924. Entre las pirámides n-gulnres regulares con longitud constante
de la arista lateral, hallen .la pirámide con el volumen máximo (calculen el
ángulo entre las arist.D.s laterales y el plano de la
base de la pirámide).
925. Entre las pirámides n-gulares regulares
con área constante ae la cara lntnal, hnllon la
pirámido con el volumen máxima (calculen el ángulo
entre la cara lateral y el plano de la base).
926. «La casita. está constituida por dos prismas
rectos con bases cuadradas y trlangula:res, con la
particularidad de que la base trianguíar es un triángulo rectángulo isósceles (flg. 163). ¿Cuál debe ser
el lado dol cuadrado para que el \•olumen de da casita• sea el máximo si sabemos que el perímetro de su
Flg , 169
baso es igual a 24 m?
927. En una pirámide cuadrangul11t regular se
trazan secciones pnrnlolns a dos nrisLns no intcrse·
cantes. Oom.ucstren quo b sección que pnsa por la linea media de la base do
a pirámide es la que ll ene ln máxima árell.
928. La base de la pirámído .MA BC es el triángulo rectángulo is6scolesADC
(AB
BC). Las caras MBC y MAB son perpendiculares al plano do la base,
J\fC = 2V2cm. ¿Con qué altura de la pinimide el área de la sección que pasa
por los puntos B y M y que divide por la mitad A C, será la ·m.!xima?
929. Por la diagonal do la base de un prisma cuadrangular ·r egular se ha
trazado una sección que con la otra base tiene por, 1.o meno.s un punto común.
Hallen el área máxima y mtnlma de semejante sección si la s o.ristas del prisma,
que salen de un mismo vértice, son iguales a 3
3
y 2 cm.
980. Demuestren que do to~os ·Jos· paralelepípedos rectangulares con base
cuadrada', inscritos' en· una semieslero, el volumen máximo lo tícno el cubo.
931. Demuestren que de tod11s las p'lrámides que tienen en su !laso un triángulo isósceles y quo están inscritas en un cono de volumen prefijado, el volumen
máximo lo tíene una pirámide .regular.
932. Hallen ·el área mfüdma de la sección de un cono con un plano quo pasa
por su vértice si el radio de la base del cono es igual a R y su altura, a H.
933. Un cilindro termina por arriba en una semiesfera. El volumen dol cuerpo es igual a V. ¿Con qué radio de la semiesfera el área total 1lel cuerpo será la
=
vz.· y2
mínima~
984. El perímetro do un trhingulo isóscclos e~ igual a 2p. ¿Cuáles deben ser
sus lados para ·que se.a el má¡d mo ol volumen dél' cuerpo obtenido debido a la
rotil.cióñ de dicho triángulo; a) alrededor de· la base; h) en tórno . al la altura
trazada a la base?
935. Del circulo dado s.e·cor.\.a un sector:¡, se ~nrolla 11n forma ,de un embudo
cónico, ¿Cuál debe.elegirse. el ·áng'ulo central del SJctor para que el volumen del
embudo sea el 'máxímo?
..
~36. ~n e,l cono d1!do insciiban el cilindro de ,volumen máximo.
937'. ·En una esfera de radio R está. inscrito uu cili'ndro. ¿A qu6 será igual la
altura.de .un cilindr.o gu<r-tonga: e,)cl "{olu'men máximo; b) el área later~l máx i ma?
938. ¿Con que altura del cono inscrito en In esfera ·dada" do radio R: a) el
voh,imcn •de!' cono S!lrá el máii;lmo; b) él ái:ca fateral del cono aorá la máxima?
989., ·un.cono .est~ círc;unsérito a una o~fera: do radio R. ¿A q!lé será. igual la
altura del cono que tiene: a), el volumen .m1nlmo; b) o! .área nún1ma lateral?
940. Hallen:'Ja altura..\lel cono de máximo volumen circunscrito a la semi~sfera de radio R.
·
9ftf. En la esfera dada est.á inscrito un cono de volumen máximo en el quo,
§ 16. Valor.r m6ztmo1 11 mínimo•
a su vez, e3lA Inscrita una esfera. H allen la razón entre los volúmenes de las
esreras.
942. En una esfera está inscrito un cono de volumen máximo. A au vez,
en el cono está Inscrito un cilindro de vol umen máximo. Hallen la razón entre
la altura del cilindro y el radio .do la esfera.
943. En un cono con generatriz constante está Inscrito un prism a hexagonal
regula r, cúyas aristas son igua.les. ¿C!>n qué valores del ángulo -entre J.a generatriz
del cono y ol plano de la base el área lateral del prii¡ma.será la rnaxlma?
9'4. En une pirám_ide cuaaranguJar regular está inscrito un cilindro de
forma que la circunferencia de su base superior os· ta11gento ii todas las caras
latera lea de la pirámidl!, mientr~s ,uo la base hiferior- yace en el pliino de la
base de la pirámide. .¿Qué parte du a ·altura. do.la pirámid,!l debe conetltulr la
alturn dol cilindro para que el volumen de ósto se·a ·el máximo?
'
945. En u na semiesfera do radio R ·está. lnscrlt~. un p rism.a triangula r regular
de rnodo que una de sus baee11 yace en el pl11no del círculo mayor d.e la semloS·
fe.ra, en tanto que el vértice de la otra baso pertenece a la superficie de la semiesfera. ¿Con quó a ltura del prisma la· suma de las longitudes·de todas sus aristas
será la máxima?
946. Hallen el volumen máximo de una pirámide hexagonal regular Inscrita en una esfera de radio R.
947. A una esrera está circunscrita una pirámide n-gular con el á rea lateral
mlnlma. Hallen el ángulo di! inclinación de su arista lateral al plano de la base.
948. En una esfera ·está inscrita una pirámi de cuadrangular regular, en tan·
to quo en ella está inscrito u n prisma cuadrangular regular de forma que una
de sus bases yace en el plano de la·hase do .la pirámide y los vértices de la ntra
base del prisma pertenecen a las.aristas laterales de la pirámide. E l lado de la
hase y la a lturo del prisma eon igua les a 2a y a, respecti'val1\onte. Hallen el
valor mlnimo del radio de la esfera. ¿Con quó altura de la pirámide se alcanza
dicho volor mlnlmo?
M9. La base de' la pirámide /lf ABC es el triángulo rectángulo ABC en el
que 4.C = 90°, LA = 60° y A C - 6 cm. La ariete SA es perpend icular al pin·
3 cm. En In pirámide SABC estA inscrita una pirámide
no de la base, SA
con vértice A, cuya base es la secci6n de 111 pirámide dada con un p lano l?arnlelo
a las nrlstas SA y BC. ¿A qué será Igual el volumen m&ximo de la pirámide
inscrita?
950. La altura de una pirámide cuad rangular regular es dos veces mayor
quo las diagonales de su base, el volumE;n de la pi rá.mide es igual a V. Se con·
sideran prismas cuadrangulares regulares Inscritos en la pi.rámldo de forma
que sua aristas lateralos son paralelas a las diagonales de la base de la pirámide.
una cara la teral pertenece a dicha base, los vértices de la cara opuesta yacen en
la superficie lateral de la pirámide. Hallen el máximo volumen del prisma.
951. El volumen de una pirámide cuadrangular rer'Jar es igual a V, el
ángulo entre la arista lateral y el plano de la hase, a 30 . En la pl.rámido está
inscrito un prisma triangulnr re~llar de forma que una de las aristas laterales
yace en le dlagunal de la base de la pirámide, una de las caras 'laterales es para·
lelo. o In hose de la pirámide, en tanto que los vértices de esa cera yacen on las
caras laterales de l a pirámide. Hallen el máximo volumen del prisma.
=
1 '-0200
SOLUCIONES E lNOI CAClONES
Capítulo I
ºV li .
2. m, m 113- y 2rn. 3. 56 y 42 em. 4. 9 ,vr 5 cm, 8 ,I'¡10
ew . S. ab
a+b
G.
mn(m+n)
1
m+~
7 . :10 cm.
m(m+11lv'2
]/m•+,.i
8. 15, 20 y 25 cm.
( 1
)
a
m(m - n)if2
v~+~
9.
:1+ 2 1/i
n
10. nrctg, T tga -T· 11. T±arccos
4
Designen LA ~ 2'i y AB =c. Expresen con e y ., los caletas del triángulo y,
a continu11ci6n, la bisectrii. 12. Designen LACK-LACB -a (CK es la altura, CM, la mediana, AC <BC), CK - h, LAC8=%. Exp~esen con h, a .
>:los segmentos AK, BK, MK y bagan· uso de que 2MK=BK -AK (véase el
15. 6 ero.
ejemplo 10 del § 3).
19.
21.
23 .
2scn(•w+ ~
)cos(45°-
v1+scos2 a
4 coaa
1
3 ;
17. 9
1 cm.
3
20 .
ª:) ·
22. arceas
2 ]/3 (p•+q•+ pq)
3
VW cm.
16.
]/3
arec os - 3 -
18. 7,2 cm.
2 sen (cr. -30")
cosa
\/3
arccos - 3 -
y
24. Por el punto E t racen lo. recta EP 11 AB (el punto
F ya.ce en AC) y bagan las necc39rias conclusiones del análisis del par o.lelo-
gr111110 DBEP. 25 . .; (y!'>+1).
linea ·· mcdia., 27.
1
(
acosa
26. t: 2. En el triángulo ADC tracen la
3a}
sen 45° + -
2
.
]121
28. arcsen - 7-
'Vil
; arc.seo-1-4 - .
11
72 .
. 1as perpendi •
29.• arctg· 13
. ••30. arcsen g¡.
31. SSº. DeJ pun to M ba¡en
cularcs MK, ; ML y MN a los lados AB, AC y BC, respectivamenle, designen
L.. BMC =>:,.IAM=k y expresen, p.ej., JI.IN como el clcmen~o de referencia
con· k y z ooo ayud~ de dos procedimientos. 37. 40°, 259. 88. 4, 6 y 8 cm
o bien 2'V~. 4-,Íii y 6 cm; obtáogulo. 39. 0,75. 40. Rectángulo.
41.
4.5 .
- -v - .5- . 4.2 .'Vb<b
+cJ.
'/
_k_
•'
arcscn J
2 (k-t) s1 k > 2;
, . / 41,.¡-b•
43.!0cm.
44. a-VS.
no h11y solucionés
to
si
1
y
ttco:i.
<
k ~ 2.
2H
S oluc.lon~• • lnclic<tci.ont1
· V<t•+i,•_:2alrcoset
'i6. · ·
i;cn et·
. t,1. ·45º. 54. Consideren ·los casos culindo los triápgulos
S-On rcgúfar, is6scol~. rool.á¡igúlo. Para el triáii8 ·gulo arl,liLrario hagan •\IS.o .d.e la afirm:icíón· der
ejeniP.]o .8 dol' § ?. 55 . . Djsponghil el trián~ulo
como ·se !JlU.c~trá en Jo flg; i64 y tracen C·D lt
BC. 'llagan µso ~e la soli\ejnnZá, do lps trián~ulos
AC'.Q. ·Y' :ABC;.56 .. Dc'signen los ángulos con x ,-2z
y 4z y con ayud·a dol tcore.ma úe los senos c:'tpres1>n
los lados mayores• dol irián~úl.o mediante '('¡
menor. 57. 90°, ~2°30' y .67°;!~ . Sc:.in . CH, ,CD Y,
C·M la alturih la l)jsccLriy,· ~ · la m.cdiann, rc.~pcc­
D e
tlvamento, del t.riángulo 4BC. l)esignon LC =
=4z, (:H = h,, hagan usó rle . que AU· + Mil=
Ftg. 164
BJI - MH y exprc~ñ todos los clcmcnLos de
cst.11 Igualdad con h y z. 58. A
l1
90° o b ien
\A - l11 = 00º . Expresen AD y DD con ayudo.
de la altlJra h y los ángulos A y B. Analicen dos casos: A es ápg11 lo agudo y A
<?S obtuso.
:!sen ct+~ cos et -~
2
• . 2
á!l.
l!Cll ~
•. 1'r11ccn por el p1mto K In rr,c!.n /lf P 11 AC (el
punlo M yncc en AB, el punto P, en HC) p inl.rodm.can la dcgign11ci(111 M K =
=KP=PC=t1. Del trló.ng\llo K.PC, co11 a, a y fl, c:q>rt':,-cn 111 80gmcnLo KC
y del Lrinngul o MEK, e\ l'Cgmcnto EK.
GI •. Un rcctún¡¡ulo; ri-b.
62. 1/2n(m + 11}; 1/':t(2m 2
3mn
112 ).
63. i7, 10, 2! y 1/3¡¡7 cm.
M.
l /p•+q•+2pqcoset
G6
ascna
65 . 2 1/2cos n(m~n)
sena
4(m+n)
· bfacosa
7
bsena
4-k'
4-k• si
67 • arccos 18
68. aTcsen -k-.<t+ b cos CL
y n- arcsen -,.-.-
=
+
+ =
+
V.2
Vi~ k < 2; no hay solucione3 si k <
o bien, k > 2. 6!1. Pc~igncn lns
lados del paralelogramo con a y k<t y las d1agonalca, con d y hti. Empleando
Ju fórmula que Jiga las diagonales y l os lados. establezcan la dependencia entre
@e
A
K
Fig. 165
D
a
A
K
.M
l"ig. 166
D
a y d y, n continuación, utilicen dos vec<?~ el \corcmn de los seno.~ . 70. Dcsi~·
nen
L .APB .= a , L .AI!B = ~ y dcsmucstrcn 11110 Lg (ce.+ ~} = 1.
71. Va•+b'+2b (1/ a•-bi sen• CL·cos a+b• sen• a). Con el fin cic buscar e l Ól\g'tlo
obtuso entre el lado menor y Ja diagonal menor empleen .el tcorcmn ele los
senos y, seguidamcnt.c, hallen la dia.gonRI con ay11da del teorema de lo~ cosenos.
(p•+q•) (n•-m.•)
. (p'+q•) (n•-m•)
.
72. arccos
y n-arccos
Designen l os
2 pq(n' + m•)
2pq(n'+m•)
l11dos del paralclograruo con px, q:r y las diagonales, con my, 11y; hagnn uso ele
la fórmula que liga· los lados y las diagonales del paralelogramo, después a¡>li·
qucn el \coremn de los cosenos para ex presar una de la:; diagonoleR con l os
1~ ·
212
S oluciont.s
~ i ndicacion~i
2
2
l11dos. 73. 3arecos i;,k y n- 311rccos i;,k si k> 2: no hay solueiolWtl si
k "'2. Sea BC=a, AB-b¡ AC=d (lig.165). Tracen l& bisectriz CK en el
A AIJC. Del A A CK isósce es expresen AK y CK con et y L ACK=x. De la
S<imOjAllUI de los A ABC y A CKD deduzcan la dependencia
: =2cos x.
Aplicando al triángulo ACD el teorema de la bisectriz, deduzcnn la dependencia d-2o. cos z-·b. Añadiendo a las relaciones obtenidas la condición del problc111e d (a+b), excluyan de les igualdades a, b y d y obtengan la relación
!
k+2
cos :r=--u;y
be
a+b ,
78. 15 cm .
82. 14;
•en OIXSIX
.:¡._c'
79. t6 cm.
12,&;
)•
80. 2 cw.
y 16,9 cm.
29,4
b 1 +ab-
"
8 1. a+b
1
d
a+b
84. t ,S.
83. h. ctg a.
c2
M. a~lg (
86.
b
• 87 . Del triángulo regular AOD
2
~tablezcan que KD .L AC y, entonces, KP=0,5 CD. Hagan una conclusión
.análoga para MP. R8. AC• -= AD•+CD'-2AD·CD·eosD, BD 2 -AB1 +An•-2All·AD·eosA,(lig. t 68). Sumen estas i~unldadcs y, oxprosando AK y MD
<:on AB, CD y Jos ángulos A y D, tengan en cuenta que MD+AK - AD-BC.
89. Por el punto M do inU:rseeeión de las diagonales del trape<:io tracen la
.
BE
EM
CF
MF
recta EF f1 AD UBC (!tg. 167). De las razones
A8 = AD = "'Cf) = A.D
establezcan que EM=MF. 90. o.~bb • Véase la Indicación al anterior problema. 91. 40 cm. Demue!lren que los ángul os AKB y CED son rectoe y que
la línea media del trapecio yace e¡¡ KE (véase el .p roblema 76). 92•
49
r
2 cm.
El segmento OK. se halla del triángul'o J.1lCi&n1ulo OCD . En el triángulo OAB
Hacen OP .L AP y, después de considerar los angt.ilos formaClos demuestren que
4al>
,/5
Ollf-AM y OM=BM. 96. a+b , 97 . 9; 9 "3 y 18 cm. 98. T x
- - T5 ,y m•+n•.
/--
X V m•+n•;
t OI. 2sen (IX+ · ~
99. 4
,I'r 2
y
18 cm.
100.
2 115
arccos - - 5
).
102. Hagan uso de que PKLM es un paralel ogramo (l os
puntos P, K , L, M son los puntos med ios de 10$
s
lados conseeutivos y, además , de que PELF es
un poralr.logramo (E y F son los 'Puntos medios
de las diagonales). 103. Empleen los rnionamientos en la solu ción del anterior problema.
104. Sup.ongnn lo contrario, qtio uno de l ois ángulos entre Jos diagonales es agudo y e l o~ro, obtuso
y hagan. uso del teorema 9 en el § 1. 105. Tracen la diagona l y demuestren que su punto med io
ynco en el segmento pre!ijado. 106. 2a y?.
A ..__ _ _ _ _ _-"" D 107. 150º, ~Oº. 108. arctg ~- Calculen las t11ngen·
Flg. 16'l
tes de lo.!!. ángu~os BAO, OAD,ODA yODC y con su
ayudn hallen la .t.angonte del iíngulo .M. 10~. arctg ;.
=
Introduzcan l;is <lesignaciones: AB = 2.r, B C = 3.r, BD
411 2.%. Del t r iánguexp~esen AD con .r según el teorema de los cosenos: establezcan qu e
Jo ABD
2l3
S.olucio1ie• • Lndtcaclonc•
LBAD es obtuso, hallen este• ángulo de acuerdo con el téoreroa de los senos.
110.
m•+m+·
1 :· .
(m+t)• . Designen: . ,¿
a
·
.
.
BAE"=1;1,, .LKAD=P., L. _EJAK=:r; AD=a,
y m lo.s !Segmentos· BE y . KD .. a, cóljtinuación, ·tg a:
AB=.ma. ·$:¡<.presen· con
tg fl -y hagan uso de que tg·.i=:; c~g (1;1,+ fl). 115. 36°, 69º, 108º y ·155•.
a .·
-
.
VH8--R• -r•
-
cm. U7.
6 (3 ± V3). t18. 2 V Rr. q9.
2,Y3
·'
·
1+5'!n1;1,
2asen1;1,cos(fl-a)
120. 14n+J2 ')/3. 121. 30 cm. 12~. 1.-.sena:. 123.
:sen ll
·
si
a.<ll· 124. Hagan uso de la semejanza de los \r.i.ángu)os ABC y AIJD ,
125. Tracen la cu!!rda AP 11 CD y ell.\pl!?Cn ~l ·que AC= PD. 126. 'Designen
U6.
24
M
Fig. 168
Flg. 169
OA=OB=R, LAOM=et (fig. 168) y expresen con ayuda dc.l teorema de los
.
.
20
15
cosenos AM del triángulo OAM y BM del triángulo BOM. 127. 3 Y T cm.
Demu~lren
que L BAC=90º. Tracen una tangente interior común.
R
, /- , / R
.~
¡128. T (8-3 " 3-,.., 7), T (3 ·., a-1 7-2). Hagan 0 B=:r y apliquon al
1
triángulo 0 20 1/J el teorema de los cosenos para OiB (fig. 169).
129. 2Vb'((b-a}cosa:-a)1 • Designen O'l..K=x (fig. 170). Tracen o.PllAB
y del triángulo 0 00 1 P, en el que Lu,0 1 P=ct., expresen xcona, by cr..
2a (lfa•+h'-aP
135. 36°, 36° y 108°. 136.
h'
137. 30 y 48 cm.
138. 12n ci;n. 139. 3VS cm. 140. arccos
a
142. b tgi T.
n-2nrccos
1~3.
Si
t-Yt - 2k
2,
arccos
141.
1-vt-2k
2
Va'+b•-2abcosy
2seny
, arccos
1-V1 - 2k
2
y
."
.
·
a+ b- c
. 1<14. Empleando. las formulas r=
, h=
2
scnA + cosA-1
=-¡,
T· a la formo.
sen 2A
. Más adelante, hagan uso de que sen 2A .P. V(senA+cos AP-1. 145. Dcm1,1~stren que el
ort.ocQntro es simétrico n cierto punto de Ja ciccunfcrencin con relación al lado
del triángulo. 146. Construyan en A M el punto K d~ forma que M K = BM
y demuestren que los triángulos ABK 11 BMC son igun les. 147. Empleen la
afirmación del problema anterior. 148. Designen el radio de la c ircuferencia
ab
2
Je~
, /T
V 3.
.
transformen la fracc16n
r
Soluelon~&
214
t indlcaeione•
con R, LA CK=tt y LKCB = ~. De acuerdo con el leoroma de los cosenos oxpre·
sen A K. , KC, AB y KB con R, a. y~. 14\l. Hallen fa relación entre las fraccione~
en el primer miembro de la lgu1tldad y las cotangentM de los ángulos del trián·
gulo. 150. Sea la circunferencia circu11scrita al triángulo ABC, el punto M
$Cha tomado en ()} arco AB, M K .L AC, .MF J. AB y MD J.. BC. Haciendo
uso de q_ue a los· cuadriláteros M A KF y ftfFBD so pueden circunscribir circun·
ferencias, demtjestren que L...AFK = L..DFB. 151. Circunscriban una circun·
lcrencia al triángulo ABC y contln(1en la bisectriz CD hasta su intersección con
la circunferencia en el punto P. Empleen la semejanza de los triángulos BCD
Flg. 170
Ftg. 171
y ACP y, además. el que BD ·AD~· CD ·DP. 152. Demuestren que L.ABJJ =
LOBC (!ig. 171}. A continuaci!$n,. hngan uso de la se¡nejanza de Jos triánlos ABC y BDE (D ·y E son las bases de las alturas) y la semejanu de los trián·
gulos BU.A y BDK. 153. (3 .::¡..
cm. Empleen el hecho de que CD ·AD=
= BD' y apliquen el teoreí:na de los cosenos al triángulo BAD .
=
vm
3R1151
lfh'+6k+t-k-1
155.
120".
156.
2 arctg
;
2
19
Y~+M+1-k-1
2
M
n-2 arctg
t57. arccos 3. 159. t2 cm. 1 60. a+c .
2
161. 2 'V3. 4
y 6 cm. 162. v'a•+ab+ b•. 1 6 • 2 cm. 11>4. 2 v'ii cm.
154.
v'a
165. 2 'Y 3 cm. 166. 30", 30" y 120". 167. a (n- ~-y)-
169. 30°, 30º
y 120"¡ .
~'3
y
~j
son
sen·
r~~ y~
cm. l ~O. 3$· ~ cm. Determinen el tipo del
... ¡o, 171. - e lfj
•1 )
ºd
.M y DE. se m
. tersecan en ~ 1
tr1angu
2-. - . Sea '~' ,e centro} e, C
punto P. }fagi!a. uso dG quil MP·l/C = Pl!.'·DP. 172. a•+ b' = 2c'. Empleen
el resultado del an~rior problomn ·y h?g;iñ uso- clo la fórmula. para calculnr
la ·m~diana ·n•c .c.on aY,uda del lado ·del 'triángulo. 173. 2 y 4 cm·. Hagnn uso. de
que el ·perírilétro del triángulo BDE no depende del Ja elección del punto do
'ºs
s-}
fangen'c ia ·y apl!qul!ll el teorema do
cosenos al trlángtilo DDE. 174.
cm.
Demuestren quo los 'triángulos DEC y ABC son .semejantes, cn~olices, DE=
·= BC = - 15 cm. Hagan U!l.O de cjuo ~I centr!> de )a -circlinfereacia ya~e ea.el punto
de fater~occión de AD -co¡¡ Ja perpendicular lrat!da.a. DE por su punto medio
Sol11rionts e lndteaelonet
215
= H·
175. 11 ·b •ctg ll. Haga o uso de que el diámetro de
la circunferencia es el segmento BH (H es el ort~Mro) y de qµe 811 =- 20 K
{véase el e jemplo 8 del .§ 2), donde el punto O es el centro de la circun!ercncln
clrcuoscrlta ál ·triáng\\lo .A.BC y OK, la perpcndloular del punto O o AC.
5
v a•+b•+2ab'éos•a
. 88
a
t17 . 10 9 cm. 178.
2sen2a
119 .
180. 2cos•-¡-
y, ademb de que seo A
s·
k - .i
k- 1
182
hen ct+p cos a - ll
2
2
nsenascn6
183. a) Supongamos que una do las diagonales Corma
con los lados los ángulos a, 1\, l y 6. Expresen a, b y
B
C
e, m, d 17 d1 con el radio de Ja circun!ere.ocla y los ~gur-- -=--==-i
los, a, p, y y 6 según el teorema de los senos. 184. Sen
quo los alturas BD, AF y CE se cortan en el punto
11 . Habiendo circunscrl\o una clrcun!erenc1a al
cuadrilátero A EHD, demuest ren que AC-CD=CEX
x C/I = ab cos C. Por onnlQgia. demuestren que
JJD·Dlf=atcosB, AF·All= bc cos A y , a conUnunción, apliquen el teprema de los cosenos a cada unr> de
los lados a, by c. I S5. i.5 y 20 cm . Circunscriban unn
circunferencia al triángulo dado. 186. t7 cm. Apliquen el toorelllll de l?itágoras al triángulo OKM (llg.
Ptg. 112
t 72). 187. 2 y 2 cm. Hagan uso de que el porímt>tro dol
triángulo APQ no depende de la elección dol punto
de tangencia y apliquen el te<>rema de los cosenos al t r iángu lo A PQ .
t8t. uccos - k-. n - arceos-k- sí k;¡;.t.
•
+v +
t .. / i
t
4k'
188. arcson-¡; V
si k:;;;. V2 . Designen el radio de la clrcu112
fereocia inscJ:>ita con r. el áogudo del t rapecio con •:. Exprel!eo con r y o: los
lados del trapecio, la diagonal y, a cont inuación, el radio de la circuoferencin
inscrita, 189. 2V4tg•a+3. Demuestren que EM es la mcdinoa dcltrlángulo
i
30 tga
CED y, por ello, EM = -:¡ CD. 190. , /
• Demueslren que El!l.
f' 25 +36 tgi a
V~·
l.AD. 191.
Demuestren que son L (A+B)<O. Ssto quiere decir que
las rectas AD y BC se cortan en el punto K que yace n la iwuierdn do AB
(fig. 173), en tanto que la cireuoforeocia de In que hablo.mos t\Stó lnscr itn en
el triángulo K.CD. Hallen los ángulos del triángul o ODC. 193. 12 y!! cm.
46
r - cm. 196. Tn '(yr.-7 -t). 1!!7. t +sensona a.
194. 249 cm . 195. 1 tO
a
Rson
198.
T
199.
sen(~+;)"
,/ -
4R Y 3seni
200.
3
Ta
R
2a
. 201. T ({acosa+ y3+ sen'a) . 202.
b
a
a
a
204 . ytg3 T. 205.
4Rsen•
cos
.206.
203.
3 cm.
207.
6 - 4v2
cm.208.
3
8
4
a1/ 3+r- Y4r•+2ari/3
3
')13
11;;¡;
b
a.
O,G
bx
,r;;;i.-y-a.209.7
216
S oluclon.• • lndicaclon.,
( ~-~).
210. _ __ b_ _ _
(t.
4.!!en<t costT
212. &a que las circunferencias circunscritas a los triá»gulos ADM y BDK
se intersecan eo el punto P. Haciendo uso de que LDAM+LDPM=180"
e
Flg. 179
y LDBK+LDPK=i80º•demuestren que LMCK+LMPK=iBff'. 213. f 20".
Expresen el radio de la circunferencia iosc~ita, que es el elemento de referencia,
con los elementos lineales conooidos y LAOB~2= de dos triángulos: 0 1 Pll
y 0 1 MB, donde P es el punto ,medio de KH, M ,. el punto medio de A.8.
214. R
(V
1 - t cos•
~ -cos ~
) si et;;;;.. : • Expresen OC con R Y
<t.
o
A
D
M P
Ftg. 175
·Y apliq~en el teorema ile lJ>s cosenos al triángulo ODC (fíg. 174) en el que
OD=R., CD=:z:, LOCD=t50º. 215. 2rtg ~ ( 1-tg ~). Apliquen el teorema de los cosenos al triángulo A001 , donde los pu.o tos O y 0 1 son los c;entro11 de las circuol~reocias. 216. tg'
(~ Vr 2 +arcseo <t. -
sen2
Véanse las indicaciones al anterior problem.a .
r-
11
ctg
~)
.
217
Soluclonts e l¡idlcacion.<1
sen
2l7.
i
{b·cos '~
-'V4r1 ....,b1
sen~ ~2rsen ·~)
Hallen el
. ·• o:
coseno
2
cos
del ~gulo .OA0.1 y apÜque'n el te.otcma de los cosenos nl triángulo 0A0 1 (los
·
,
. b;t-c-2 ')/be cos·o:
;Puntos ·O· y 0 1 son los.>centros d.e ·las clrcuiiferencias). 218. · · 2 senª.cz
•
Hagan uso de que AM'="<IC·ÁB (Íig., i.7.S) y · de que A..V=AD+DM.
b'-at ·
a 1 +b•-2abcoscz
·
·
219. DE= - b-, R=
Zb · .·
. Hagan 11so·de queOC'~QD·OE.
seno:.
..
.
.
Apliquen Jos teorem.a s de lDll senos y cosenos a. los tiiánguloa O~C, ODG
y· CED. 223. 360 cm•. 224. a)' 9 cm'; b) 3 cm1 ¡ e) i2 cm 2 •. 225. 4 cm•.
226.
a.•
2
. .,
(3+21(2).
a
229. mªcos•-zctga.
227.
zao .
i80 'Vli'
-
- - . 228.
19
l
4
c2 (q1-Í)
si
b)
..
tg
c)
r a -y )
.2
sen a sen y
sen(a+v>
1a-y1
2
a:+y
2tg-2-
soo '(a.-11)
2V3cosa.+scnc:t
11 • 232 •
· 233. a)
sen a
2 cosa sen
seo (c:t-"y)
2sen(a+i'l
-
y2.
mscnll rV••-m•sen•fl+mcosll>·
t<>
231.
1· <q~
234.
l (a+b)
4ab
X
9'V-
x 'V 4ci•b•-1• (a+b)1 • 235. ~. 236. Expresen los lado5 del triángulo
con R y Jos ángulos A, 8 y C (según el teore!Da de los serios) y demu~trcn
, ..
3
que sen A sen B sen C.¡;;; 4. 237. ab V3. Expresen el área del triángulo con
dos procedimientos: con ·l a fórmula de Moren y con S ._,, pr. 238. ('){Si+
'VS.>"· Introduzcan las designaciones: BQ x, CQ =y y hagan uso de que
SABC
( z+11
S1
~r;;-s-=
--· Y 7
=ztVS- 1)2. Véanse las lndi2 • 239. (VS1+ v S 2
/1
'
•
y
720
caciones al anterior problema. 2~0.• 17. Determinen el tipo del triángulo.
241. 200 cmt. Sean AK y CM las perpendilula.res a .la tangente, BD la altura
del triángulo ABC. De la semejanza de los triiingulos ABD y BMC establettBD
AB
,
can que 16=Bc y de la semejanza de los triangules AKB y BDC, que
BD
BC
abe
25=Aif · 242. mkc+ nkb+mna· Haciendo uso de que SDsp=SMDE+
+
)2
=
+
+sMEF+sMDF· .expresen SDEF con m, n y k y los senos de losóngulosdel
triángulo ABC. A contim1aci6n, los senos de los ángulos ~el triángulo ABC
sean expresados con Sll área y sus lados. 243. 2 cos c:t cos ~ cos i'· líaciendo uso dl' que los triángulos ABC y BFD con semejant.cs (véase ol ejemplo 11,
§ 3), estable:tcan que FD = b cos 11; por analogla, DE
ecos y. Habiendo
hecho uso de que LABE = LFDA = LADE = LFCA (véaAe el ejemplo
3 del § i), expresen el ángulo FDE con ayuda del ángulo BAC.
3
244.
lnt rodu:tcan las designaciones AC= b, BC =a. Aplicando el
=
r2. .
218
S olucion.s e
Lndlco.clonc~
teorema de los cosc:nos a los triángulos ABC, ACD, BCD
{ 2a'+"'=i2
a'+h'+ab~9.
obt~nemosel
sistema:
245. 3 lf2. Vé3115e !ns indicaciones al anterior problema.
27
246. ¡¡g cm1 . Vé~se el ejemplo 5 del § lo. 247. 2'10 cm•. Sean OP .L AC
(0P=10 cm), BH, In altura del triángulo ABC y OK J. BH. Del triángulo
BHD hallen B/1-20 cm. Entonces, en eJ triángulo OBK BK = 10 cm,
L ODK ¿_ BDH; de e~tc trlflngulo hallen el radio de la circunferencia y,
a continuación, AP, AC y, según Ja fórm11h1 CD·AD=BD' hallen CD.
=
249.
(a-b); senª
254 .
147 cm•. 255. 12G
, /-
a•.
250.
251. 135 cm1 . 252. 48 cm•. 253.
SR'
v'a
4
cm'. 2.S6. 1476 cm'. 2.57 . 336 cm'. 258. 168 em•.
2:'í!I. 25 v 3 cm•. 260. 150 ern•. 261.
21
(a'-b') Lg a.
262.
2R 2 sen 2a: X
a+~
a-fl
4 sen 2 cos-2n•
xscn• a. 263.
n sen asen ll
. 264. 2 sen fl·cos (a-y). 265. Trll·
ccn 111 nlturn DII y l1ngon coincidir el punto M con el punto JI. 26U. (11+11)•.
Snoc
BO
11
Demuestren que SAoo = ScoD y hagan uso de que Scoo = OD = -;;- ·
267. 20 cm•. Las rect113 divid<?ll el rombo en 9 partes de lns cuitles 4 son triángulos. Designen el área del triángulo pequcilo con z, muestren que el área del
cuadrilátero, adyacente al lado, es igual a 3.z y cicprcscn el área del triángulo
•
8ab yab"
ABN con el área del rombo. 268.
a+b . Véase el ejemplo 7 del § 3.
~~~: = ~~ . 270. 40 cm•. Designen KD .,.
son Jos puntos de tangencia do In elrcunfo-reocia con AD y All.
ffneicn<lo uso de quo PD•=CD·KD, expresen conzlos segment.os PD, OP
y /11 K. 271. 20; 10,4. Introduzcan las de!lgnaciones: AB=z, B.C:=2x,
L.. ABM =a. Apliquen ol teorema de Jos cosenos para AM (en el tnangulo
AD/11) y CM (en el triángulo BMC) y, a continuación, hagan u.w de la !6r·
mula :;cn•a+cos'a=i. 272. 4S+2c2. 273. 926 cm•. 274. 2a2 (¡{Í- 1).
a
n•
31
275. 2º'· 276. 280 (113+1) cm=. 277.
- 4- (8 ·¡{3-9). 278. 64·
2G!l. 70 cmi. Hago11 uso de que
= z, P,
Jlf
279. 11 em•. lntroduzeon las dcsignacionCll:
y
BF~z.
AF=2z, BK=y, CK-3y
comparen Jus árcps de los triángu los BKF y
281.
2a•4 (3 ·V3;
.-
5
n) . 282.
' a-b
x(a' -b')arccos 4 +b; b)
n•
i
(
v'3Mn -18
. 283. a)
ABC.
280.
-
nb'
b)'
286. a) -¡¡-(7-4l/'S)(2Va-n); b) R1 (3-2l/'2)(4-n);
~~~
'(2-
t
e)
(n-2).
zn•
- 9- x
2
n• (ct+sen o;). 288. n;• . 289. tOIP ( ¡(3+ ;
4
4
V3}. 29 L ( n- 3 -v;~c +v''i) . 292. DcmuestrP.n
X(3 l/'3 - n). 287.
290.
7
(11+b)Vab-.,--2x
a•
a•
Vana•b•Vb . 28-\.15
(3l/'3 -n). 285. T
a+
7 ~~ •
CD'=AC·BC. 293.
~· ctgjl(fl-scn~cos(2a+~)).
Hagan
) .
que
uso de que la
219
Soluciones t in<!Lc.adon.s
.
Ctrcun(ere~cla
na.•
( otga+ n.- - nn- ) , don.pasa por ef punto C. 294. t+tg~a
2
do a "= ~.
Ln "estrella " es la diCcrenoio entre el n-gono, con. los vórtices en
n
·
'
·
na• cosi a (
l os centros de los c(rculos, y los n sectore11. 295. 4 (l.+5ena)' ctg a+ n -
~n
) , dondeª'"" :
30l. 75 cm.
304.
, /
V
302.
. :vb~ la
2
, /
S
para e! añt.er ior
+;
a
2abcos
ctg T. 303. a)
11 +¡¡
V S
a.•sena+2ahcosa. +212h.
sen e
'J/2Ssen cuen ~sen y
sena
iodicaci6n
31>5.
..
problema.
11b.!eny
b)
/-2-
e
1
<t
-b1
V 4-i\. 3Q6. arctg ---¡;s.
7 V145
cm.
-V3 cm. 309.
5
310. lntroduuan lM rici1ignnclonc:1: bc;,a-{-d, c=11+2d. cxrreicn el ñrcnScon
a y d según la (órmuln do H.cron y, n conti11uoció11, hngon uso ric los Umnuttbc
• 's
.. / 11'+b•
307.
508.
1200.
v --
las R=-¡¡s- 1 r=-p· 31t .
2- - . Sea que la al~ura del t rapecio
se divide con el segmento z, sc)>re el que se hobl n en el problema, en lns parb+ z
i a+b
z-b
k
tes h y kh (k> 1). Entonces - -M= - -Ck+1)h y a=b= k+t
2
2 2
De este sistema hallen z. 3 12. 14,4 cm. Hngan uso de la semejanza de los
t r i6ngulos AKD y CKM, donde K es el punto de intcrsecd64 de AM y CD.
313. 4,5 cm. Véase el ejempló 11 del § 3. 31'. 24 om. Véase el ejemplo 11
.
2 vs.cs,+s,)
dol § 3. 315. 4 cm. Véase el eJemplo 4 del § 4. 316.
Y4ST -SI
·
ln\roduzcan Ju dcsignaoiooes: AC-..,, AB= y. Expresen conz e y el área del
,
11
s,
.
triangulo ABC. Demuestren que - = y . 317 . 8,25 cm. lfallen el radio de
de la afirmación en el cjomplo 8
la circunferencia circunscrita y h:gan
riel§ 2. 318. -
8
y3
26
- , 3V3
3
Jo
y
tOJ/3 cm. Demncs\renquett·:b:c -=4: 13: 15
2 cos++a
e Introduzcan los designaciones: 11 = 4z, b- 13z, e= t5%. 319.
1-{-Gcos+
Introduzcan el parámetro auxiliar AC - b, entonces Bé = 3b. Hagan uso del
.
yi
.r
método de arens: SAco+SDc11~SAcx+ R1cco. 320. - 9- {4+ v 7 ). Véase el
.
33+12 Vü
c¡emplo 13 del § 4. 321.
. V~sc el ojemplo 13· del § 4. 323 . Los
25
triángulos A 18 1C 1 y PQR son slm6trlcos con rcloci6n ni punto M . 324. /\nalicon la simetria con respecto 111 pu.oto modio de aquol lado, cuya looRitud es
incógnita, entonces el triángulo, en el quo las longitudes de los Indos son iguaa- b
·
.
l es a a, b y 2m, puedo ser construido; - - .- ' donde m es la
< m < -o+b
2
2
longitud de la mediana. 325. F.stableic.11n que el punto de inter~ección do fas
~olucloMs
220
e lndlcacionu
medianas del Lriángulo es ol centro do simeLrla r¡uo traspasa P.l triángulo ABC o
trián,gu\o construido. 32G. Consideren la simetri" con rolo.ción al punto P.
327. Hagan uso· de que el punto de Intersección de las diagonales del paralelogramo es su centro de simeLria. 328. T racen una recLa por el cenLro de simetría
del paralelogramo. 329. Hallan uso de que el centro de la circunlerenola es asimismo su centro de sime\r1a y con la sime\ría central, las correspondientes
rectas son paralelas. 330. El centro de la circunferencia es también el centro
de slmetda del hexágono. 331. Bstableican que el punto O de intersección de las
d iagonales AD y BE es el centro de simetría del h~xágono A BCDEF. Dospués,
SI> COA
SI> OAF•
St; COE = St; OEF 1 St; EOA = SI> ODE ·
Suman•
SADCDEI'.
do estas igualdades, término por término, Oblenem~: SI> ACE =
=
+
332. Sea el punto A ' slmdtrico al punto A con relación al punto M (fig. 176). Entonces, el ángulo BKA'
es conocido (K os el punto do intersección de La reclll 8 X con la recta 1). De aqul so desprendo 111 construcción: on el segmento A'B, donde A'= ZM(A). construyn n el scgmonlo quo contiene ol ángu lo iguol
n 180°-a, K e~ ol p1111to do lnt.!rsc.:cicín del nréo tld
segmento l. El problema tiene dos soluciones. 3.13. La
recta m buscadn pasa por los puntos s imétricos con
relación ni punlo M y que pertenecen 11 los lodos del
ángu lo A DC. Para ·demostrar oste hecho oslable1.can
que el área dol triángulo cortado por cierta recta l,
que contiene ol punto M y distinta de 111 rect.n rn, es
mayor que el 6reo del tri6ngulo cortado por la recta
m. 334. Hngan uso de la.simotrio, cuyo centro coincide
A
con el centro de lo circunfel'1!ncla. 335. El punto medio
Ftg. 1 76
del lado B C, os decir, el punto M. es el centro de simotrla del parnl elogr~mo 8 XCY (fig. 177). Por ello,
La transforlJlnción quo traspasa "' a y, es una sii;netría con rell).ción al centro M
o bien, lo que •es lo mismo, 13 :homotecia H.. con centro en M y con la ru6n
k,
1. Como XY = TI, A Z = 2 MY. Por lo lant.o, la transformación
quo traspnsa Yo Z es 13 homotecia H i con centro S en el punto do intenección
de las rectas ZY y A M y con la rozón k 2 = 2. La composicio.n de estas homotecíe.s
H 2 • Hb es cierta homotccia /1 con la rnión k = k ·k ~ - 2. Hallemos el
centro . Como 111(111) - M, /1 (M}= /Ji(ilf)•.fl1 (M) - }1 2 1tM)= A . Do modo que
DA = - 2 OM. Esto quiere decir que O es ol punto de Intersección ele las media·
nas del triiingulo ABC. AsJ, pues, 111 trans·
formación z_,. i es una homotecla con centro
en el punto do jntersección de las medianas del
e
S
triángulo ABC [coli la roión k= - 2. 336. 11) Sea
ABC,D el parail~~ogrnmp bu,sl(ado insc.rlto en el
X
cuadrlláterp d!ido 'LMNK , ol purilo O es el
f"'=::::J'--:i~-...:::::~y
centro· de simé\ria del panilelogTamo, los
puntos B y D perte'ncccn, respcctl vnmenle, a
MN y KL (fig1 178)- Es evidente que 11 es el
punto do lntorileccion de OML y la imagen do
A--- --.::..-.::::::,,.iz
M L con la shnctría con relnción nl centro O,
en taoto quo 9 es Ja. intcrsceción de M-K y la
Fig. 1 77
im.agen <!o Mi con I~ slm9tfia cpn centro O.
Sea que el pun~o A. pertenece .a / ,M y 8, a M N.
Como TAij (A). =< By T.W (D) = C, Ces el punto do intersección do KM y la
imageri"deMliOBSERVACION. En este problema se supone que los v~rtices del
paralelogramo buscado pueden pertenecer o los lados del cuadrilátero o a 5us coolinµaciones. 337. Designemos: BC:; 11, A C =b y, entonces, según el planto·
=-
221
o> b. Det.engá.mono• en dos procedimentos para resolver ol problema.
1 procedlml•'*>· Construyamos el punio: D slmétritamenle al punto C con rela·
ción al punto medio M del lado A B y unamos el punto D epn el vértice A
(fig. i79). El triángulo A·DM construidc;i es ~imétrico al ABCM con relación al
punto M., ·por lo' que ¡tD = BC,c: o y LBCM - LADM:;: p·. Ob.tenemos el
triángulo A.CD qu,e contiene lo~ 'ángulos <i. l'.· P que nos lntereSe.n, con la parllcularid.o'd ().e qu~ los lado~·;, D y Á C·, .opµe~tos. a. ell os, ·~on iguales.!\ o. y b·¡ respecU·
vam 0ntli. De acuerd?~c'o~ el planteam¡en,to. a> b. por lo qüe ci> 6. ll proudlmlt11·
amiento,
e
AL
~
A~--8
D
Ftg. 178
Ftg. 179
to, Unnn el punto medio E del lado AC con el punto M. So forma el triángulo
i•
CEM con los lados EM =
CE={. Los án.g ulosdcltrliÍtlgul!)CEJlf,opuestos
a dich os lados, son Iguales a a y p, respectivamente. Esto 110 demuestra con facílídad emJ!leando la pi:oplodad de la línea media de un \riángiilo. 338. a) Tomen un vertlce en el eje de simetría, otr<1s dos no en el eje y con5truyan los simétricos a ellos; b} e) pentágono que tiene dos ejes de simetría es rc~lar. 339. Hagan
uso de la simetr1a axial que traspasa una recta 11 la otra . 34.Q. Const ruyan In
imagen del lado menor con In simetrfa respecto do la bisect,rii del.ángulo opuesto
al la.do dado. 341. 'Des'pu ó~ do aplicar la simetrla con relación· a la perpendicular
wadla, trazada al tercer lado, el froblema se reduce a In construcción de un
triángulo conociendo dos lados y e ángulo comprendido
1
entre ellos. 343. Véase la fi~. 180. 344. La áemostraelón ee puede roalb.ar partiendo de Jo contrario. Si
8
la recta en la que yace Ja diagonal del penUgono
es su ele de simetría, en H yacen dos y sólo dos vér·
t ices del polígono (ya que el pentágono es convexo).
Los tres vértices restantes deben distribuirge en iguol
número por cada uno de los semlplañós con limite l.
A
e
Pero, para tres vértices esto es imposible. Esto significa que tal pentágono no existe. 345. 'La solución es
o
análoga o. la del problema anterior. 346. Construyan
un triángulo auxiliar de acuerdo con loa dos la.dos
conocidos y el ángulo entre ellos, a continuación,
Ftg. 180
hagan uso de la siroetrla con relación a la perpendicular
media al tercer lado. El ángulo se toma bien el dad o
o bien el adyacente a él. 347. Véase la indicación al problema 346. 348. a). Construyan cualquier diámetro de una circunferencia y el diámetro perpendicular
al primero de la otra circuoCcren cia: b) continúen el diámetro. ÁB de la circun·
ferencia menor basta su inter~~ión con Ja circunlorcncia mayor en el punto C.
Construyan fos ejos de simetría do los segmentos A C y BC. 349: La composición
de las almotrias·axiales S, • s,, o S P es una simotrlo. con 4ll eje l iy S, o Sq • S P X
X(A)=A, por lo tanto, S1 lA)-A, es decir, A porlonece a ·l. Construyan la
L(l, r) . Cualquier punt1> de la
recto l tomando en consideración q\JO L(p, q)
recta 1 puede servir de vértice A. 350. Tra~¡i por el centro de !In circunferencia
tres rectas perpendiculares " las dadas. Hagan uso del rosu.ltad o del proble·
mo. 349. 351. Supongamos que la recta D/l corta la recta CO en el punto //'
=
222
S oluclones e indltadonet
y que C' es diamctrahncnlo opuesto a C (IJg., 181). Loe rayos CJI y CO son
simétricos con relación a e M , por consiguiente, CD : CM = CH' : CC' =
= Clf : CC' = 2 R cos C : 2 R = cos c. 352. SI OM = p, ON = q, OP ~ r,
OQ
1, <1 = S ¡ o S, o Sq o S,. es un movimiento idéntico, )'ª que 11 (0) = O,
o(A) = A. Por !o tanto, Sq o S 1, = S, o S 1 o bien L(p, q) - L(I, r). 353. Con·
sideren la composición li = S 1 o S , o Sq o Sp, donde p, q, r y l son rectas quo
contienen las bisectrices do los ángulos de 108 cundrllát.eros. Como 6 (AD) =
=AD, 6 (0) =O, 6 es unn transformación idéntica y Sq_o s,. = S,oS1; esto
quier~ decir quo L(p, q) = L(I, r). De este modo, LCOu
L..AOB = 180º
(lig. 182). 35G. Pcsi!(oemos las bases del trapecio con BC y A.D, el.eje do simetría con l. Entonces, S1 (A)= D, S 1 (B) = C, S¡ (AB) = DC. Esto significa
que S 1, (AB) =·De, St (AC)~ DB. Poro el punto de intersección de la recta y do su imagen con la simetría axial pertenecen al eje. Por e llo, el punto de
=
+
o
.11
Fig. 181
Flg. 182
inlcrseccíón de las rectas AB y DC _porlcnece a l y el punto de iµtersección de los
segmentos BC y AC per\encce.oslmismq a l. 357. Sea. el punto O el cenuo do una
circunfercncin, -A B y CD.; cuerdas paralelas de dicha. circunferencia, M, el p11ntf)
medio de la cuerda AB, N el punto medio de I~ cuerda CD. Como AO = OB
y A M= MB, OM es el eje do simetría de Jo5 nuntos A y 8, ele donde se desprende que OM J. .AD. Por nnalogfa , O'N C!! el eje de simotria de Jos punt-Os C y D
y ON .1 CD. Teniendo en cuenta que AB tl CD, obtenemos: OM 11 ON y, por
ello, 0111
ON. 358. Sea O el centro de Ja cirounlerencia F 1 y 0 1 , él de las circu nferencias F 2 y F ~· Las circunferencias F 1 y F 2 se intcl'3ccnn en los puntos A
y B, en tanto que f 1 y F 3 , en Jos puntos C y D. Entonces. 00 1 es el cjo du simetrin de la figura constituida por In intersección do In~ circunferencias F,, F,
y F 3 • 35!). Se.i M el punto coinún de las clrcun!ercncins dadas w1 , w2 , ui 3 . A, 8
e con.
los segundos.puntos de intersección w, y ro3 • c.> 3 y ro1. c.>1 y Q) • • Annlicomos
la c~·ll)¡ig6ici6n '.d e ·trcs simctríns ·axiale's .S.M-~ ,,SM 8 , .S·M.c que cs. ul)a s,i'oíe~rla- con
l!io M.0 2 , dona¡¡-.0 2. es él cent~o de l~ ci,rc.unfe¡:e~cia ,ro,. Si el pun~o P es dia·m etralmente .oP.uesto al purítd ,M e¡t ro 2 , ·en t'al C'.as!i S l>!A '(P.)
Q. S.MB (Q)
= -R, 'Sy.c (R). ;="P., con la:_ partic1ililrida~ de.que .FA='· AQ,'QB ·= BR , RC =
=<;P . .Por consiguiente, AB, 8'(J,.OA $On las.,líneaN·médias·en ol triángulo POR
y, Ji.Or· esta. ra7.Ón, Ja' cir.cunfercncia :cii-c11Jis.~ ri~il oJ. tri~l!gu lo A/JC ticn() el mismo
radip qu'e ta cirqli1ferencia circunscritl\ d'l-:t~iñngulo QAlJ. 360. Véaso 111 solución
del, problema•359. 361. Sea A el.punto dadQ, ¡,In.roela dada y AO J. l. Es'·fócil
ad v~rlir q11e el. Iugnr ge.omélr\có. F bu~c;;ad.o e'~. simétrico con relación a la recta
AO y, por ello, c.'·su'flcicnte $9lo· C<i~idera.r el semi11lano a lo. derecha de AO ,
ineluida· Ja propia recta A-O . Al Juga·r .geométrico F pertenece, por lo visto, el
2 KO. Por el puntó K trazamos In
punt.o K (K pcrl~nccc a AO) tal que AK
·rocln.m-.p aralcla a l. Suppngnrnos que el punto M os el centró del triáng11lorcg11lar
ABC Y. yace má.s arribo. qµe Ja reqtn ·m, AD os la aH.u ro do é>·AB,C. Como l!)s áng~­
los A'OB y ADB son rec~os. Jns puntos O y D yac!m en la c;;ircunferencia de diametro AB y, por ello, L..AOD = LABD "" 60º. Pero, Jos triángulos OAD
=
=
=
=
Soluciones .t tndicadones
y KA M son ·semejant·e s (su~ ·1ados ~omólogos son proporcfont1les), .de modo que
LA .K M= 60º .. _s¡¡
- un1-o M ·yacé, por ,debnjo d·e la recta m., por ahaiogia·, obt.e nell1oS 11ue 4--A.K f. . "':'· HOº,,:Es 'fá~il. ~erCiora~so de l.ó c·ontrario; todo p~n~o M
tat· que LAKM sea 1gu11l J! 60°.o .bien_ a t20º pertenece al -lugar gCQmetr1co F
buscáa() .. Dy!fµitiv~ent$. obti\Pii!ll.ºª que el lugar geom~~ríco 'f' son <\os rectas·
<¡u.e pq~in ppr el punto_ K: bajo. 'ángulos .de' 60º y 12.0° hacia· la ·recta -A·O o .b_ien
.bajo a¡¡gulos de 3ú 0 •yj5.9º.haéfo,·Ia rcéta dada d, lo qúe e_s lo.mi11mo.. ~62 . Súi>onga·mos quíi::A llS el ptin~9 d·a d_o, l 1 lá f~ta.·dadn, A', un ·punl.o simótdco a A co n
.relacipñ il ·i ,""m, una recta •q\IO pá~!l.por .A ',y·que·es para'léla a l. ·EirfáciJ·v.er que el
Jugar geoiiíé~rico ile pu,ntos F. 'b uscád'o ·ea· simétrico c<in ,rchi_c;ión n la recta AA'
.y; por ello, sófo podemqs examl_rtai: ºet.soin1plano situad(! li: la !lcrccl~n de dicha
.recta {qmitiendo és~). s·eali. c~-.un punto ·pertenecfor\fo al . lug'a~. geiimét~ko F
y que yace más arriba 'd.e 111- recta m, B; ol segundo vértjco d~I ~r,iángillo ·regul~r
¡4·8C'. Entonces, la' circunferencia de rad io AB; .con ci:ntrp <frr 131. pu'nto ,.,B ~ pasa
pDr Jos puntos A, A' y C. Do 1Dm4;ldiato. de ll!Jui-se dcdu.co. que LAA 'C = 30°.
Pero, si el punto C yace por debajo de la rcctn, LAA'C: = 150º- 'l'aplbién es
evidente que en la recta m al lugar geométrico F sólo pertenece el punto A'. Do
este modo, los puntos del lugar geométrico F, situados' a Ja derecha de A A' cid
semi plano, yncen en dos rayos q11c salen del punto A' y que C\ln A A ' forman los
ángulos do 30° y 150°. Domucstron quo eslr. pnr do myos es In mitnd dcrccltn del
conjunto F de puntos buscado. Dcfinitrvomcnto obtenemos: el lugnr genm6trico ¡:
son dos rectns que pasnn. por. e.! punto A' bajo ángulos clq 30° y 150° hnci11 AA'
o bien, lo que es .lo m·ismo, 'bajo ángu los de 60° y 120° hncia In recto dada l.
363. Construian el ·punto M' correspondiente a M ol· girnr alrededor de O n un
ángulo de 90 . La distancia desde el punto. O hasta ·M' N ~s igual a la mitad de
Ja longitud· del lado del cua drado. ·354, Construyan la imagoñ de unn de Jns
circunferencins-al girar a 600, siendo su ccn\ro·el.p untoO. El pun.to de jntcrsección
de Ja segunda de las ·oircu.nferencias dadas con la construida es el segundo
vértice del triángUlo. 365. Construyan la cuerda de la ci rcunf~rencin de longitud prefijada y de la circunferencia concéntrica a la dada, quo pqsa por el punto
dado. Consideren el giro en torno al centro de. las circunferencias con el que ni
punto dado corresponde el punto de interscccfón de la cuerda· y la i:ircunfercncin construidas. 366. El giro alrededor del centro do un triángulo a un ángulo de
12.0º tnslada M a N; Na P, P a M; MN
- . 367.
-= -4¡13
3
El giro en t<>rno al cuo·
avw
drado a un ángulo de 90º traslada P a Q, Q a R, R a S, S a/'; PQ=-- .
4
370. Los segmentos AD y AEpueílenser trazados con ayuda de dos procedimlentos
En uno do los casos no se cumplen los condiciones del problema. En el otro,
existe una circunferencia que pasa por los puntos B, e, D, E (su centro yace en
el eie de simetría l de la •mariposa• Dl:.'ABC). 371. Señn O el jlunto de intcrscc·
ción de las,diag9nales A C y BC del cuadrado A BCD, M N y K:L, In h1tct$CCción
del cuadrado con las rectas dadas (los puntos M, N, K y L pertenecen a los lados
AB, CD, BC y AD del cuadrado, respectivamente). Entonces-:' R~o· (AB) =
= AD . La imagen del punto M será un ¡nmto M' lnl del segl)lenlo AD que
LMOM'
90º, es decir, el punto L. Por o.nalogia, R~o· (N) = K. Por consiguíento, R~0-º (MN)
KL y, por ello, MN = KL. 372 .. t?ara <lemoslrnr la
igualdad de los segmentos hay qué hallar el movimiento con el que uno do los
segmentos pesa ~l o~ro. Como el ángulo ontre las rectas, a l:1s quo pertenecen
los mencionados segmentos, es lguo.ln ·60°, es natural considerar el giro en torno
al punto O, Tomando en consideración que el giro alrededor del punto O a 120°
traslada el triángulo a si mismo, llegamos a la conveniencia ile examinar el giro
alrededor del punto O a 120°. Con el giro quo exnminamus el punto A pasa ni
B , B al C, Cal A, AB a BC, BC a CA, CA a AB- El punto E, perteneciente
a AC, pasará al punto M que pertenece a AB (Iig.183); el punto F, perlenec iento
=
=
Soluclonos e indicacioneJ
224
a AB, pasará al punto N, perteneciente a BC (LEOM = 120° , LFON = 120º).
Por consiguiente, EF pasará a MN. Esto significa que 8F
MN. 373. Sea
el C.PKL el buscado (fig. 184). Entonces, los puntos K y L se hallan a igual
distancia del P,unto P, pertenecen a las rectas a y b, respoctivainente, y se -vendesde P bajo un ángulo de 60°. Como el punto Les la imagen del punto K al girar
alrededor del punto P a 60°, él pertenece a la imagen de la recta a durante el
mencionado giro (es decir, L es el punto común de Ja rocta o' = R~o· (a) y de
la recta b). E l, punto K es la preimagen del punto L. Si b
R~ºº to), el problema tiene un número infinit-0 de soluciones. E n los demás casos el problema
tiene no más de dos soluciones, ya que. la recta b Uene n o más de un punto do
intersección con lá recta a' y no más de un punto de Intersección con la recta
a" = P¡; 60 °(q,). 374. La perpendicularidad de dos rectas estará demostrada si
trasladamos una de ellas a la otra mediante el giro a 90". Analizando las condl·
ciones d(ll pro'blema, advertimos que los puntos M y 8 se encuentran a iguales
=
=
B
p
a
K
Ftg. 183
Ftg. 184
=
=
distancias del punto A y 4..MAB = 90°. An~logametrte, AC
AP y LCAP
= 90º. Esto , quiere deciT, que el giro en tot'oo al punto A a 90" en senUdo
horario trasla'.d a el plinto M, a) By el punto Cal P . 375. Consideran el giro del
plano alrededpt del punto Mal ángulo de 90°. 376. Analicemos la t:omposiclón de
dos giros: R'lt y R~70 º • Obtenemos la ~raslación TAC· Con es\a traslación el
punto D pasa a P, con la particularidad de que DP = Ac: Pero, LPDE
45º
=
A
.~r\
L !~
w~
Ftg. 185
Plg. 186
.. ,N
D
M
y, por ello, asimismo el ángulo buscado es igual a 45º. 378. Sea TCD(B) = B'.
/$,el punto··médio del segmento AB'. P·o r los puntos A.• B, C, D tracen rectas
paralelas a 8$. 379. Sea "JJC 11 AlJ. Examinen la i.magen de )a diagoQal AC
durante Ja .traslacióq TADº 380. Séa: BC 11 AD. Consideren las imágenes de los
·segmentos A B y CD con las traslaciones .T BM y TCM. Demuestren que la blsoc,lriz de1 ábgulo del triángulo obten.ido e5, simultáneamente, su mediana (fig. 185).
381, Analicemos Ja 'traslación. 'l'"Xé. Hábiendo construido para el punto K el
correspondiente punto L, obtenemos: A. K 11 CL. LKCI, = 90° y, por consi-
22?
S olllclones • lndicaclone_s
=
guientcr LAKC
00º, 382, Mediant.e' l;:i trasla-ciót.1 d!ll.lado. laLeraJ en el scn\ido
y a la ais~!tcia det.erm.inl!dOS pc¡r-:111, !>Me del trapecio, · obtcril!rán un 'tdóngu_l o.
Calculen .la al~ura ·d~l t riá.11g1,llo, 383_'. :Ad.viertan- que la su.mo,_do las rJístnnc1tis
desl!e· Jos vgrt ices opu.c.sto~f ·de.l. l?"ralclo. rnmo OABC hasla cualqu.ier recta .son
igu.~)es •. ya :q~~: d lcbas ,s~mas soli: igµa·les.,al dóbl.e. d~ la:_dlstanc í~ ·des<lo .el punto
de 'lJllel.'seccioil· de las. ·d 1agonal8l!' hast.a .Ja ·mencionada r41cta , 384 . .Construyan
una circilniereilcia_ail¡tfliar. igual"ii fa da.da~ es ~angenLe a ello, y pas·a por·.cl punto M' (fig. 186). Noten. quo ·OOi .-=. 2Jl, M.0. 1 =R.. 386. Seah las bases del trapecio ABCD .A.D y BC, .M, el.punto medio del segmento BC., 'N, el. punto med io del
segmento AD. ·eon la trru¡la.ci'ón paralela TsM e1·.punto B ·pasnrá a ltf., .A n A" .
Cón la tra~lación parafola TcM él punt-0 c · pasará al ·l>U.ri~ó M,.D .a D, ~ F:ntónccs, A 1N = AN - AA,= A'N - BM (t),. N,/) 1 "" ND - DiD. =: ND - ,MC.
(2l. Sumando las lgut1ldades \1) y (2), obLencmos:A¡N
ND 1 = A /V+ ND -(BM+ MC)=AD1-BC. Pero, AtN
ND·1 = A 1,D;, Lo que signi~ica que
A 1D 1 =AD ·- BC (3). Como A 1 N = ND 1 , MN es la mediana 'del ~rlá_ngúlo
rectángulo formado A 1MD 1 ; Por esta razón, MN
A 1N
ND 1 • Toniend.o
+
+
=;
en c11enta la igualdad (3), obtenemos:
/lfN
+
=-} A,D, =
{(AD -
387. Designemos los vértices del Lrnpeejo con A. B. C. D (AC,,., 13 c.m, BJJ
BC).
= 20 cm,
AD + OC :;;= 21 cm). Entonces, S..t.Dcli = ~ (AD
BC)h, d~nd!l /1 es la altura
del trapecio. Examinemos la traslac.ión parahila T ·n c· Con esta trnslación el
puntóB pesará el punto C, Da Í)'. El área del t.rlángufo.AC.Q' cs. i.gual a~ AD' fl.
Pero, AD' = AD+ DD' =AD
BC, por ello·. asimisl)lo, el área del LrB.Pccio A BCD es igual a la del triángulo A OD' .. 388. Designemos lqs cel)l ros d o las:
circunlercncias dadas con 0 1 y O,. Enton~s. la ·traslación paralela T o,o, trnslada
la circ1mfcrcncia coa. centro 0 1 a la eircunferencia con centro O., Con cstn Lr~s­
lacióu el punloA pesará al punto C. mientras que el ·punto H al D. Por lo tanto
AC = BD
0 10 1 = d. 389. Supongamos qu.e el cuadrilátero ABCD buscado
cstli construido (fig. 187). Realicemos Ja traslación paralela r 15Ñ del lado DA
y la traslación paralela T CN del lado CB. Ahora, del punto N snlcn tl'cs >.cgmcn\os NAi. NM, NB 1 de longitud conochln. Es fácil mostrar qué Mes el punto·
medio del segmento A 18 1 • En efecto, las longitudes de los segmentos AA 1 y JJB 1
>on lguules a DC y estos segmentos son paralelos a DC. Por esta razón, el cuu1lrilátero A 1 .AB1B es ún parnlelogramo. M es el ¡iunto medio de su diagonal A 8.
Por e llo , M pertenece a la diagona l 11 1 8 1 y es Sil punlo medio. Así, pues, cr1 el
b.NA,B1 son conocidos los Jaaos NA., NB, y la mediana ca¡npr~ndida entre
dios. Con el !in do constru ir esto triángulo, morquemos el ¡1unto N 1 simétrico
a N con relación a M. Es e\·ldente que A>N, = N.fJ,. El triángl)lo N N 1A1 puwe
S-Or construido por su8 tres lados conoc1dos: NA)= DA, A1 1'{1 - NB 1
CB
'Y N N = 2 N M. Ahora construyamos el cun.drilnlcro. Con el punto M dividimos el aegmcnto NN 1 en dos segmentos iguales. Construimos el punto B 1 simétrico ni punto A,· respecto a M. Con sus tres lados construimos los lriá11glllos
A 1 MA y B 1 JlfD. Traslil.dnndo el segmento AA, con ayuda de la traslación TA,N
y el segmento BB, con ln traslación de TB,N' obtenomos los cuatro vértices del
cuadrilátero ABCD. Es fácil 1110.strar la unicidad de Ja solución. 391. Consideremos el caso general: el triangulo ABC es irregular y, por lo la nto, l!lS rintos O. JI,
M son dist.intos. Le homotccia con centro en M y razón k = - l traslada el
triángulo ABC al triángulo A· B' C', cuyos vértices son los puntos medios de los
+
+
=
i
=
lf2
1~- 0290
Soluciones e indicaciones
226
lados del triángulo dado. Los correpondientes lados de dichos triángulos son
paralelos. Las' alturas AAi. BB1 , CC} del triángulo ABC pasan a las altura~
A'A,;,,B'B;, C'. Cí del triánguloA'B'C con la_ particularidad de que A 'A;, B'B;
y C C, ~on perpe'!dt<:u,lares a los lados del t~!ángulo ABC trazadas por sus pun·
tos medios. Esto s1gmftca que el punto H de mtersccción de las alturas del trián-
gulo pasa, con Ja homotecia indicada, a l ceotto O de la circunferencia circuns-
B
Ftg. 188
Fig. 187
crit.a al triángulo ADC. De aqui ee desprende que los puntos M{centro de bomotccia), /f y O (puntos correspondientes en la ho111otecia) yacen en una misma rectu
1ty /110 ~ --zMH, de donde OM = -;¡;MH. Si el triángulo ABC es regular,
O= M = H y la recta d<> Eulcr es indeterminada. 392. Sea el segmento KD el
1
buscado, es decir, KM: MD= 1: 2 (lig. 188). Entoncos la homotecia /{~2 tras·
ladará el.punto D al punto K. Como D pertenece a BC, K pertenecerá a B' C',
Fig. 190
Fig.189
n;
1
don~e B' C' =' 12 (B.{i). Por lo tnnto, K es el pu,n to de ·lnter:<ccción entre B A
y B'C'-. .Hnbiundo construirlo ehpunto K, hallamo.s en BC el punto D que es
1
la ··prcimagen dol ,pµnto K. ~oh la }1oi:ootec.l a .H ;,.2. '393. Circunferencias tangent.os en el punto M soq bomQtec1!1$ con relación. a ~icho .punto. Analicemos
.Ja· ·homot.C!Ci¡t con la que (J) pa·so a ro 1 • Esta homo.t ecia· traslad¡¡ ol pµnto A al C
(fig. i !19) ·Y el punto B ·al. D . .Haciendo u~o didas' propicdades do la homotecia,
obtenern.os ¡l.B 11 CD. '!94'. Sea M el ,punt<i de. ~l}ngencia ·de las circunferencias ro
eón cenlrii en O y radio r y>c.¡1 .cón c;entro en.0 1 y rarlio r1 , a, la secantj) que corta
l~ ·éircunf9~encia~ ·p,or segunda y~z en los puntos A y A, (fig. 190). Es necesa·
rio dernos\ra r que 0 1A 1 U OA .. E¡camincmos lo. homotecia JIM que traslada el
punto O. al 0 1• Con se1nojante ·homo~ecia la t-ectil."11 pasa así misma, ya qui\ pasa
por el,centro, •mientras que la circunferencia ro ·pasa a ro 1 • El punto A pertenec ien·
te ·a la foters ección de a y w, pasa a un punto pertenocíontc a la intersección
Solucio!JtS
~
227
lndlcoclont•
a y c.> 1 , pero distinto de M. es decir..'al punto A 1 • Cornq 0 1 A 1 ~s la i!Mgen del segm onto OA con la homotocla 1 dJchQS seg111entos son paralelos-. 395. Deslgoemos
los punt.os de.intcrsocci 6n X de las rect!Js ·MP y NQ, Y de lns.rectns MR y NS,
Z de 111.s-·rectas PR y QS. Ea fécil <!_e ed~ertir que In composicilSn de les homote·
e les H.~ ; P~ y H'f; R) os . la ho:inotecia .//~: R>o11'1;1: P) qtfo significa lo bomotccia con el ce.o tro en X_. que traslada el pu'nto Mal/!. P!:r.Q, ~i.la composlG i.ó n
o
Ffg , 192
Ftg. 191
de dos bomotecias es una homot.ccia, los centros de las tres hqmotoclas perlener.en a uoa misma recta, es decir , .Y pcrt~ece a ZX. La segunda parte del problema
se resuelve por analogia. 396. Consideren lo homotecia HA que traslada la circunferencia w a la "'•· Los puntos M y N pasarán, con
!!Sta homotecia, a los puntos N,. P 1 , (N 1 es el punto
tl e lnter$cccíón de la circunferencia c.> con h1 recta
AN, P 1 , el punto de Intersección de la circunferencia
w con la recta AP) . Entonces, la recta N,P 1 , como
imagen de la reda f'I P, con la ho111otecia es paralela
11 ella. Debido a paralelism o de las rectas MQ y
M,P 1 los arcos MN 1 y QP1 son iguales, por lo quo
también lo son los ángulos inscritos MAN, y QAP.,
corresp ondientes a ellos, es decir, LMA N- L-QAP.
Plg. 198
397. S<?a el punto M el extremo común de l os segmentos A, , A., A 3 , • • ., los puntos de una, recta que son
l os otros extremos de dichos segmentos, M., Mt• M 3 , l os puntos quo dividen los
segmentos MAi. MA 1 , MA,, . .. en la razón dada)., es decir, _'1;~; = -;_;~¡ =
Aallfs
,
MM1
MM 2
MMs ·
= MM=· .. ='" Mostremos que MAs = MA = MA, = .. ·- -¡;¡-¡;¡-;=
MM1+M1A1
M,A,
MM¡
1
•
•
Jlf M
= 1+ MM ""' 1+A., entoncca, ""M'A';"'= 1 +A. . Por analog1a,
MA,
1
MM
MA: =
1
1
1
- -
1
1
- -
- -
i+ i. , ClC(. Examinemos B~t"; Hlj"'(A 1 ) =M 1 ; H~j A (Aa)=/111 ,
etc. Tomando en conside.r ación que la Imagen de uoa recl.3 con Ja
homotecia es una recta, obtene.m os que los T1Untos Mi. M., M 1 , etc,.
pertenecen a una misma roota. 413. Sea que D A corta CB en el punto O
(lig.191). Enton<.es, ODm:cx.OA, OC=cx.08. Según el planteamiento, AN 11 CM,
dQ nqul OM - ~OA, ON=~OC. Por consigulc-nte, OD=
OM_, OÑ - 5=..on,
Jo que significa que DN 11 MB. 415. K. es el puoto roedio de Ja cuer!o AC,
-'-1 por ello OK=z (OA+OC) (Cig. 192). Les PI punto medio de la cuerda BD,
p
Soluciones e indicaciones
228
-1--
por lo que OL """ T
(OB+OD),
1.
-
OM=-z <OA"+os+oc+Oñ). De Rquí,
-LM=OM-OL
- .= 21 (OA+OC).
De este modo, OK=LM y, por consiguiente,
el cuadrilátero OKM L es un paralelogramo. 4t 6. Sea CC 1 la mediana del
-1--i.-tríáugulo ABC, entonces CC 1 =2 (CA+ CB), 1CC 1 1=-z 1 CA+CB I· Como
CA y CB no son collocalcs, 1CA+CB I <1CA1+ 1 CB t. De aquí CC1 <
--i
411. Sabemos que MA+MB+MC
= O, de aquí OJlf=aX
t (CA+CB) .
< "2
X (OA+óB+oC), O es un punto arbitrario en el plano. Como los vectores
--1
OA , OB, OC no son colineales , 1OM1 < T (1OA l
f OB 1 1OC !) , de
+
+
A,
8
e,
s,~D,
D
C0 ~D0
Ao• B
e
A,
D1
A
(u)
(b)
A
8
(e)
e
Pig. 191.
donde OM
<.:} (OA+OB+OC). 41 8. Pe acuerdo con el planteamiento ABCD
y AD,C1 D 1 son paralelogramos, por lo que AC1 =AB1 +AD1 yAc=AB+
+AD (fig. i93) . .Sustrnyamos de '1a primera igualdad vectorial la segunda:
AC 1 -AC~AB¡-AB+AD 1 -AD y, por lo tanto , Cc1 ~TB1 +.D.ii 1 , de donde
CC 1 .;;;.BB1 +D.P,. 419, Sean A0 , 8 0 C0 y D 0 los puntos medíos de los segmentos
AA 1 , BBi. CCi. DD 1 , respectivamente (fig. 19'., a) . Entonces, para el punto
t ---1--t
arbitrario o del pl ano OA0=2 (OA, +oA,), OBo =z {OB+OB1), OC0 = 2 X
- - -·· 1 - -
---1-
X(OC+.OC1). OD0 =2(0D.+OD 1 )_. De nqu\, A 0 B0 =0B0 - 0A 0 - -z(OB- · - ·-0A
- )- =2(AB+A1 ~ ·- ·B ), D
- C ~0C
t -- -OA+O,B
1
1
1 1
0 0
0 -0D¡,=2" (OC - OD+OC1-0D1)
=Í (pC,,f-D C
1
1). ·Según
.el p]áritcamiento .itBCD y A 18 1C 1D 1 son parale-
log_ramos.y;. p.o r.cllo;· AB=D·G y AiB1 =-D 1 Ci.. por coMiguleT>tc, AoB0 =D0C0 y
el cu¡1,!lt.ilatero·A0 B0C0 D0_es un paralelógramo. úos casos par.ticulares de dispos!ción
'tnutua de l_c¡S pariilelogramos dados se exponen -en la flg. 194 ~. c. 420. Sea
AP
BQ
CR
· DS
.
.
AP
PB "'"" e
'ííD' =' SA - k y O, el punto a r bitrarlo en el plano.
= k,
·Q
·
.
de. donc;!c
=
Of-OA
OB-,OP
-=PB
= k,
.
lo q_uo significa que OP
= OA+kOB
k+t
• Por analogía,
S oluclo11e1 e 111.djcac/.011e1
Oii+wc
- .=
,;+:1 , -O!l. = oc+kOD
k-l~f
. OS
OQ=
PQ=SR. En efecto, PQ
-
1Aif
OD+kOA
•
.
k+i
. Es Jacll ndverL11 que
=,Oo :- OP' = k~1
1<- .-
229
(OB -
OA)
--
1
+ .k~·t
......__
(Pe- OD) +
-08) .. k+i A + k+i BC, ·SR_.= ·oR k+t
· k
-1
k ·-k-f'-1 (OD-OA)= k+I DC+ k+i · AD. P9ro. AB=.DC Y.
os=
(OC -
-
+
,___
BC=AD,
por ello, PQ=SR y, por consiguinte .. PQRS · es. uo. parelel!>grllJllo.
ttZf. Desigrieniós los pliotos' de. intorse~ción do·Jas mcdia_nas . ~e ll)s triáqgul os.
ABC y· A 18 1 C1 con G y G1 • ~espect1vamonté. Sean AG ·ll-H 1C 1 , BG ll·A 1 C( y
CGllA 1 8 1 • Entonces, s~gún él pl(llltoai¡liénto, GA'=·h(G.1 01 -,-.~), GB=
= µ (G 1 C 1 -G 1 A 1), GC= y (G 1 A 1 -G1B 1 ) • . Sumando, - térmi110 :por: término,
estas igualdades, obtenemos.: G..4+GB+GC="G1 A1 (y-µ¡+0 1µ¡ (A - y)+
G1Ct(µ->.). Pero, GA+GB +GC=O y , por ello', 'Gí.A a °t'l•~µ):.¡_G,B 1 (h- V)+ G1C 1 (ll - A)= O. Por otro follo, G1A 1
G1B 1 -1- G1C 1 = Q.
+
+
De aquí , hallamos que A.=y=ic. Así, pues,
1
-
T
.-
-.-
- --
(GA - G8).,,G 1A 1 +G1 8 1 -
-2G1C1=3(G1A1+G1B1).,;,-3G1C1,
o
sea,
~ BA=;z-3G1Ctt de
donde se desprende que BAJI G1C1 • ~22. Scal) A1 , 8 1 , C 1 y .01 los puntos de
intersección de las medianas e los triángulos BCD, C.DA, DAB, ABC, respecli:t t i vamcnte¡ OA1=3 (OB+oc+oD), OB, =a (OC+oD+OA), OC1=3(0D+
+ OA + OB),
OD 1 =
+
(OA + OB +
OC)
(1).
Si
M
son,
M1
y
respectivamente, lós puntos de intersección de lns línens medias de los cuadrilá+-Oli +OC+
OD) . Tomando en
1igu,aldnd (1), obtenemos: OM = ;¡ (OA + OB +OC+ OD).
-
1-
teros ABCD y A 1 B 1C1D¡. OM = r(OA
consideración la
1
Jo que demuestra que M 1= M.. 423. Sea R el pu nto medio de 11.(¡, y es té dndo que
d 11 OR (fig. 195). P-. ej·. deroóstremos que en llll caso e 11 OE, donde E es el
punto metlío do NP. Teqel]los; OP = mOL, ON
riOM, NP 11 OR. es decir,
OP - ON = k (OM
Ql,). i>oi:lgamos en eStll, última lgualdad los valores de
OF y ON: mOL- nOM = kOM + kO.f,,, es decir, . (m - k) OL
(k
11) OM . Como Jos vectorc80L y OM no son coline.alcs, m - k =O,
k
11 =O, o sea, m = k, 11 = ~k. Entonces, OP = kO[,; ON = - kOJlf.
Analfcemos el vector {OÑ OP), collncnl 11 OE, OÑ +OP = - kOM
kO[, = k (0[, - 0,M)
kML. Por consiguiente, OE 11 Ml,. Por nnalogia,
podemos también demostrar la indicada propied'a d pnra otras rectas. 424. Supongamos que las rectas m1 y €1 ~e corlan en el punto K (l ig. 1116). Demostremos que
=
+
=
= +
+
+
=
+
+
+ -:¿SB; SK = SA + A 8 +
+ -8 K = 21SB- +2tSC- + 21SA- - 7tSJJ
+ mSB
= mS!J
+ "T1 SA
+
1 ----1i 1+ 2 SC. Por otro lado SK = SB + B A + A K = 2so + -:¡:SC + ¡¡ SA Kl'o ll
1i.
----·
KC 0 = SC 0
-
SK;
1.-t---
SC 0 = ¡¡SA
0
0
0
0
0
0
0
0
SoluciOlleS e i11 dicacLoncs
23{}
1---1-1 -111SA
nSA
T SB 2 se. Entonces, mSB T SA
-2 SA
+
+
=
+
+
+ySC =
t 1nSA +TsB+ySC o bien mSB +2SA=11SA
dd
e onde m
=z-, n= T.
t-
1-
-
t
1
Por esla raz6n , SK =
+
1
+ -;¡:SB,
1--:;_
tsA+sn+sc¡. En·
2
- .
11t t tonces, /í.C0 =<= z S A +2 SB - y (SA
SB + SC) = - T SC, debido a
Jo cual K0 0 11 SC. es decir, Ja recta n1 pasa por el punto de intersecr.lón de las
+
e
FEg. 196
Fig. 195
rectas 1, y
m 1 • 425. Vt!ase la solución dol problema 424. 426. Primero, demostremos que en cualquier cuadrilátero MN
= Tt (CD+BA.),
donde M y N
son loe pun1.0s medios de CB y DA,respectivamenl.e. MN=MC+cD+DÑ,
MN= MB+BA+AN. DP.spués de sumar estas igualdades y teniendo en
cuenta que
M"C+M8-o, iJÑ+ÁÑ=Ü.
obtenemos: MÑ =+(CD+ BA).
i
Sea que los vectores CD y BA no son coline~les, entonces MN <2 (CD+ BA),
lo que es fácil de demostrar haciendo uso de la propiedad de los lados del triángulo. Por consiguiente, CD 11 BA, es decir, el cuadrilátero dado ABCD es un
trap~io o paralelogramo. 427. AA1.+ BB1 ~::Ci= O. Por ello, OA 1 - OA+
+Qa;- on+o.c ,--.QC=O. Si O esel.puntodll lri~ersecci6n de lasmedia'!las del
·tr~iggulo 1_BC,; OA OB+. OC=1i\ .p or CQ!lsigliienle, también 0.il 1 OíJ,
PP.1 .... ·O. De·S:c¡uí.se de~prendo ~ue 9 es ~l,P,unio 'de 'µitersccci9II de las miidíaA 24 .'+ ,AC." Durailte é) gíro a 90º AtA
_nas ~el triáng2~º AhC1 • 428. C.A 2
.pasará a.·co , A:C a BBi, es decir., <;?4 2 pasará' a CB
BB 2 := CB 2 , ~e dondes~
deduc:.e que CA , .L CB¡: y CA 2 .=; CBi· 429. Sea D et punto medio de C1 C,
..._1--.(fig. 197) , CD ""2 (CC, 4 CC{)·. Al girar a ·90• los vector1is CC 1 y ce; pasarán
+
+
+
+
.=
+
+
a los ·v ec_tores CA y - CB, ~ectiva:mente. Por~ ta~ durante este gir2.Ja
suma 'de los veo~ores CC 1
ce; pasará al vector CA - CB, o sea, al vector BA •
·CCi} .L fi, ICC1 +ce; 1 = IBAI. do donde se duduce la
.Por esto, (CC1
aflr¡nación del p'toblemu.. 430. Demostremos que el vector P R se obtiene del
vector SQ con el giro a 90º (fig.198).SQ = SL
LK KQ o bien SQ S1
+
+
+
+
=
+
231
Soluc/4nu • fndlcaclon•s
+· ! (D.C+ AB) + KQ. Hagamos· girar a .90° cada uno de .los vectores del. s~gun·
t 1a -LP=:;.yAD,
~DC
na.'Eh -tal.~nso, ~Q pasará af AD+M~+PÑ+
.
-
do miembro de la última igualdad. ·Entonc:ell, SL pasará
a MR,
-~ ·.AB a PN; KQ ·•a
+ ~- Bc ... .PÑ+ ~
significa· que P R
·8&
(AD
.!
¡;¡¡ + Ñ'M
+ BC)- +, Mil .=
+ Mfl =
P.R : .Esto.
obtiene mediant~ el. giró. -de SQ a 90°. ~or lo. tantó, P.R 1.
Ci
~
e:
s;
B,
p
A,
A,
A
Fig. 197
.L SQ , I>R --
SQ . 431• Como MA
B,
Fig. 198
+2 MB__
-
Me+ Mí)
2
MA
+ MB+
+ MC + MD =O, es decir, MI> = - (MA + MB + MC). Tenemos: U=
= MA + MB, V= MA + MB + MC = - MiJ, o sea, V= - . MD.
Esto quiere decir quéM es el punto medio del segmento D V. Expresemos el área
del c:uadiílátero MAUV. a) S1.uuv = sl!-fAV + SMuV = §MAJJ + S,_.1cDi
b) SMAUV = SMAY + SuA.v = SMAD + SMBC· • De aqu1, 2SMAUV =
= SI>cAB + SucD ;- SMA.D + SMBC = SA BCD· As1, pues, SA:BCD: sM....lJv =
= 2 . .\32. Sean AL y BK las med ianas del triángulo ABC y M el punto de in·
- (1). A continuación, KL
- =
tel'l!ección de AL)' BK. En tal caso, -K[,= 1AP
2
1AB,
- de donde
= -KM+ -ML (2). De (1) y (2) se desprende que KM+
ML=
2
2KM + 2ML = AB, pero, AM + MD = AB lo que significa que 2KM +
+ 2ML = ííM + MB o bien (AM - 2MZ) + (MB - 2KM) =O. Los
vectores AM - 2ML y MB - 2KM son colinealos a los Yedores AL y PK,
respectivamente, por elloAM- 2ML = Oy MF - 2KM =-0, es decir,AM =
- y MB
- = 2KM,
IAMI
IMLI
= 2ML
lo que quiere decir que ~=2 y --=- = 2.
_
lft1al
IKMI
433. Ante todo demostremos que los puntos M, N y 8 perten!!C.e!l a una ml.!lDa
recta. Con este fin hay que demostrar que los vectores MN y FlB son colineales.
MN =
MA
+ AN¡
fil =+DA; AÑ
=
~
(AB
+AD);
MN =
~DA+
Solucioneg e i11dicacio11ts
232
1-
1-
+-¡¡AlJ - l f DA =
= AB -
61AB
-
61AB
+
1 30 VA =
1
-
-
3lf (SALJ+DA);
151-"""""
6 AD=-¡¡ AB +¡¡VA. ,yB =
1
G
-
-
-
NB= NA+ A/J
=
(SAB -1-DA), Jo que
significa que N B = SM N, por coosigui<!nlc, Jos vectores N B y M N son colineales . El punto N divide el segmcMo /lfB en la rn?.í>n f : S. 434. En el Jeng\lajc
vectorial, el requisito del problema consiste en demostrar el hecho ne que Mo =
~ ON, donde O es el punto de lnlcrsocción de los segmentos, cuyos extremos son
los puntos medio$ de Jos lados opuestos del cuadrilátero ABCD, en tanto qu ~
M y N son los puntos medios de las diagonales BD y AC. MO = 7iiP +PO,
t1- ~ero M /> = T DA, lo qua quiere decir que MO=T DA+ FO (1). A continua-
-
ción, ON = OR
+ -RN,
1 -i-l!N=-z DA, por consiguiente ON= yDA+O.R (2).
Comparando {!) y (2) y tomando en consicleración que PO = 01!, obtenemos:
MO = ON. 435. Si los segmentos A 1A •y 8 18 2 yacen en lns rectas paralelas n ! ,,
B,
··~.
A .~e,
B4~C2
A,~
C,
A,
A
C
Ftg. 200
Fig. J99
en el caso cuo..ndo AJA 2 = 1J 18 2 hay \ID número infíni\o ele soluclonos: es su(icionlo
lomar en la rocl.tl .c ualquier segmunto C,C, tal que c,c,
A 1A 2 • PeTo si· los sl'g·
mcntos dados son tales que A 1 A 2 ""° JJ 1B,. es evidente que los puntos e,, C 2
requeridos no ex isten. Ahora. pnra mayor certeza, supongamos que A 1A 2 no es
paralelo al Y,_ sea A, el punto de intersección de l as rectas A 1A 1 y 11 • En la recta
B 1B 1 coostru.1mos dos puntos M 1 y Jlf, tales que 0 1 8 2 : B ~M =-= A ,,A 1 : A ,A 3
y designemos con B • aquel de ellos pára el que se cumple la condiciún: si A 1
yace entre los otTOS dos: puntos en la recta A ,A., ent.onccs B 1 yace entre lns olrns
dos iiuntos eii 11! recta B,8 2 • Ahorn unamos los puntos B 3 y As y t!acemos P?r
los puntos B 1 y B, Tectas,paro.leln! 11 A 38 3; sus puntos C1 y C, do 1ntersccc1on
.
. . .
ce
BB
A 1 A'2
1
con.la~ectal,~on· Josbuscados.Enefecto, C A·•= ·,,10 '= A A , <lo do¡ido
'
l. 3
>.'2 3
, 3
con, ay_lida de l~ proporción deducida, ·tenemos: CCiAA;i = AAiAA, . Entonces,
=
.
2
3
t
3
los .~ngulos_A,1 C1A 3 y A 1 C2 A 3 son iguales como los.ángulos correspondientes de
los triángulossemejan\e.s · A 1 C 1 A~ .y A 2 C,A 3 , de modo .q,ue A ,C1 11A 2 C 2 • 436. Con·
sldéremos l'a · solución · de est.e probll!1lla ségún el metodo vectorial (!ig. 19!1).
S~gún le, propiedad de -111- sú'ma .de vectores representados ~na <!_Uobrad'a cerr11-da,:tenemps: A(Bi
B,'Bf .+ 'B,1.C•
C2 C1
C1 A,
,1,A 1 = O. Ho.ciend<>
uso de las' propjedades ;isociativa y conmutat'fva. de la suma de véctores, .obtenemo~B2B, .. c,C 1• .+ A~A,. Ú 1B 2 B,S C1A 1) =O. Como AJi;
== A B, B,c.-= BC, O¡A", = CA y AB BC OA ó, la suma de vecto·
res entro par~ntesis es aáimismó igual al vec,tor nulo. Pc:r esta razón, JJ 1 B 1
·+
+
+
+
+
+
+
+ +
+
=
=
+
Soluciones e
1' n.dicacion~s
233
+
+C2 C1 A-1./1 1 =-Q pe.r o, entonces·, 8 2 8 1 = C1 C:-A1A 1 • Está· c)a.ro, ·ql!e
cualqúieTa de· Jos tres ;·cctorQs daaos pilede"sor representado como la d 1fcrenc1a
de los otros.-dos . Si ..estos vectoreS'-!J.O-S()n colineales el ~riá!)gulo requcrido:e:l'.iste.
Pero dos de .los tres vectores. d11dos..no p1,1edcn ser !lOJi'neales; ya que · entonces el
tercero..será.'colineal a ·ellos (éómo. lii suma· ele .dos veclo're~· coliileál~s). Noobst.ante.
es posible el, caso cuando. Jos tres vectores son,colineales'.. En efecto, sl' tolilainos
como.·er v.értÍce A 1, ·p : ·ei., del ·tercer·.pará·l!l_lo.gramo· cualqi.ti'er püntó ~o :¡9 recta
pualola a C 1C,, eri este caso cibtenem1>5, (fig. 200): 8 2 8 1.
B;,B
·.lJ.B; ~
A1A +ce,=
A;ifi,+ (ce;.:+ C,C,)""' (A.¡A; +C3<-"2) (tl2A+
+CC1 ) = A,A 1
C¡,_C 1 , Así,'pues'B 1 8 1 = · ;i,A;_:+ c,c,, es decir,. el-trlá.n~llo
requerido no existe. De este inodo , la construcció¡i do este trián,gulo sólo es posible cuand'!;> :los
.B,
vectores, que conesponden a los segmontos .indl~·
.
cados en él planteamiento, no son colincales.
·B ·
Examinemos la ·solución del anterior problemn (acerca de la posibílidad de la constrnc·
83
C
clón del triángulo) según el método de la tras'.A
~
lnción ~nr~le~enn no ~olineales los vecto_r~s
A,
C
B,B~, C1C1 , A 1 A 0 • Realicemos la
tr11slac1011
paralela T- del triángulo CC 1 C, (íig. 201).
A
et
CA
i
Entonces, CC 0 pasará a AB 3 , CCJ a AA, y
C 1 C2 a A 2 B 3 • En efecto, f!.8 1 11 cC 1 y BB, =
Fig. 201
CC 2 ; ABt 11 B,Bi y AB,
B,f) 1 • De ncuerdo con la trans1tividad de las relaciones
de paralelelismo e lgu1tldad, tenemos: C.C~ [) AB 3 y CC 2 ....: AB,._ Pero entonces,
C1C2 pns11r6. a A 2 B,. RealiUUJ.do la traslación paralola TIJA del tn:íng11lo BB 1 B,.
Jefinitiv11mente obtenemos; B~ pasará a A 1 , 8 1 11 8 2 , C, a 8 3 , C1 a A 2 y, de
este modo, el ll.A,B 3 A, tendrá los lados iguales a los segm~ntos 8 1 111 , C1Ci
=
=
(..r.;:r; +
+
=
y A 2 A 1 • 543. a)
+
+
=
q/ ~:~ ,b} q/
544. arccos { . 545.
b+~
S ·t¡¡ {;e} 2 ( t+scn {)
V ~~ri·
. 546. El valor mínimo es iguR n lt0 , el máximo, a h11 •
547. La diagonal os la altura del triángulo. 548. 4 ¡f2 ro. 549. 30 cm. 550. n)
6000 cm•; b) 108 cm2. 551. KD=MD = 5 cm; S=32 cm•. 552. a) 45 cm•; b) 36,
125 cm'.553. a), b) A C= BC. 554, a), b) El ángulo central del sec~or es igual a 2.
p
•
l f hi+anz-3h , ·
3 • ,...
555. n:¡::r. 556. Ln nllura del rcctaugulo
. ;,57. 4 R· 11.~ .
4
558. l proctdim¡e11lo . Sea que en el triángulo ABC esté dado AC ~ b, LB = ~.
Designen LBA C = z, expresen AB y BC con b, z y ~ y dupliquen la mediana ,
construyendo el triángulo hasta un paralelogramo. // procedimittllo (goométrico).
Circunscriban una circunferencia al triángulo ABC y demuestren que la mediana
BM de un triángulo arbitrario es menor (en ol caso del ángulo obtus.o , mayor)
que la mediana B 1 M del tril\ngu lo Isósceles Al11 C. 559. Seun AJ> y CK perpen·
dicnlares n I; designen: L...PBA = z. Después de los cálculos demuestr•n quo
P B = DK. 560. a)
2vs
V2'7 ; b) 3 1r 3 r' , Ocsignen con la letra z la mitad del ángu-
lo en la base del triángulo . 561. ~ cm1 . Sea PKMF, el lrnpecit> inscrito, el punto E yace en BC, el punto M, en CD. Los triángulos PAK y ECl\f son semejante~. Habiendo hecho uso de esto, introduzcan las designaciones; CM= 2z,
CE ~ 3:t. f>62. 100°. 'Véase el ejemplo 4 del § 7.
t 6-0290
Soluciones e indü:acio11es
234
Capítulo lI
63l. 60". 63.2 . 60º. 633. 115º;
35º. 634 .
300. 635.
2arccos (cos~)
COs I'
'
4
VB- . 638. 900. 659. arcscn--¡¡s6V~
G36 . arcscn5. 637. arcsen. 640 . 30º;
3
1151
4ab 1139
1115
112
nrctg---¡r . G4 l. arcsen 13 (a•+b'). 642. arctg----r5"'. 643. arcscn6- .
1/ 6
4 ')165
•
'Jfts
')115
644. arcsen 3
. 645 . arcron ----¡¡:s- • 646. arcsen -g- ; arcsen
---rn- .
Fig. 202
v3
2V3T>
2y30
(cosjl)
648. arcscn - -5- . 649. arcsen - 1-5- . 650. arcsen
1
651. 2R (2 ,v·/-3-1¡ -2). 652. SO". 653. El primer segmento forma con el segundo,
tercero y cuarto ~egmcntos ángulos de 120°, 00º y 60°, respectrvamente; el sermo.lo
con el terec'r o y cuarto segmentos, t20º y 90ª;, el terceró con el cuarto, 120 . Con
647 . arcscu
sena- .
- !)- .
s
s
e
B
e
B
Fig. 201.
Fig. 203
el fi n. de hallar. él LAR 1 C 2 (flg. 202), construyamo~ AD1 , D 1Ci...Y. B 1_D1: C<!mo
AD 11 BJC" los p1inlos A, B,,
yD 1 yacen oli un m1smo ·pln._no. l:'Or cons1gu1ente. 1LA.Ji C; = LAB,D1 + Ll). 1B 1 t: . Para ha Bar l!l ángulo entre las .rectas
Ali 1 y c 2 1:J 0 es posible em_plcnrer paralelismo.de las rectas c~D, y BA 1 • 654. 45º.
657; arccosi6. Sea CM lli mediana del trlángulo-ABC lfig. 203). E'n el plano CEM
c.
S oluclone~ e indicaciones
235
por el eunto O, trnr.amos la ;ecta O.f:{ 11 CE. ,Entónces, .LAOK ~ il!'Jl!I n_I ángulo
entre las rectas AD.. Y. CE. -Es poi¡¡ple ballar t..AOK cakulan~o los lados del
triángulo ÁOK. Confo pnrámc(r~ '!,11xilitll' es conveniente ~orna.- AB ~ a.
·s ll 102 "l , prót•dímiento.
·
u 1a me d i:1:~ª
·
d.e1 .'t ri11._
• •ngu 1<> A·oc·
658. arccos """102·
S~a Cin
(!ig. Z04). 'Por el puq_Lo O en el pl~.119 CE/lf trnamos la recta.'Of(. l_I CE. __En.ton··
ces, LKO.P es i'g_ual nl á_ngul~ iln,Lre . la~ ,r.~tQs Of!. . y CE. ,ES' p~~·~lc hall!lr el
ángulo KOD calculahoo. lo~ ·1ado~ del t.ri.angulo _KOD. H proced1m1e11to. Como
s
,,
e
D
A
Pig. 205
Fig. 206
AS .l AC, AS .l AD, AIJ J. AC y AB=AC =AS, es p 0síble introduc.ir
un s islemn carlesinno rectangular de coordcnadns, haciendo AC = i, AB =T.
2 -;;1 -: t
tAS = k. Enlonces, OD
OA +AD=
1-A ·+·AD =-3 t-3 l+z k,
3iCE = CB +BE= k. 659. arccos (seo a sen ~+cosacos~ x
2 i f
X
e~~
=
+ +2
y) .
a
660. 45°; 60º. 66J. nrccost. Haciendo DC=
hollnmos que DP
=~ DA+ Al'= j
V§
66z. arecos-¡¡.
.
663.
+ z1-k,
~
-
QC = /JC
i,
DA
= [, DD¡ = };,
+ 2'i -CC =i- + 21
1
-
k.
vm
arccos-- . 664. arctg l 1 19. En la carn SAB, por o\ pun-
5
to M, traU1mos M F 11 S K (fig. ZOS). Entonces, el ángulo bu~cado es igual a
L DMF = :r. lfacicndo AB = a (parámetro auxilfar), podemos hallar los
lados FD y MF del triángulo DMF y, a continuación, lg z. 665. nrctg
~á.
6G6. arccos~. 667. arccos (-~).Por el punto Den el plano APQ Lraiamo$ la
recta DR 11 AQ (!ig. 206). Entonces, LBDn es igual al ángulo entre los rayosDB
y AQ. Parn determinar este ángulo es posibl~ hallar los lados del triángulo BD/l.
5i tomamos como parámetro auidliar /JC
a, cll ol triángulo DDR DR =
o l/2, BR
a v'5. El lado BD puede h11llarso del triángulo rectángulo
=
=
=
a'J/S
BD/lf, donde BM=2- ,DM
alf 5
=2 - . Obtenemos; BD=a V2 y, n continua-
ción, según el teorema.do los cosenos, hallamos LBD R. 6GI!. arccos
·~ . DC$com-
poniendo los vectores BP y OL cu los vectores Í = OC , i ""O/J, k =Os
de la base rectangular ca.rlesinna y, más adelante, hollamos el coseno del ángulo
16•
S oludone$ e indicacion«!S
236
entre los vectores BP y OL. Es posíbla operar do olTO modo. Por el punto O on
el plano SBD lrar.amos la recta OM 11 BI' . Entonces, LLOM es igual al ángulo
buscado. Con el !in de determinarlo hay que hallar los lad<>s LM , OL y OM del
triángulo LOM y, seguidamente, aplicar el teorema de los cosenos.
669. arccos
(-fo). Descomponer los vectores K8 y CD por los vectores i= MA,
Í=MIJ y k= MO de la base rectangular cartcsina, donde Mes el punto medio
de la diagonal AC del cuadrado ABCD . 070. arccosi. 671. arccoa ( -
~6 ).
Por el punto S en el ¡>!ano SBC trazamos In recta 111 BD y por el punto D, la recta
m 11 S8. s~a que las rectas 1 y m ~e cortan en el punto K .. Entonces, LKS.O es
e
IJ,.
s"
Ffg. 207
Fig. 208
igual al angulo buscado. Para determinar LKSO es preciso hallar los lados KO,
S K y SO del triángulo KSO y, a continuación, aplicar el Leorema de los cosenos.
V~ sen et sen
a
072. arccos (sen et sen~}. 673. 4,8 cm. 674.
675. 2 ·
1 sen (CL ± ~) 1
p
a
1121
ab
-+··.
ah
676. - -- . 077. R; ·¡( R•+f,•scn•et. 678. a v
679. 1~
·a•+1i•
7
. . .. ,ra 2 ·scn·• cp+ll•
a ·¡t í
á 1/3
R V .,-·c!Jll 2cx
680. a V
. a'+h' , G8i . _- 2- . :682. - 3-. - . 683.. . seoet
•
684. Vb• ._il..c\g''<i, G85. V22;5~1~. lft3,5 ero, v~.5 cm. SI. cort11-mos la
j>lrámide- por .las· a·rist_ás sA , SH;. sc.;:c;léSñtrollaódo· las caras
laterales ,. COIJ\O
111
se m11ostra en lo fi"g:. 207, obtbnemos cJ ·triángulo S' S1 'S • En cfocfo·, como
d.e.sp1¡és del giro el' á.1:1guio· s.c c9n.set.vi11.. L1:= L3·, L2= L4, y, por lo ·i anto.
L3+ LBA.C.+ 4:.4= .180"',_ es .deéir, el ·p11nto A yace ep. el Indo S' S". De forma
análo'ga, el.punto B yo.ce en el l¡ido S'.S'" y et punto C, en el ladQ S'"S'". ·En el
tri~ngu_l<í ·S' $" S'" AD, AC y Qc·son.las lineas .medills Y.• por ello, los triángulos
.SÁB., SBC, SAC y ABf:.: sbn l~s re11roscntaolbnesde cuatro triángulos igu;Hes·, Disponga.ñ, ·l! cqntrñuac<ión, la :pirámide dada enun.p·aralelepípedo, como se muestra
0ñ la fig . .208, y, -<!cspués dé demostrar que esUi p.11ralelepípedo. es rectangular,
hallen las disltincins ·buscadas. 690. GOº. 691. 90°. 692. 45°. 693. arccos (ctg1 ó:).
-694. ' arceos ~seri ·~ 'cos et}. donde. 00 <a·< 90°. .
'
237
S oludo11ts e indicacione&
695.
211 3
)
arcscn ( - 3-sc!lcr; . . 696.
col! ct-c~ ~ cos.y.
scii~scny
, fh.=
ex~=nrcjl(ls
~
cos.:v-co~a:cos·~ . . :.
, /aH-o'
sen' ex seo. v.
• '1'1.- arccos sen.ex 5cnjl . 697• arc~n
~.
sen cr; )
(. ·y z·
·}·
.
21/6;
698. arcscn ( · sc~jl . 699. arcsen '.2scno: . 700. arclg-3- · • Baj11mos del punto K Ja· p1;r:pend·icu)ar K [, al. pl\lno ABC Y¡ después del p\lqto L
la perpendicular [;M a Ja ~édíona AD 'f unimos cJ .punto M c:Qn el. plmt.o K.
=arccos
cos~~cQScr;cos'y
v·
Fi¡¡. 209
El ángulo KM L ~érá e) ángulo lineal del diedro buscado. Cmi el fi n de determinar L KML es posible hallar los catetos KL y ML del lriñn¡:~ulo recl.iingulo
KML.
70t.
VZI 702.
arctg-ll-.
704. 300; 60°; i20"¡ 60". 705 . arctg (
t1
arcco.~35·
21 tg cr; )
703. 2arclg (
. 706. arccos
• /
V
1 ) •
SéñCi
1-2cosa;
.
3
V.r t-cosa
• /t+3cos' o: )
707. 2orcsen ( V
• 'i'09. arccos
a+coscr;. 710. 45°. 13aje2 .
mos del punto L la perpendicular LN al plano A.BCD y, 11 continuación, del
punto N ln perpendiculru: NM a la rectn FK. Unan lps ;puntos L y N.
El ángulo LMN será el ángulo lineal del diedro buscado. ,Vara determinar
L LMN podemos 'hallar los cnlclos LM y /11N del triángulo rcclángulo LM N.
180"
180°
) .
711. . arccos(-cos• a),
712.
713.
arce<>s ( ctg-n-ctga
-n-·
180" clg a. ) .
714. nrcsen ( ctg -,,715 . tg x=
cosa;
·1/ -cos (.;;.+a.) cos ( : -a)
.
1
716. arccos5-.
En eJ
plano ABC,P1 • por el punto A, trozamos unn rcctn parnlcln a BD 1 y dcslg·
nnmos con vf 110 punto de intersección con la. rectn C1 D, (fig. 209). El ñngulo
CAD, será e buscado. Purn determinarlo, haciendo, p. ej ., AB=n (parámetro
auxiliar), podríamos ha.l lar Jos lados CDt, AC y AD, del triángulo CAD,
y después, aplicar a éste el teorema de l os cosenos. 717. arccos
2+ 3·tf2
8
•
238
Solucionu t indícacionts
.
3VÍ-2
av58
718. 45°. 719. arccos
. 720. nrcscn 29. Bajemos la perpendicular
8
CP del punto C al plano SAB. Entonces, el segmento DP será Ja ptoyección
de CP en el plano SAB, y por lo tanto, L CDP será el buscado. Con el fin
de determin~r L CDP es posible, haciendo AB=a (parámetro auxiliar), hallar
· .dos 1a dos de1 tr1angu
..
1o eDP. 721. arccos -112- ; arccos116
cua1esqu1cra
-.
722.
a.rccos
ab
yii )
( ---¡¡- .
¡12
723.
724.
vi
3a'. 729.
Sa2
- 1- 6
727.
4
y?
2arctg-3- ·
726.
- -4
730 ·
Jf• sen 2o:
8seo(30º- o:)sen(30"+o:)
728.
731.
a,
2
4
1
3ª· 3ª· 725.
3a 1/ 17
_1_7_
a
a·
sa
732.
s
e
D
A
Fig, 211
Flg. 210
?a• V 113
ai llTií
• <>1 1151
_ 1
733.
7M.
735. - - · Ssec - 2BD·MK,do.ndeMKJ_
8
72
10
.l. BD. La construcción MK .l. BD se puede realizar del modo siguiente
(fig. 210): 'i ) por el punto O en el plano SAC se traza OM.11 SA; 2) por el
pu11to M ..en .e~ ºplnno SllC se baja la pcrpendióulár MNº a AC¡ •. 3)' del" ·punto
f: . se ·}¡aja la perpeodJc.ular .CL a BD; 4). por: .e.l ·punto. N se traza' N KIJCL;
p) se uoch l os puntos. jlf y K : ·La lon~ltud: del seg1~ient!> MK• ~ ·puede hallar
.del
.
triángulo
,
737,
.
rectángulo
tgf!
· .
,2 cos 3cx
atC<_0.5«
.
7J8.
. , - ·2aH
'X .r1 a--:+ct. 741. o/'g¡..1
+4111
MNK.
73~.
al 2 l /.3 COS <X 'J/ 1
·
·
2
+ 3 COS· c:t
2S :J/3
S(c+ll)
b
. 740. -6 X
En - .,crn•. 7(19.
2
..,
.
111 i/3
2b 113
742. ~, <loilde 4~ --3- ctgc:i (b es
a• sen_2_2c:i cos f!
'
42 113
at 113
743 .
reo~ (a+f!> .· 744. - -3745.
4
.
· Es P.osi.bie construir fa sccció)l roqüeridn del . modo siguiente "(fig. 211):
• · 1) coiistruy¡m.la mediana OM del. .triángulo AAiC (es evidente, QUI\ e.o tat caso
CM .l. AA,r 2) hallen el punto O, de intersección de las rectas CM Y 00 1
.
1.o. arista 1.a lcral),
S oluclones e tndícaclonos
239
3) por. el punto 0 2 ·cu el plano BB 11) 1 P tracen la recta N KJI BD; 4) el
cuadrilátéro 'CKMN es la sección buscada.
74G.
a.'2.
-
.,. 1/li
4
X
.
-z ;¡/5en(60º+a)'scn (SO~."""?ªl ,. donde 00<a<45°. 747.
x(1+ ..V/
·E\n+ 3
m
m".. )
~ lfS (1~,/0m+:!n)
---¡;v--,.- · •
2
b ..
te~
o
P<
donde
t.
H lf6 (JI es lo altura de la· pir~mide). 748.
Como· AC?BC=a y A!J=
= a 112 (fig. 212),
L. ACB = 90° y en' oL ·triángulo rectángulo
SAC. SC - a V8. !llientras. que en· el .(riaagúlÓ recl.ángulÓ SAB SB: = 2a.
La construcción de !a sección requerida se pµede realizar de Ja forma sigu!eiltc:
1) construir CD que es la mediana del triángulo Á BC; !?l construir la .recta K"L 11
11 CD"(es ·evidcnlc que. entonces, AB J. KL y ·S.B J.' "f(L): 3) construi(~P quo
es la mediana del triángulo SAB; 4) por oL punto f, en el' plano· SAB traiar la
recta LN 11 AP; 5) desde el pun~o A en ol piano SAC bajar la rerpendicular A f
a la arista SC (para l a construcción es posible hallor que CF : CS = 1 : 3);
6) por el punto N en el plano SBC se train la recta N M 11 P fl; 7) unan los puntos
<
vs
/lf y K. E l cun<lríiátcro KLN M es la sección buscada. 71i!I, arctg--¡,-.
7!'>0.
4 1/ f<í
11-3
9
753. arctg (
3 tg
761.
a) .
ll7
arctg-- .
757.
v2t
nrccos (tg
6
762.
96 cm•.
771.
3
1
144 cm',
y 21/li
S sen
4.8 cmt,
T
( a. lt \
sen T+T)
2111 cos4Lcos (
773.
775.
777.
•
772.
+++)
arctg-- .
4
tg ~
~
llB.
759.
763.
sen• 3ct
G l l(¡ fl2 ctg a sen (45° +a)
sena
769.
755.
n• sen• 2o. cos et
764. d~ 1/2 sen 2~ cos (45º - et). 7G::i.
766.
a).
51113
arct~--- .
758.
2
·¡1ia• + 4b1
211+lf2a•+4b'.
11
3
754.
1115
752.
arctg-- .
751.
nrctg - -- - .
lid~
760.
SCll ct
y~
2cos z770.
+
l.
768.
a
288 cm•.
11: t 2.
l/~ ·
11 cosa
767.
2: :1.
756.
f[2,
11 S cos oc sen et
2
a• (1 + sen a+coso:)
coso:
cos (
-%--{)
sena sen~
4 (1 +sen a ) 11 son (a + ll) sen (a - ~)
776.
sen 2o. sen ~
f +2 sen a.
n
n 1/ifi
778.
780.
--312 sen a
cosoc
17t..
4
781. arccos " .
,,
240
Solucion.s • tndicacto11et
783.
270n cro 2 •
a.
2nci' cos' 2
78.5.
788.
4nRZ (2 + cosa+ 2 sen a)
sen a.
789. 4n: 113. wo. v2 +L 191. a• ¡1+
cos a.
;t(5+411 3)
786.
.
2
")IS )2
+-
-
787.
4n.
y 4 +2 v2 l
.
a•sena.
2
792. 120°. 793. ( VS 1
. 794. eosq> • Sea SO la altura de la
2pirám ide (fig. 213). Del vértice S bajamos la Perpendicular SN a la arista
s
e
B
e
Jlig. 213
Fig. 212
AB y In perpendicular SL a la arista BC. A contim1nción, tracemos las reotas NO y f,O y designemos con M y K sus puntos de inrersección con los
lados CD y AD, respectivamente. Entonces, LSNO - LSLO=LSMO=
=LSKO = q¡, el ASON=t>SOL=t::.SOM=t::.SOK, es decir, ON = OL=
=OM=OK 'Y· por lo tanto el punlo O es equidistante a los lados de Jaba.ce . .
'
di sefl 2et cos.'
795.
q>
T
cos·ip
a? 113
.
• 796', ·acos.et (1+y1+as~n•a) . 797. 7.a'+2ax
x'V4b• - ·a• : 796. ·ab."(l!1 Í.+1). ' 799-. 3 veces.
XV sen Sa.sen a.
xy
V
X
c·os (
802.
aoc-V -cos 2a. · •
.!f+qi) cos ( ·~ -<¡>).
'jn
5en•li:cósi:.t • 807 ·
803.
805.
.8 00.
a3
2.
801.. 2a3 sen ax
a 3 sen
am•.
80".
806.
2
vaa• sen 2a. sen a.
- 2 • 808. - - - -- - 43
2cos•
45° -
X
sen q>
a'(b+c)
(
a
T sen a.
~
)
• 809 •
vsx
...
Tx
S qluclones e indicaci.ones
x
sen~·
_H3
8f L
V sen ( W+ ~ Jsen (~O"-~ ) .
y~ sell. {-f+aoo },$eh (+-:-ao
.
. ..i
cos .. 2·
815.
P(4+tg•a¡ v12+a.1g'··ª
1Hg1 a
,r
1
12 b 3
V
)
. 812. -TM
.~Ii.3~
2
2PQ
2. 818. ah •
821. 1 ; 31. 822.
V
9y•
tg a lg 1:1
•
a
stbc(/1-f-c)scna cosT
t
T;Stgl:lf(S:;cna . 82.5.
+a V2·- ).
832.
tgsa .
cp .
cos..·F coscp.
+
813.
824.
V
.t
0
• ./liV2 Ya <t cos ci>
V tg•a ·
cos~
817.
2;
810.
a3-b>
4
..
827.
2nS
4S 2
Jt/
27.
--cos 2a.
V-
6 cos a
83fi.
1
826. 2nb•(5 +
3
828.
1: 2.
nR• J/2
6
830.
V•· _ b
l'b
e
v¡;4-. V.""b'.
R~a• "V4n•-a•
2'11t•
835.
3Vli+t
naS
a3
836. 48 (4Y2+Yi0). 837. sena.
838. 1:4. 839.
26
840. !)7: Hit. 841. 25; 47. 842. i: 47. 8~3. 7: 20. 844. 7: 17. M5. rn: 23
846. 7:20. 847 . 8 : 37. 848. i :ili. 849. 3:5. 850.
852.
855.
5 : 24.
853.
i
2a V R'-d' . 854.
'Vi
¡¡-.
/a•
n•
J; 8 +-y,
5J/2
- 7- · 851. 3:1.
ilondc
a,¡¡; 2R.
-3 nR lg a tg 2a, donde 90" <a< H2º30'. 856. nRt (t+cos• Za+
3
3
+ 2 sen a sen 2a). 821.
• /
V
S 1 (S 2 -S 1)
acs.+ s,) ·
:itR•ctg (
45º-T)
cos• a
~
1 ) . 860. 2arccos ( --acos
) . 861. arccos (J/2-1). 862. 1: 1; 9: 7.
859. arccos ( - 3
cos4
m- n
2cos•a
n- 1n
, - .
0
863. 2 ercscn-+ . 865. 60.
866. cos 2a lg a . 867. axcscn-+
. 868. v1 3 . t.
m n
11
rn
1
869. 2 arctg
870.
ctg 400 : c\g ano: ctg 20º. 872.
11rclg 112.
7n.
r
4
a
2R'V 1--¡¡sen'T (fig. 214) . Ya que SA=SB= S~ y t...ASB=
=LBSC=LCSA~a, SABC es una pirámide trlangul11r regular. SeaSC=:t.
a
a
a
2
Entonces, CD=:t:SCJ!T, BC=~sen7, AD=YJ:r:scn2y AP=-¡¡ AD=
873.
242
Soluclonu
a
=a2 ,vr-3., sen T.
874.
t
(
2 son
fndicacion''
a~ena
~
b
875.
24 j-1n
'•8
ctgy+i)
1
_
1.
_
"(2- 11a)
876. 12na•(t5-8V2). 877.4a(3-113).878.
Sea que
2
las dos csfer:ss son t.angt-ntcs a los caras quo tienen el vértice común D
(fig. 215). Entonces los puntos 0 1 y O,, que son Jos centros de dichas cs[e-
r as. yacen co lo diagonal 8 1D del cubo. Coroo la esfera mayor es
t~ngentc
e,
Flg. 215
Flf. 214
n lns basca supcrlor e lolcrlor, O,M
n
=z,
a ]12
0 1 l =- /lfD= - 2- .
Hocicndo,
para abreviar, O,N=z, de la scmcjanw. de los tri:lngulos O~KD y 0 1 LD
obtenemos que
880.
0 1 K=zV2y, más adelante, OiD..., zy3. 879.
Tª¡/41.
t
.ryR
(2+ 1¡ -2). 881. 21 a(3-lf3).
882. a (3 ,vr 2-2
v 3).
20 ]fa R
. 884. 17: i25. Sen que la csfora es tangente a fo lJn~supcrior
27
del cubo en el punto T y o las aristas AB, BC, CD y DA de
la bas& Inferior. Los puntos do tangencia do la esfera con las aristas
isoo los puntos modios de ésl.as. Consideremos 111 S()cclón de la eombino.clón do la
esfera con· el cubo con el plano TPQ. donde Pes el punto medio de la orlsta AB;
Q. el l!UDlo medio do la arisl.i CD (flg. 216).. Esta seecí6n también pasa por el
centro ' de 'Ja eslera, el punto O. lntroduzca!DOS el pnrámotro auxiliar OT - R
y, par¡¡ abrevinr, hagamó~ la arista del cubo igu al 11 :c. Entonces del triángulo
883.
OM.Q ·olitenem.o's
~fér!ciis:
qu~ :c = ~. n. I··~cra
del cubo so encuentran cinco segmentos
uoo bajo la base ael cubo y cuatro (quo son iguo.les) por los lados de
las caras !alero.les clel cubo. 885. 8 : 1141. Consideremos la sc~i6o de Ja combinación de una esfera co¡¡ un cubo c.on el plano ORS que pasa por R y S que son
los pun tos medios de dos ·aristas opuestas de la .base del cu bo y el punto O, centro
de la esfera (lig. 217). (P ·y Q son los puntos medios do dos o.ristas opuestas de la
baso inferior t ban quedo.do suspcndirlos-, ya quo el radio do la esfera ·es mayor
que lci:mitod do la arista del cubo.) Introducimos .un parámetro auxiliar, haciendo 'la arista del cubo igual a a. Entonces, M F = al 12. Soguidament.e, analicen
243'
Solucione$. e Indicaciones
los tri~ngulos OKF y OMS. 886. Tt n(5 ',/v 2-6). 887·.
,r
"'
et.
v 2sen 2« cos 2 x
x ·cos ( 45º- ~ )
t
6
.
888. 2a rctg 2
6
.
889. arcse.nññl; n-accsen·ñiñ · 890
vi "891.
. -r-·
a
d
,y¡¡
8
m
.
N
Flg. 216
a lga
893.
2
Flg . 217
(
OI
45o --¡-
X ( scn<:J.+
, sen«
)V
-ª
6VtgT.
• /
t
V f-ycos• o:
)
900.
• 902.
~
•
.898.
tg
1tH<:J.
~
45°ccs
bnse
904. ·
n-1
arccos ,.
+2
en
,
sen(~ - ~)
lo.
<:J.
a
/
48 J
tg
vCQ;'(;
24a
,
a•
b•-2
~.
•
vrns 2 +45a•
895.
a.-n
3
9 tg-• {j
-~
CtgS~
tg<p
2
4
896. 1: 1; 9: 7. 897. aR3
Xtg
2v3mg~
{j
8!1'.
v' 3-4 sen• o:
F
89!l.
t
2 x
901. 2vzn•cosa.x
903.
En la arista de la
2
nristn
lateral
9os.1f ibscn%-sen (45º-f) (íig. 218). ScaFL=:r,
sen(~+~)
r.nll•nccs también OL=:r.. Del triángulo MNK (en el que cslñ inscrita lo. cir·
cunCorencia que es la base del cilindro) LK.=z ctg ~
y
riel triángulo SAO,
AO=bcosa. En tal caso, AL=bcosa. - z. Pero, AL=LK, o sea,
ex
•
2<:J.+ll
]{ sen -.-2-
2a.-ll
cos - - 2 -
3 112 -
bcosa-z~
=zctg
scn•a
. 907. ----¡¡-ª· Examinemos
2 . 906.
la sección del tetraedro con el plano que pasa por los vértices A, B y el punto
medio F de la arista SC (lig. 219). En el t.iángulo ABF AB= "· Entonces,
Soluciones e indicaciones
24~
AF=BF=
"r¡j
y DF= a.
rz .
Como el triángulo ABF está inscrito en una
circunferencia, O e.~ el plmto de int.crsccoi6n de las perpendiculares a los punt os medios de sus lados. De la i;cmejanza de los triángulos FOM y FBD
.
OF
MF
(M es el punto medio del lado BF) obten~mos: Bf<' = DF • 908. G cm.
5
e
Ftg. 219
Fig. 218
909.
1: tg ª
9
3
4(lf 3+tga)3
. 910. L
a V3 _ _ , R =
a V3
.
- - -'---'-....;....
. Exa·
CI.
r
CI.
a .rC(ls-z+l 2scny
ctg-z+ y 2
minemo$ la sección diagonal del cubo. Ella será, además, la sección axial del
B
p
D
D
A
Fig. 221
Fig. 22Q
cono inscrito .cil el .cubo ((ig-. 220) (el punto M no yace cu la arista DDtt ya
que el con!J"no c:s tangente a las aristas del cubo). Es evidente que BB1 =a,
B.O=ciV2, a·,:z>=alf3-. Sea .PQ.=:r. .Entonces, del triángulo rectángulo B 1 PQ
tendremos B1Q""Z ctg -~ y de la. semejanza cie los triángulos DPQ y DBB 1
.fQ ~ DQ
obtyndremos
!'81 - BD • ~ ll .
n2 V6
4
912.
nH' scn(2c.-P)!lC!nP
3sen•c.sen•(c.-P) •
245
Soluclonts t indicaciones
v'Qb
913 arccos - -
914·
a v12+n1
(!ig. 221). Como BC=B0.-1-0 10,+
·
" · · · 21/3n
+o,c=i' t'12 + 2;
=.~ y AD=a, ,BC=.AD,
+:
es d,ecir, las circun.feroncias
y ro2 son tangentl!s a l o!! lados laterales del rcct.~ngulo. De 1.a scmejania
de los triángulos 0 1M P y 0 1M N se ·désprcnde que ·g:~
.g:~ dE\ donde
<ili
,
=
O,M =
~ . Pero,
O,N=: , en
tal caso LO,MN·=S-0.0 y , ·por lo tanto, si se
construye 1~ recta KN 11 BC y la recta Pkll AB1 en el triángulo
PNK LPÑK =;:
Q .-----~------.;.:,.
B
D
Á
p
L
(a)
FI¡;. 222
= 30° . A continuación, 0 1 M =
PN = a
9 15
•
9 19.
b
3
rs
VS
150
V
y
de~
a
.
a 113
6 , 1lcspuésPM = i 2 y
t riángulo PKN KN= ;
a
Jf ii
MN = -1.-,-, osca
y, más adelante, PK =
ª r~.
917 (
nbc
)~
¡,yzscn2a.
918
48 ·
·
ab+ac+bc
·
4sen(45º+a) ·
·
(fig. 222, a, b). Intro<lui.camos el 1>arúmctro auxiliar, haciendo
3
916
·
5a
'
Bn'
3n•+4
PL=Za. Entonces, AB = a. Como el cuadrado PQML $C enrolla en un cilindro,
2nOP=2a, de donde OP=~ y, por lo tanto, AB=~. Al enrollar el
n
n
a
Jl3
cuadro la longitud del segmento CD no varin, o sea, CD= - ·- - . A conli·
2
nuación, del lriángulo CFD CF= ~ VSn'+4. 92f, a) 1/ 2 cm•; h) 6 cm 2 •
922.
P ¡15
-¡o-.
,r
112
-
923. 3 v 3 cm. 924, arctg-- . 925. arctg V2,
2
928. 2 cm. 929.
3V3 cm•,
12 cmi. 932. Rll si Rs:;.H:
nitui
926.
8 m.
si R>Il-
Soluciona e ln<Ucaclones
246
933.
..
/3V
V 5ñ.
~
93G. 11=
937. a) ZR
961.
946.
9~9.
9M. a)
p
3p
3p
2 ' T .T ;
b)
3p
T .
3p
4p
/2
-5- • 5. 935. 2n '), 3.
, donde Ji es la altura del cilindro, /l, la altura del cono.
rj ;
b) R
vi.
8
,)3
270
3-1 . 942.
X3 R'.
32
16113
- -3
4
.
9
943.
947. arctg(cos:
>
CUI •
9a0,
":~.
038. a), b)
16
81 V.
951.
939. n) 4R; b) R
-15°.
944.
y2(V2+ i)).
1
U
Y.
1
3.
v2. 940. RV3.
r.
94a,
948. R=
-
R
Vi3
1- 3- .
~. 11- aa.
BIBLIOGRAf lA
i. .IJuM1u<_un JI. JI., Mop8icosu~ A. I'·. , l<ynu1f1CaJ1 E •.•C .• MaTeMaTnqcc.1<uií
a11anna. ~111fl<jiepeB11l!lln.i.noé· 11c'U1cnca1ic. M.: flpoc!'~l.l\et;u1e, 1984,, ps c.
(N; Ya. V._ilenkiu, ¡<I. G. .Mordkóuich, E. S. Kut1tiská.ya. Análisis matemátic():.
Cálculo dilcrenclal.)
2. I'eo.1tempu11. Y'le6noe ooco6ne AJill 6-8 nnaGCoa/noA peJJ,. A. H. ltont.1oropóBS. M.: flpocoe11tee11e, 1982, a84 c. (Geometrla. Manual didáctico para
los grados 6-8. Dirigido po1 A.' N. Kolmogórov.)
3. Jlnmau1UH.ico B. /l. Jl_pa1<T111<yM no pe1ucu1110 aannq w1<oni.uoA MaTe~mTlll<U. reo•1erp11il M.: Ilpocoe~enne, 1982, t60 c. (V. N. Lllulnenko. Práclicas para resolver problemas matemáticos en Ja escuela. Geometría.)
4. Hecmtpt1u10 10. B., O.te.r:nuie C. Jl. 1 Uomanoa M . 1(. 3ana'lu scTyneTenbe1>1x a11au1eoon no \1&Te)1aTnl\e. M.: Hay1<a, 1983, 448 c. (Yu. V. Nesltrenko,
S. N. Olejnik, M. K. Potápo11. Problemas para Jos c:icámencs de ingreso do
matemáticas.)
5. Doaopt.toa A . B. reo•1erp1111 e 6-10 1<naceax. M.: npocseUJ,emie, 1983,
288 e. (A. V. PogorUlou. Geometría para los grados 6-10.)
6. C6opnu" 11oaKypc1r1o1x aa11aq no Ma:ro~taTH KO ¡vtR no c-ryoalO~X ao
BTY3J.1/00A pc11. M. C1<11eaa11, 4-o 113,11.. M.: B b1cwa11 w11011a, 1980, 542 c. (Compendío de problemas de concurso de matemáticas para los estudíanles que ingrusan a Jos C:!!ntros do cnse!lanza superior. Dirigido por M. Sknnnví.)
A N UESl'ROS LECTORES:
cl\fi r• edita libros soviéticos traducidos al español, inglés,
írancé~, árabe y otros idiomas extranjeros. Entro cUos figuran
las mcjorc.s obras de las distintas ramas de la ciencia y la técnica,
manuales para Jos centros de enseñanza supel'ior y escuelas tecnológicas, literatura sobro cícncias naturales y médicas. También
se incluyon monografías, libros de divulgncion cinntífica y cioncía-ficción.
Dirijan sus opinio11Cs a la l~ditorial l\!ir, l Hízhskl pcr., 2,
1291!20, Moscú, I-HO, GSP, Ul\SS.